Olimpiade? Insyaallah, Ada Jalan {Cerita Sebelas Sahabat} SABAR SITANGGANG
Cetakan Pertama, Desember 2014
Penyunting: Aba Rayhan Ilustrasi Sampul: Aba Rayhan
Diterbitkan oleh Penerbit Pustaka ar-Rayhan Jl. Mandor Basyir II 28 M RT 04 RW 08 Kukusan, Beji – Depok Jawa Barat 16425 Surel:
[email protected]
Perpustakaan Nasional: Katalog Dalam Terbitan (KDT) Sitanggang, Sabar Olimpiade? Insyaallah, Ada Jalan {Cerita Sebelas Sahabat/Sabar Sitanggang; Penyunting, Aba Rayhan, Depok: Pustaka ar-Rayhan, 2014-12-13 ISBN 978 – 979 – 18028 – 6 – 4 I. Judul
II. Aba Rayhan
Isi Novel Awalnya ........... 1 SMP Shidiqqin 9 “Makhluk” Apa Itu? 13
What’s in a Name? 19 Kakek Penjaga Musholla 24 Selepas Shalat Ashar 33 Tujuh Prinsip Dasar 38
Wow, Fantastic Appetizer! 45 Dari Delapan Penjuru Angin 70 Surat Misterius RRHS 114 Kesebelasan Genk OSN 142
l’histoire se répète 164 Kombinasi yang Indah 181
The Queen of the Problems 247 Tabel, Grafik dan Coba-Coba 317 Lima Lontar yang Mengharukan 405 Akhirnya ........... 460 Sumber Inspirasi 467
“Mathematical proofs, like diamonds, are hard as well as clear, and will be touched by nothing but strict reasoning” --JHON LOCKE--
“If I have seen farther than other men, it is because I have stood on the shoulders of giants” --ISAAC NEWTON--
1
Awalnya...........
“As-salaamu’alaikum!”terdengar ucapan salam bersamaan dari beberapa orang di telingaku. “Wa alaikum Salaam! Eih teman-teman Rayhan. Masuk Nak, masuk! Wah ada apa ini, pagi-pagi sudah rombongan dan pada rapirapi lagi?”jawabku. “Alhamdulilllah, kebetulan nih, Paman sedang di rumah. Justru kami kemari mau ketemu sama Paman,”jawab salah seorang teman anakku yang kutahu bernama Naufal. “Duduk dulu, nanti kita bicara enak. Tapi, lesehan ya! Ini a la Jepang! Tanpa kursi,”jawabku pada teman-teman anakku itu. “Mak, ada teman-teman sekolah Rayhan datang. Buat minuman, ya!”ujarku pada istriku yang sedang di dapur, tak jauh dari ruang kami berbincang. “Paman, sebenarnya kami kemari dengan harapan besar. Padahal Rayhan sudah bilang “janganlah!”. Tapi kami memaksa untuk kemari. Ini serius lo Paman,”kata Saffana, satu-satunya perempuan dalam rombongan itu. “Kelihatannya serius banget, sampai serombongan datang begini,”pancingku. |1
“Iya, Paman. Ini memang serius. Dan tampaknya Paman harus bantu, dan hanya Paman yang kami kira bisa membantu ini,”kali ini Zaidan angkat bicara. “Jangan berlebihan, anak-anak. Kalian boleh bicara sepuasnya, insyaallah Paman akan dengarkan!”jawabku pada anak-anak itu. Tampak kelima anak itu saling pandang. Tampaknya mereka akan mengutarakan sesuatu yang besar, setidaknya menurut mereka. Aku hanya menunggu. “Begini
Paman.
Sebenarnya...,”Naufal
mulai
membuka
perbicangan serius. “Wah, kita kedatangan tamu besar, nih!”tiba-tiba istriku menyela pembicaraan yang mulai kami lakukan. Istriku datang membawa senampan besar gelas dan teko serta penganan kecil. “Pokoknya isi perut dulu, baru bicara. Bicara itu soal gampang. Tapi, kalau perut kosong bisa berabe!”kata istriku pada anak-anakku. “Nah, yang ini aku setuju seratus persen Bibi!”tiba-tiba Saed yang sedari tadi diam, mulai bicara. “Dasar kau Saed, kalau sudah soal makanan, otakmu langsung encer!”sindir Saffana. “Ha...ha..ha. Tak apa. Bibimu betul juga, dan Saed pun tak salah. Banyak soal besar selesai di meja makan, tapi banyak persoalan kecil jadi masalah besar hanya karena soal makanan pula, anak-anak!”kataku menengahi. “Nah, apa kubilang. Betol, kan! Soal makan, itu soal penting, Kawan!”komentar Saed lagi. |2
Kami menikmati sajian pagi itu bersama. Teh Manis Kental Hangat dan Pisang Kepok Rebus. “Ayo, silahkan! Silahkan dinikmati anak-anak. Oh ya, mana Rayhan Mak?”tanyaku pada istriku. “Tadi mau jemput Yaseen katanya, Pak. Mungkin beberapa saat lagi sudah kembali,”jawab istriku. Kulihat ada hal yang mengganjal pada anak-anak itu. Mereka sepertinya memendam sesuatu yang besar. Hanya Saed yang tampaknya tak terpengaruh. Dia menikmati Pisang Kepok Rebus dan Teh Manis Hangat itu seperti menikmati alunan musik klasik. “Coba ceritakan pelan-pelan apa yang menjadi persoalan kalian, anak-anak!”tanyaku datar. Kali ini tampaknya Saffana yang akan jadi juru bicara. Perempuan kelas 8 Sekolah Menengah Pertama ini menarik nafas dalam-dalam, mengeluarkannya perlahan, dan mulai menggerakkan bibirnya. “Sebenarnya begini, Paman. Insyaallah 4 bulan ke depan sampai pertengahan tahun akan digelar Olimpiade Sains Nasional, OSN. Olimpiade ini diselenggarakan oleh kementerian pendidikan dari tingkat Sekolah Dasar sampai Sekolah Menengah Atas. Bahkan kami dengar, konon sampai tingkat Mahasiswa, Paman. Mendengar berita itu, kami bertujuh kebetulan sedang membahas sesuatu, dan akhirnya sampailah ke topik OSN. Kesimpulannya, kami berharap bisa ikut dalam ajang kompetisi nasional itu,”papar Saffana sopan. “Wah, bagus itu anak-anak!”jawabku singkat. “Ya, tapi ada masalah Paman!”kata Saffana lagi. |3
“Wajarlah setiap hal mengandung masalah. tinggal bagaimana kita menyelesaikan masalah itu, Saffana!”jawabku lagi. “Masalahnya, sumber daya kami terbatas, Paman. Guru-guru yang ada, seperti Paman ketahui sendiri, kan! Sementara kompetisi berupa olimpiade itu membutuhkan persiapan ekstra dan latihan yang serius. Kami belum menemukan jalan keluarnya. Sampai....”Saffana berhenti menjelaskan. “Sampai apa Saffana?”aku mulai penasaran. “Sampai ayah Pakavi berkata pada kami. Katanya, Paman dulu waktu kuliah, satu kost dengan beliau di Jogja. Dan katanya Paman Jago Matematika. Ayah Pakavi meminta kami datang ke Paman untuk bersedia memberi petunjuk dan arahan. Syukur-syukur Paman bersedia membimbing kami untuk mempersiapkan diri menghadapi OSN
itu.
Oh
ya
Paman,
kami
fokuskan
pada
bidang
Matematika,”papar Saffana. “Ah, ayah Pakavi itu berlebihan. Paman hanya gemar Matematika saja. Dan itu pun dulu, sekarang bukan apaapa,”jawabku masih datar. Kelima anak itu salin pandang. Mereka tampak bingung. Entah kenapa tiba-tiba keterangan Saffana dan kedatangan rombongan anak-anak teman sekolah anakku itu membuatku menarik nafas panjang. Kupandangi wajah-wajah polos yang hadir di hadapanku pagi itu dalam-dalam. “Apa benar kalian serius, anak-anak?”tanyaku. “Insyaallah kami serius, Paman. Tekad kami sudah bulat. Itulah sebabnya kami memberanikan diri datang ke sini, meski Rayhan sebenarnya melarang kami,”jawab Zaidan sigap. |4
Tampak Naufal, Pakavi, Saed mengangguk. Sementara Saffana hanya tertunduk. “Kalian sudah sholat Dhuha, anak-anak?”tanyaku. “Belum, Paman!”terdengar jawaban serentak. “Sekarang kalian berwudhu’, kemudian sholat Dhuha di Masjid depan itu. Nanti, selepas sholat, kalian membuat dua kelompok, terserah siapa masing-masing anggota kelompoknya. Setelah
itu
kembali
kemari,
insyaallah
Paman
punya
kejutan!”kataku. Kulihat kelima anak itu saling pandang. Tapi tak lama, mereka segera beranjak meninggalkan rumahku menuju Masjid yang hanya beberapa meter persis di depan rumahku. “Mau Bapak uji apa mereka? Kasihan, Pak! Bantulah mereka, luangkan waktu!”pinta istriku sambil berjalan mendekat. “Insyaallah, Mak. Tapi, mereka harus melalui proses yang tak mudah. Sudah kau siapkan kopiku? Biasa, kopi tanpa gula, Mak!”tanyaku pada istri. “Alhamdulillah sudah. Itu di samping Bapak!” Menghirup secangkir kopi di pagi hari adalah kebiasaan yang aku warisi dari Bapakku di kampung. Bedanya, kalau kopi Bapakku harus Panas, Kental dan Manis, aku tidak. Aku memilih kopi tanpa gula. Soal jenis kopi, aku beruntung lahir dari keluarga petani kopi. Bapak dan Mamakku adalah petani kopi di Kintara, sebuah tempat di daerah penghasil kopi terbaik di Pulau Sumatera. Sampai mereka harus meninggalkan kampung itu, tradisi ngopi tetap mereka bawa. Dan aku dilahirkan dengan kebiasaan seperti itu. Hmmm… aroma kopi Arabica ini sungguh nikmat. |5
“As-salaamu’alaikum!”terdengar
suara
anakku,
Rayhan,
mengucapkan salam. “Wa alaikum salam. Dari mana, Rey?”tanyaku. “Jemput Yaseen, Pak! Kami langsung kemari. Katanya, kawan-kawan Rayhan datang kemari, ya Pak?”tanya anakku balik. “Betul Paman, tadi Rayhan ke Rumah Yaseen. Paman sehat, kan? Oh ya, ada salam dari Ayah buat Paman,”kata anak muda berkulit putih bertubuh ceking ini. “Wa
alaikum
salam.
Alhamdulillah,
Paman
sehat
Yaseen,”jawabku. “Kawana-kawan Rayhan ada di mana, Pak?”tanya anakku. “Mereka sedang di Masjid, sholat Dhuha. Kau sudah sholat Dhuha?”tanyaku. “Belum, Pak!”jawab Rayhan. “Sekarang, kau dan Yaseen gabung dengan kawan-kawanmu, sholat dan segera kembali!”kataku pada anakku. Rayhan dan Yaseen berjalan menuju ke Masjid. …∑πχαΩ…
Sekitar 15 menit berlalu, Rayhan dan keenam kawannya kembali ke rumah. Kali ini tampaknya mereka sudah siap dengan’ujian’ yang akan aku berikan. “Paman, seperti perintahmu, kami sudah membagi menjadi dua kelompok. Kelompok 1 terdiri atas Saffana, Yaseen, dan Naufal. Sementara Kelompok 2 terdiri atas, Pakavi, Zaidan, Rayhan dan aku sendiri, Paman!”kata Saed yang tampaknya dijadikan Juru Bicara. |6
Aku menarik nafas panjang. Kupandangi satu per satu wajahwajah polos di hadapanku. “Baik, sebagai bentuk keseriusan kalian, Paman akan ajukan 2 pertanyaan. Masing-masing kelompok mengerjakan 1 soal. Setelah yakin selesai, beritahu Paman! Kalian harus menjelaskan di hadapan kita semua!”kataku. Ketujuh anak itu tampak heran. Tapi aku tahu, mereka tak punya pilihan. Dan aku hanya menguji mereka. Istriku tampak senyum-senyum sambil menggeleng-gelengkan kepalanya. “Baik, Paman. Silahkan berikan kedua soal itu kepada kami. Insyaallah kami akan mengerjakannya sebaik mungkin,”jawab Saed. Aku berjalan menuju rak buku tua. Aku ingat, di antara susunan buku-buku itu, aku pernah menyelipkan selembar kertas yang berisi catatan tentang soal Matematika. “Anak-anak. Ini soal untuk Kelompok 2. Dalam bahasa Inggeris, ya! Dengarkan baik-baik, kalau perlu catat! ‘The five-digit number 2xy89 is the square of an integer. Find the two-digit number xy’. Dan soal untuk Kelompok 1 adalah, ‘predict the next term in the sequence: 1, 14, 51, 124, 245, 426, ...! Paman percaya kalian mengerti maksudnya dan bisa memahami arah pertanyaan kedua soal itu. Selamat mengerjakan!”kataku. Ketujuh sahabat itu langsung memecah menjadi dua kelompok. Masing-masing sibuk membaca, menganalisa dan mencoba-coba. Aku hanya memandang tinghkah-polah mereka dengan senyum. Istriku, sekali lagi geleng-geleng kepala dan senyum-senyum. Kali ini gelengan itu ditujukan hanya untukku. |7
Tiba-tiba, menyaksikan ketujuh anak-anak ini bekerjasama menyelesaikan soal yang kuberikan, membuatku teringat akan peristiwa yang telah lama berlalu. Lama sekali. Peristiwa, dimana saat itu, usiaku pun seusia dengan anak-anak yang hari ini duduk bersila di hadapanku. Inilah ceritaku, cerita tentang sebelas kawan yang cinta pada Matematika, juga cerita tentang sebelas kawan yang berusaha mewujudkan mimpi-mimpinya. Dan aku beruntung bisa berada di tengah-tengah mereka, menjadi saksi kesebelas anak jenius ini berdiskusi, berdebat, bermain, dan bercerita. Dan inilah ceritaku, cerita tentang Ayra, Amer, Bily, Bintang, Denan, Fateh, Ikram, Iman, Saver, Yodha, dan Sabiq. Sebelas anak “jenius kampong” yang mencoba untuk merangkai asa, menggapai mimpi, dalam satu bahasa, Matematika!
…∑πχαΩ…
|8
2
SMP Shidiqqin Untuk ukuran sebuah Kecamatan, daerah ini tergolong luas. Bukan hanya terdiri atas banyak Desa, tetapi memang wilayah tiap desanya pun luas-luas. Dari segi sumber daya, Kecamatan ini beruntung. Ia berada pada posisi bordertown, sebuah posisi yang strategis bagi perubahan sebuah kawasan. Tapi, tidak hanya itu. Kekayaan sumberdaya alam yang melimpah, pertanian dan perkebunan khususnya, telah menjadikan kawasan ini menjadi target perkembang kota. Terletak sekitar 45 km ke arah Timur dari pusat kota, desa itu begitu pas untuk melukiskan segala hal yang disebut kampung. Hamparan sawah yang luas, dengan padi yang sebagian mulai menguning pada waktu-waktu tertentu dan bagian lainnya masih tampak hamparan hijau nan luas, menjadi pemandangan yang lazim. Perempuan-perempuan setengah baya dan laki-laki gagah yang mengangkat cangkulnya adalah pemandangan yang umum dijumpai. Belahan sungai yang membentang dari Utara ke Selatan, seolah membagi rata desa itu, dengan aliran air yang jernih melewati persawahan dan perkebunan tebu rakyat, seolah menjadi magnet yang tak akan melepaskan besi, bagi mata yang memandangnya.
|9
Makin jauh ke desa, terdapat perkampungan yang dihuni cukup padat untuk ukuran orang desa. Wajah-wajah polos anak-anak desa yang berlarian di pematang sawah dan pinggiran perkebunan tebu akan menjadi pemandangan lain, saat lembayung mulai tampak di cakrawala. Bila gelap menyapa, maka alunan suara anak-anak mengaji dan lantunan Selawatan di beberapa surau dan langgar adalah musik abadi sepanjang hari, bulan, dan tahun. Terus begitu, begitu, dan begitu. Saat hari berganti pekan, pekan menjadi bulan, dan bulan menjadi tahun. Pemandangan rutin lain yang tak terlupa, seiring dengan derasnya perkembangan dan kemajuan serta tingkat pertumbuhan penduduk yang tinggi, tampak sejak fajar mulai muncul di ufuk, rangkaian mengular ribuan anak manusia usia pekerja mengayuh sepeda mereka, seakan berlomba dengan semburat cahaya mentari bergerak dari Timur ke Barat. Di desa ini hanya ada dua SMP Negeri, SMP Negeri 1 dan SMP Negeri 2. Sepanjang tahun, persaingan antara keduanya menjadi ajang yang mudah dirasakan oleh seluruh siswa, guru dan orang tua. Posisi yang dekat dengan pusat desa lebih menguntungkan bagi SMP Negeri 1, sementara SMP Negeri 2, cenderung underdog! Meskipun begitu, masih ada beberapa sekolah partikelir alias swasta yang dibuat oleh masyarakat, mulai dari tingkat Sekolah Dasar (SD) sampai Sekolah Menengah Atas (SMA). Dan di antara sekolah-sekolah menengah itu, ada sekolah menengah tingkat pertama yang bernama Shidiqqin. Entah apa yang melatari pemiliknya memberi nama itu, yang pasti ia termasuk pendahulu | 10
lahirnya sekolah-sekolah swasta menengah di desa itu, dan memilih nama yang tak lazim untuk zamannya! SMP Shidiqqin terletak di tengah sawah. Di bagian Barat dibatasi oleh sungai, sementara di sebelah Timur dan Utara terhampar persawahan, dan jalan desa menjadi batas sekolah itu di bagian Selatan. Sebagai sekolah partikelir yang memang pas-pasan, maka tampilan sekolah ini pun tak jauh berbeda dengan sekolahsekolah partikelir lainnya. Deretan Mahoni tua, beberapa pohon Jati, dan pohon Pisang Kepok, tampak di sekeliling sekolah yang hanya berpagar kawat berduri yang sudah berkarat itu. Tak ada yang istimewa dari SMP Shidiqqin ini, kecuali 2 hal, yaitu: Tambahan pelajaran kewajiban mengahafal beberapa juz al Qur’an bagi siswa-siswinya dan Pak Tumanggor! Soal kewajiban menghafal beberapa juz ayat al Qur’an bagi setiap siswa dengan pola tertentu. Pola menghafal dimulai dari Juz 30, Juz Amma, dan seterusnya maju ke juz-juz berikutnya. Tak heran beberapa di antara siswanya ada yang sudah hafal 6 juz, 5 juz, 4 juz, 3 juz dari 30 juz al-Qur’an. Sekolah ini percaya bahwa semakin banyak
hafalan
Qur’an
siswa-siswinya,
Insyaallah,
prestasi
akademiknya meningkat. keyakinan yang berisiko untuk zaman itu! Sementara soal keistimewaan kedua, karena Pak Guru ini memang Istimewa. Pak Tumanggor adalah Guru Matematika sekaligus Guru IPA. Dan, bukan hanya itu, Pak Tumanggor adalah juga Kepala Sekolah SMP Shidiqqin! Tumanggor adalah jenis marga dari etnis suku Batak, khususnya Batak Fakfak. Batak Fakfak merupakan kelompok suku Batak yang habitat asalnya di Kabupaten Sidikalang. Diantara suku | 11
Batak, selain Batak Mandailing, suku Batak Fakfak umumnya dalah beragama Islam. Dan Pak Tumanggor, adalah salah satu diantaranya. Senyum khasnya, dengan tampilan yang susah marah adalah ciri yang melekat dalam diri Pak Tumanggor yang postur tubuhnya, kurus tinggi dan berkulit putih itu. Tapi, lebih dari semua itu, sesuatu yang lain, juga sedang bergeliat di sekolah ini! Lebih tepatnya, ada semangat yang membara mendesak keluar dari dada beberapa orang siswanya yang rindu akan perubahan. Perubahan menuju ke arah yang lebih baik. Ya, sebelas anak-anak “jenius kampong”, berkumpul, membincangkan masa depan mereka dan sekolah mereka, melampaui usia dan desanya, mereka
bicara
tentang
persoalan
event
nasional,
bahkan
internasional. Jenius-jenius lokal yang lahir dari kesederhanaan dan kepolosan yang berpadu dengan rasa ingin tahu dan penasaran mendalam. Dan jenius-jenius lokal itu, ternyata memang punya catatan tersendiri. Kemampuan masing-masing mereka memang berbeda, namun masing-masing pun memiliki keistimewaan sendiri pada bidang tertentu. Tapi, di atas semua itu, mereka memiliki satu kesamaan. Kesamaan yang tak bisa ditawar-tawar! Kesebelas anak jenius kampong itu, masing-masing memiliki beberapa juz hafalan surat dalam al-Qur’an di kepala dan di hati-hati mereka. Ya, paling sedikit, mereka menghafal 2 Juz! …∑πχαΩ…
| 12
3
“Makhluk Apa Itu?” Seorang siswa laki-laki sebuah sekolah menengah sedang memasang telinganya serius. Tampaknya, ia sedang mendengarkan sebuah perbincangan yang menarik baginya. “Senang ya kalau kita bisa ikutan dalam lomba bergengsi tingkat nasional itu,”begitu terdengar suara itu jelas di telinga anak laki-laki itu. Perbincangan beberapa kawannya yang sedang duduk di bawah pohon Mahoni tua di halaman samping sekolah ternyata benar-banr mengusik pikirannya. “Lomba Bergengsi Tingkat Nasional,”pikir anak laki-laki itu. “Ya, tapi ndak bakal mudah untuk bisa lolos seleksi, Kawan! Ketat sekali persaingannya, Bro!”terdengar lagi suara dari seorang yang sangat dikenal anak laki-laki itu. Suara Ikram, kawannya sekelas. Karena penasaran, anak laki-laki itu pun akhirnya mendekat ke arah kawan-kawannya yang sedang asyik berbincang tentang sebuah lomba bergengsi “Assalaamu’alaikum, Bos!”sapa anak- laki-laki itu agak keras hingga membuat kawan-kawannya yang sedari tadi asyik berbincang terkejut. | 13
“Wa alaikum salam, Biq. Haaa… pas kali kau datang, Kawan. Ada hal serius yang harus kita diskusikan,”kata Amer. “Haa… serius, soal apa pulak itu?”tanya anak laki-laki yang dipanggil Biq itu. “Ini soal lomba bergengsi tingkat nasional, Kawan. Olimpiade Sains Nasional. OSN, Bro!”terang Amer lagi. “OSN, makhluk apa itu, Kawan?”tanya Sabiq memasang wajah penasaran. “Aaah, kau terlalu ini banyak baca buku. Kurang gaul!”Ikram kembali menimpali. “Aku pun tak tahu pastinya. Tapi intinya lomba pelajaran tingkat nasional untuk beberapa mata pelajaran. Ada tingkat SD, SMP, SMK, dan SMA. Bahkan sampai tingkat Mahasiswa, Biq. Nah, kami sedang diskusi tentang bagaimana caranya sekolah kita bisa ambil bagian. Dan bukan hanya itu, tapi bagaimana caranya agar bisa menang Kawan!”papar Amer bersemangat. Anak-laki yang dipanggil Biq itu mulai merasakan aroma semangat kawan-kawannya menyebar dari bawah pohon Mahoni tua samping sekolah itu ke segala penjuru. Ya, nama lengkap anak lakilkai yang dipanggil Biq itu adalah Sabiq, dan dia mendapat tempat khusus di antara kawan-kawannya. Meskipun kemampuannya, hanya ‘biasa-biasa saja’. “Oooh, begitu. Itu tampaknya memang hal serius, Mer,”jawab Sabiq. “Lantas, apa pendapatmu, Biq?”tiba-tiba Yodha, anak laki-laki lainnya yang juga duduk di bawah pohon Mahoni tua samping | 14
sekolah, yang sedari tadi hanya mendengar dan asyik dengan permainan kubiknya bersuara. Tampak Sabiq berusaha berfikir. Ia berfikir keras, dan berfikir. “Kita
harus
ajak
dua
teman
kita
lagi
untuk
memecahkannya?”ujarku. “Maksudmu, Biq?”tanya Ikram “Untuk soal OSN ini, kita harus libatkan Fateh dan Ayra, Kawan!”kata Sabiq. “Aku setuju itu. Itulah manfaatnya kau ada dalam diskusi ini, Kawan.
Kapan
konkretnya
kita
ajak
Ayra
dan
Fateh
berembug?”tanya Amer. “Sesegera mungkin, Kawan. Sesegera mungkin. Lebih cepat, lebih baik,”ucap Sabiq sambil tersenyum. “OK, kalau begitu selepas sekolah nanti, kita temui Ayra dan Fateh,”kata Ikram penuh semangat. Bunyi lonceng khas sekolah di sekolah itu, lonceng yang dibuat dari potongan pipa baja sepanjang 45 sentimeter
dan
berdiameter 15 sentimeter yang digantungi potongan besi terdengar keras. Dua-kali dua-kali. “Teng-teng, Teng-teng”. Hal itu menjadi tanda bahwa waktu istirahat kedua sudah selesai, dan pelajaran terakhir hari itu akan segera dimulai. Keempat anak laki-laki yang terlibat diskusi ringan tapi serius itu pun tampak bubar. Masing-masing menuju kelas. Terlihat wajahwajah keempat anak itu begitu cerah. Ya, mereka akan berbuat sesuatu. Sesuatu yang tak pernah mereka bayangkan seperti apa nantinya. | 15
…∑πχαΩ…
“Teng, Teng, Teng, Teng, Teng, Teng….,” lima kali bunyi lonceng sekolah itu dipukul. Artinya, pulang! “Alhamdulillah,”suara riuh terdengar hampir serempak dari masing-masing ruangan yang ada di sekolah itu. Di salah satu bagian samping halaman luar sekolah, di bawah pohon Mahoni tua yang rindang, tampak empat anak laki-laki sedang menunggu sesuatu. “Aah, lama sekali kawan-kawan kita itu,”Amer mulai tampak bosan. “Sabar, Kawan. Pada saatnya, mereka insyaallah akan muncul juga dari gerbang itu,”kata Ikram. “Kau sudah lapar, Mer?”goda Yodha pada Amer. Dan benar saja. Tak sampai hitungan menit, seorang perempuan muncul dari gerbang utama. Dia, Ayra. “Ayra, tunggu!”suara khas Amer terdengar jelas diantara riuh suara murid-murid yang keluar dari gerbang sekolah. Perempuan yang dipanggil Ayra itu berhenti. Ia mencari asal suara yang memanggil namanya. “Ayra, di sini Kawan!”kembali Amer bersuara sambil melambaikan tangan ke arah perempuan yang dipanggil Ayra. Kali ini Ayra menatap ke arah lambaian tangan. Seperti telah paham maksudnya, perempuan itu segera menuju ke arah kami. “as-Salaamua’alaikum,
kawan-kawan.
Ada
apa
ini?
Tampaknya serius sekali,”tanya Ayra kepada kami. “Pokoknya serius!”celoteh Amer. | 16
“Apa hal serius itu, kawan?”tanya Ayra tampak bingung. Sabiq, mendapat tugas menjelaskan apa rencana yang tadi telah disepakati. Setelah beberapa saat. Tampak perubahan di wajah Ayra. Seulas senyum tersungging di bibirnya. “Insyaallah, aku bersedia. Bahkan, ini yang aku tunggu. By the way, Fateh bagaimana?”kata Ayra. “Ya, kita memang sedang menunggunya. Dia belum tampak keluar,”kata Yodha menimpali. Selang beberapa saat setelah kelima sahabat itu berbincang, seorang laki-laki seumuran mereka tampak keluar dari pintu gerbang. Dia Fateh, laki-laki yang ditunggu oleh kelima sahabat itu. “Fateh, kemari dulu kawan!”terdengar suara lantang Amer. Tak perlu waktu lama, Fateh bisa mendengar dan tahu asal suara panggilan itu. Dia menoleh dan segera beranjak menemui sahabat-sahabatnya itu. “Wah, ada hajatan apa ini? Lima Jawara Sekolah berkumpul. Pasti ada sesuatu yang seru, nih!”Fateh memulai pembicaraan. “Justru itu, Kawan. Kita mau melengkapkan menjadi enam. Dan itu, kau!”ujar Amer. “Tunggu, tunggu dulu! Ada apa ini, kok namaku masuk daftar. Bukan daftar Teroris yang dicari-cari, kan?”canda Fateh. “Sudahlah, kawan, cukup bercandanya. Saat ini kita serius. Dan untuk ini, Sabiq yang akan menjelaskannya padamu. Ayo, Kawan!”kali ini Ayra mengambil inisiatif. Seperti biasa, Sabiq menjelaskan apa yang telah mereka rencanakan sebelumnya. Tak perlu waktu lama, Fateh tampak | 17
mengangguk-angguk dan memasang wajah berseri-seri. Tampaknya ia pun setuju dengan ide ini. “Alhamdulillah, akhirnya kita bisa belajar bersama. Bismillah, aku ikut dalam tim,”ujar Fateh mantap. “Alhamdulillah! Mudah-mudahan ini awal yang baik untuk langkah kita ke depan, kawan-kawan. satu langkah sudah selesai. Selanjutnya, kita perlu bertemu khusus untuk mematangkan langkah. Ada usul?”tiba-tiba Ikram yang sedari keluar sekolah hanya diam angkat bicara. Bukan hanya itu. Langsung mengajukan usul kongkret kepada kawan-kawannya. “Seperti apa kata Sabiq. Lebih cepat lebih baik. Bagaimana kalau malam ini selepas Isya kita berkumpul di rumah Ayra, mematangkan langkah?”usul Yodha. “Setuju!”seperti
dikomando
keenam
sahabat
itu
mengucapkannya hampir bersamaan. …∑πχαΩ…
| 18
4
What’s in a Name?
Suasana hening dan nyaman terasa dari sekitar rumah itu. Penerangan cukup yang sampai ke halaman depan yang luas membuat suasana malam yang makin merambat terasa lambat. Derik suara Jengkrik dan sesekali suara motor yang melintas berpadu dalam kesatuan yang harmonis. “As-salaamu’alaikum!”terdengar suara salam yang tampaknya berasal dari beberapa orang. “Wa alaikum salam!”jawaban yang terdengar dari dalam rumah. Tampak seorang perempuan setengah baya keluar menyambut lima anak laki-laki yang berdiri di luar. “Ooo, kawan-kawan sekolah Ayra, ya! Silahkan masuk, Nak!”ajar perempuan itu yang ternyata ibunda Ayra. “Terima kasih, Bu. Tampaknya lebih nyaman di teras ini, Bu. Lebih luas pemandangan dan terang juga, kok!”diplomasi Sabiq. “Baiklah, tak apa-apa. Ibu panggilkan Ayra dulu, ya!”kata perempuan itu sambil bergerak masuk ke dalam rumah. Beberapa saat keenam sahabat itu menunggu, Ayra muncul dari dalam rumah. “Wah, sudah lengkap nih!”Ayra memulai pembicaraan. | 19
“Jadi, kita nongkrong di teras aja, nih? Tunggu ya, aku ambilkan tikar, biar lebih enakan ngobrolnya,”kata Ayra sambil masuk kembali ke dalam rumah. Tak berapa lama ia telah kembali dengan selembar tikar pandan. Kami pun akhirnya duduk bersila di atas tikar, merencanakan langkah-langkah selanjutnya. Kali ini, Ayra lebih banyak mengambil inisiatif. “Baik Kawan-kawan. Apa kiranya langkah kita ke depan untuk persiapan OSN itu?”ujar Ayra memulai pembicaraan. “Menurutku ada beberapa hal yang harus kita lakukan, kawankawan,”kata Sabiq. “Apa saja beberapa hal itu, Bro?”tanya Ikram. “Ya, langkah awal sudah kita selesaikan, membuat kelompok. Itu sudah tepat. Selanjutnya…,” belum sempat Sabiq melanjutkan kalimatnya, Amer memotong pembicaraan. “Aih, jangan sebut kelompok, Kawan! ndak keren. Sebut saja Genk! Tapi bukan Genk kekerasan atau vandal yang sering bikin rusuh. Ini Genk OSN!”usul Amer. Kami saling pandang. Yodha memicingkan matanya, Ikram sedikit menarik kepalanya ke belakang, Fateh yang biasa suka bercanda tampak membuka mulutnya. Sementara Ayra tampak agak berkerut keningnya. Dan Sabiq hanya tersenyum tipis. Usul Amer memang agak aneh. Suasana masih diam. Hening, sehening malam dan kicauan Jangkrik yang bersahutan. “Hmm…. usulan yang unik juga, ya!”Sabiq berusaha memecah kebekuan. | 20
“Mungkin ada usulan lain?”tanya Sabiq “Menurutku…,”tiba-tiba Yodha menghentikan kalimatnya, karena Ibu Ayra datang dengan sesuatu di piring. “Stop dulu diskusinya, anak-anak! Ibu sudah siapkan Singkong Rebus untuk anak-anak pemberani ini,”kata Ibu Ayra sambil meletakkan 2 piring Singkong rebus di hadapan kami. “Bagaimana
Kawan?
apa
yang
mau
kau
sampaikan
tadi?”tanya Amer pada Yodha. Yodha hanya diam. Kawan-kawannya yang lain juga diam. Cukup lama suasana itu berlangsung sampai akhirnya Sabiq mengucapkan sesuatu.
“William Shakespeare, seorang penyair besar Inggris pernah berujar kawan. Katanya, ‘What’s in a name? That which we call a rose by any other name would smell as sweet’,” kata-kata Sabiq memecah kesunyian malam itu. “Apa maksudnya itu, Biq?”tanya Amer “Apalah arti sebuah nama? Sesuatu yang kita sebut sebagai Mawar, meskipun digelari dengan nama lain, akan tetap tercium harum. Begitu, maksudnya kira-kira, kan Biq!”jelas Ayra. Sabiq hanya tersenyum. “Aih, tampaknya OSN harus ada bagian Bahasa dan Sastra. Dan aku pasti usulkan Sabiq mewakili sekolah kita,”kata Amer. “Ha….ha….ha,”keenam sahabat itu tertawa lepas. “Jadi, kita sepakat bahwa menyebut sebagai Genk untuk persiapan OSN ini! Ya, usulku Genk OSN,”kata Sabiq “Oke. Genk OSN. Sebutan yang keren,”timpal Amer puas. | 21
“Waah, tampaknya pada seneng, ya! Ini Ibu bawakan Wedang Jahe, anget-anget lho,”kata Ibu Ayra yang muncul tiba-tiba dengan seteko Wedang Jahe. “Alhamdulillah. Ini namanya pucuk dicinta ulam tiba,”kata Amer. “Dasar, tukang makan! Begitu ada makanan langsung otakmu encer, Kawan,”Fateh menyambut Amer. “Ha…ha..ha,”keenam sahabat itu tertawa bersama. …∑πχαΩ…
Malam semakin larut. Bulan separuh pun makin tinggi. Keenam sahabat itu masih asyik berdiskusi tentang langkah apa yang akan mereka susun menghadapi OSN. “Baik. kawan-kawan. Kita masuk ke bagian inti. Apa langkah konkret yang akan kita lakukan?”tanya Ayra. “Kita harus punya pembimbing, Ayra!”usul Fateh “Koordinasi ke sekolah juga harus, kan?”kata Yodha “Waktu pertemuan rutin kita harus terjadual, Kawan!”usul Amer. “Oke. Semua itu perlu. Tapi, apakah masih ada yang lain lagi? Ikram!”tanya Ayra. “Usul yang masuk akal dan mungkin dilakukan,”tanggap Ikram. Semua
diam.
Suasana
tambah
hening.
Meski
dalam
keheningan itu, sesungguhnya Amer sudah menyelesaikan gelas kedua Wedang Jahe dan potongan ketiga Singkong rebus buatan ibunda Ayra. | 22
“Aku pikir, hal yang paling utama kita berlatih materi OSN itu sendiri. Bagaimana teknis pelatihan dan ketersediaan bahanbahannya, akan sangat menentukan sekali persiapan kita menghadapi OSN, Kawan-kawan,”usul Sabiq memecah kebekuan diskusi malam itu. “Nah,
kata-kata
seperti
ini
yang
aku
suka
darimu,
Kawan!”kata Amer sambil mengangkat wajahnya dan menepuk bahu Sabiq. “Baik. kita listing semua usul itu. Kita buat skala prioritas. Selanjutnya kita jalankan. Setuju?”tanya Ayra. “Setuju,
setuju!”keenam
sahabat
itu
saling
memberi
persetujuannya. “Untuk hal teknisnya, sebaiknya Ayra saja yang menyusun semua usulan tadi. Kita sepakat saja. Dan, lebih cepat lebih baik kita mulai persiapan ini, kawan-kawan!”kata Sabiq. Malam itu pertemuan enam sahabat berakhir. Setelah berpamitan, kelima sahabat Ayra pulang dengan langkah mantap. Satu langkah telah dilakukan. Ditunggu langkah berikutnya. Fa idzaa faraghta fanshab! …∑πχαΩ…
| 23
5
Kakek Penjaga Musholla “Pada prinsipnya Bapak senang sekali dengan tekad kalian, anak-anak!”ucapan itu yang terngiang dari wejangan Kepala Sekolah saat keenam sahabat Genk OSN menghadap dan mengutarakan maksud mereka. “Pasti ada tapinya, ya Pak?’potong Amer dengan ekspresi wajah cemberut. Pak Kepsek tak langsung menjawab. “Anak-anak, sepengetahuan Bapak, OSN itu khusus dan membutuhkan tenaga ekstra,”terang Kepala Sekolah. “Maksud Bapak dengan khusus dan membutuhkan tenaga ekstra itu, apa Pak?”tanya Sabiq tenang. Kepala Sekolah berdiri dan berjalan ke arah jendela. Ia tampak memandang jauh ke depan. Entah apa yang akan disampaikannya. “Satu hal yang bisa Bapak sampaikan kepada kalian bahwa soal-soal olimpiade Matematika memiliki karakteristik tidak umum, non-rutin, dan bukan hanya memerlukan pengetahuan Matematika tingkat sekolah menengah tetapi juga kematangan lanjutan,”terang Kepala Sekolah. Keenam sahabat itu tampak tekun mendengar apa yang disampaikan oleh Kepala Sekolah mereka. | 24
“Materi soal-soal olimpiade Matematika yang dikelompokkan ke dalam Aljabar, Geometri, Kombinatorika, dan Teori Bilangan memerlukan keluasan wawasan, kecermatan, kejelian, kecerdikan dan pengalaman. Dan orientasi soal adalah pemecahan masalah alias problem solving ,”terang Kepala Sekolah lagi. “Hal itu kami sadari sepenuhnya, Pak. Justru kami mendiskusikannya dengan Bapak untuk mendapat arahan bagaimana sebaiknya agar kami bisa survive menghadapi OSN nanti,”Ayra mencoba membuka diskusi. “Di situlah masalahnya anak-anakku. Ketersediaan sumber daya guru di sekolah kita sangat terbatas. Bukan saja jumlahnya, tapi kualifikasi untuk itu,”terang Kepala Sekolah lagi. Amer mulai gelisah. Fateh pun tampaknya sama. Ikram memandang serius ke depan. Yodha memandangi langit-langit ruangan, sementara Ayra berusaha tenang. Sabiq mendengar tanpa ekspresi. Suasana hening. Keenam sahabat itu tampak serius menunggu jawaban Kepala Sekolah. “Apa jalan keluar yang Bapak tawarkan dengan kondisi seperti ini, Pak?”tanya Ayra lagi. “Sebagai Kepala Sekolah Bapak akan memaksimalkan yang bisa sekolah berikan. Fasilitas perpustakaan bisa kalian gunakan 24 jam dan 7 hari seminggu. Bahkan kalau anak-anakku ingin menggunakan salah satu ruangan di sekolah, Bapak akan usahakan memberi izin. Tapi,”Kepala Sekolah menghentikan kalimatnya. “Tapi, apa Pak?”potong Yodha | 25
“Sumber daya pengajar untuk itu, sangat tidak memadai. Kalian harus menemukan jalan untuk memecahkan masalah ini, karena justru itu persoalan pokok persiapan untuk menghadapi OSN, anak-anakku!”terang Kepala Sekolah. “Alhamdulillah, Pak. Doa dan dukungan Bapak atas niat kami berenam, rasanya cukup menjadi bekal kami melangkah. Insyaallah ada jalan keluar atas hal ini, Pak!”tiba-tiba Sabiq berkomentar. “Bapak berdoa untuk kalian, anak-anakku!”tegas Kepala Sekolah. Pertemuan pagi itu begitu menyesakkan. Setidaknya tantangan baru muncul di hadapan keenam sahabat yang sedang semangat itu. …∑πχαΩ…
“The show must go on, Bro!”kata Ayra. “Caranya, Ayra. Caranya!”Amer mulai gelisah. “Insyallah ada jalan! Begitukan tadi katamu, Biq?”tanya Ayra. “Aih, dia itu Satrawan, Kawan. Bayangkan saja. Dia pakai lagu Maher Zein untuk menjawab keraguan Kepala Sekolah!”kata Amer tambah ketus. “Ya, itulah ciri Sabiq, Mer. selalu optimis,”timpal Yodha. “Sekarang kita tanya Sabiq saja. Apa kiranya arti Insyaallah Ada Jalan yang dia sampaikan pada Kepala Sekolah tadi?”kali ini Ikram nimbrung. Fateh masih diam. Tampaknya mendengar penjelasan Kepala Sekolah tadi pagi Fateh Hilang Lucunya. “Jadi, apa maksudmu tadi dengan kalimat itu, Biq?”tanya Ayra datar pada Sabiq. | 26
Sabiq diam. Dia menarik nafas panjang. Tampaknya, dia akan mengutarakan sesuatu yang berat. “Kawan-kawan. Kita tadi sama-sama sudah mendengar dengan jelas apa yang disampaikan oleh Kepala Sekolah tentang ketersediaan yang ada di sekolah untuk menghadapi OSN. Hal itu pun sebenarnya telah kita sadari sebelumnya. Tapi, bukankah awalnya pun kita memulai Genk OSN ini dengan modal semangat? Dan tidak pernah memikirkan apa yang disediakan oleh sekolah? Lantas kenapa saat ini hal itu menjadi sesuatu yang membebani kita?”papar Sabiq. “Konkretnya, apa maksudmu Biq?”kali ini Fateh memasang wajah serius. Sabiq tak langsung menjawab. “Allah tak akan meninggalkan hamba-hamba-Nya sedetik jua. Apatah lagi bagi mereka yang mempunyai niat yang tulus, Kawan!”terang Sabiq. “Aku percaya itu Boy, tapi…”kalimat Amer terputus. “Sabar, Bro! Adzan Ashar telah berkumandang. Sebaiknya kita menuju musholla terdekat. Diskusi ini kita lanjutkan selepas sholat Ashar,”kali ini Ikram menengahi suasana. Memang, tak jauh dari tempat mereka berkumpul, sekitar 5 menit berjalan kaki, terdapat sebuah musholla kecil namun tampak bersih dan terawat baik. Keenam sahabat itu segera beranjak menuju musholla. Tentu dengan perasaan masing-masing.
…∑πχαΩ… | 27
“Nah, sekarang apa penjelasanmu, Kawan?”Amer memulai diskusi lanjutan beberapa saat setelah sholat Ashar selesai. Masih di serambi musholla yang asri itu, keenam sahabat melanjutkan diskusi mereka yang tadi tertunda. “Ya, seperti tadi aku katakan bahwa Allah tak akan meninggalkan hamba-hamba-Nya sedetik jua,”papar Sabiq. Keempat
sahabat
Sabiq
serius
mendengar
apa
yang
diutarakannya. Tapi tidak dengan Amer. Dia mulai gelisah. “Insyallah niat kita adalah tulus. Kita ingin ambil bagian dalam OSN apa pun hasilnya tak penting. Mudah-mudahan yang terbaik yang bisa kita persembahkan untuk sekolah. Nah, kita punya Yodha, yang diantara kita tahu, dia lebih mahir soal geometri. Kita juga punya Ayra, soal operasi bilangan, dia seperti Srikandi dalam memanah. Tentang, Kombinatorika, serahkan pada Fateh yang gemar dengan hal itu. Sementara soal-soal Aljabar, bukankah kita punya Ikram!”terang Sabiq bersemangat. “Dan jangan lupa, soal menghubungkan hal-hal yang tak terpikirkan, Amer ahlinya. Dan aku sendiri, hanyalah pecinta Sastra dan buku-buku unik, yang kebetulan juga gemar Matematika, Kawan. Itulah kenapa tadi aku sebut, Allah tak akan meninggalkan hamba-hamba-Nya
sedetik
jua.
Aku
yakin
itu,
kawan-
kawanku,”terang Sabiq lagi. “Kau benar sahabatku. Pertanyaannya adalah, kapan dan bagaimana kita tahu bahwa Allah akan menunjukan tanda itu pada kita?”kali ini Ayra angkat bicara. Sabiq hanya memasang wajah datar. | 28
“Aku tak bisa menjawab pertanyaan itu, Kawan. Aku hanya bisa meyakini hal itu. Bahwa kalau kita mufakat akan membawa berkat,”jawab Sabiq. “Tapi kita butuh usaha untuk mencapai ke arah itu, Kawan! Usaha yang konkret,”timpal Fateh. Diskusi semakin panas dan terus berlanjut. Masing-masing mengemukakan kegusarannya. Suasana sore di serambi Mmusholla sejatinya begitu hening dan tenang. Namun tampaknya, tidak bagi keenam sahabat itu. Bagi mereka berenam, seolah hembusan angin pun berhenti. Ada suasana yang menyesakkan dada pada keenam sabahat itu. Tiba-tiba, sayup-sayup terdengar suara pelan memecah kebuntuan. “Apa yang kalian perdebatkan begitu tajam anak-anak muda?”tiba-tiba tampak seorang laki-laki yang tak bisa disebut muda lagi menghentikan pembicaraan keenam sahabat itu. Laki-laki yang berusia di atas 60-tahunan itu terus menghampiri keenam sahabat yang masih diam. “Kakek Penjaga Musholla di sini! Tampaknya serius betul perbincangan kalian tadi?”tanya orang tua itu sopan. “Orang tua ini Penjaga Musholla al-Attiqii ini,”pikir Ayra. Kali ini Sabiq memberanikan diri menjawab pertanyaan Kakek ‘Penjaga Musholla’ itu. “Mohon
maaf
bila
diskusi
kami
telah
mengganggu
kenyamanan Kakek dan Musholla ini, ya Kek!”ujar Sabiq sopan.
| 29
“Ah, tidak apa-apa. Justru Kakek senang masih ada anak-anak muda yang mau mendiskusikan persoalan mereka di Musholla,”kata Kakek Penjaga Musholla itu. Selanjutnya, Sabiq menceritakan masalah yang sedang dihadapi oleh keenam sahabat itu kepada Kakek Penjaga Musholla. Tampak Sang Kakek begitu serius mendengar dan menyimak setiap kalimat yang disampaikan oleh Sabiq. “Sampai kami akhirnya bertanya ‘kapan dan bagaimana kami tahu bahwa Allah akan menunjukan tanda itu pada kami’, Kek’?”tanya Sabiq lagi. Kakek Penjaga Musholla itu tampak tertegun. Ia memegangi dagunya yang ditumbuhi jenggot halus yang sudah tampak memutih di beberapa helainya. Di pandangi satu per satu wajah anak-anak yang berdiri lesu di hadapannya. Pandnagan yang aneh! “Niat dan tekad kalian sungguh mulia, anak-anak. Benar kata teman kalian bahwa kesungguhan dan kerja keras penting. Tapi lebih jauh, itu saja tidaklah cukup. Kepasrahan dan keyakinan pada Allah bahwa Dia akan menolong hamba-hamba-Nya, dan itu termasuk kalian juga di dalamnya, harus tetap bertahta di hati kalian. Dan, jangan pernah berhenti bersabar terhadap janji Allah itu. Dan mungkin, inilah saatnya….,”tiba-tiba Kakek Penjaga Musholla itu menghentikan ucapannya, berbalik ke arah buntelan yang sedari tadi tergeletak di salah satu sudut musholla. Keenam sahabat itu hanya bisa diam, terpaku. Tak sepatah kata pun terucap. Mereka hanya saling pandang. Kakek Penjaga Musholla itu kembali kepada keenam sahabat yang masih terpaku diam dan heran. Di hadapan keenam sahabat itu, | 30
Kakek Penjaga Musholla membuka buntelan yang sudah tampak lusuh di makan usia. “Kakek tak tahu, apa isi dalam buku tua ini. Tapi waktu Kakek sekolah dulu di kampung, kami biasa menyebutnya sebagai Aljabar, atau Ilmu Hitung, atau Sepren. Apakah ini bisa membantu kegelisahan kalian atau tidak, kalianlah yang lebih tahu. Kalau kalian berkenan, Kakek bermaksud meminjamkannya sementara pada kalian. Bila bermanfaat pelajarilah. Setelah itu, tolong serahkan buku tua ini ke perpustakaan sekolah kalian, untuk menjadi koleksi bacaan yang bisa dinikmati oleh teman-teman kalian lainnya,”terang Kakek Penjaga Musholla. “Maksud Kakek, kami...”tanya Ayra terputus. “Ya, Nak. Saat ini tugas kalian untuk menjaga catatan tua ini. Mudah-mudahan bermanfaat. Dan kau, siapa namamu, Nak?”tanya Kakek Penjaga Musholla itu. “Ayra, Kek. Ayra!”jawab Ayra santun. “Nak Ayra, menjadi tugasmu untuk menjaganya dan menitipkannya ke perpustakaan bila sudah selesai!”pinta Kakek Penjaga Musholla. “Insyaallah, Kek!”jawab Ayra mantap. “Satu hal lagi. Mohon catatan ini jangan langsung dibuka. Tunggulah 3 hari sejak hari ini. Selepas kalian sholat Ashar berjamaah, buka dan pelajarilah!”pesan Kakek Penjaga Musholla itu. Seperti tersihir, keenam sahabat itu tak mampu berkata sepatah pun. Mereka hanya diam menyaksikan Ayra menerima catatan yang dikeluarkan dari buntelan lusuh milik Kakek Penjaga Musholla itu. | 31
Amer yang biasanya tukang protes, bibirnya rapat terkunci. Fateh, yang biasanya ceria penuh canda, hanya bisa memandang Kakek Penjaga Musholla dengan mulut sedikit terbuka. Yodha yang biasanya asyik dengan permainan kubik dan catatan matematikanya, kali ini seperti terbius, kaku. Ikram, nyaris tertunduk dan tak bisa memandang apapun selain mendengar dengan hikmat. Ayra pun hanya takzim mendengar semua pesan Kakek, sambil mengulurkan tangan menerima catatan itu. Sementara Sabiq hanya terpejam dengan tangan kedua tangannya menopang dagu, entah apa yang dipikirkannya. “Itu saja pesan Kakek. Mudah-mudahan kalian berhasil. Kakek permisi dulu, ya! Mau keluar sebentar memberi keperluan Musholla,”tiba-tiba perkataan itu memecah suasana hening. “Baik, Kek. Terima kasih atas nasehat dan catatannya,”jawab keenam sahabat itu serentak. Bersamaan dengan beranjaknya Kakek Penjaga Musholla itu, keenam sahabat itu pun bergegas pula meninggalkan Musholla. Kali ini secercah optimisme mulai timbul. Optimisme yang bercampur dengan penasaran. Dan hal itu hanya akan terjawab tiga hari lagi. Tiga hari sejak hari ini, selepas sholat Ashar berjamaah! “Hei Kawan! keyakinanmu diuji tiga hari lagi!”kata Amer sambil mengepalkan tinju kanannya ke arah Sabiq. “Insyaallah ada jalan dan semua akan baik-baik saja. Allah tak akan
meninggalkan
hamba-hamba-Nya
meski
sedetik
jua,
Kawan!”jawab Sabiq enteng. …∑πχαΩ… | 32
6
Selepas Sholat Ashar… Tibalah waktu yang ditungu-tunggu oleh keenam sahabat itu. Batas waktu tiga hari yang dipersyaratkan oleh Kakek Penjaga Musholla untuk membuka catatan berupa buku tua itupun, akhirnya tiba. Seperti sedang menunggu sesuatu yang sangat diharapkan, keenam sahabat itu tampak duduk rapi, di bangku panjang yang terbuat dari bambu dan disusun berbaris saling berhadapan. Di tengah-tengahnya tergeletak buku titipan Kakek Penjaga Musholla yang tertutup rapi di atas sebuah meja lebar yang terbuat dari sisa batang bawah dan akar pohon jati tua. “Ayra, tampaknya waktunya telah tiba. Dan kau yang diberi amanah
untuk
memegangnya,
aku
rasa
lebih
pantas
membukanya,”kata Sabiq memecah kebisuan. “Ya, aku setuju. Kau lebih tepat membukanya Ayra!”kali ini ini Ikram menegaskan dukungannya. Ayra masih diam. Dia menoleh ke Yodha. Tampaknya sahabatnya itu pun setuju agar Ayra segera membuka buku itu. “Betul, kaulah yang lebih pantas mendapat kehormatan itu, Ayra!”kata Yodha. “Baiklah kalau begitu. Tapi, sebaiknya kita saling mendekat ke arah buku,”kata Ayra. | 33
Keenam sahabat itu segera merapatkan tubuh mereka menuju ke tengah. Begitu dekat hingga tampak setiap orang akan jelas melihat apa isi buku. “Bismillahirrahmanirrahim,”Ayra memulai aksinya sambil meraih buku itu dengan tangan kanannya. Dan, lembar pertama pun tersingkap. Seperti dikomando, keenam sahabat itu memicingkan mata menuju ke sudut kanan bawah halaman buku. Di situ tertulis jelas, dengan tulisan tangan yang indah, untaian kalimat,…. “Karya ini dipersembahkan oleh para pendidik yang berhimpun pada suatu waktu di satu tempat pada suatu waktu di bagian timur pulau yang padat, bagi mereka yang cinta akan kreativitas, kesederhanaan, kejujuran, ketekunan, kesabaran dan kerja keras, yang kesemuanya itu dimulai dengan kebersihan hati serta terangkai dalam satu kata indah Matematika!” “Apa maksudnya kalimat itu, Biq?”tiba-tiba Ikram bertanya kepada Sabiq. Dia yang ditanya, hanya diam. Tak seperti biasanya. “Teruskan membuka lembar berikutnya, Ayra!”pinta Yodha. “Subhanallah! Contoh-contoh soal Matematika, Kawankawan!”seru Ayra bersemangat. “Bagaimana Kawan?”kali ini Sabiq menunjuk ke arah Amer. “Gila kau, Boy. Pucuk dicinta ulam pun tiba!”seru Amer. “Bukankah benar bahwa, Insyaallah ada jalan dan semua akan baik-baik saja pada akhirnya, Kawan!”tegas Sabiq pada Amer.
| 34
“Alhamdulillah. Tampaknya kita harus segera atur waktu untuk memelajarinya, Kawan-kawan!”kata Ayra dengan wajah berseri. Seperti mendapatkan durian runtuh, keenam sahabat itu begitu bergembira. Tanpa mereka sadari sekitar 300 meter dari tempat mereka berdiskusi, sejak hampir setengah jam lalu, seorang tua sedang mengamati tingkah mereka. Seulas senyum tersungging di bibir tuanya. “Allahummanshurhum
wa
yasirhum
wa
laa
tu’asirhum,”sepotong doa keluar dari mulut Kakek tua itu sambil berlalu meninggalkan keenam sahabat yang tampak terus asyik membuka lembar demi lembar buku titipan itu. …∑πχαΩ…
“Subhanallah! Isinya luar biasa, Kawan-kawan!”seru Ayra. “Ya, hanya kata itu yang pantas tampaknya,”sambung Amer. “Aku setuju, Kawan,”timpal Yodha Sementara Ikram begitu senangnya, hingga tampak butiran bening menetes tepat di bagian bawah dari kelopak mata sebelah kirinya. “Aku pikir juga begitu. Buku ini sangat bermanfaat untuk kita,”tandas Fateh. Sabiq memilih bersikap datar. Ia bahkan tampak berfikir keras. Seperti ada yang mengganggu pikirannya. “Biq, tampaknya kau punya pendapat berbeda, Kawan?”tanya Ayra. | 35
“Hai Sastrawan, apa pulak yang kau pikirkan, Bro!”tanya Amer. Sabiq masih diam. Dia berusaha tersenyum. Namun, senyum itu masih menyisakan misteri. Dan kelima sahabatnya tahu, bahwa senyum itu berarti ‘ada sesuatu yang lain’. “Apa menurutmu sesuatu yang lain itu, Biq?”kali ini Fateh mendesak dengan pertanyaan yang straight to the point. “Masih ingat apa kata Kepala Sekolah, saat kita bertemu beliau,
Kawan-kawan?”Sabiq
balik
bertanya
kepada
semua
sahabatnya. “Maksudmu, Biq”Ayra bertanya balik. “Beliau pernah berkata bahwa materi soal-soal Olimpiade Matematika
dikelompokkan
ke
dalam
Aljabar,
Geometri,
Kombinatorika, dan Teori Bilangan, dan memerlukan keluasan wawasan, kecermatan, kejelian, kecerdikan dan pengalaman. Bukan hanya itu, tapi orientasi soal adalah pemecahan masalah alias problem solving,”terang Sabiq “Lantas, apa hubungannya dengan buku ini, Bro?”tanya Amer penasaran. “Perhatikan baik-baik. Buku ini, buku yang dititipkan oleh Kakek Penjaga Musholla itu, baru sekadar pengantar! Lebih tepatnya pembuka ke arah semua persoalan dalam memasuki olimpiade matematika, Kawan-kawan!”terang Sabiq lagi. Kali ini tampak Ayra, Amer, Yodha, Ikram, dan Fateh, seperti tersadar. “Kau benar, Sastrawan!”komentar Amer seketika. | 36
“Artinya kita harus lebih serius dan giat berlatih dan mencari jalan untuk menambah bekal persiapan kita, Kawan-kawan. Tapi, kita memang bisa mulai dari buku titipan ini. Dan saranku, segera saja!”tandas Sabiq. “Setuju! Dan kita sudah siap sekarang, insyaallah!”kata Ayra. …∑πχαΩ…
| 37
7
Tujuh Prinsip Dasar “Tak satupun yang dapat menggantikan ketekunan untuk mencapai keberhasilan. Bakat pun tidak. Tidak ada yang lebih umum daripada orang berbakat yang tidak berhasil. Para orang pandai pun tidak, dunia penuh dengan orang gagal yang berpendidikan. Hanya kegigihan dan ketetapan hatilah... (Calvin Coolidge).” Begitu tertera sebaris kalimat pada halaman cover bagian dalam buku titipan itu. Ayra terus membuka lembaran berikutnya. “Sebelum lebih jauh memulai membaca buku ini dan berlatih Matematika, beberapa prinsip dan amalan dasar sebaiknya menjadi perhatian!”begitu kalimat pembuka pada bagian pertama isi buku. “Inilah tujuh prinsip yang harus menjadi pegangan bagi para pembaca dan yang ingin mengambil manfaat dari buku ini!”paragraf berikutnya tegas menulis. Pertama, luruskan niat. Karena niat adalah fondasi yang akan dibangun di atasnya pengetahuan dan kemampuan serta kemahiran bermatematika. Niatkan semata-mata untuk lebih meningkatkan kemampuan diri dalam membaca dan memelajari fenomena alam, khususnya matematika, sebagai maha karya Allah, Tuhan Yang Maha Pintar dan Bijaksana, dan bukan sekadar menjadi juara atau yang terbaik di antara manusia. | 38
Kedua, jagalah kebersihan dan kesucian anggota tubuh. Karena ilmu adalah cahaya dan kemuliaan. Cahaya dan kemuliaan yang bersih hanya akan bersemayam di tempat yang bersih pula. Seperti Islam yang mengajarkan berwudhu, menjadi penting memulai pelajaran dengan berwudhu dan tetap menjaga serta mempertahankannya selama belajar. Ketiga, mulailah dengan membaca Basmallah, melibatkan Allah, Tuhan Seru Sekalian Alam, Sang Penguasa Pengetahuan, dalam setiap tindakan yang dilandasi niat yang tulus, insyaallah akan bernilai ibadah. Matematika bukan hanya sekadar ilmu, tetapi ilmu yang mampu mendekatkan diri pemiliknya, kepada Sang Pencipta, Sang Pemilik Ilmu itu sendiri. Keempat, akhiri dengan ucapan Hamdalah. Karena awal yang baik haruslah pula ditutup dengan akhir yang baik pula. Rasa syukur atas pengetahuan yang diperoleh sekecil apapun, akan berdampak baik bagi pemahaman dan pengertian yang lebih bagi para pencarinya. Kelima, mulailah dari yang sederhana. Karena dalam kesederhanaan banyak tersimpan kekuatan yang seringkali kurang disadari oleh pemiliknya. Bukankah perjalanan bermil-mil dimulai dari satu langkah? Begitu Orang Bijak Dari Timur pernah berujar. Keenam, jatuh-bangkit, gagal-ulangi, lagi dan lagi. Karena tak ada yang pandai dalam matematika. Tapi ketekunan dan kesabaran serta perilaku pantang menyerah untuk terus mencoba adalah rumusan terpenting yang harus diketahui oleh para pembaca buku ini.” | 39
Ketujuh, berusahalah keluar dari kebiasaan yang membatasi kreativitas dan imajinasi. Karena sering kali tradisi yang usang terus dipertahankan dan mengungkung kreativitas, dan pada gilirannya mematikan imajinasi. Padahal dalam matematika, kreativitas dan imajinasi merupakan dua hal yang penting, dalam mencari dan menemukan pemecahan masalah. Out of box thinking kata orang zaman sekarang. Inilah tujuh prinsip dasar yang seharusnya menjadi pegangan bagi siapa saja yang ingin mendapat manfaat dari buku ini,”kalimat penutup pada pengantar buku yang dibacakan oleh Ayra. “Wah, unik juga buku ini,”komentar Amer memecah keseriusan enam sahabat itu. “Wudhu’mu belum batal, kan Mer?”balas Ikram tiba-tiba. “Aku ragu,”jawab Amer singkat. “Kembali berwudhu-lah, agar segera kita mulai pengembaran ini!”kali ini Ikram berkata serius. …∑πχαΩ…
“Ayra, apakah bisa kita mulai? Adakah contoh yang diberikan oleh buku yang kau pegang itu?”tanya Amer penasaran. “Ya, ada Mer,”jawab Ayra singkat. Ayra kembali memandangi lembaran buku di tangannya. “Nah ini dia, Kawan-kawan!”seru Ayra. “Coba kamu bacakan untuk kita, Ayra!”pinta Ikram. “Appetizer. Selesaikanlah…” Belum selesai Ayra membaca, Amer memotong. “Apa, Ayra! App…pe..?”tanya Amer. | 40
“Appetizer, Kawan! Bahasa Inggris untuk menyebutkan pembuka. Dalam istilah bahasa Indonesia yaitu ‘hidangan pembuka’. Orang Perancis menyebutnya Hors d’oeuvre. Appetizer merupakan hidangan pembuka sebelum hidangan utama yang disajikan dengan tujuan membangkitkan nafsu makan atau selera makan. Biasanya juga Appetizer merupakan penghantar untuk menikmati hidangan utama, sebagai hidangan pembuka yang menimbulkan selera makan karena itu porsinya kecil dengan rasa asam, asin atau pedas, Sahabatku Amer!”terang Sabiq. “Aih, Sastrawan kita ini boleh juga soal istilah,”puji Yodha. “Lanjutkan, Ayra!”pinta Sabiq “Baik. Selesaikanlah beberapa persolan di bawah ini:
Nomor 1: Tentukanlah panjang dan lebar dari sebuah persegi panjang yang luas dan kelilingnya sama besarnya. Nomor 2: Seseorang memiliki sejumlah koin senilai 1000 Rupiah. Setelah diperhatikan dengan seksama, ternyata koin yang dimilikinya terdiri dari tiga macam koin di antara 4 macam koin yang sekarang masih berlaku (500-an, 200an, 100-an dan 50-an). Selidiki dan tentukan berapa banyak kombinasi koin yang mungkin dimiliki oleh orang tersebut. Nomor 3: Tentukan angka satuan bilangan 19971991 jika dibagi 41. Nomor 4: Hitunglah nilai N dari persamaan berikut:
| 41
1 1+2 1+ 2 𝑁𝑁 = 1 + 𝑥𝑥 1 1+ 1 1+ 1,5
7 1 [5 − (1 + 0,405: 0,945) + 1,25 − 0, 1� (5 − 0,32: 0,35): ] 8 9
Nomor 5: Bila diberikan nilai A dan B masing-masing: 1 1 1 𝐴𝐴 = + + ⋯+ 1.2 3.4 1997.1998 dan 1 1 1 𝐵𝐵 = + + ⋯+ 1000.1998 1001.1997 1998.1000
Maka buktikan bahwa (A/B) merupakan bilangan bulat. Nomor 6 : Hitung luas daerah yang diarsir pada gambar di bawah bila ST merupakan garis singgung lingkaran kecil yang panjanganya 36 cm dan memotong lingkaran besar di dua titik: A. 324𝜋𝜋 B. 81𝜋𝜋 C. 129𝜋𝜋 D. 162𝜋𝜋 E. Tak bisa ditentukan.
“Selesai!”tiba-tiba Ayra menghentikan bacaannya. “Wow, menantang sekali!”Yodha berseru. “Bagaimana, Kawan-kawan! Jelas persoalannya?”tanya Ayra. “Jelas sekali soal-soalnya, Ayra. Yang masih kabur, bagaimana solusinya?”jawab Ikram. “Apa tak diterangkan solusinya juga, Kawan?”tanya Amer.
| 42
“Tampaknya benar-benar hidangan pembuka yang menantang Kawan. Hanya diberikan pentunjuk kunci jawabannya saja. Mungkin maksudnya…”kata Ayra. “Maksudnya kita diminta untuk mencoba menemukan jalan penyelesaiannya, Kawan!”terang Sabiq. “Bagaimana menurutmu, Ayra?”tanya Ikram “Aku sependapat dengan Sabiq. Kita harus menemukan jalan penyelesaiannya,”jawab Ayra. “Nah,
sudah
cocok
ini.
Kita
berenam,
sementara
App…pe..,”kata Amer terbata-bata. “Appetizer, Kawanku Amer!”sambung Ayra. “Ya, Appetizer-nya pun disediakan 6 Nomor. Kita bagi rata saja, Kawan. bagaimana?”tawar Amer pada kelima sahabatnya. “Setuju! Bukankah kita punya Yodha yang mahir soal Geometri. Itu artinya kau kerjakan Nomor 6, Yodha! Fateh, hobi otak-atik kombinasimu kali ini diuji. Kau selesaikan Nomor 2. Dan Ikram, yang gemar Aljabar kebagian Nomor 3. Aku sendiri, insyaallah akan kerjakan Nomor 5. Tinggal Nomor 1 dan Nomor 4. Berbagi antara Sabiq dan Amer,”usul Ayra. “Kau mau kerjakan nomor berapa, Kawanku?”tanya Sabiq pada Amer. “Aku usahakan menyelesaikan Nomor 4, Biq!”jawab Amer pelan. “Baiklah, kalau begitu Nomor 1 bagianku, Kawan!”kata Sabiq “Oke, tugas sudah terbagi. Dan suara adzan Ashar pun sudah berkumandang. Kita sudahi saja pertemuan hari ini. Malam ini kita | 43
kerjakan masing-masing tugas yang sudah kita sepakati. Dan besok kita saling berbagi hasil pekerjaan itu,”pungkas Ayra. Keenam sahabat itu bubar, setelah sebelumnya sama-sama mengucapkan doa penutup majelis. …∑πχαΩ…
| 44
8
Wow, Fantastic Appetizer! “Baik, siapa yang akan maju lebih dulu memberikan hasil pekerjaannya kepada kita sore ini? Dan Kawan-kawan, kita hanya punya waktu 2 jam,”kalimat Ayra meluncur begitu jelas terdengar memecah suasana diskusi sore di bawah pohon Mahoni tua samping sekolah. Keenam sahabat itu memang tampak tak kenal menyerah. Meski hanya ditemani oleh papan tulis tua, dan beberapa penggaris kayu yang telah patah pula serta sekotak kapur tulis, namun semangat mereka seperti badai Tsunami yang seakan ingin menggulung semua yang ada di pinggir pantai. Semilir angin sore dan gemericik suara daun bambu diterpa angin, berpadu sesekali dengan dengusan sapi yang sedang dihela oleh anak-anak di padang lapangan bekas panenan padi petani beberapa pekan lalu. “Aku Ayra! Aku akan mencoba kemampuanku. Mohon dibetulkan bila ada yang tak pas, Kawan-kawan!”sodor Amer. “Silahkan,
Kawan.
Kami
siap
mendengarkan
dan
memerhatikan serta belajar darimu, Kawan!”jawab Ayra. “Persoalan Nomor 4 biasa dikenal sebagai operasi pecahan bertumpuk. Hanya saja untuk kasusku, ia merupakan kombinasi | 45
antara pecahan bertumpuk dengan prioritas pengerjaan operasi pada bilangan dalam matematika,”Amer memulai penjelasannya. “Kalau kita perhatikan, sebenarnya nilai N merupakan hasil dari perkalian dua kelompok, sebut saja Kelompok A dan Kelompok B. Hal ini berarti bahwa: 𝑁𝑁 = 𝐴𝐴 𝑥𝑥 𝐵𝐵
Dimana nilai A-nya adalah hasil dari operasi pecahan bertumpuk: 1 1+ 1+ 2 2 𝐴𝐴 = 1 + 1 1+ 1 1+ 1,5 Sementara nilai B-nya merupakan hasil dari operasi bilangan: 7 1 𝐵𝐵 = �5 − (1 + 0,405: 0,945) + 1,25 − 0, 1� (5 − 0,32: 0,35): � 8 9 Kawan-kawan,”papar Amer bersemangat. “Mantap, Bro!”tiba-tiba Ikram menyela “Terima kasih, Ikram. Tapi, simpan dulu pujianmu. Biarkan kuteruskan dulu usahaku ini, Kawan!”pinta Amer. “Nah, hal itu berarti pula bahwa kita harus terlebih dahulu menyelesaikan bagian A, yaitu: 3 3 1+ 2 1+4 2 𝑁𝑁 = 1 + = 1+ 1 1 1+ 1+ 1 2 1+ 1+3 1,5 3 3 1+4 1+ 4 =1+ 𝐴𝐴 = 1 + 1 3 1+ 5 1+ 5 3 | 46
7 35 𝐴𝐴 = 1 + 4 = 1 + 8 32 5 “Itu nilai akhir dari A, yaitu: 1 + (35/32). Sementara nilai B dicari dengan prioritas operasi yang dibatasi oleh tanda-tanda operasi dalam matematika. Sehingga…”lanjut Amer “Tunggu dulu, Bro!”Sela Ikram lagi. “Ada apa, Kawan?”tanya Amer. “Tadi kau bilang, ‘dicari dengan prioritas operasi yang dibatasi
oleh
tanda
operasi
dalam
Matematika’.
Apa
maksudnya?”tanya Ikram Kali ini Amer pasang aksi, seolah guru yang profesional, dia memutar-mutar kapur tulis yang ada dijarinya. “Ya, memang ada aturan sederhana yang disepakati dalam operasi matematika. Operasi itu terkait dengan skala prioritas pengerjaan alias mana dulu yang harus dikerjakan bila terdapat kombinasi tanda operasi matematika dalam satu persamaan. Dalam kaitan dengan soal Nomor 4 bagian B, maka kita lihat ada beberapa tanda, yaitu: Kurung, Kali, Bagi, Tambah, dan Kurang. Nah pertanyaannya,
bagian
dioperasikan?
Mari
manakah
yang
sama-sama
kita
lebih
dahulu
perhatikan,
harus Kawan-
kawan!”terang Amer dengan aksinya. Tampak Ayra, Yodha, dan Fateh senyum-senyum. Sementara Ikram sedikit cemberut. Sabiq, ia masih memasang wajah tanpa ekspresi. Dia serius mengikuti apa yang diuraikan oleh Amer. “Lihat persamaan ini, Boy!
| 47
7 405 1000 5 32 100 � �5 − 𝐵𝐵 = �5 − 𝑥𝑥 �1 + 𝑥𝑥 � + − 0, 1 𝑥𝑥 � 𝑥𝑥9� 8 1000 945 4 100 35
Prinsip pertama, selesaikan dulu operasi dalam kurung.
Sehingga, akan kita peroleh persamaan B sebagai berikut: 7 405 1000 5 1 32 100 𝐵𝐵 = �5 − 𝑥𝑥 �1 + 𝑥𝑥 � + − 𝑥𝑥 �5 − 𝑥𝑥 � 𝑥𝑥9� 8 1000 945 4 9 100 35 3 5 32 7 𝐵𝐵 = �5 − 𝑥𝑥 �1 + � + − �5 − �� 7 4 35 8
32 7 10 5 + −5+ � 𝐵𝐵 = �5 − 𝑥𝑥 35 8 7 4 10 5 32 𝐵𝐵 = �5 − 5 − + + � 8 4 35 32 𝐵𝐵 = 35 Sehingga, bila nilai B yang kita peroleh dikalikan dengan nilai A yang kita dapatkan sebelumnya , maka akan didapati persamaan: 𝑁𝑁 = 1 +
35 32 𝑥𝑥 32 35
Nah, kembali kita temukan skala prioritas pengerjaan, antara penambahan dan perkalian. Karenanya, berdasarkan prinsip tadi kita mendahulukan operasi perkalian. Persamaan N dapat ditulis ulang sebagai: 35 32 𝑁𝑁 = 1 + � 𝑥𝑥 � 32 35 𝑁𝑁 = 1 + 1 = 2
Akhirnya, akan diperoleh nilai yang dicari yaitu 𝑁𝑁 = 2,”terang Amer mengakhiri penjelasannya. “Mantap, Bro!”puji Fateh.
| 48
“Tapi, tunggu dulu. Tadi dalam proses perhitungan nilai B, � setelah suku 1,25 itu kau ganti dengan pecahan 1/9, kenapa nilai 0,1
Kawan?”kali ini Ayra tampak serius.
Amer tersenyum. Dia tampak puas. “Tepat sekali. Aku memang mengganti 0,1� dengan 1/9.
Karena memang 1/9 itu adalah bentuk pecahan dari 0,1� ,”terang
Amer.
“Bagaimana caranya, Kawan?”Fateh ikut bertanya. “Baik. Pertama-tama harus dipahami arti dari tanda garis datar di atas angka di belakang koma. Hal itu berarti bahwa berulang terus � hal itu berarti sama dengan menerus. Sehingga, kalau dituliskan 0,1
0,111…. Karenanya, jika kita misalkan nilai 0,111… sama dengan p. Sehingga akan terbentuk persamaan: 0,111 … = 𝑝𝑝
Kalau kedua ruas, kanan dan kiri, kita kalikan dengan 10, maka: 1,111 … = 10𝑝𝑝
Bukankah nilai dari 1,111 … itu sama dengan 1+0,111…, dan 0,111 itu adalah sama dengan p. Sehingga: 1 + 0,111 … = 10𝑝𝑝 1 + 𝑝𝑝 = 10𝑝𝑝
1 9 Kira-kira begitu, Kawan-kawan. Sehingga hasil akhir perhitunganku 9𝑝𝑝 = 1 ⇔ 𝑝𝑝 =
nilai N adalah sama dengan 2! Ayra, apakah sesuai dengan petunjuk kunci jawaban yang diberikan dalam buku itu?”tanya Amer.
| 49
“Coba kita lihat. Ya, sama Mer. Petunjuk kuncinya menuliskan
[2]
sebagai
jawabnya.
Selamat!
Kau
berhasil
memecahkan persoalan ini,”puji Ayra. “Plok, plok, plok,”terdengar tepuk tangan dari kawan-kawan Amer sebagai bentuk selamat atas usaha Amer. “Baik, selanjutnya siapa, ya?”kembali Ayra memfokuskan perhatian keenam sahabat itu. “Biar aku coba kerjaku tadi malam, Ayra!”Fateh kali ini maju ke depan dengan penuh semangat. “Silahkan, Kawan. Sepenuhnya waktu menjelaskan adalah milikmu,”jawab Ayra. “Persoalan Nomor 2 sebenarnya persoalan sederhana saja. Hanya,
membutuhkan
kejelian
dan
kesabaran.
Aku
menyelesaikannya dengan cara membuat tabel kemungkinan dari kombinasi atas ketiga koin, dari koin-koin yang ada,”terang Fateh kepada kawan-kawannya yang kali ini tampak sangat serius. Memang, dalam soal-soal, Kombinatorika berupa statistika, kombinasi dan permutasi, Fateh dikenal lebih cepat memahami dibandingkan teman-teman sekelasnya. “Pertama, kita tentukan dulu kata kunci dari persoalan yang menyatakan bahwa seseorang memiliki sejumlah koin senilai 1000 Rupiah. Setelah diperhatikan dengan seksama, ternyata koin yang dimilikinya terdiri dari tiga macam koin di antara 4 macam koin yang sekarang masih berlaku (500-an, 200-an, 100-an dan 50-an)”. Hal ini berarti bahwa: (1) Nilai koin orang itu 1000; (2) Terdiri atas 3 macam koin; (3) Terdapat 4 macam koin yang masih berlaku,”terang Fateh bersemangat. | 50
“Kedua, membuat tabel kemungkinan kombinasi harga koin yang ada untuk menghasilkan 1000 dengan 3 koin. Sebagai berikut: Nomor
Nilai Koin yang Digunakan 500-an 200-an 100-an 50-an 1 1 1 14 2 1 2 12 3 1 3 10 4 1 4 8 5 1 5 6 6 1 6 4 7 1 7 2 8 2 1 10 9 2 2 8 10 2 3 6 11 2 4 4 12 2 5 2 13 3 1 6 14 3 2 4 15 3 3 2 16 4 1 2 17 1 1 3 18 1 1 6 19 1 2 1 20 1 2 2 21 1 1 8 22 1 2 6 23 1 3 4 24 1 4 2 Dari tabulasi di atas terlihat bahwa terdapat 24 kemungkinan kombinasi dari 3 koin yang bernilai 1000 dari 4 jenis koin yang masing-masing bernilai 500-an, 200-an, 100-an, dan
50-an.
Sehingga jawabanku adalah 24 kemungkinan,”terang Fateh kepada kawan-kawannya. “Subhanallah! Soal kombinasi memang kau orangnya, Kawan,”puji Amer. | 51
“Terima kasih, Kawan. Kita bersyukur mendapat Guru yang pandai mengajar di sekolah, Kawan-kawan,”balas Fateh sambil tersenyum. “Bagaimana petunjuk jawabannya, Ayra?”tanya Yodha. “Sebentar, aku periksa dulu. Soal Nomor 2 kunci jawabannya 24 kemungkinan. Jawabanmu benar, Kawan! Terima kasih, Fateh,”kata Ayra. “Plok, plok, plok,”kembali tepukan meriah diterima Fateh dari kelima kawannya. “Terima kasih atas apresiasinya, Kawan!”jawab Fateh. Suasana semakin seru. Aroma semangat yang terpancar makin terasa seiring mentari yang semakin condong ke Barat. “Alhamdulillah, sudah selesai 2 persoalan. Berikutnya, aku tunjuk saja, ya? Kita minta Ikram menyampaikan cara penyelesaian untuk Appetizer Nomor 3!”tegas Ayra. “Baik, Ayra dan kawan-kawan sekalian. Kepadaku diberikan persoalan Nomor 3 tentang angka satuan dari bilangan berpangkat. tepatnya 19971991,”kata Ikram. Ikram mulai menyampaikan cara yang dikerjakannya untuk menemukan angka satuan bilangan berpangkat besar itu. “Prinsipnya bahwa untuk mencari angka satuan dari bilangan besar berpangkat besar, perhatian kita fokuskan pada angka satuan bilangan itu dan kelipatan dari pangkatnya. Sehingga bilangan 19971991 dapat ditulis menjadi: Angka satuan 19971991
≡ sisa pembagian 19971991 oleh 10;
Angka 19971991
≡ (1990+7)1991 ≡ (199x10+7) 1991 | 52
Selanjutnya perhatikan bahwa (199x10) pangkat 1991 angka satuannya adalah nol, sehingga: ≡ (199x10+7)1991 mod (10)
Angka 19971991
≡ (7)1991 mod (10) ≡ (7)4x497+3 mod (10) ≡ (74)497 x 73 mod (10) ≡ (2421)497 x 343 mod (10) Angka 2421 yang bersatuan 1, yang bila dipangkatkan 497 tetaplah bersatuan 1. Karenanya tinggalah 343 mod (10) yang berarti (340+3), sehingga: ≡ 1 x 3 mod (10) ≡ 3 mod (10) Jadi, angka satuan dari bilangan 19971991 adalah 3. Dan itu adalah hasil penyelesaianku, Ayra!”terang Ikram menyelesaikan tugasnya. “Ada beberapa hal yang harus kau jelaskan lebih jauh Ikram,”tanya Yodha. “Bagian yang mana itu, Kawanku?”balas Ikram. “Pilihanmu pada angka 7 berpangkat 1991 yang kau uraikan menjadi perkalian (4x497+3) itu, Kawan!”Yodha kembali mengejar. “Baik. Intinya pada 7 pangkat (4x497+3) itu, kawan-kawan. Coba perhatikan kelipatan pangkat 7 berikut: •
7 pangkat 1
=7
•
7 pangkat 2
= 49
•
7 pangkat 3
= 343,
•
7 pangkat 4
= 2.401,
•
7 pangkat 5
= 16.807, | 53
•
7 pangkat 6
= 117.649.
Terlihat bahwa setelah perpangkatan ke-4, terjadi perulangan satuan, yaitu dari …1 ke …7. Karenanya, angka 4 merupakan perulangan untuk perpangkatan 7,”terang Ikram. “Lalu, bagaimana dengan 497, Kram?” “Ya, pangkat 1991 merupakan kelipatan dari 497, dimana = (1991/497) = 4 sisa 3 Karenanya, setelah berulang untuk 497 kali, maka masih ada 3 kali lagi pangkat atas angka satuan 7 yang tersisa, sehingga: = 73 = 343 Sehingga, angka satuan terakhir yang tertinggal adalah 3. Dengan kata lain penyelesaian dari angka satuan dari 19971991 adalah 3. Begitu, Kawan-kawanku! Oh ya, apa petunjuk kunci jawaban menurut buku itu, Ayra?”tanya Ikram penasaran. “Tunggu dulu, aku buka bukunya. Di sini disebutkan bahwa penyelesaian dari angka satuan 19971991 adalah 3. Jawabanmu benar, Kawan. Hebat!”puji Ayra pada Ikram. “Sudah tiga persoalan kita selesaiakan. Tampaknya rehat dan secangkir teh bagus juga untuk memulihkan syaraf dan konsentrasi kita. Bagaimana Sabiq?”usul Amer. “Setuju. Tadi aku perhatikan Ayra bawa-bawa tempat minuman yang agak besar. Apa isinya, Ayra?”tanya Sabiq pada Ayra.
| 54
“Alhamdulillah, tadi Ibu bekali kita dengan Wedang Jahe. Cukuplah untuk 10 porsi. Jadi kita rehat 15 menit, ya Kawankawan!”tegas Ayra. “Oh ya. Tadi juga saat berangkat Mamakku membekali dengan beberapa buah Pisang Rebus. Coba kulihat ada berapa. Satu, dua, tiga,…., ada delapan, Kawan!”tiba-tiba Sabiq teringat akan bungkusan yang dititipkan Mamaknya saat pamitan akan berangkat untuk diskusi kelompok. “Ini namanya harmoni yang indah: Secangkir Wedang Jahe, Sepotong Pisang Rebus dan Semilir Angin Sore,”ujar Yodha. “Wah, sudah jadi satrawan juga kau, Boy!”celetuk Amer. …∑πχαΩ…
“Sudah segar, Kawan-kawan? Bisa kita teruskan?”seru Ayra sambil bangkit dari tempat duduknya. “Insyaallah,”jawab kawan-kawan Ayra hampir bersamaan. “Nah, sekarang giliran siapa ini?”tanya Ayra. “Aku usul Nomor 5, giliranmu Ayra!”kata Sabiq. “Setuju,”sahut Yodha. “Bagus juga,”sambung Fateh. “Jadi, bagaimana Ayra?”tanya Sabiq lagi. “Baik, aku selesaikan tugasku. Perempuan satu-satunya dalam Genk OSN itu maju dengan mantap. Diraihnya kapur tulis ketiga yang tinggal separuh itu. “Pada prinsipnya, persoalan Nomor 5 adalah persoalan penjumlahan pecahan biasa. Hanya saja dia membentuk deret sedemikian rupa, yang dikenal sebagai penjumlahan teleskopik | 55
(telescopic sum). Secara umum, terhadap bentuk penjumlahan sebagai berikut, berlaku hubungan sedemikian rupa sehingga: 1 1 1 = − … … … … … … … … … … … … … … … . . (8.1) 𝑘𝑘(𝑘𝑘 + 1) 𝑘𝑘 𝑘𝑘 + 1
1 1 1 1 = � − � … … … … … … … … … . … … . . … . (8.2) 𝑘𝑘(𝑘𝑘 + 𝑚𝑚) 𝑚𝑚 𝑘𝑘 𝑘𝑘 + 𝑚𝑚
1 1 1 1 = � − � … … . . . . (8.3) (𝑘𝑘 + 1)(𝑘𝑘 + 2) 𝑘𝑘(𝑘𝑘 + 1)(𝑘𝑘 + 2) 2 𝑘𝑘(𝑘𝑘 + 1)
Karenanya, bila soal Nomor 5 disusun ulang dengan memerhatikan pola teleskopik pada persamaan (8.2) di atas akan diperoleh nilai: 1 1 1 + + ⋯+ 1997.1998 1.2 3.4 1 1 1 1 1 𝐴𝐴 = 1 − + − + ⋯ + − 2 3 4 1997 1998 1 1 1 1 1 1 1 1 + − 2� + + ⋯+ � 𝐴𝐴 = 1 + + + + ⋯ + 1997 1998 2 4 1998 2 3 4 1 1 1 1 1 1 1 + − 1 − − ⋯− 𝐴𝐴 = 1 + + − + ⋯ + 1997 1998 2 999 2 3 4 1 1 1 1 1 1 − �+ +⋯+ 𝐴𝐴 = (1 − 1) + � − � + ⋯ + � 999 999 1000 1998 2 2 1 1 1 1 1 + + + ⋯+ + 𝐴𝐴 = 1997 1998 1000 1001 1002
𝐴𝐴 =
Dengan sedikit trik dan modifikasi, bila nilai A kita gandakan,
dikalikan 2, akan diperoleh: 1 1 1 1 1 1 2𝐴𝐴 = � + �+� + �+ ⋯+ ( + ) 1000 1998 1001 1997 1998 1000 1997 + 1001 1998 + 1000 1998 + 1000 �+� �+ ⋯+ � � 2𝐴𝐴 = � 1001𝑥𝑥1997 1998𝑥𝑥1000 1000𝑥𝑥1998 1 1 1 + + ⋯+ � … . (8.4) = 2998 � 1998𝑥𝑥1000 1000𝑥𝑥1998 1001𝑥𝑥1997
| 56
Sementara itu, berdasarkan data soal diketahui bahwa: 1 1 1 𝐵𝐵 = + +⋯+ 1000.1998 1001.1997 1998.1000
Maka, bila diperhatikan persamaan (8.4), khususnya pada bagian penjumlahan suku dalam kurung, akan diperoleh hubungan antara A dan B, yaitu: 2𝐴𝐴 = 2998 𝐵𝐵
Akhirnya, nilai dari (A/B) adalah: (A/B) = (2998/2) = 1499 Jadi, terbukti bahwa nilai yang diperoleh dari hasil perbandingan A dan B, adalah bilang bulat, yaitu: 1499. Demikian paparanku, Kawan-kawan. Aku tak tahu apakah nilai ini sama dengan kunci jawaban yang disediakan atau tidak,”terang Ayra. “Coba kau lihat Mer!”pinta Ikram pada Amer. “Baik. Di sini dituliskan bahwa angka perbandingan A terhadap B adalah sebesar 1499 dan merupakan bilangan bulat. Subhanallah! kau berhasil Ayra!”terang Amer bersemangat. “Alhamdulillah!”ucapan yang keluar dari mulut Ayra. “Tapi, tunggu dulu, Kawan. Bagaimana kau menjelaskan ketiga bentuk persamaan teleskopik itu, Ayra?”tanya Fateh serius. “Ya, aku juga bingung soal itu,”timpal Amer. “Baik, untuk menambah pemahaman kita tentang penjumlahan teleskopik ini, mari kita kerjakan 2 soal berikut, 1 soal dikerjakan oleh Amer dan 1 soal dikerjakan oleh Fateh. Oke?”usul Ayra. “Setuju!”jawab Amer dan Fateh bersamaan. “Misalkan terdapat persoalan berikut: ‘Find the sum: | 57
and,
1 1 1 1 1 + + + + 5𝑥𝑥7 7𝑥𝑥9 9𝑥𝑥11 11𝑥𝑥13 13𝑥𝑥15
1 1 1 1 1 + + + + 10 40 88 154 238 Bagaimana kita menyelesaikannya? Kita bisa memanfaatkan sifat penjumlahan teleskopik di atas. Kau harus mencoba Ayra! Kau juga bisa Fateh!”ujar Amer. “Biar aku dulu!”Fateh langsung maju. Dia menuliskan beberapa coretan. Lebih tepatnya penjumlahan dan pengurangan. “Kalau kita perhatikan bilangan penyebut untuk pola yang pertama, terdapat hubungan: 5𝑥𝑥7 = 𝑎𝑎𝑥𝑥𝑏𝑏 ⇒ 𝑎𝑎 = 5 𝑑𝑑𝑎𝑎𝑛𝑛 𝑏𝑏 = 7 ⇔ 𝑏𝑏 = 𝑎𝑎 + 2
7𝑥𝑥9 = 𝑎𝑎𝑥𝑥𝑏𝑏 ⇒ 𝑎𝑎 = 7 𝑑𝑑𝑎𝑎𝑛𝑛 𝑏𝑏 = 9 ⇔ 𝑏𝑏 = 𝑎𝑎 + 2
9𝑥𝑥11 = 𝑎𝑎𝑥𝑥𝑏𝑏 ⇒ 𝑎𝑎 = 9 𝑑𝑑𝑎𝑎𝑛𝑛 𝑏𝑏 = 11 ⇔ 𝑏𝑏 = 𝑎𝑎 + 2
Dan seterusnya berpola sama. Sehingga memenuhi persamaan: 1 1 1 1 = � − � 𝑘𝑘(𝑘𝑘 + 𝑚𝑚) 𝑚𝑚 𝑘𝑘 𝑘𝑘 + 𝑚𝑚
Dengan m=2, dan k = 5, 7, 9, 11, dan 13. Karenanya: 1 1 1 1 1 1 1 1 1 1 1 = �� − � + � − � + � − � + � − � + � − �� 2 5 7 7 9 9 11 11 13 13 15 1 1 1 = � − � 2 5 15 1 = 15
Bagaimana, Kawan?”tanya Fateh.
| 58
“Mantap!
Tepat
sekali,
Kawanku
Fateh.
Bagaimana
denganmu, Ayra?”tanya Amer lagi. Ayra maju, di tangannya sudah ada alat tulis. “Kalau kita perhatikan penyebutnya, maka terdapat hubungan diantaranya pada angka 10, 40, 88, 154, dan 238, yaitu: 10 = 2𝑥𝑥5 = 𝑚𝑚𝑥𝑥𝑛𝑛 ⇒ 𝑚𝑚 = 2 𝑑𝑑𝑎𝑎𝑛𝑛 𝑛𝑛 = 5 ⇔ 𝑛𝑛 = 𝑚𝑚 + 3
40 = 5𝑥𝑥8 = 𝑚𝑚𝑥𝑥𝑛𝑛 ⇒ 𝑚𝑚 = 5 𝑑𝑑𝑎𝑎𝑛𝑛 𝑛𝑛 = 8 ⇔ 𝑛𝑛 = 𝑚𝑚 + 3
88 = 8𝑥𝑥11 = 𝑚𝑚𝑥𝑥𝑛𝑛 ⇒ 𝑚𝑚 = 8 𝑑𝑑𝑎𝑎𝑛𝑛 𝑛𝑛 = 11 ⇔ 瘼 = 𝑚𝑚 + 3
154 = 11𝑥𝑥14 = 𝑚𝑚𝑥𝑥𝑛𝑛 ⇒ 𝑚𝑚 = 11 𝑑𝑑𝑎𝑎𝑛𝑛 𝑛𝑛 = 14 ⇔ 𝑛𝑛 = 𝑚𝑚 + 3
Dan seterusnya dengan pola yang sama hingga m =14 dan n =17. Karenanya, pola di atas memenuhi bentuk persamaan (8.2), yaitu: 1 1 1 1 = � − � 𝑘𝑘(𝑘𝑘 + 𝑚𝑚) 𝑚𝑚 𝑘𝑘 𝑘𝑘 + 𝑚𝑚
Sehingga,
1 1 1 1 1 + + + + 10 40 88 154 238 Dapat ditulis ulang menjadi:
1 1 1 1 1 + + + + 2𝑥𝑥5 5𝑥𝑥8 8𝑥𝑥11 11𝑥𝑥14 14𝑥𝑥17 1 1 1 1 1 1 1 1 1 1 1 = �� − � + � − � + � − � + � − � + � − �� 5 8 8 11 11 14 14 17 3 2 5 1 1 1 = � − � 3 2 17 5 = 34 | 59
Jadi, hasil akhir perhitungan jumlah teleskopik itu adalah 5/34, Kawan-kawan,”pungkas Ayra. “Bukan main! Indah dan menarik sekali,”puji Ikram. Ayra hanya tersenyum tipis. Di tampak puas. “Plok, plok, plok,”kembali terdengar applause. …∑πχαΩ…
Sementara keenam sahabat itu sedang asyik dengan persoalan mereka, tepat di atas kepala mereka tampak 2 ekor anak burung Kutilang yang tampaknya baru mendapatkan kemampuan terbang perdananya, berlompatan dari satu dahan ke dahan lainnya. Kegirangan kedua burung Kutilang muda itu tergambar dari kicauan mereka yang begitu meriah, meski masih terkesan suara kekanakkanakan. Tampaknya kehadiran enam sahabat, tepat di bawah keduanya, tak mampu mengusik keasyikan bermain dua Kutilang muda itu. Dan sebaliknya, keenam sahabat itu pun tak terusik, bahkan menikmati kicauan kedua burung yang beranjak dewasa itu. “Tampaknya giliran kali ini, tidak bisa tidak, harus Yodha. Kita bahas soal Geometri. Bagaimana Yodha?”tanya Ayra. “Aku? Insyaallah, Ayra,”jawab Yodha datar. Yodha bangkit, ia menuju papan tulis tua itu. Diterimanya kapur tulis dari Ayra, dan dia pun memulai aksinya. “Untuk persoalan Appetizer lingkaran dalam kasus ini membutuhkan sedikit kejelian,”kalimat pembuka Yodha pada kawan-kawannya.
| 60
Tampak kelima kawannya itu mengangguk. Bahkan kali ini Sabiq yang biasanya tenang, tampak ikut juga menganggukkan kepalanya. “Sepertinya jawaban E akan banyak dipilih oleh yang membaca dan mengerjakan soal ini. Hal ini wajar karena dianggap tidak terdapat cukup informasi untuk menjawabnya. Tapi tunggu dulu! Seperti Bu Guru sering sebutkan, bahwa selalu ada jalan untuk penyelesaian Matematika,”terang Yodha lagi. Kelima kawannya masih tampak sangat serius. “Dari gambar yang disajikan pada Nomor 6, bila kita modifikasi sedikit dan lengkapi sehingga akan menghasilkan gambar yang lebih utuh, di mana R dan r merupakan jari-jari lingkaran luar dan lingkaran dalam. Selanjutnya akan terlihat sketsa sebagai berikut,”terang Yodha.
“Perhatikan gambar di atas. Dari data garis ST yang diberikan pada persoalan, kita tarik garis dari titik C, yang merupakan titik pusat kedua lingkaran, ke titik P yang menjadi titik singgung garis ST pada lingkaran kecil. Sebagaimana sifat Garis Singgung pada Lingkaran terhadap Jari-jari, maka garis CP akan tegak lurus. Selanjutnya, dari kedua titik potong garis ST terhadap Lingkaran Luar, kita akan memeroleh dua buah segitiga siku-siku di P, yaitu | 61
SPC dan TPC, di mana sudut P = 90°. Karenanya, dengan dalil Phytagoras, akan terlihat hubungan: 𝑅𝑅 2 = 𝑆𝑆𝑃𝑃2 + 𝑟𝑟 2
1 𝑆𝑆𝑃𝑃2 = 𝑅𝑅 2 − 𝑟𝑟 2 ⇔ 𝑆𝑆𝑃𝑃 = 𝑆𝑆𝑇𝑇 2
182 = 𝑅𝑅 2 − 𝑟𝑟 2 … … … … … . (8.5)
Sementara itu, untuk menemukan luas daerah yang diarsir, adalah sama dengan menemukan selisih luas lingkaran besar dengan lingkaran kecil, di mana: Luas Lingkaran Besar = π R2 ...................(8.6) Luas Lingkaran Kecil
= π r2 ....................(8.7)
Selanjutnya, bila Persamaan (8.6) kita kurangi dengan Persamaan (8.7), akan diperoleh Luas Daerah Arsiran, yaitu: Luas Daerah Arsiran
= π (R2 – r2)
Dengan memerhatikan bahwa nilai (R2 – r2) pada Persamaan (8.5), akan kita peroleh: Luas Daerah Arsiran
= π (R2 – r2) = 182π satuan luas = 324π satuan luas.
Akhirnya, jawaban luas daerah yang diarsir yang tepat berdasarkan perhitunganku adalah 324π satuan luas. Selesai!”papar Yodha. “Subhanallah! Geometri, memang kau orangnya Yodha. Beruntung kami punya kawan sepertimu,”puji Fateh dan Ayra hampir bersamaan. “Terima kasih, aku lebih setuju pendapat kawan kita sebelumnya, bahwa kita beruntung mempunyai guru matematika
| 62
yang baik dan pandai membimbing murid-muridnya seperti kita ini, Kawan-kawan,”balas Yodha bersahaja. “Bagaimana kawan-kawan, ada pertanyaan untuk penjelasan Yodha ini?”tawar Ayra. Tak ada jawaban. Semua tampak puas. “Cukup. Penjelasan dan uraian Yodha sudah sangat terang, Kawanku Ayra!”jawab Amer mewakili. “Plok, plok, plok,”kembali terdengar tepuk tangan meriah dari kawan-kawan Yodha untuk penyelesaian yang diajukannya. “Tapi, tunggu dulu. Bukankah kita belum melihat kunci jawaban yang diberikan?”tanya Yodha pada kawan-kawannya. “Ah, aku pikir tak perlu. Aku yakin jawabanmu betul, Yodha,”balas Ikram. “Ya, aku setuju dengan Ikram,”timpal Amer. “Meskipun
begitu,
kita
perlu
melakukan
verifikasi,
Kawan,”tegas Yodha lagi. “Baiklah, aku lihat di buku ini dulu, ya,”Ayra memotong pembicaraan. Beberapa saat hening. Semua menunggu. “Kunci jawaban menyebutkan bahwa luas daerah yang diarsir adalah 324π satuan luas. Cocok!”terang Ayra. “Alhamdulillah,”ucap Yodha tenang. Keenam sahabat itu pun tampak gembira. Semua merasa puas dan dapat saling berbagi atas kerja masing-masing. “Akhirnya kita sampai pada persoalan Nomor 1. Dan itu berarti giliran Sabiq. Silahkan kawan!”pinta Ayra. | 63
“Kali ini, kita akan mendengar penjelasan Sastrawan kita tentang Matematika,”Amer berkomentar. Sabiq maju sambil menerima kapur tulis dari tangan Ayra. “Sebelum aku mencoba menyelesaikan persoalan ini, jujur saja, aku mau bertanya pada semua kawan,”ujar Sabiq memulai. Kelima sahabat itu saling pandang. Heran. “Apakah ada di antara kawan-kawan yang telah mencoba untuk memecahkan persoalan Nomor 1 ini? Selain Nomor yang menjadi tugas masing-masing tentunya,”tanya Sabiq Kali ini, kelima sahabat itu seperti tersengat listrik. Ekspresi wajah mereka tampak kaget. “Apa maksudmu, Kawan?”tanya Fateh tenang. “Aku yakin, kalau ada diantara kawan-kawan yang mencoba, akan menemukan satu keanehan. Akan sangat bermanfaat bila kita sharing, Kawanku Fateh,”terang Sabiq. “Ha...ha...ha... kau ini Sastrawan tampaknya merangkap Dukun, Kawanku. Baik aku mulai. Aku memang mencoba mengerjakan persoalan Nomor 1 itu. Dan aku menemukan sesuatu yang aneh. Tapi, apa ya.....,”tiba-tiba Amer dengan suara yang khas itu memecah kebingungan kawan-kawannya. “Aku
juga
menyelesaikannya.
harus Tapi,
katakan, aku
bahwa
terbentur
aku
dengan
mencoba
keterbatasan
persamaan yang tersedia dari data yang disajikan. Akhirnya, aku berhenti,”terang Ikram. “Yodha, kawanku?”tanya Sabiq.
| 64
“Aku juga begitu. Sampai pada titik krusial. Memeroleh satu persamaan
linier
dengan
2
variabel.
Bagaimana
memecahkannya?”kata Yodha. “Dan, Fateh kawanku yang kritis?”pancing Sabiq. “Sabiq kawanku. Tak ada yang bisa aku sembunyikan darimu, Kawan. Ya, aku bahkan mencoba berkali-kali. Tapi, selalu buntu. Butuh sesuatu yang bisa membantu untuk keluar dari lingkaran persoalan sederhana tapi sulit,”terang Fateh. ‘Dan, kau....”belum sempat Sabiq meneruskan kalimatnya, terdengar balasan. “Ya, Biq. Aku pun seperti kawan-kawan lainnya. Berusaha mencoba. Hanya saja, aku bisa menemukan penyelesaiannya tapi tak tahu caranya?”terang Ayra. “Bagaimana bisa begitu, Ayra?”tanya Amer penasaran. “Ya, aku menemukan nilai panjang dan lebarnya dengan trial and error. Hanya saja bagaimana caranya, aku bingung,”terang Ayra lagi. Entah kenapa keenam sahabat itu saling pandang satu sama lain. Begitupun Sabiq. Hanya saja kali ini wajah Satrawan plus Dukun itu agak tersenyum puas. “Terima kasih sudah jujur kawan-kawan. Sejatinya itulah maksud pertanyaanku tadi. Meski untuk itu, Amer menambah namaku jadi Dukun,”kata Sabiq enteng. “Tapi, rasanya julukan itu tepat juga. Setidaknya dalam kasus Nomor 1 ini, Kawan!”balas Ikram. “Baiklah, aku mulai saja pembahasan Nomor 1 ini. Sesungguhnya tidak ada yang istimewa dalam persoalan yang | 65
disajikan. Pertama Ayra sudah tepat, melakukan trial and error alias coba-salah-ulangi. Itu pun bagian dari cara menemukan solusi permasalahan dalam matematika. Bahkan dalam ilmu pengetahuan secara umum. Dan jawabanmu berapa, Ayra?”tanya Sabiq langsung. “Eeee...panjangnya 6 satuan dan lebarnya 3 satuan. Atau sebaliknya, Kawan!”jawab Ayra. “Ya, aku pun menemukan seperti itu, Ayra”balas Sabiq. “Caranya bagaimana, Biq?”kali ini Yodha penasaran. “Baik. Masih ingat petuah orang-orang tua kita dan itu selaras dengan ajaran agama kita, Islam? Bahwa kembalikan-pinjaman dan bayarlah-hutang!”terang Sabiq. “Tentu, kita semua tahu itu, Kawan. Tapi, apa hubungannya dengan sebuah persamaan linier dengan dua variabel dan harus ditemukan solusinya, Kawan?”tanya Yodha. “Itulah prinsipnya. Keterbatasan persamaan sehingga terkesan tak bisa diselesaikan, berputar-putar, hanya mungkin ditebak. Tapi, sesungguhnya, kita bisa melakukan sedikit manipulasi,”terang Sabiq “Manipulasi?”tanya Ayra “Manipulasi terhadap persamaan sedemikian rupa agar menjadi
persamaan
yang
bisa
difaktorkan
dan
dijelaskan
kemungkinan penyelesaiannya, Kawan. Dan disitulah berlaku prinsip kembalikan-pinjaman dan bayarlah-hutang!”terang Sabiq. Seakan tersadar, kelima kawan Sabiq saling pandang dan mulai terenyum. “Mari kita lihat persoalan konkretnya! Dikatakan bahwa sebuah persegi panjang yang luas dan kelilingnya sama. Misalkan | 66
panjang persegi panjang itu adalah p dan lebarnya adalah l. Dari informasi awal kita peroleh data tentang Persegi Panjang: 𝐿𝐿𝑢𝑢𝑎𝑎𝑠𝑠 = 𝑝𝑝𝑙𝑙
𝐾𝐾𝑒𝑒𝑙𝑙𝑖𝑖𝑖𝑖𝑖𝑖𝑖𝑖𝑖𝑖 = 2(𝑝𝑝 + 𝑙𝑙)
Karena Luas Persegi Panjang dan Kelilingnya sama, maka akan diperoleh persamaan: 𝐿𝐿𝑢𝑢𝑎𝑎𝑠𝑠 = 𝐾𝐾𝑒𝑒𝑙𝑙𝑖𝑖𝑙𝑙𝑖𝑖𝑖𝑖𝑖𝑖 𝑝𝑝𝑙𝑙 = 2(𝑝𝑝 + 𝑙𝑙)
𝑝𝑝𝑙𝑙 − 2𝑝𝑝 − 2𝑙𝑙 = 0 … … … … … … … … . . (8.8)
Sampai di titik ini, apa yang disebut Yodha, satu persamaan linier dengan 2 variabel menjadi benar. Solusinya? Ingat prinsip kembalikan-pinjaman dan bayarlah-hutang. Gunakan!”terang Sabiq serius. Tampak kelima kawannya pun serius mengamati penjelasan Sastrawan-Dukun itu. “Bila terhadap Persamaan (8.8) kita meminjam angka 4 yang kita tempatkan di kedua sisinya, akan diperoleh persamaan: p l – 2p – 2l + 4 = 4 Selanjutnya kita susun ulang menjadi: p (l-2) – 2(l-2) = 4 kita keluarkan kedua suku yang sama pada persamaan, yaitu (l-2), sehingga akan diperoleh persamaan: (l – 2 ) (p – 2) = 4 .................................. (8.9) Sampai di titik ini, pada Persamaan (2), barulah langkah trial and error yang digunakan oleh Ayra berlaku dan bisa menemukan rasionalisasinya, Kawan-kawan,”terang Sabiq. “Cek, cek, cek. Gila kau, Bro!”terdengar celotehan Amer. | 67
Tampak wajah Ayra, Yodha, Ikram, dan Fateh dan Amer, tentunya, berseri-seri . “Selanjutnya tinggal melihat kemungkinannya. Kemungkinan itu diperoleh dari faktor yang dikandung oleh 4, yaitu perkalian dua bilangan yang menghasilkan nilai 4! Sehingga, kemungkinankemungkinan itu adalah: Kemungkinan I (1,4), hal ini terjadi dengan kondisi: (l – 2) = 1 (p – 2) = 4
⇔l=3
⇔p=6
Kemungkinan II (2,2) hal ini terjadi dengan kondisi: (l – 2) = 2 (p – 2) = 2
⇔l=4
⇔p=4
Kemungkinan III (4,1) hal ini terjadi dengan kondisi: (l – 2) = 4 (p – 2) = 1
⇔l=6
⇔p=3
Selanjutnya kita periksa dan sesuaikan dengan data. Bahwa bentuknya adalah persegi panjang, sehingga panjang dan lebarnya berbeda. Karenanya, Kemungkinan II yang menghasilkan p=4 dan l=4 tidak memenuhi. Sehingga jawaban yang benar adalah Kemungkinan I dan III, untuk nilai p=6 dan l=3 atau p=3 dan l=6. Bila nilai ini diverifikasi terhadap luas dan kelilingnya, menjadi: Luas
= (6 x 3) atau (3 x 6) = 18 satuan luas
Keliling
= 2 (6+3) atau 2 (3 + 6) satuan = 18 satuan | 68
Yang berarti bahwa nilai besaran luas dan kelilingnya adalah sama. Karenanya persyaratan Nomor 6 terpenuhi. Demikian penjelasanku, Kawan-kawan. Sekian semoga bermanfaat!”pungkas Sabiq. Tampak keceriaan yang besar dari wajah-wajah enam sahabat itu setelah selesai mereka menemukan penyelesaian atas soal-soal Appetizer dari buku titipan Kakek Penjaga Musholla. “Biq, suatu saat kau harus menjelaskan lebih jauh tentang prinsip
kembalikan-pinjaman
dan
bayarlah-hutang
itu
pada
kita!”pinta Fateh sambil menepuk bahu sahabatnya itu. “Insyaallah, Kawan,”janji Sabiq. Tepat lima belas menit sebelum pukul 17.00, keenam sahabat itu bubar. Sejak sore ini, seberkas optimisme mulai muncul dan terus tumbuh dari masing-masing sanubari mereka. …∑πχαΩ…
| 69
9
Dari Delapan Penjuru Angin
“Setelah menikmati Appetizer yang membangkitkan semangat untuk mengembara di alam Matematika, pada bagian berikut, akan ditampilkan sejumlah catatan yang dihimpun, setidaknya dari tempat-tempat yang jauh, panas dan dingin serta mewakili 8 penjuru angin,”begitu tulis pengantar dalam buku yang dititipkan oleh Kakek Penjaga Musholla itu saat Ayra membuka lembar berikutnya. “Tunggu, Ayra! Apa maksudnya tempat-tempat yang jauh, panas dan dingin serta mewakili 8 penjuru angin itu?”Ikram memotong. “Aku tak tahu, Kawan. Coba aku teruskan dulu kalimat selanjutnya. Mungkin akan menjadi jelas setelahnya,”jawab Ayra. Ayra kembali membuka lembaran itu. “Tempat yang jauh karena memang bukan dari bumi Nusantara ini asal catatan itu. Panas dan dingin karena di beberapa tempat asal catatan ini, udaranya bisa membekukan sementara di tempat yang lain, bisa membakar kulit. Dan catatan itu memang mewakili 8 penjuru angin,”tegas kalimat penjelas dalam buku itu. “Tapi,…”Ayra menghentikan bacaannya. “Tapi, apa Ayra?”kali ini Fateh yang menyela. | 70
“Tapi, demi menghormati pengirim catatan itu, sengaja dituliskan naskahnya dalam bahasa aslinya, sementara penjelasannya dalam bahasa Nusantara. Karenanya pula, sebagai penghormatan atas kiriman catatan itu, akan dituliskan asal lahirnya catatan itu,”kalimat akhir yang tertulis dalam paragraf akhir buku itu. “Aih, kalau begitu bertambah tugas Sastrawan-Dukun kita. Kau pun harus jadi Penerjemah, Kawan!”kembali Amer berceloteh dengan suara beratnya yang khas itu sambil menoleh ke arah Sabiq. “Wah, baru 3 hari Sabiq sudah punya 3 gelar; Sastrawan, Dukun, dan Penerjemah,”kali ini Yodha berkomentar. “Lanjut, Ayra!”pinta Fateh. Ayra kembali konsentrasi pada catatan dalam buku itu. “Baik, kita mulai dari Afrika Selatan. Perhatikan gambar di bawah ini! “Two identical squares meet at a vertex as shown. The size of the angle marked x is…. (Pretoria) (A) 1050 (B) 1200 (C) 1300 (D) 1500 (E) 1600
“Perhatikan bahwa segitiga yang terbentuk antara 2 bujur sangkar adalah segitiga sama kaki, sehingga sudut yang terbentuk pada bagian atas segitiga itu haruslah: 1800 – (2 x 650) = 500 Selanjutnya perhatikan bahwa sudut x merupakan bagian dari sudut utuh yang melingkar pada pertemuan 2 bujur sangkar (kiri dan | 71
kanan), segitiga dan ruang terbuka x, sehingga besar sudut x dapat dicari dengan membuat persamaan: = 3600 – (2x900) – 500 = 3600 – 1800 – 500 = 1300 “Mantap, Ayra!”komentar Amer “Berikutnya,
masih
dari
asal
yang
sama.
Perhatikan
persoalannya: ‘A rectangle has one corner placed at the centre of a quarter-circle and the opposite corner on the circumference of the circle. If the rectangle has width 6 cm and length 8 cm, then the shaded area in cm2 is… (Pretoria) (A) 9π–48 (B) 16π–48 (C) 25π–48 (D) 36π–48 (E) 49π–48
“Kata kunci pertanyaannya pada frasa the shaded area, Kawan! Kira-kira maknanya, luas daerah yang diarsir,”jelas Sabiq. “Perhatikan gambar di atas baik-baik. Bahwa jari-jari dari seperempat lingkaran merupakan diagonal dari persegi panjang. Dengan bantuan rumus Phytagoras, kita bisa menghitung panjang diagonal persegi panjang sebagai: r 2 = 62 + 82 r = √100 cm
⇔ r2 = 36 + 64
⇔ r = 10 cm
Selanjutnya, bahwa luas daerah yang diarsir adalah sama dengan luas seperempat lingkaran dikurangi oleh luas persegi panjang. Sehingga: Luas Arsiran
= (1/4 π102 – 8x6) cm2 | 72
= (100/4 π – 48) cm2 = 25π – 48 Sehingga jawaban yang tepat adalah pilihan (C),”tulis penjelasan atas soal itu. “Ternyata tak sepanas daerah asalnya, Kawan!”celoteh Amer. “Jangan sombong, Kau Bro!”tegur Ikram. “Catatan dari mana lagi, Ayra?”tanya Yodha. “Dari Afrika Selatan, kita bergerak ke Jamaica!”ucap Ayra. “Wah, Negeri Regge, donk!”celetuk Fateh. “Perhatikan hal berikut:“What is �𝑝𝑝�𝑞𝑞
3
if,
𝑝𝑝 = 1𝑥𝑥2𝑥𝑥4 + 2𝑥𝑥4𝑥𝑥8 + 3𝑥𝑥6𝑥𝑥12 + ⋯ + 2014𝑥𝑥4028𝑥𝑥8056
𝑞𝑞 = 1𝑥𝑥3𝑥𝑥9 + 2𝑥𝑥6𝑥𝑥18 + 3𝑥𝑥9𝑥𝑥27 + ⋯ + 2014𝑥𝑥6042𝑥𝑥18126
Persoalan di atas memerlukan kejelian dan kehati-hatian untuk menemukan titik terangnya,”baca Ayra. “Tunggu dulu, Ayra! Ada komentar, Biq?”tanya Ikram. Sabiq hanya terdiam dan memandang terpaku ke arah persoalan. Tenang dan serius. Tak lama kemudian, seulas senyum tersungging. “Aku setuju! Butuh kejelian untuk menemukan pola yang menarik, Kawan,”ucap Sabiq. “Coba aku teruskan dulu membacanya, ya!”kata Ayra. “Oke, Ayra. Lanjutkan!”kata Yodha. “Mula-mula, perhatikan pola pada p = (1x2x4)+(2x1)(2x2)(2x4) + (3x1)(3x2)(3x4) +… | 73
+ (2014x1)(2014x2)(2014x4); = 13(1)(2)(4)+ 23(1)(2)(4) + 33(1)(2)(4) + · · · + 20143(1)(2)(4); Sehingga nilai p dapat disederhanakan menjadi p = (13 + 23 + · · · + 20143) (8) ……………..….(9.1) dengan cara yang sama kita bisa perlakukan terhadap nilai q, sehingga akan terlihat bahwa: q
= (1)(3)(9) + (2)(6)(18) + (3)(9)(27) + · · · + (2014)(6042)(18126); = (1x3x9) + (2x1)(2x3)(2x9) + (3x1)(3x3)(3x9) +…+ (2014x1)(2014x3)(2014x9); = (1)(3)(9)+ 23(1)(3)(9) + 33(1)(2)(4) + · · · + 20143(1)(2)(4); = (13 + 23 + 33 + · · · + 20143) (27) ……… (9.2)
q
Karenanya, dari Persamaan (9.1) dan Persamaan (9.2), di atas: (13 + 23 + 33 + ⋯ + 20143 )(8) 𝑝𝑝 = 3 𝑞𝑞 (1 + 23 + 33 + ⋯ + 20143 )(27)
8 𝑝𝑝 = 𝑞𝑞 27
Sehingga,
1� 3
8 �𝑝𝑝�𝑞𝑞 = � � 27
3
23 = � 3� 3
1� 3
�𝑝𝑝�𝑞𝑞 =
3
2 3 | 74
“Ternyata, betul, Kau Biq! Butuh kejelian untuk menemukan pola tersembunyinya,”komentar Amer. “Dan persoalan berikutnya, dikatakan bahwa, “A solid box is 15 cm by 10 cm by 8 cm. A new solid is formed by removing a cube 3 cm on a side from each corner of this box. What percent of the original volume is removed? (the University of the West Indies),”ucap Ayra. “Adakah pilihan jawabannya, Ayra?”tanya Yodha. “Tidak ada Yodha. Soalnya isian singkat,”jawab Ayra. “Oh ya, disertakan gambar seperti ini, Kawan-kawan!”seru Ayra lagi.
“Apa lagi catatannya, Ayra?”tanya Yodha lagi. “Ada pada bagian berikutnya. Sebelumnya, apa kira-kira inti pertanyaannya, Biq?”tanya Amer. “Diminta menjelaskan berapa persen bagian volume kotak yang hilang saat masing-masing sudutnya dihilangkan seukuran kubus juga yang sisinya 3 sentimeter,”terang Sabiq. “Ooo, begitu,”ucap Amer. “Baik, di sini dijelaskan bahwa volume awal kotak itu adalah: = 15 cm × 10 cm × 8 cm = 1200 cm3. Selanjutnya ke delapan ujung kubus dihilangkan
atau dipotong
berbentuk dan seukuran kubus yang masing-masing volumenya: = 3 cm × 3 cm × 3 cm = 27 cm3. | 75
Karenanya, volume kotak yang hilang di ujungnya adalah: = 8 × 27 cm3 = 216 cm3 Sehingga bila dibandingkan antara volume ujung kotak yang dihilangkan dengan volume awal kotak adalah sebesar: = 216 cm3/1200 cm3 = (216/1200) = (18/100) = 18% “Sederhana dan jelas, ya Kawan-kawan!”tanya Ayra. “Alhamdulillah, clear Kawan,”balas Ikram. “Tampaknya Jamaica mengirimkan 3 persoalan,”Ayra berseru. “Masih ada, Ayra?”tanya Yodha. ‘Ya, Yodha. Dan tampaknya ini akan lebih seru,”balas Ayra. “Apa katanya, Ayra?”tanya Fateh. “Baik. aku baca, ya! ‘A wall clock strikes every hour. The number of strikes corresponds to the time. For example, at 10am and 10pm you will hear 10 strikes. The clock also strikes once at the halfhour mark. How many strikes can be heard in one 24-hour period?”bacaan bahasa Inggris yang lancar keluar dari mulut Ayra. “Nah, kalau ini, jujur aku belum paham maksudnya. Pak Penerjemah bisa bantu kita?”pinta Amer kepada Sabiq. Sesaat Sabiq memandangi catatan itu. Tak perlu lama, ia mengangkat kepalanya, dan berkata kepada kawan-kawannya. “Kira-kira begini sederhananya, Kawan. Berapa kali jam dinding berbunyi dalam 24 jam, jika setiap jamnya ia berbunyi, yang jumlah bunyinya sama dengan waktu yang ditunjukkannya. Misalnya | 76
sekali berbunyi untuk jam 1, 2 kali bunyi untuk jam 2, dan seterusnya. Dan juga, setiap setengah jam, jamnya juga berbunyi sekali,”terang Sabiq. “Apa saja pilihan jawabannya, Ayra?”tanya Yodha. “Pilihannya: (A) 90 kali; (B) 135 kali; (C) 180 kali; (D) 200 kali, dan (E) 120 kali, Yodha,”jawab Ayra. “Tebakanku 90 kali,”kata Amer. “Ya, mari kita lihat cara penyelesaiannya, Kawan,”kata Ayra lagi. “Dalam persoalan ini, dapat dimulai analisanya dari pukul 00:01 setelah lewat tengah malam. Jam dinding akan berbunyi setiap jam 1, jam 2, jam 3, dan seterusnya sampai jam 12 siang. Hingga jumlah bunyi yang terdengar adalah: = 1 + 2 + 3 + …+ 12 = 78 kali. Selanjutnya, jam dinding juga akan berbunyi sekali untuk setiap setengah jam. Hal ini berarti bahwa sejak pukul 00.30 sampai 11.30 siang , jam dinding akan berbunyi sebanyak: = 1 (00:30) +1+1+1+1+1+1+1+1+1+1+1 (11:30) = 12 kali. Sehingga selama 12 jam, sejak 00:01 pagi sampai dengan 12:00 siang. Jam dinding telah berbunyi sebanyak: = (78 +12) kali = 90 kali. “Yess! Tebakanku tepat, kan!”tiba-tiba Amer memotong bacaan Ayra. | 77
“Tunggu
dulu,
Sabar
Kawan.
Ayra
belum
selesai
membaca,”tegur Sabiq Ayra melanjutkan bacaannya. “Karenanya selama 24 jam, dari pukul 00:01 pagi sampai pukul 12.00 siang, dan dari pukul 12:00 siang sampai pukul 00:00 tepat tengah malam, jam dinding itu berbunyi sebanyak: = 2 x 90 kali = 180 kali. Jawaban yang tepat adalah pilihan (C), 180 kali jam dinding itu berbunyi selama 24 jam,”pungkas Ayra. “Nah, bagaimana Mer?”tanya Sabiq. “Oh ya, aku lupa menggandakan siang dan malam. Sorry, Bro!”jawab Amer sambil tersenyum. “Ha….ha…ha..,”terdengar lepas tawa keenam sahabat itu. “Oh ya, tak terasa sudah jam 10 lewat 5 menit, lho!”kata. “Ya, kita selesaikan mumpung hari ini libur, kan Ayra?”balas Yodha. “Ya, tapi, sebaiknya yang masih punya Wudhu, sholat Dhuha dulu, deh!”kata Ayra lagi. “Oh, iya. Saking asyiknya kita diskusi, Dhuha hampir lewat, Kawan. Kita Dhuha dulu, yuk!”kata Fateh. …∑πχαΩ…
Bentuk syukur atas rahmat yang diterima seorang hamba dari Sang Khaliq beragam wujudnya. Kerelaan melakukan aktivitas ibadah bukan sekadar karena perintah dan kewajiban adalah salah satu bentuk kesyukuran itu. Wajah-wajah bersih dan bercahaya dari enam | 78
sahabat itu seolah berlomba dengan sinar mentari yang terasa semakin hangat pagi itu. Doa pagi yang dipanjatkan oleh enam sahabat kepada Penguasa Alam dan Pengetahuan seakan menguatkan tekad yang telah mereka tanamkan beberapa hari lalu. Doa waktu Dhuha umumnya dihubungkan dengan rezqi seorang hamba. Tapi, doa dhuha dari keenam sahabat ini berbeda. Sebagai waktu Tuhan, Dhuha adalah waktu keagungan, keindahan, kekuatan dan penjagaan oleh Sang Khaliq. Mendekatkan yang jauh, memudahkan yang sukar dan mensucikan yang haram, adalah inti dari harapan rezqi berupa ilmu yang sedang didalami oleh keenam sahabat itu. Mereka yang berkumpul di waktu Dhuha untuk memelajari sebagian dari ilmu Tuhannya, Allah. “Kali ini, dari mana asal catatan yang akan ditunjukkan oleh buku itu, Ayra?”tanya Fateh pada Ayra. “Coba aku lihat dulu, Fateh,”jawab Ayra singkat. “Puerto Rico, Kawan!”seru Ayra bersemangat. “Berapa catatan, Ayra?”tanya Fateh lagi. “Satu saja. Langsung, ya?”tanya Ayra. “Oke, deh. Langsung saja, Ayra!”kata Yodha. Ayra membetulkan letak duduknya. Dia lebih santai dan kali ini begitu bersemangat. “In the equation 1 + (n2 + n)(n2 + 5n + 6) = 1812, where n is a whole number, what is the value of n(n + 3)?”ucap Ayra. “Masih seputar kejelian dan kecermatan memperhatikan susunan dan pola, Kawan,”ucap Sabiq. “Lantas apa kata buku itu selanjutya, Ayra?”tanya Ikram | 79
“Bila persamaan di atas kita susun ulang dengan sedikit modifikasi, akan terlibat sebagai: 1 + (n2 + n) (n2 + 5n + 6) = 1812 Bila angka 1 dipindah ke sisi kanan, dan tidak mengubah nilai persamaan, akan diperoleh: (n2 + n)(n2 + 5n + 6) = 1812 − 12 n (n + 1)(n + 2)(n + 3) = 1812 − 12 Memanfaatkan sifat operasi kuadrat, bahwa: a2 – b2 = (a+b) (a−b) Maka, 1812 − 12
= (181 − 1) (181 + 1) = 180 (182)
Selanjutnya perhatikanlah bahwa sisi kiri sesungguhnya merupakan perkalian dari 4 bilangan bulat berurutan. Sementara di sisi kanan, merupakan faktor dari: = (12 × 15) (13 × 14) = 12 × 13 × 14 × 15 Akhirnya,
dengan
menyusun
kembali
deretan
angka
dan
perkaliannya, akan didapati bahwa: n (n + 1)(n + 2) (n + 3) = 12 × 13 × 14 × 15 Hal ini pun berarti bahwa nilai dari n (n+3) dapat diketahui besarnya sebagai berikut: n (n+3) = 12 × 15 n = 180, sehingga, hasil perkalian n (n+3) nilainya adalah 180,”kata Ayra “Ya,
ketelitian
dan
kecermatan
pada
intinya,”Ikram
berkomentar. | 80
“Setelah selesai dengan catatan dari Puorto Rico, berikut adalah kiriman catatan dari Saskatchewan, Kanada,”kata Ayra. “Nah, dia boleh juga itu negara,”komentar Amer. “A square has four sides and two diagonals. A pentagon has five sides and five diagonals. How many sides does a regular polygon have, if it has 35 diagonals?”kata Ayra. “Sudah clear, kan! Tidak ada yang perlu aku terjemahkan. Sederhana kok bahasanya, ditanyakan berapa banyak sisi dari suatu poligon yang memiliki 35 diagonal,”Sabiq berkomentar. “Dilampirkan juga gambar sebagai ilustrasi,”kata Ayra.
“Ada
yang
mau
berkomentar
tentang
kemungkinan
penjelasannya?”tantang Yodha. “Rasanya aku pernah mendengar penjelasan dari seorang teman yang pada intinya menerangkan tentang hubungan antara jumlah sisi sebuah polygon dan jumlah diagonal yang bisa terbentuk dari masing-masing titik sudutnya,”tiba-tiba Fateh berkomentar. “Penjelasan siapa? Dan apa isi penjelasan itu, Fateh?”kali ini Sabiq bertanya serius. “Aku lupa tepatnya. Namun kalimatnya masih tegas kuingat. Dia menulis tentang Teorema Fermat yang menurutnya bisa diselesaikan dengan selembar kertas kerja,”terang Fateh. “Teorema Fermat yang sudah berumur ratusan tahu itu?”tanya Yodha penasaran. | 81
“Ya. Dia menulis begini, ‘Saya menghitung jumlah diagonal ruang dari prisma segi-n. Saya melihat dari sini bahwa setiap satu titik sudut di bawah, menghubungkan (n-3) titik sudut di atasnya yang berhadapan, sehingga jumlah diagonal ruang pada prisma segi-n adalah n (n-3). Saya definisikan ini sebagai a…’,”terang Fateh. “Ya, tepat seperti yang disampaikan oleh Fateh. Catatan ini menulis bahwa terdapat 2 cara untuk menjawab persoalan ini. Cara 1: dengan teknik trial and error yang konkretnya melukiskan gambar pentagon dan menarik diagonal dari setiap sudutnya ke sudut yang menghasilkan
diagonal.
Sehingga,
bila
persoalan
di
atas
ditabulasikan, maka hubungan antara jumlah sisi dan diagonal yang bisa ditarik, akan terlihat sebagai tabel berikut: Sisi
4
5
6
7
8
9
10
Diagonal
2
5
9
14
20
27
35
Karenannya, akan terdapat 35 diagonal yang mungkin dihasilkan dari polygon dengan lima sisi alias pentagon!” “Bagaimana dengan Cara 2-nya, Ayra?”tanya Fateh. “Tampaknya ada persamaan yang eksplisit menggambarkan hubungan diagonal dan jumah sisi pada sebuah poligon Kawan,”kata Ayra. “Dikatakan bahwa bila terdapat n titik pada suatu polygon, sebuah garis diagonal hanya dapat dihubungkan ke (n-3) titik lainnya, dan tak bisa terhubung pada titik itu sendiri.. Sehingga dimungkinkan
terbentuk n (n-3)
garis
diagonal,”kata
Ayra
membacakan keterangannya pada kelima kawannya. | 82
“Ya, ada kemiripan dengan tulisan RRHS yang aku ingat itu,”komentar Fateh. “Teruskan, Ayra!”pinta Ikram. “Meskipun begitu, karena setiap diagonal dihitung atau dibaca dua kali. Misalnya dari satu titik p ke titik q, dan satu kali dari titik q ke titik p, maka hasil akhirnya harus dikoreksi dengan membaginya dengan 2. Sehingga, secara umum dapat dituliskan hubungan jumah sisi dan diagonal pada poligon sebagai: 𝑑𝑑 =
𝑛𝑛 (𝑛𝑛 − 3) 2
Dengan semikian, dari persoalan yang dimukakan, dapat ditentukan menjadi: 𝑛𝑛(𝑛𝑛 − 3) 2 𝑛𝑛(𝑛𝑛 − 3) 35 = 2
𝑑𝑑 =
70 = 𝑛𝑛2 − 3𝑛𝑛 ↔
(𝑛𝑛 + 7)(𝑛𝑛 − 10) = 0
𝑛𝑛2 − 3𝑛𝑛 − 70 = 0
Sehingga diperoleh n = -7 yang tidak memenugi dan n = 10, bisa memeuhi. Terlihat bahwa ternyata hasil tabulasi dengan trial and error sama nilainya dengan hasil hitungan menggunakan formula hubungan sisi dan diagonal pada poligon,”ujar Ayra sambil tersenyum. “Alhamdulillah, dan indah betul solusinya,”tiba-tiba Amer berkomentar. “Indah? Maksudmu apa, Mer?”tanya Yodha.
| 83
“Ya, ketekunan mencoba dan salah, serta coba lagi, terkonfirmasi
dengan
formula
yang
ditentukan
sedemikian
rupa,”jawab Amer. “Itulah Matematika, Kawan. Insyaallah, selalu ada jalan!”kali ini Sabiq berkomentar. “Selanjutnya catatan dari negeri mana, Ayra?”tanya Ikram. “Australia, tepatnya Queensland, Kawan!”jawab Ayra singkat. Ayra kembali meneruskan membaca. “Ada dua pertanyaan. Pertama, ‘how many positive integer solutions x and y are there of the equation x2 - y2 = 75?’ Dan kedua, ‘suppose x, y and z are numbers satisfying: x+y+z
= -2
2
2
2
= 122
3
3
3
= 142
x +y +z x +y +z
What is the value of xyz?”jelas Ayra. “Penyelesaian atas persoalan ini membutuhkan sedikit keluwesan dalam membangun dan mengembangan persamaan yang terbentuk. Misalkan saja, bahwa dari persamaan x2 - y2 > 0, x dan y merupakan bilangan bulat positif di mana x > y. Bila persamaan yang terbentuk difaktorkan kembali dengan mengingat bahwa: x2 –y2= (x –y ) (x + y) Maka akan diperoleh: (x -y) (x + y) = 75 Dari persamaan di atas dapatlah disebut bahwa kedua suku di sisi kiri “pastilah” merupakan faktor bilangan bulat positif dari 75. Dengan sedikit pengubahan formulasi persamaan akan diperoleh bahwa: x = 1/2[(x - y) + (x + y)] | 84
Sehingga, bila nilai x di atas dibuat tabel kemungkinannya, akan diperoleh hasil pasangan x dan y sebagai berikut: x-y
1
3
5
x+y
75
25
15
(x, y)
(38, 37)
(14, 11)
(10, 5)
“Untuk solusi atas pertanyaan kedua, dapat kita selesaikan dengan memperhatikan hubungan dari bilangan kuadrat serta kubik. Dengan mengingat bahwa: (𝐴𝐴 + 𝐵𝐵)3 = 𝐴𝐴3 + 3𝐴𝐴2 𝐵𝐵 + 3𝐴𝐴𝐵𝐵2 + 𝐵𝐵3
Karenanya, akan dapat disusun persamaan:
(𝑥𝑥 + 𝑦𝑦 + 𝑧𝑧)3 − (𝑥𝑥 3 + 𝑦𝑦 3 + 𝑧𝑧 3 ) = 3A + 6xyz
= 3(𝑥𝑥 2 𝑦𝑦 + 𝑥𝑥 2 𝑧𝑧 + 𝑦𝑦 2 𝑥𝑥 + 𝑦𝑦 2 𝑧𝑧 + 𝑧𝑧 2 𝑥𝑥 + 𝑧𝑧 2 𝑦𝑦) + 6𝑥𝑥𝑦𝑦𝑦𝑦
Pemberian suku A pada persamaan, seolah-olah merupakan bentuk lain dari perkalian antara (x+y+z) dengan (x2+y2+z2). Di mana: (𝑥𝑥 + 𝑦𝑦 + 𝑧𝑧)(𝑥𝑥 2 + 𝑦𝑦 2 + 𝑧𝑧 2 ) =
= (𝑥𝑥 3 + 𝑦𝑦 3 + 𝑧𝑧 3 ) + (𝑥𝑥 2 𝑦𝑦 + 𝑥𝑥𝑦𝑦 2 + 𝑥𝑥 2 𝑧𝑧 + 𝑦𝑦 2 𝑧𝑧 + 𝑥𝑥𝑧𝑧 2 + 𝑦𝑦𝑧𝑧 2 )
Bila dihimpun suku-suku yang diketahui, dan dimisalkan: 𝑥𝑥 + 𝑦𝑦 + 𝑧𝑧 = 𝑝𝑝1 = −2
𝑥𝑥 2 + 𝑦𝑦 2 + 𝑦𝑦 2 = 𝑝𝑝2 = 122 Maka,
𝑥𝑥 3 + 𝑦𝑦 3 + 𝑦𝑦 3 = 𝑝𝑝3 = 142
1 𝑥𝑥𝑦𝑦𝑦𝑦 = (𝑝𝑝12 − 3𝑝𝑝1 𝑝𝑝2 + 2𝑝𝑝3 ) 6 1 𝑥𝑥𝑦𝑦𝑦𝑦 = ((−2)3 − 3(−2)(142) + 2(142) 6 1 𝑥𝑥𝑦𝑦𝑦𝑦 = (1008) = 168 6
| 85
Penyelesaian dengan menggunakan persamaan seperti tersebut di atas, dalam matematika sering disebut sebagai Identitas Newton, (Newton’s Identities),”tutup Ayra mengakhiri bacaannya. “Tunggu dulu, Ayra!”tiba-tiba Amer mengangkat tangan kanannya. “Ada apa, Kawan?”tanya Ayra. “Jujur, aku masih bingung dengan penyelesaian terakhir yang kau baca tadi,”kata Amer lugas. “Aku juga belum sepenuhnya paham, Kawan,”balas Ayra. “Mungkin kau punya penjelasan, Pak Dukun?”tanya Amer lagi. Kali ini pada Sabiq. Sabiq tak langsung menjawab. Seulas senyum terlukis di wajahnya. Senyum khas yang misterius itu. “Insyaallah. Sekadar pengantar untuk Newton’s Identities, kira-kira seperti mencari atau menemukan hubungan antara operasi bilangan berpangkat sedemikian rupa. Misalnya antara pangkat 1 dengan pangkat 2 dan pangkat 3, atau pangkat yang lebih tinggi lagi, Kawan-kawan.”ujar Sabiq tenang. Tampak Amer dan Ayra bersemangat mendengarkan, begitu pun Yodha. “Tapi, ada sedikit catatanku atas penyelesaian kedua itu, mungkin catatan ini berguna,”Sabiq melanjutkan kalimatnya. “Catatan apa itu, Kawan?”tanya Ayra “Catatan tentang bentuk kuadrat dan kubik tadi yang kau bacakan, hubungannya dengan bentuk linier, Ayra!”jawab Sabiq. “Nah, di bagian itu pula aku bingung, Biq. Kau jelaskanlah catatanmu itu!”pinta Amer. | 86
“Baiklah. Menurutku sebelum menyelesaikan soal kedua itu, kita harus ingat tentang proses perpangkatan dan koefisienkoefisiennya,”jelas Sabiq. “Segitiga Pascal, maksudmu, Biq!”sela Yodha. “Tepat, Kawan! Segitiga Pascal. Akan sangat membantu dalam proses penulisan koefisien dalam pemangkatan,”jawab Sabiq. “Bisa kau jelaskan kembali pada kita semua, Kawan?”tanya Amer. “Insyaallah, Kawan! Setiga pascal secara sederhana dapat digambarkan dengan ilustrasi berikut:
Bagian kiri (x+y) menunjukkan proses perpangkatan variabel, sementara bagian kanan menunjukkan koefisien masing-masing variabel, sehingga: (𝑥𝑥 + 𝑦𝑦)0 = 1
(𝑥𝑥 + 𝑦𝑦)1 = 1𝑥𝑥 1 + 1𝑦𝑦1
(𝑥𝑥 + 𝑦𝑦)2 = 1𝑥𝑥 2 + 2𝑥𝑥𝑦𝑦 + 1𝑦𝑦 2
(𝑥𝑥 + 𝑦𝑦)3 = 1𝑥𝑥 3 + 3𝑥𝑥 2 𝑦𝑦 + 3𝑥𝑥𝑦𝑦 2 + 1𝑦𝑦 3
(𝑥𝑥 + 𝑦𝑦)4 = 1𝑥𝑥 4 + 4𝑥𝑥 3 𝑦𝑦 + 6𝑥𝑥 2 𝑦𝑦 2 + 4𝑥𝑥𝑦𝑦 3 + 1𝑦𝑦 6
Sehingga berdasar atas segitiga Pascal di atas bentuk persamaan kubik x3+y3+z3 dapat ditulis menjadi: | 87
(𝑥𝑥 + 𝑦𝑦 + 𝑧𝑧)3 = 1(𝑥𝑥 + 𝑦𝑦)3 + 3(𝑥𝑥 + 𝑦𝑦)2 𝑧𝑧 + 3(𝑥𝑥 + 𝑦𝑦)𝑧𝑧 2 + 1𝑧𝑧 3
= 𝑥𝑥 3 + 3𝑥𝑥 2 𝑦𝑦 + 3𝑥𝑥𝑦𝑦 2 + 𝑦𝑦 3 + 3(𝑥𝑥 2 + 2𝑥𝑥𝑦𝑦 + 𝑦𝑦 2 )𝑧𝑧 + 3(𝑥𝑥 + 𝑦𝑦)𝑧𝑧 2 + 𝑧𝑧 3
= 𝑥𝑥 3 + 𝑦𝑦 3 + 𝑧𝑧 3 + 3(𝑥𝑥 2 𝑦𝑦 + 𝑥𝑥𝑦𝑦 2 + 𝑥𝑥 2 𝑧𝑧 + 𝑦𝑦 2 𝑧𝑧 + 𝑥𝑥𝑧𝑧 2 + 𝑦𝑦𝑧𝑧 2 ) + 6𝑥𝑥𝑦𝑦𝑦𝑦
Dan bila kita telusuri lebih teliti, ada terlihat hubungan antara penjumlahan linier pangkat 1 dengan kuadrat dan kubik, sehingga: (𝑥𝑥 + 𝑦𝑦 + 𝑧𝑧)(𝑥𝑥 2 + 𝑦𝑦 2 + 𝑧𝑧 2 ) =
= (𝑥𝑥 3 + 𝑦𝑦 3 + 𝑧𝑧 3 ) + (𝑥𝑥 2 𝑦𝑦 + 𝑥𝑥𝑦𝑦 2 + 𝑥𝑥 2 𝑧𝑧 + 𝑦𝑦 2 𝑧𝑧 + 𝑥𝑥𝑧𝑧 2 + 𝑦𝑦𝑧𝑧 2 )
Selanjutnya, dengan memisalkan bahwa:
𝐴𝐴 = (𝑥𝑥 2 𝑦𝑦 + 𝑥𝑥𝑦𝑦 2 + 𝑥𝑥 2 𝑧𝑧 + 𝑦𝑦 2 𝑧𝑧 + 𝑥𝑥𝑧𝑧 2 + 𝑦𝑦𝑧𝑧 2 )
Maka, dari bentuk persamaan kubik sebelumnya, terlihat hubungan antara penjumlahan 3 variabel linier, kuadrat dan kubik sebagai: (𝑥𝑥 + 𝑦𝑦 + 𝑧𝑧)3 − (𝑥𝑥 3 + 𝑦𝑦 3 + 𝐶𝐶 3 ) = 3A + 6xyz
Di mana,
𝐴𝐴 = (𝑥𝑥 + 𝑦𝑦 + 𝑧𝑧)(𝑥𝑥 2 + 𝑦𝑦 2 + 𝑧𝑧 2 ) − (𝑥𝑥 3 + 𝑦𝑦 3 + 𝑧𝑧 3 )
𝐴𝐴 = 𝑝𝑝1 𝑝𝑝2 − 𝑝𝑝3
𝐴𝐴 = (−2)(122) − (142)
𝐴𝐴 = −244 − 142 = −386
Sehingga nilai xyz dapat dicari dengan memasukkan nilai: 1 xyz = ((𝑥𝑥 + 𝑦𝑦 + 𝑧𝑧)3 − (𝑥𝑥 3 + 𝑦𝑦 3 + 𝑧𝑧 3 ) − 3(−386)) 6 1 1008 xyz = �(−8) − (142) + 1158� = 6 6 xyz = 168
Inilah kira-kira sedikit dari penggunaan bentuk identitas Newton itu, Kawan-kawan!”jelas Sabiq. | 88
“Nah, dengan penjelasanmu itu aku menjadi lebih paham maksud dari persoalan kedua yang dibacakan Ayra. Intinya, kita menemukan hubungan ketiga penjumlahan yang terdiri atas penjulahan 3 bilangan berpangkat 1, tiga bilangan berpangkat dua atau kuadrat, dan tiga bilangan berpangkat tiga atau kubik. Dan jembatan untuk menemukan hubungannya itu yang disebut dengan Newton’s Identities. Bukankah begitu, Biq!”tegas Amer. “Tepat sekali, Kawan. Semakin bertambah cerdas saja kawanku ini,”puji Sabiq. “Ha….ha….ha, semakin mantap profesi sebagai Dukunmu itu, Kawan,”tiba-tiba Ikram tertawa lepas. Keenam sahabat itu pun saling tertawa. “Ngomong-ngomong,
kepala
sudah
agak
terasa
berat,
dan..,”tiba-tiba Amer memecah tawa. “Aaallaah, paling kau mau bilang perutmu keroncongan, Mer!”kali ini Yodha berkomentar. “Betul kawan, ada bagian tubuhku yang meminta haknya,”kata Amer. “Gaya kau Boy! Bilang saja kau sudah kelaparan!”balas Yodha. “Ha....ha…ha….,”kembali terdengar tawa lepas keenam sahabat itu. “Baiklah,
kita
rehat
sebentar,
sambil
mengisi
perut,
Oke!”tawar Ayra. …∑πχαΩ…
| 89
Semangat individu dan kerja sama tim merupakan dua hal yang bila bertemu, bisa memecahkan batu karang sekalipun! Ya, dalam diri keenam sahabat itu dua hal itu sedang dan akan terus berproses. Semangat perubahan untuk menjadi lebih baik, telah bertemu dengan kebersamaan yang timbul karena kesadaran atas keterbatasan. Lewat Matematika, mereka berusaha mewujudkan mimpi. “Berikutnya, masih dari Kanada, tepatnya sebuah kompetisi rutin yang biasa digelar di Negara itu. Gauss Contest!”kata Ayra memulai session berikutnya. “Gauss Contest, Ayra?”tanya Fateh. “Betul, Kawan. Tampaknya panitianya mengirimkan catatan kepada Si Empunya buku ini,”jelas Ayra. “Ooo, begitu. Seperti apa persoalan kiriman itu, Ayra?”tanya Fateh lagi. “Begini. Ada 2 persoalan. Nomor 1 dikatakan, “the area of square ABCD below is 64 and AX = BW = CZ = DY = 2. What is the area of square WXYZ? (A) 58 (B) 52 (C) 48 (D) 40 (E) 36.
“Apakah disertakan ilustrasi gambarnya, Ayra?”tanya Fateh. “Ya, seperti terlukis itulah,”jawab Ayra. Selanjutnya Ayra membuka penjelasan atas persoalan itu. “Ada dua cara untuk menyelesaikan persoalan di atas. Cara pertama dengan memperhatikan bahwa luas bujur sangkar ABCD | 90
adalah 64, sehingga masing-masing sisi dari bujur sangkar itu tentulah 8. Karena AX = BW = CZ = DY = 2, tentulah AW = BZ = CY = DX = 6. Selanjutnya, masing-masing segitiga XAW, WBZ, ZCY dan YDX merupakan segitiga siku-siku dengan panjang masing-masing sisinya 2 dan 6. Karenanya, luas masing-masing segitiga kecil dari keempat segit tiga yang terbentuk adalah: = 1/2 (2) (6) = 6 satuan luas Selanjutnya, luas bujur sangkar WXYZ adalah sama dengan luas bujur sangkar ABCD dikurangi oleh luas keempat segi tiga kecil, atau sama dengan: = 64 − 4(6) satuan luas = 40 satuan luas,”terang Ayra. “Wow, keren boy!”celetuk Amer. Ayra kembali memerhatikan keterangan pada buku itu. “Cara berikutnya untuk menentukan luas bujur sangkar WXYZ adalah memanfaatkan perumusan Phytagoras!”terang Ayra. “Phytagoras?”tanya Amer. “Ya, begitu tertulis di sini. Perhatikan bahwa luas bujur sangkar ABCD adalah 64, dan panjang sisi bujur sangkar ABCD adalah 8. Sementara AX = BW = CZ = DY = 2, dan AW = BZ = CY = DX = 6. Dengan rumusan Phytagoras akan dapat diketahui bahwa panjang sisi XW = WZ = ZY = YX adalah = √22 + 62
= √40 satuan panjang
Selanjutnya, luas yang terbentuk dari bujur sangkar WXYZ adalah (√40) (√40) = 40 satuan luas,”kata Ayra. | 91
“Ya, betul-betul Phytagoras. Sisi miring keempat segitiga adalah juga sisi-sisi dari bujur sangkar yang ditanyakan,”komentar Yodha. “Ini persoalan Nomor 2. Dikatakan bahwa, ‘Kira can draw a connected path from M to N by drawing arrows along only the diagonals of the nine squares shown. One such possible path is shown. A path cannot pass through the interior of the same square twice. In total, how many different paths can she draw from M to N? (A) 5 (B) 6 (C) 7 (D) 8 (E) 9,”kata Ayra.
“Wah,
ini
tampaknya
menarik,
Kawan.
Bagaimana
Kram?”tanya Yodha pada Ikram. “Setuju, Kawan!”balas Ikram. “Penyelesaian terhadap persoalan ini dilakukan dengan menggambarkan tabel kemungkinannya. Pertama untuk menghindari terjadinya jalur yang terulang, dan kedua, agar lebih tampak perubahan atas jalur yang ditempuh dari asal ke tujuan,”penjelasan awal dari catatan itu. “Lanjutkan, Ayra!”pinta Sabiq. “Perhatikan penggambaran lintas yang telah dilukis ulang sebagai berikut:
| 92
Lintasan MADN seperti ditunjukkan anak panah pada gambar di bawah hanyalah 3 kali melalui diagonal. Karena lintasan yang simetris pada diagonal MN, semua lintas di satu sisi terefleksi pada bagian sebelahnya, sehingga merupakan satu lintasan berpasangan. Selanjutnya, bila dibuat tabel berpasangan sebagai berikut, di mana lintasan dari M ke N yang hanya melalui diagonal, adalah sebagai berikut: Pj. Lintasan
Nama Lintasan
Lintasan Refleksi
3
MADN
sama
5
MABCDN
MAFEDN
9
MABCDEFADN
MAFEDCBADN
MABCDAFEDN
MAFEDABCDN
MADCBAFEDN
MADEFABCDN,
Sehingga banyaknya lintasan berbeda yang dapat digambar dari M ke N adalah sebanyak 9 lintasan. Meskipun sejatinya bisa 11 lintasan. Coba tebak! Namun, karena pilihannya yang terbesar adalah 9 lintasan, maka jawabnya adalah E,”papar Ayra. “Wah, keren!”ujar Ikram bersemangat. “Masih ada 4 sumber lagi yang mengirimkan catatannya, Kawan-kawan! Sementara waktu mendekati Dzuhur. Bagaimana sebaiknya?”tanya Ayra pada kawan-kawannnya. “Sebaiknya kita skor untuk sholat Dzuhur. Nanti kita selesaikan catatan yang berasal dari empat tempat itu,”usul Sabiq. | 93
“Setuju!”ucap keenam sahabat itu serempak. Keenam sahabat itu sepakat bubar sementara. Mereka beranjak menuju Mushola terdekat untuk siap-siap menyongsong Dzuhur. …∑πχαΩ…
Jauh berjalan banyak dilihat, lama hidup banyak dirasa; begitu peribahasa Melayu mengajarkan tentang makna dari seseorang yang sudah
memiliki
banyak
berpengalaman.
Jauhnya
perjalanan
terhubung dengan banyaknya tempat, dan setiap tempat tentu memberi pemandangan yang berbeda. Begitu pun Matematika. Ketelitian, jumlah latihan, dan persoalan yang beragam dari berbagai tempat merupakan teladan yang baik meningkatkan pemahaman terhadap matematika itu sendiri. Dan hari ini, keenam sahabat itu sedang menikmati jauhnya perjalanan para pendahulu yang terhimpun dalam tumpukan catatan kiriman dari 8 penjuru angin. “Kali ini catatan kiriman persoalan dari London, Inggris kawan-kawan!”seru Ayra. “Wow, Negeri Sang Penyair, Shakespeare!”komentar Sabiq. “Ada 3 persoalan. Pertama, ‘What is the largest digit that appears in the answer to the calculation (3 × 37)2? Kedua, what is the sum of all fractions of the form (N/7), where N is a positive integer less than 7, dan ketiga, Find a fraction (m/n), with m not equal to n, such that all of the fractions: 𝑚𝑚 𝑚𝑚 + 1 𝑚𝑚 + 2 𝑚𝑚 + 3 𝑚𝑚 + 4 𝑚𝑚 + 5 , , , , , 𝑛𝑛 𝑛𝑛 + 1 𝑛𝑛 + 2 𝑛𝑛 + 3 𝑛𝑛 + 4 𝑛𝑛 + 5
can be simplified by cancelling,”baca Ayra bersemangat
| 94
Keenam sahabat itu serius. Masing-masing memerhatikan dengan sungguh-sungguh. “Jelas maksud dari masing-masing soalnya, Kawan?”tanya Ayra. “Ya, cukup jelas, Ayra,”jawab Yodha mewakili kawankawannya. Keempat sahabat lainnya menganggung tanda setuju dengan ucapan Yodha. “Pertama bila kita perhatikan bahwa: 3 × 37 = 111, dan (3 × 37)2
= 1112.
Sementara bentuk, 1112
= (111 x 111) = (1+10+100) x 111 = (1 × 111) + (10 × 111) + (100 × 111) = 111 + 1110 + 11100 = 12321.
Sehingga angka terbesar dari nilai 12321 adalah 3,”kata Ayra. “Selanjutnya, terhadap persoalan nomor 2 yang menanyakan jumlah dari semua pecahan dalam bentuk (N/7) untuk nilai N yang kurang dari 7 adalah sebagai berikut: = (1/7) + (2/7) + (3/7) + (4/7) + (5/7) + (6/7) = (21/7) = 3. Kedua persoalan di atas terkategori sederhana. Tidak rumit, hanya butuh sedikit trick dan ketelitian. Tapi, untuk persoalan nomor 3, dibutuhkan konsentrasi agak lebih dari biasanya,”ucapan Ayra membaca catatan itu. | 95
“Misalkan bahwa n > m dan dituliskan n = m + k, di mana k merupakan bilangan bulat. Maka keenam bentuk pecahan dapat dituliskan sebagai: 𝑚𝑚 𝑚𝑚 + 1 𝑚𝑚 + 2 𝑚𝑚 + 3 𝑚𝑚 + 4 𝑚𝑚 + 5 , , , , , 𝑚𝑚 + 𝑘𝑘 (𝑚𝑚 + 1) + 𝑘𝑘 (𝑚𝑚 + 2) + 𝑘𝑘 (𝑚𝑚 + 3) + 𝑘𝑘 (𝑚𝑚 + 4) + 𝑘𝑘 (𝑚𝑚 + 5) + 𝑘𝑘
Bentuk
pecahan
seperti
ini
secara
teoretis
dapat
saling
menghilangkan pada operasi suku-suku berikutnya di mana k adalah merupakan kelipatan dari masing-masing bilangan bulat itu. Selanjutnya nilai m akan bergerak sebagai: m, m + 1, m + 2, m + 3, m + 4, m + 5 Misalkan diambil untuk nilai m=2. Sehingga akan terdapat nilai dari bentuk umum kelipatan dari 2, 3, 4, 5, 6, 7. Nilai itu adalah pada k = 420. Selanjutnya, keenam pecahan yang terbentuk adalah: (2/422), (3/423), (4/424), (5/425), (6/426), (7/427). Karenanya, untuk nilai m=2 dan n = 422 adalah jawabannya! “Subhanallah!
Persoalan
yang
ketiga
membutuhkan
konsentrasi yang memang lebih, Kawan!”kata Sabiq memecah keseriusan kelima sahabatnya. “Ya, aku pun merasa seperti itu, Bro!”balas Fateh. “Aku juga, Kawan!”timpal Amer. Keenam sahabat itu tampak mengangguk-angguk. Anggukan kepuasan atas penjelasan catatan kiriman titipan Kakek Penjaga Musholla. “Berikutnya, tinggal 3 dari Korea Selatan, Selandia Baru dan Singapura, Kawan-kawan!”seru Ayra “Ya, mudah-mudahan bisa kita selesaikan semuanya siang ini, Kawan,”kata Ikram. | 96
“Selanjutnya, aku bacakan kiriman catatan dari Seoul, Korea Selatan. Begini persoalannya: Pertama, ‘A function f has the following properties. What is the value of f(243)? If f(3) = 1 and f(xy) = f(x) + f(y) for any real numbers x and y: (A) 15 (B) 12 (C) 10 (D) 5 (E) 1; Kedua, Evaluate 12 − 22 + 32 − 42 + · · · − 9982 + 9992. Ketiga, The graph of a function f(x) = (a/x2) passes through two points (8,2) and (b, 2b). Find the sum of a and b! (A) 4 (B) 32 (C) 68 (D) 128 (E) 132; dan Keempat tentang Geometri. Disebutkan, ‘the interior angles of a quadrilateral inscribed in a circle are α, β, σ and γ as shown in the figure. Which of the following is always true?’ (A) α+β+σ +γ = 1800
(B) γ+σ = 900
(D) β+σ = 900
(E) α+γ = β+σ,”kata Ayra.
(C) α+β = 900
“Apa pendapatmu terhadap kiriman catatan dari Negeri Ginseng ini, Kram?”tanya Sabiq serius. “Sederhana, lebih pada pemahaman konsep dasar tentang fungsi dan prinsip garis dan sudut pada lingkaran, Kawanku Sabiq,”jawab Ikram singkat. Ayra masih menunggu reaksi teman-temanya. “Apa ada yang mau jadi volunteer ?”tanya Ayra. “Maksudnya apa, Ayra?”balas Amer | 97
“Ya, aku pikir ini soal konsep dasar fungsi dan sedikit sudut pada lingkaran. Yodha atau Ikram? Bersedia?”tanya Ayra lagi. “Ikram saja, tampaknya dia lebih pas,”balas Yodha. “Kau didapuk oleh kawan-kawan, Bro!”bisik Sabiq. “Baiklah. Aku coba semampuku. Mudah-mudahan bisa sesuai harapan,”jawab Ikram sambil maju ke depan dan meminta kapur pada Ayra. “Persoalan pertama sebenarnya sederhana saja, sebatas kita bisa memanfaatkan sifat dan data yang tersedia. Disebutkan bahwa f(3) = 1 and f(xy) = f(x) + f(y) for any real numbers x and y. Itu kata kuncinya. Artinya fungsi yang memuat perkalian diubah menjadi bentuk penjumlahan fungsi. Sehingga persoalan f(243) dapat ditulis sebagai: f(243)
= f(3x81) = f(3) + f(81) = f(3) + f(3x27) = f(3) + f(3) + f(3x9) = f(3) + f(3) + f(3) + f(3x3) = f(3) +f (3) + f(3) + f(3) + f(3)
Karena nilai f(3) diketahui sebesar 1, maka: f(243) = (1) + (1) + (1) + (1) + (1) + (1) =5 Jawaban untuk soal nomor 1 adalah D,”terang Ikram tenang. Ikram menarik nafas agak panjang. Kelima kawannya tampak menikmati, serius tapi santai. “Untuk persoalan kedua, kita perlu sedikit trik!”papar Ikram. “Trik seperti apa maksudmu, Kawan?”tanya Amer. | 98
“Lihat saja. Penjumlahan 12 − 22 + 32 − 42 + · · · − 9982 + 9992, Bolehkan, kalau aku ubah menjadi: 12 + 32 − 22 +52− 42 +· · · +9992− 9982,”kata Ikram “Oke, tak ada masalah. Lantas?”desak Amer “Perhatikan baik-baik. kalau persamaan itu aku pisahkan dengan tanda kurung seperti ini: = (12) + (32 − 22) + (52− 42)+· · · +(9992− 9982) Dengan memanfaatkan sifat operasi bilangan berpangkat 2, kuadrat, di mana (a2 –b2) = (a–b) (a+b), maka; = 1+ [(3-2)(3+2)+(5-4)(5+4)+ …+(999-998)(999+998)] = 1+ (1)5 + (1)9 + (1)13 + …+ (1) 1997 Selanjutnya perhatikan bahwa suku yang baru membentuk barisan aritmatika 1, 5, 9, 13,…, 1997, dengan beda tetap 4. Sehingga jumlahnya menjadi: 𝑛𝑛 2
𝑆𝑆𝑛𝑛 = (𝑈𝑈1 + 𝑈𝑈𝑛𝑛 )
Di mana n adalah jumlah suku, yaitu 500; U1 suku pertama bernilai 1 dan Un sebagai suku ke-n besarnya 1997,”terang Ikram. “Tunggu dulu, Kawan!”tiba-tiba Amer memotong penjelasan Ikram. “Ada apa, Boy?”tanya Ikram. “Kenapa jumlah sukunya tinggal 500?”tanya Amer lagi. Terlihat senyum mengembang di wajah Ikram. “Perhatikan baik-baik, Kawanku Amer. Awalnya jumlah sukunya adalah 999. Dimulai dari 12 dan berakhir di 9992. Tetapi setelah kita pisahkan berdasarkan kedekatannya dan sifat operasi | 99
pangkat 2, kita lihat (32-22), (52-42), (72-62) menjadi satu suku baru yang membentuk deret itu. Sehingga suku yang baru banyaknya tinggal (999-1)/2 atau sama dengan 499 suku baru. Dan ingat dalam suku baru itu, angka 12 belum terhitung. Sehingga jumlah sukunya menjadi (499+1)=500 suku, Bro!”terang Ikram pada Amer. “Ooo…., begitu. Aku paham,”balas Ikram “Aku teruskan lagi keterangan yang terpotong tadi. Sehingga jumlah barisannya 1+5+913+…+1997 menjadi: Sn
= (500/2)(1+1997) = (250)(1998) = (250)(2000-2) = 500.000 –500 = 499.500,”terang Ikram.
“Plok, plok, plok! Hebat kau, Kawan!”puji Fateh sambil bertepuk tangan. “Terima kasih, Kawanku Fateh,”balas Ikram. “Bagaimana dengan persoalan ketiga, Kram?”kali ini Ayra mengajukan pertanyaan. “Persoalan nomor 3 masih terkait fungsi, kaitannya dengan garis lengkung. Disebutkan bahwa function f(x) = (a/x2) melalui dua titik yaitu pada titik (8,2) dan (b, 2b). Hal ini berarti bahwa bila nilai pada titik yang dilalui itu disubstitusikan ke dalam fungsi f(x) = (a/x2) akan terpenuhi: Pada titik (8,2)
↔ 2 = (a/82) ↔ 2 = a/64 ↔ a = 128 ……………..……..(9.3)
Pada titik (b,2b)
↔ 2b = [a/(b)2] | 100
↔ 2b = a/b2 ↔ a = (2b) b2 ↔ a = 2b3 …………………(9.4) Dengan memasukkan nilai a pada Persamaan (9.3) ke Persamaan (9.4), akan diperoleh persamaan: ↔ 128 = 2b3 ↔ b3 = 64 ↔ b3 = 43 Jadi, nilai
↔ b = 4 …………………… (9.5)
Sehingga jawaban atas pertanyaan the sum of a and b adalah jumlah antara Persamaan (9.3) dan Persamaan (9.5), yaitu: =a+b = 128 + 4 = 132 Dus, pilihan (E) adalah pilihan jawaban yang tepat,”terang Ikram. Tiba-tiba Ikram mengalihkan pandangannya ke Yodha. “Kawan, jujur, aku tak mau melangkahimu dalam soal Geometri. Kau adalah orang yang lebih tepat untuk menjelaskan persoalan nomor 4 ini, Kawan!”kata Ikram pada Yodha. “Tidak, kawanku. Kita semua dalam proses belajar. Aku yakin, kau pun akan mahir dalam soal geometri. Silakan diteruskan!”pinta Yodha. Memang, di antara keenam sahabat itu, mereka menaruh hormat atas pengetahuan Yodha dalam soal Geometri. Yodha seperti begitu menikmati dan indah bila menjelaskan persoalan terkait ilmu ukur ruang dan bidang itu. Tapi, kali ini adalah bagian Ikram. Dan | 101
Yodha meyakinkan kawannya bahwa ia juga mampu menjelaskan dengan baik kepada kawan-kawan lainnya. “Baiklah, aku mencoba menyelesaikan persoalan keempat. Ini soal yang disebut sebagai the opposite angle of a cyclic quadrilateral, Kawan-kawan,”terang Ikram memulai. “Salah satu prinsip dan menjadi sifat dalam the opposite angle of a cyclic quadrilateral bahwa sudut yang berseberangan atau berhadap-hadapan dalam sebuah segiempat yang berada dalam lingkaran
adalah
merupakan
supplementary
alias
saling
berpelurus,”terang Ikram. “Perhatikan ilustrasi berikut:
Sebut misalnya bahwa dalam lingkaran yang berpusat di O terbentuk segi empat sedemikian rupa yang keempat sudutnya (A, B, C, D) terletak pada bagian dalam lingkaran. Sehingga akan berlaku: “Sudut BOD adalah 2 (dua) kali besar sudut BCD” BOD = 2y Dengan cara yang sama, akan kita peroleh bahwa: “Sudut BOD adalah 2 (dua) kali besar sudut BAD” BOD = 2 x Sehingga, 2x + 2y = 3600 Karenanya, | 102
x + y = 1800 Dengan prinsip yang sama, maka pernyataan yang benar pada persoalan keempat adalah: (E) yaitu α+γ = β+σ. Sekian!”pungkas Ikram. “Bagaimana Kawan-kawan! Bukankah penjelasan Ikram soal Geometri juga mantap?”tiba-tiba Yodha berseru. “Manteb full, Bro!”balas Amer. “Selamat, Kawan, kau berhasil menjelaskan pada kami,”kata Yodha pada Ikram. “Terima kasih, Kawan, Aku sampai berkeringat untuk bagian geometri ini,”balas Ikram sambil menyeka keringat yang tampak mengalir di kening dan samping dekat telinganya. “Tapi, tampaknya kau terlalu serius sehingga lupa beberapa sifat dasar yang melekat pada lingkaran,”sela Yodha. “Sifat dasar? Dalil maksudmu, Kawan?’tanya Ikram. “Ya, dalil yang berhubungan dengan lingkaran,”tegas Yodha. “Jujur, aku lupa Kawanku,”balas Ikram. “Kau
harus
sempurnakan
dengan
ajukan
dalil
tentang
dalil
itu,
Yodha!”Fateh memotong. “Baik.
Dalil
itu
adalah,
Segiempat
Talibusur,”tandas Yodha. Semua mata tiba-tiba terpaku pada ucapan terakhir Yodha. “Dalil Segiempat Talibusur?”Ayra berkomentar. “Ya, Ayra. Segiempat Talibusur adalah segiempat yang keempat titik sudutnya terletak pada satu lingkaran,”terang Yodha lagi. | 103
Kelima sahabatnya masih menunggu sesuatu yang disebut dalil itu dari Yodha. “Dalilnya menyebut bahwa ‘pada segiempat talibusur, jumlah sudut sehadapan sama dengan 1800. Berlaku juga bahwa jika jumlah sudut sehadapan sama dengan 1800 maka segiempat tersebut merupakan segiempat talibusur,”tegas Yodha. Tampak kelima sahabat Yodha mengangguk-anggukkan kepala mereka perlahan. Sepertinya mereka paham apa yang dimaksud oleh Yodha dengan segiempat talibusur itu. “Selanjutnya, karena sudut-sudut α, γ, β, dan σ semua terletak di dalam lingkaran, atau seperti penggambaran Ikram tadi, bahwa titik-titik A,B, C, dan D semuanya terletak pada suatu lingkaran maka ABCD adalah segiempat talibusur. Karenanya pula berlaku akibat, bahwa: Sudut BAD + Sudut BCD = 1800 Sudut CDA + Sudut ABC = 1800, atau dengan kata lain berlaku hubungan (α+γ) = (β+σ), Kawan,”tegas Yodha. “Subhanallah! Memang tak salah gelar yang kami berikan padamu, Kawan Yodha!”puji Amer. Yang dipuji hanya tersenyum simpul saja. “Terima kasih sudah mengingatkan dan melengkapinya menjadi lebih sempurna, Kawan,”ujar Ikram pada Yodha. “Sama-sama, Bro. Kita sedang belajar bersama, kan! Take and give, saling beri dan terima. Tentunya saling melengkapi satu sama lain. Dan inilah kekuatan kita,”kata Yodha lagi. | 104
“Dan ternyata, kau bisa berkeringat dingin juga, Kram,”sindir Amer. “Ha…ha…ha,”terdengar tawa lepas dari keenam sahabat itu memecah siang yang semakin beranjak sore itu. …∑πχαΩ…
Kata orang pintar, yang sering menyebut diri mereka sebagai psikolog, seringkali rasa rendah diri pada seseorang jatuh pada perilaku Minderwaardigheids Complex alias Inferiority Complex alias MC. Dalam bentuk yang sederhana, misalnya, perilaku menyerah sebelum bertanding. MC adalah salah satu bentuk kelainan atau penyimpangan jiwa. Biasanya seseorang yang terlalu lama hidup dalam ‘keterkungkungan’ dan ‘keterbelakangan’', khususnya dalam berpikir, yang ‘tertekan’ oleh nilai-nilai sosial yang dianut masyarakat dimana mereka tinggal, terjangkit syndrome ini. Sebenarnya, MC tidak terlalu berpengaruh pada kadar kesadaran seseorang. Hanya saja jika menjangkiti seseorang, orang tersebut jadi sedikit atau bahkan banyak terhalang untuk melakukan sebuah proses sosialisasi atau interaksi secara baik. MC biasanya melanda orang yang merasa bahwa ada orang yang lebih baik dan tahu tentang sesuatu hal daripada dirinya. Padahal, bila ia berusaha untuk berbuat hal yang sama itu, bisa jadi akan menghasilkan yang jauh lebih baik dari orang yang dianggap super itu. Sekurang-kurangnya ia bisa menyamainya. Di sisi yang lain, pemilik kemampuan yang lebih itu, berpotensi menjadi pribadi yang sombong. Kemampuan mengelola sikap “rendah diri’ dan ‘kemampuan lebih’ ini menjadi cermin | 105
kedewasaan seseorang dan kematangan dalam pengendalian emosi diri. Bagi pemilik kemampuan lebih, bila mampu menahan untuk tidak show of force, bahkan memberi kesempatan orang lain untuk mencoba, akan melahirkan simpati publik, dus kematangan jiwa dan pribadinya pun bertambah. Dari sisi underdog, keberanian mencoba adalah jawaban atas sikap menyerah sebelum mencoba itu. Ikram merasa bahwa soal geometri Yodha adalah jagonya. Karenanya, ia sempat minder saat diminta mengerjakan soal-soal terkait geometri dan meminta Yodha untuk mengerjakannya. Tapi, Yodha, menunjukkan sikap yang patut diacungi jempol. Bukan hanya karena sudah menjadi tugas Ikram menyelesaikannya, ia mengembalikan permintaan kawannya itu dengan kalimat motivasi yang justru membangkitkan semangat! “Kita semua dalam proses belajar. Aku yakin, kau pun akan mahir dalam soal geometri. Silakan diteruskan!” begitu kalimat singkat itu. Dan akhirnya, Ikram bisa melewati batasannya. Batasan yang lahir dari sikap membatasi kemampuan dan rendah diri terhadap Geometri. “Selanjutnya dari mana, Ayra?”tanya Sabiq “Singapura, negeri tetangga kita Biq!”jawab Ayra. “Oke, silahkan dilanjutkan, Kawan!”pinta Sabiq. “Baik, Kawan. kiriman catatan dari Singapura itu mengatakan begini: ‘There is three-digit number A, Average of all possible arrangement of All the digits of A will still be the three-digit number A. How many three-digit numbers satisfy the given condition?”Ayra membaca dengan lancar dan jelas. “Apa pendapatmu, Biq?”tanya Fateh. | 106
Sabiq
tidak
langsung
menjawab.
Seperti
biasa,
dia
memandangi agak serius beberapa saat persoalan itu. “Bagaimana, Satrawan? Kali ini apa pendapatmu, Bro!”sergap Amer. “Ketelitian, kesabaran, dan open minded, kata kunci bagi penyelesaian soal ini,”jawab Sabiq datar. “Nah, kalau begitu, kau saja yang jelaskan!”pinta Amer. “Aku…?”tanya Sabiq heran. “Ya, siapa lagi! Kaulah, Bro!”balas Amer. “Oke. Sebelumnya, aku tanya Ayra. Apakah catatan itu memberi penyelesaian dan caranya?”tanya Sabiq pada Ayra. “Sebentar aku periksa dulu,”jawab Ayra. Tampak Ayra sibuk membuka beberapa lembar catatan dari Singapura itu. Dan… “Tampaknya, mereka hanya mengirimkan guidance berupa kunci jawabannya saja, Kawan!”jelas Ayra. “Nah, semakin tepat kau yang jelaskan jalan penyelesaiannya, Kawan,”kata Amer lagi. “Baiklah, aku coba,”Sabiq berkata sambil maju ke depan. “Mohon sama-sama kita perhatikan dan tolong di-cut saja bila ada langkah yang membingungkan atau tak tepat, Kawankawan!”pinta Sabiq. “Insyaallah,
Kawan,”jawab
Yodha
mewakili
kawan-
kawannya. “Misalkan angka A itu adalah A=abc, maka berdasaran informasi soal bahwa all possible arrangement of all the digits of A, | 107
maka: abc + acb + bac + bca + cab + cba. Dan kalau kita susun ulang akan dipenuhi persamaan sebagai berikut: abc + acb + bac + bca + cab + cba = 6abc Dan bila persamaan ini diuraikan berdasarkan posisi ratusan, puluhan dan satuannya, akan akan diperoleh: abc
= 100a+10b+c,
…………. (9.6)
acb
= 100a+10c+b,
…………. (9.7)
bac
= 100b+10a+c,
…………. (9.8)
bca
= 100b+10c+a,
…………. (9.9)
cab
= 100c+10a+b,
…………. (9.10)
cba
= 100c+10b+c,
…………. (9.11)
6abc
= 6(100a+10b+c)
…………. (9.12)
Bila ketujuh persamaan di atas disederhanakan akan diperoleh persamaan baru: 222 (a+b+c) = 6 (100a+10b+c) 37 (a+b+c) = 100a+10b+c (37-10) b+ (37-1) c = (100-37) a 63a = 27b+36c
……….…… (9.13)
Bila Persamaan (9.13) kedua sisi dibagi dengan 9, akan diperoleh persamaan: 7a = 3b+4c
…………… (9.14)
Persamaan (9.14) di atas, dengan sedikit trik dan konsep hutangbayar, pinjam kembalikan, dapat dimodifikasi menjadi: 7(a – b) = 4 (c – b)
…………… (9.15)
“Tunggu dulu, Biq! Kau gunakan kalimat itu lagi. Kau benarbenar berhutang janji untuk menjelaskannya, Kawan!”potong Amer. | 108
“Tentu Kawan, aku sudah berjanji. Insyaallah saatnya akan aku jelaskan. Tapi tidak dalam sesi ini. Oke?”balas Sabiq. “Tapi, biar kau tambah penasaran, ini sedikit trik itu. Coba kurangkan kedua sisi persamaan (9.14) dengan 7b, Mer!”pinta Sabiq, “Baik. Hal itu berarti bahwa: 7𝑎𝑎 − 7𝑏𝑏 = 3𝑏𝑏 + 4𝑐𝑐 − 7𝑏𝑏
7𝑎𝑎 − 7𝑏𝑏 = 4𝑐𝑐 − 4𝑏𝑏
7(𝑎𝑎 − 𝑏𝑏) = 4(𝑐𝑐 − 𝑏𝑏)
Wah, sama dengan persamaan (9.15), Biq. Apa artinya ini?”tanya Amer penasaran. “Itu contoh prinsip bayar-hutang, kembalikan-pinjaman, Kawan!” “Baik, kalau begitu aku tunggu kelengkapannya. Lanjutkan pembahasannya, Sastrawan!”pinta Amer. “Baiklah. Selanjutnya, dengan persamaan (9.15) di atas, akan terdapat beberapa kemungkinan. Karena bila hanya mengandalkan Persamaan (9.14), untuk nilai a=b=c, semua nilai akan terpenuhi. Karenanya, Persamaan (9.15) adalah alternatifnya. Dengan asumsi bahwa a = b = c, misalnya, maka: a – b = 4;
c–b=7
b – a = 4;
b–c=7
⇔0≤b≤2
⇔7≤b≤9
…… (9.16) …… (9.17)
Sehingga, jumlah kemungkinan dari variasi tiga angka yang memenuhi bentuk di atas adalah sebanyak 15, yaitu: Dari kondisi 0 ≤ b ≤ 2 untuk
b=0
untuk
b=1
⇔ a = 4 dan c = 7 ⇔a = 5 dan c = 8
A = 407; A = 518; | 109
b=2
⇔a = 6 dan c = 9
A = 629;
untuk
b=7
A = 370;
untuk
b=8
⇔ a = 3 dan c = 0
untuk
b=9
⇔ a = 5 dan c = 2
A = 592;
untuk Dari kondisi 7 ≤ b ≤ 9
⇔ a = 4 dan c = 1
A = 481;
Lainnya diperoleh dari Persamaan (9.15), dengan substitusi nilai a, b dan c, sehingga dipenuhi nilai A sebagai: 111, 222, 333, 444, 555, 666, 777, 888, 999 Untuk itu, jawaban atas ‘How many three-digit numbers satisfy the given condition?’ adalah fifteen alias 15!”papar Sabiq. “Terima kasih, Kawanku Sabiq yang Satrawan. Betul-betul butuh ketelitian, kesabaran, dan open minded seperti yang kau bilang itu, Kawan,”komentar Amer. Tampak keenam sahabat itu begitu puas. Wajah-wajah mereka seperti tersiram air sejuk setelah seharian terpapar terik sinar matahari. “Selanjutnya, dari Selandia Baru. Kiriman catatan terakhir, Kawan-kawan!”tiba-tiba Ayra berkata. “Ada berapa persoalan, Ayra?”tanya Ikram. “Hanya satu. Dan nampaknya sederhana. Tapi tunggu dulu, biar aku bacakan: ‘Find all integer solutions of the equation: xy + 3x - 5y = – 3. Selesai!”ucap Ayra. “Begitu saja, Ayra?”tanya Amer penasaran. “Ya, begitu saja, Mer,”jawab Ayra pendek. “Ada jalan pemecahan dan jawabannya, Kawan?”kali ini Fateh ikut bertanya. | 110
“Tidak.
Hanya
disebutkan
terdapat
12
pasangan
bilangan,”jawab Ayra lagi. “So…?”Ikram ikut bingung. Tampak ekspresi wajah Sabiq berpikir keras. Suasana sepi. Amer
gelisah.
Ayra
masih
memerhatikan
catatan
kiriman
mamastikan akan ada tambahan keterangan. “Tampaknya kali ini profesimu sebagai Dukun akan diuji lagi, Biq!”tiba-tiba Ayra berkata lembut. Tidak seperti biasanya. Semua mata tertuju keada Ayra, dan sesaat kemudian ke arah Sabiq. Tampaknya mereka sepakat dengan pendapat Ayra. “Aku tahu, kalian pasti berpikir ada yang harus pakai trik, kan?”tanya Sabiq. Kelima kawannya itu mengangguk setuju. “Tidak sepenuhnya keliru. Memang kita butuh sedikit trik dalam persoalan ini. Tapi bukan lagi trik hutang-bayar, pinjam kembalkan. Trik susun-kembali menjadi sederhana. Baiklah, aku coba,”kata Sabiq mantap. “Tunggu dulu, kau bilang bukan lagi trik hutang-bayar, pinjam kembalkan, berarti trik itu juga bisa digunakan, kan?”tanya Ayra penuh selidik. “Ya, bisa, hanya saja kita akan lihat cara lain, cara yang lebih sederhana dan umum bagi sebuah persamaan. Bila misalnya kita nyatakan y sebagai fungsi x, maka persamaan xy + 3x - 5y = – 3, dapat ditulis dan disusun ulang menjadi: xy - 5y = – 3 – 3x, y(x-5) = – 3 – 3x, | 111
−3 − 3𝑥𝑥 15 − 3𝑥𝑥 − 18 18 = = −3 − 𝑥𝑥 − 5 𝑥𝑥 − 5 𝑥𝑥 − 5 Sehingga, akan terlihat bahwa (x – 5) merupakan pembagi atas 18. 𝑦𝑦 =
Dengan kata lain merupakan faktor yang habis membagi 18. Dan faktor itu adalah ±1; ±2; ±3; ±6; ±9; ±18, Kawan-kawan,”papar
Sabiq serius.
Tampak bahwa kelima sahabat yang lain begitu antusias dan serius mengamati setiap coretan dan keterangan Sabiq. “Selanjutnya, bila dibuat tabulasi nilai persamaan berdasarkan masing-masing faktor 18 di atas, dan akan diperoleh 12 kemungkinan penyelesaian seperti terlihat pada tabel di bawah ini: 𝑦𝑦 = −3 −
18 𝑥𝑥 − 5
Maka tabel yang bisa dihasilkan adalah: (x – 5)
-1
1
-2
2
-3
3
-6
6
-9
9
-18
18
x
4
6
3
7
2
8
-1
11
-4
14
13
23
y
15
-21
6
-12
3
-9
0
-6
-1
-5
-2
-4
Sehingga, jawaban atas all integer solutions adalah: (x,y) = (4, 15), (6, -21), (3, 6), (7, -12), (2, 3), (8, -9), (-1, 0), (11, -6), (-4, -1), (14, 5), (13, -2), (23, -4). Itu saja yang bisa aku pikirkan dan selesaikan, Kawan-kawan!”pungkas Sabiq “Alhamdulillah! Akhirnya selesai. Dan penutupmu itu manis sekali, Sastrawan yang Dukun. Eh… Dukun yang Sastrawan!”puji sekaligus sindir Amer. “Tapi, Biq. Andai kau gunakan hutang-bayar, pinjam kembalkan, Bagaimana?”kembali Ayra penasaran. “Baiklah, aku singkat saja, kita akan bicarakan ini lebih serius,”Sabiq merespons dan terus menuju alat tulis. | 112
“Kawan-kawan, mohon perhatikan langkah penyusunan persamaan di bawah ini: 𝑥𝑥𝑦𝑦 + 3𝑥𝑥 − 5𝑦𝑦 = −3
𝑥𝑥𝑦𝑦 + 3𝑥𝑥 − 5𝑦𝑦 − 15 = −18
𝑥𝑥(𝑦𝑦 + 3) − 5(𝑦𝑦 + 3) = −18
(𝑦𝑦 + 3)(𝑥𝑥 − 5) = −18 … … … … . … … (9.18)
Persamaan (9.18) ini berarti bahwa kita diminta menemukan pasangan perkalian (y+3) dengan (x-3) yang hasilnya adalah – 18. Itu artinya adalah faktor-faktor dari – 18 itu sendiri! Yaitu: ±1, ±2, ±3, ±6, ±9, ±18. Hasil ini sama dengan tabel sebelumnya!”terang Sabiq “Dasar kau, Biq! Kau berhutang penjelasan!”ujar Amer “Ha….ha..ha…,”terdengar tawa lepas enam sahabat itu. Enam sahabat yang selesai membaca catatan kiriman dari 8 penjuru angin. Tanpa mereka sadari, beberapa ratus meter dari tempat mereka berdiskusi, seorang kakek dengan janggut yang agak memutih, cermat dan awas mengamati keenam sahabat itu. “Subhanallah! Kalian anak-anak berbakat! Andai kalian tahu bahwa apa yang kalian diskusikan itu adalah contoh dari beberapa kompetisi
dan
olimpiade
matematika
tingkat
nasional
dan
internasional di beberapa Negara! Mudah-mudahan kalian bisa mendapat manfaat dan tidak berhenti sampai disana,”terdengar ucapan pelan namun jelas dari mulut Kakek berjanggut putih itu. Dari kejauhan, tampak keenam sahabat itu pun sepertinya sepakat untuk mengakhiri diskusi mereka hari itu. Mereka berpisah dengan
wajah
berseri.
Terpancar
kepuasan
atas
tambahan
pengetahuan yang mereka peroleh hari ini. …∑πχαΩ… | 113
10
Surat Misterius RRHS “Sabiq,
Ayra,
Fateh,
Yodha,
Amer,
ada….ada…ada…,.ada….,”terdengar suara Ikram terputus-putus memecah kesunyian pagi itu di sekolah. “Sabar, Bro! Tenang, Tenang! Pelan-pelan saja. Ada apa, Kawan?”tanya Fateh pada Ikram yang mendekat sambil berlari-lari kecil. Ikram tampak menarik nafas panjang. Terlihat kucuran keringat di beberapa bagian wajahnya. Meski hari masih terlalu pagi untuk disebut hangat. Mentari belum sepenuhnya menghangatkan bumi. Pagi itu, hari sekolah seperti biasa. Dan kebiasan enam sahabat ini untuk hadir lebih cepat dibandingkan siswa lain di sekolah mereka. Selalu saja ada yang mereka bicarakan. Apatah lagi setelah mereka sepakat membentuk Genk OSN. Intensitas pertemuan mereka terasa semakin padat. “Setelah kau tenangkan diri, dan nafasmu kembali teratur, ceritakan apa persoalannya, Kawan!”ujar Fateh lagi. “Begini, Kawan-kawan. Kemarin, ada surat yang diantar ke rumahku. Tapi, tak jelas siapa yang mengantar. Anehnya, tertulis, | 114
‘Kepada Ikram dan Lima Sahabatnya: Ayra, Fateh, Amer, Yodha dan Sabiq di suatu tempat’,”terang Ikram. “Ah, yang benar kau, Bro! Mana pulak ada surat dengan tujuan alamat seperti itu!”bantah Amer. “Itu
dia
persoalannya.
Seperti
itu
tertulis.
Dan
ini
suratnya!”jawab Ikram. “Tunggu dulu! Maksudmu, surat ini ditujukan untuk kita berenam dari seseorang yang tak jelas siapa yang mengantarnya, dan siapa pengirimnya?”tanya Fateh selidik. “Ya, benar kawan. Karenanya, aku segera memberi tahu kalian tentang surat ini,”jawab Ikram. Ayra tampak sedikit memicingkan kedua matanya, Amer melotot sambil membuka sedikit mulutnya, Yodha lebih tenang, tapi tak bisa menyimpan rasa penasaran di wajahnya. Fateh, dia tampak serius menanggapi, sementara Sabiq hanya diam dan tampaknya berpikir keras. Entah apa yang dipikirkannya. “Jadi bagaimana? Kita apakan surat ini, Kawan?”tanya Ikram. Fateh memandang Ayra, Ayra memandang Yodha, Yodha melirik ke arah Amer. Amer masih melotot, mulutnya sedikit terbuka, heran. “Bagaimana menurutmu, Biq?”tanya Fateh. “Ya, namanya surat, ditujukan kepada kita, tak penting siapa yang antar dan belum jelas pengirimnya, ya kita buka saja! Mungkin ada petunjuk lebih jelas di dalamnya,”jawab Sabiq enteng. “Begitu saja?”tanya Amer.
| 115
“Ya, begitu saja. Kenapa tidak kita buka saja surat ini agar lebih jelas apa maksud Si Pengirim, Kawan-kawan?”tawar Sabiq “Baiklah, kalau begitu bagian Ayra yang membuka dan membacakannya kepada kita semua,”lanjut Fateh tak sabar. Ikram memberikan surat itu kepada Ayra. Sigap, Ayra menerima dan langsung membuka segel dalam amplop besar berwarna coklat itu. “Bacalah Ayra! Mungkin ada petunjuk di dalamnya,”pinta Sabiq. “Baik, aku akan membaca apa adanya isi surat ini,”kata Ayra
“As-Salaamu’alaikum W.W Semoga Allah senantiasa memberikan kesabaran kepada kita dalam menjalankan aktivitas sehari-hari beribadah kepadaNya. Sebelumnya saya mohon maaf, telah membuat adik-adik bingung dengan datangnya surat ini. Langsung saja! Saya mendengar dari seorang teman, bahwa di suatu tempat di dekat Masjid al-Attiqi, terdapat 6 orang sahabat yang gemar dan sedang mendalami Matematika serta berharap bisa ikut dalam event-event olimpiade baik nasional maupun internasional. Niat itu adalah suatu niat yang mulia semoga bisa tercapai. Sebagai seorang yang pernah berkecimpung dalam perlombaan matematika seperti itu, perkenankan saya ikut berpartisipasi. Mudah-mudahan partisipasi itu bisa membantu adik-adik mencapai niat mulia itu. Untuk itu beberapa hal perlu saya sampaikan dan mudah-mudahan menjadi perhatian bersama. | 116
Pertama, semua yang adik-adik niatkan dan impikan harus dilandasi dengan ke-IKHLAS-an. Ikhlas adalah melakukan suatu perbuatan sesuai dengan aturan Allah dan Rasul-Nya serta meniatkan perbuatan tersebut semata-mata mencari atau memperoleh keridhoan Allah. Kedua, manusia ibarat dua sisi dari satu mata uang, pastilah mempunyai sisi baik dan sisi buruk. Senantiasalah menjaga prasangka baik atas segala sesuatu yang telah, sedang, dan akan dihadapi dengan tetap mawas diri dan berhati-hati. Ketiga, bahwa berbuat baik secara berjamaah akan jauh lebih baik daripada berbuat baik secara sendiri-sendiri, apalagi untuk mewujudkan suatu cita-cita yang besar. Dan kelompok matematika yang adik-adik buat ini langkah awal yang baik. Keempat, berharaplah agar sukses di dunia dan akhirat! Untuk itu, “tiga jangan” yang harus dilakukan yaitu (1) jangan sombong, (2) jangan percaya diri tapi percaya Allah (3) jangan punya keinginan. Ini mungkin terasa aneh. Tapi, percayalah, pengalaman telah mengajarkan pada saya tentang “tiga jangan” itu. Kelima, gunakan hati untuk menyelesaikan masalah, hati yang senantiasa terpaut kepada Allah! Selama ini kita mengandalkan otak untuk menyelesaikan segala macam masalah, ketika menghadapi masalah yang sangat sulit otak menjadi stress, karena masalah yang ada di luar kapasitas kemampuan otak. Tetapi, jika menyelesaikan masalah dengan menggunakan hati yang terpaut kepada Allah, maka masalah itu Allah yang akan menyelesaikan. Keenam, percayalah bahwa hati yang selalu terpaut kepada Allah, mengingat dan memuji Allah, lebih dahsyat kemampuannya daripada kemampuan otak yang sangat cerdas. Oleh karena itu, latihlah hati untuk selalu bersih dari berbagai penyakit hati agar ia bisa berperan maksimal. Akhirnya, setelah belajar dengan giat dan serius. Pada saat kompetisi matematika berlangsung, teruslah banyak mengingat dan memuji Allah, karena dalam kompetisi tersebut akan banyak soal yang sulit dipecahkan. Semoga, dengan hati yang selalu terpaut kepada Allah, maka Ia akan membantu memecahkan soal yang dihadapi. | 117
Sebagai bentuk partisipasi saya kepada adik-adik, terlampir 9 persoalan matematika, yang insyallah bermanfaat dan akan memperkaya bahan- untuk persiapan ke depan. Juga diselipkan jawaban atas persoalan itu. Harapan saya akan memacu adik-adik untuk menemukan jalan penyelesaiannya” Wassalam, RRHS Berikut adalah 9 soal yang sebaiknya adik-adik selesaikan bersama-sama. Semoga bermanfaat! Soal 1. 10 orang siswa telah berhasil memecahkan total 35 soal. Diantara mereka ada siswa yang hanya memecahkan 1 soal, ada siswa yang hanya memecahkan 2 soal dan ada yang hanya 3 soal. Buktikan bahwa paling sedikit ada hanya 1 orang bisa memecahkan 5 soal. Soal 2. What is the units digit for the sum: 13 - 23 + 33 - 43 + 53 - 63 + ... +193 ? [0] Soal 3. Suppose x, y, and z are positive integers such that x > y > z > 663 and x, y, and z satisfy the following: x + y + z = 1998; 2x + 3y + 4z = 5992. Find the values of x, y, and z. [x = 667, y = 666, z = 665] Soal 4. Carilah seluruh nilai X dan Y yang mungkin apabila bilangan enam digit 123X4Y habis dibagi oleh 4 dan 9. [(X,Y) = (8,0); (4,4); (0,8), dan (9,8)] Soal 5. Let a, b, c be positive integers such that: ab + bc = 518, and ab – ac = 360. Find the largest possible value of the product abc. [1008] | 118
Soal 6. Misalkan a, b, c adalah bilangan real di mana a + b + c = 20 dan a2+b2+c2= 318. Tentukan nilai ab + bc + ac! [41] Soal 7. Terdapat 10 buah topi. Setiap topi berbeda bahan dan harga. Dua topi terbuat dari bahan katun (Rp. 30.000,-/topi), lima topi terbuat dari bahan kulit (Rp. 50.000,-/topi), dan tiga topi terbuat dari bahan wol (Rp. 10.000,-/topi). Berapa banyak cara untuk membeli 5 buah topi (dengan memperhatikan perbedaan bahan topi) di mana total biayanya yang harus dikeluarkan lebih dari Rp. 101.000,- tetapi kurang dari Rp. 149.000,-? [35] Soal 8. Palindrome adalah bilangan bulat positif dimana angkaangkanya sama ketika dibaca dari depan atau dari belakang. Contoh 4554 adalah palindrome 4-angka dan 73237 adalah palindrome 5-angka. Berapa pasangan palindrome 4-angka yang jika dijumlahkan menghasilkan palindrome 5-angka? [Contoh satu pasangan adalah 2882 dengan 9339 menghasilkan 12221]. [36] Soal 9. Jika P = 32008 + 3-2008 dan Q = 32008 – 3-2008. Maka nilai P2 – Q2 = … [4] “Subhanallah! Siapa orang ini?”tanya Sabiq tercengang. “Dia
seperti
tahu
yang
sedang
kita
perlukan,
Kawan,”komentar Ikram. “Jangan-jangan,…
dia
adalah,…”Ayra
menghentikan
kalimatnya. “Dia adalah siapa Ayra?”tanya Ikram penasaran. Yodha masih tampak berpikir keras. Ia seperti memandang jauh ke depan. | 119
“Kau ingin mengatakan bahwa jangan-jangan orang ini adalah sahabat Kakek Penjaga Musholla itu, Ayra?”potong Yodha. Ayra mengangguk. “Atau jangan-jangan orang ini adalah Kakek Penjaga Musholla itu sendiri, Kawan!”komentar Sabiq yang membuat kelima sahabatnya itu terperanjat. “Sudahlah itu semua tak penting. Yang penting bahwa kita mendapat kiriman soal matematika dari orang yang tak dikenal. Siapapun orang itu, kita berterima kasih padanya. Dan kita diminta menyelesaikannya bersama, Kawan-kawan,”kata Ayra. “Setuju. Hal itu jauh lebih penting,”tegas Yodha. Tiba-tiba lonceng sekolah berbunyi tanda bahwa jam sekolah dimulai. Tampak para siswa sibuk berlarian masuk ke ruang kelas masing-masing. Keenam sahabat itu pun bersiap-siap masuk kelas. “Baik, kawan-kawan. nanti soal ini kita diskusikan setelah jam pelajaran sekolah usai. Kita bertemu di Musholla al-Attiqi. Oke!”usul Ayra. “Baik. sepakat,”serempak keenam sahabat itu mengacungkan jempol ke udara. Tak selang beberapa menit, halaman sekolah yang tadi riuh rendah dengan suara dan hilir mudik para siswa, tampak lengang. Suasana pagi itu kembali sunyi seperti semula. Hanya kicauan Prenjak dan sesekali Kutilang terdengar bersahutan di antara pohon Mahoni tua dan Mangga yang ada di sekitar sekolah. …∑πχαΩ…
| 120
Hari itu proses belajar mengajar berjalan seperti bisa, lancar, dan tidak ada sesuatu yang istimewa. Guru mengajar dan siswa mendengarkan. Tapi tidak bagi keenam sahabat yang terpisah dalam dua kelas itu. Ayra, Ikram, dan Amer yang berada satu kelas terlihat beberapa kali saling pandang. Mereka tampak gelisah. Begitupun Fateh, Yodha, dan Sabiq di kelas lainnya. Ada perasaan yang berkecamuk untuk segera bisa bertemu. “Teng, Teng, Teng, Teng, Teng,”terdengar suara lonceng sekolah berbunyi, tanda bahwa jam pelajaran terakhir sudah selesai. Terdengar suara riuh dari seluruh ruangan kelas. Murid-murid tampaknya seakan sepakat bahwa tanda bunyi lonceng itu seautu yang sangat dinantikan. Begitu pun terpancar kegembiraan dari wajah keenam sahabat. Seperti dikomando, keenam sahabat meluncur ke satu tempat yang telah mereka sepakati menjadi Markas Besar Genk OSN. Terlihat Amer dan Yodha berlari begitu bersemangat. Ayra dan Fateh menyusul, sementara Ikram memilih berjalan santai. Tapi, tak terlihat di mana Sabiq. “As-salaamualaikum,
Kawan-kawan!”sapa
Amer
kepada
keempat sahabatnya. “Wa alaikum salam, Mer!’balas mereka serentak. “Lho, Sabiq di mana? Kok belum muncul?”tanya Amer. “Kami pun tak tahu. Tadi kami sama-sama keluar. Aku dan Fateh langsung ke sini, sementara Sabiq kulihat menuju ke ruang guru,”kata Yodha. | 121
“Ke ruang guru? Ada apa, ya?”tanya Amer. “Ya, kita tunggu saja. Nanti kalau sudah tiba kita tanya Sabiq ada apa?”Ayra memotong. Tak berapa lama, Sabiq muncul di markas, membawa secarik kertas di tangannya. “As-salaamualaikum!”Sabiq mengucapkan salam. “Wa alaikum salam!”terdengar jawaban serentak dari kawankawanya. “Waduh, mohon maafkan aku, Kawan-kawan. Tadi aku dipanggil oleh Pak Tumanggor, Guru Matematika kita, untuk menghadap. Ada hal yang ingin beliau sampaikan katanya,”jelas Sabiq. “Tak apa-apa, Kawan. Santai saja, Bro!”jawab Ikram. “Apa yang disampaikan oleh Guru Matematika tadi, Biq?”tanya Yodha. “Pak Guru tanya tentang perkembangan Genk OSN kita. Aku jawab saja ‘alhamdulillah, lancar, Pak!’ Dan beliau memberitahukan bahwa sekitar 3 bulan lagi, seleksi tingkat Kabupaten akan digelar untuk OSN Matematika, Kawan-kawan!”terang Sabiq. “Wow, akhirnya datang juga waktu itu,”komentar Amer. “Pertanyaannya yang terpenting adalah, ‘bagaimana persiapan sekolah kita? Lebih tepatnya persiapan kita, Kawan!”Ayra menimpali. “Apa hanya itu yang disampaikan Pak Guru, Biq?”tanya Yodha penasaran.
| 122
Tampak ragu menjawab. Sabiq menggerak-gerakkan jari telunjuk kanannya, seperti ada sesuatu yang mengganggu pikirannya untuk disampaikan. “Biq, rasanya ada sesuatu yang lain?”kali ini Ikram menajamkan. “Memang, ada kawan-kawan,”jawab Sabiq pelan. Kelima sahabatnya sedang menanti kelanjutan keterangan Sabiq. Mereka menunggu. Sampai akhirnya, Sabiq menarik nafas panjang sekali, dan melepaskannya. “Pak Guru minta kepada kita agar Genk OSN ini diperbesar jumlahnya. Konkretnya siswa-siswa kelas khusus diharapkan bisa bergabung,”papar Sabiq pelan. “Bah, bagus kali itu. Itu baru berita, Kawan!”seru Amer dengan suara yang khas itu. “Apa maksudmu, Biq. Kita diminta menambah pasukan ini? Itu berita yang sangat enak didengar kawan,”timpal Ayra. “Jadi,….kalian… setuju!”tanya Sabiq heran. “Kenapa tidak “Alhamdulillah, justru aku khawatir kalian keberatan. Itu yang menjadi beban pikiranku, Kawan-kawan,”ujar Sabiq menumpahkan kekhawatirannya. “Oke, soal ini nanti kita bahas lain waktu. Kita kembali ke pokok persoalan tentang persiapan kita sendiri,”Ayra mengalihkan ke fokus. “Ya,
pertanyaan
tadi,
pertanyaan
tentang
‘bagaimana
persiapan kita’, adalah mendasar. Karenanya, perbincangan tadi pagi | 123
harus
kita
tuntaskan
sekarang.
Bagaimana
menurutmu,
Yodha?”tanya Sabiq. “Menurutku bahwa waktu belajar kita semakin sempit. Untuk itu semakin banyak dan sering kita berlatih, akan semakin baik bagi persiapan kita, Kawan! Dan aku usul agar 9 persoalan yang dikirimkan oleh RRHS segera kita diskusikan,”usul Yodha. “Setuju! Aku usul sebaiknya malam ini kita diskusi di…,”Ayra menghentikan kalimatnya. “Di rumahku saja!”tiba-tiba Yodha menawarkan tempat. “Setuju. Jam berapa kita mulai?”tanya Fateh. “Kita sholat Isya berjamaah di Mushola dekat rumah Yodha. Bagaimana, Kawan?”usul Ayra. “Setuju. Insyaallah malam ini kita bertemu di rumah Yodha, dan sebelumnya sholat Isya berjamaah di mushola terdekat. “Insyaallah!”terdengar ucapan serentak keenam sahabat itu. …∑πχαΩ…
“Ini bagaimana menyelesaikannya, Biq?”tanya Ayra. “Ooh, ini yang dikenal sebagai Pigeon Hole Principle, Kawan,”terang Sabiq “Pigeon Hole Principle, apa pula itu Kawan?’tanya Amer lagi. “Kira-kira begini, Kawan. Prinsipnya menyebutkan bahwa ‘if k+1 letters are posted into k pigeon-holes, then at least two letters will share the same hole’”terang Sabiq. “Sederhananya, bagaimana Kawan?”tanya Amer lagi.
| 124
“Hmm…hmm. Bagaimana, ya? Begini deh contohnya. diantara 3 orang manusia normal paling sedikit terdapat 2 orang yang berjenis kelamin sama. Atau dalam satu kelompok yang terdiri atas 13 orang, paling sedikit 2 orang lahir pada bulan yang sama,”terang Sabiq lagi. “Nah, kalau ini aku sudah paham. Boleh juga kan aku sebutkan bahwa salah satu terapan dari Pigeon Hole Principle itu, bahwa ‘dari 367 siswa di sekolah kita paling sedikit tedapat 2 orang yang punya hari ulang tahun yang sama dalam setahun’,”komentar Amer. “Cerdas, kau kawan!”puji Sabiq pada Amer. “Karenanya, kalau kita perhatikan soal nomor 1 kiriman RRHS, maka dari 10 siswa, 3 orang yang disebutkan pertama telah mampu menyelesaikan persoalan sebanyak: = (1+2+3) = 6 soal sudah diselesaikan Sehingga, soal yang tersisa untuk dikerjakan oleh 7 siswa lainnya: = (35 – 6) = 29 soal Misalkan, tidak ada siswa yang mengerjakan 5 soal, dan ke-7 siswa mengerjakan 4 soal, maka jumlah sisa soal yang dikerjakan: = 7 siswa x 4 soal/siswa = 28 siswa Karenanya, dari 29 soal yang tersisa terdapat = (29 – 28) soal = 1 soal, yang tersisa.
| 125
Karena ternyata 10 orang siswa telah berhasil memecahkan total 35 soal, dengan terapan dari Pigeon Hole Principle dapat dipastikan bahwa sekurang-kurangnya (minimal) 1 orang siswa mengerjakan 5 soal,”papar Sabiq kepada teman-temannya. “Oooh, begitu maksudnya,”komentar Amer. Dan, kawan-kawan yang lain sibuk mencatat beberapa hal tentang prinsip lubang merpati ini. Sabiq bersiap untuk melanjutkan. “Selanjutnya, untuk yang nomor 2, siapa?”tanya Sabiq “Aku pikir Ikram adalah orang yang tepat,”jawab Ayra. “Silahkan, Kawan!”kata Sabiq Ikram maju ke depan, tenang dia melangkah. Ditatapnya Kawan-kawannya. Dan mulalah dia beraksi. “Soal ini hanya butuh kesabaran dan ketenangan, Kawan. Disebutkan what is the units digit for the sum: 13-23+33-43+5363+…+193?”kata Ikram memulai aksinya. Kawan-kawannya terlihat khusuk. “Untuk menentukan angka satuan dari penjumlahan pada persoalan di atas, dapat ditelusuri dengan menguraikan satu per satu dari angka yang dipangkatkan 3, yaitu: 03 angka satuannya
= 0x0x0
=0
1 angka satuannya
= 1x1x1
=1
23 angka satuannya
= 2x2x2
=8
33 angka satuannya
3
= 3x3x3
= 27
3
= 4x4x4
= 64
3
5 angka satuannya
= 5x5x5
= 125
63 angka satuannya
= 6x6x6
= 216
4 angka satuannya
| 126
73 angka satuannya
= 7x7x7
= 343
8 angka satuannya
= 8x8x8
= 512
93 angka satuannya
= 9x9x9
= 729
103 angka satuannya
3
= 10x10x10
= 1000
3
= 11x11x11
= 1331
3
12 angka satuannya
= 12x12x12
= 1728
133 angka satuannya
= 13x13x13
= 2197
143 angka satuannya
11 angka satuannya
= 14x14x14
= 2744
3
= 15x15x15
= 3375
3
16 angka satuannya
= 16x16x16
= 4096
173 angka satuannya
= 17x17x17
= 4913
183 angka satuannya
= 18x18x18
= 5832
= 19x19x19
= 6859
15 angka satuannya
3
19 angka satuannya
Terlihat ada pola yang berulang untuk angka satuan dari 0,...,9. Karenanya, untuk langkah selanjutnya, kita dapat membuat tabel satuan untuk nilai i3 yang bergerak dari i = 0 sampai i = 9, sebagai berikut: i
0
1
2
3
4
5
6
7
8
9
Unit digit i3
0
1
8
7
4
5
6
3
2
9
Sehingga, angka satuan dari jumlah satuan (the units digit for the sum) seperti persoalan di atas, adalah: = (1 – 8 + 7 – 4 + 5 – 6 + 3 – 2 + 9) x 2 – 0 = 10 =0 Seperti itu yang mampu aku pahami dari persoalan nomor 2 itu, Kawan-kawan,”tutup Ikram. | 127
“Kuerren, Bro!”komentar Amer atas penjelasan Ikram. “Terima kasih, Kawan,”balas Ikram. Terlihat beberapa orang sibuk mencatat ulang apa yang ditulis oleh Ikram di whiteboard. “Selanjutnya, soal nomor 3. Apakah ada yang mau menjadi sukarelawan?”pancing Ayra. “Logikanya butuh keseriusan tingkat tinggi, Bro!”komentar Sabiq. “Setuju, dan aku minta agar Fateh mencoba kali ini?”kata Ayra lagi. “Aku? Apa aku bisa?”sindir Fateh. “Insyaallah kau bisa, Kawan. Janganlah merendah terus. Kau itu hebat, Kawan!”puji Amer. “Baiklah, aku coba,”kata Fateh. Fateh melangkah ke depan. Dia terima kertas kiriman itu. Dibacanya serius. Sesat dipandanginya atap-atap langit. Dan… “Untuk menemukan penyelesaian atas soal ini, perhatikan kata kuncinya, bahwa x > y > z > 663. Itu artinya baik nilai x, nilai y, atau pun nilai z, ketiganya haruslah lebih besar dari 663!”kalimat pembuka Fateh. “Untuk itu, bila kita misalkan bahwa, x + y + z = 1998
....................(10.1)
2x + 3y + 4z = 5992
....................(10.2),
di mana Persamaan (10.2) dikurangi oleh 2 kali Persamaan (10.1), akan diperoleh: 2x + 3y + 4z = 5992 | 128
2x + 2y + 2z = 3996 – y+2z = 1996
....................(10.3)
Dengan memperhatikan syarat bahwa x> y > z > 663, kita dapat melakukan proses trial and error alias coba-coba terhadap beberapa kemungkinan nilai z, misalnya dengan z = 664. Sehingga: Kondisi 1 untuk
z = 664
y + 2 (664) = 1996 y = 1996 – 1328 y = 668
Sementara nilai x dapat dicari melalui Persamaan (10.1): x = 1998 – 668 – 664 x = 666
(x,y,z) = (666, 668, 664)
z = 665
y + 2 (665) = 1996
Kondisi 2 untuk
y = 1996 – 1330 y = 666 Sementara nilai x dapat dicari melalui Persamaan (10.1): x = 1998 – 666 – 665 x = 667
(x,y,z) = (667, 666, 665)
Akhirnya terlihat bahwa: (1) Kondisi 1 di mana nilai (x,y,z) = (666, 668, 664), terlihat bahwa syarat x > y > z > 663 tidak terpenuhi karena nilai x ternyata lebih kecil dari nilai y! (2) Kondisi 2 di mana nilai (x,y,z) = (667, 666, 665), terlihat bahwa syarat x > y > z > 663 terpenuhi.
| 129
Sehingga, nilai yang dicari adalah x = 667, y = 666, z = 665,”terang Fateh mengakhiri penjelasannya “Konkret, Bro. Keren Abiz,”lagi-lagi Amer berkomentar. “Selanjutnya, soal nomor 4. Siapa yang akan…”tiba-tiba kalimat Ayra terpotong. “Anak-anak istirahat dulu, ya! Ibu sudah buatkan Teh Manis dan Pisang Goreng di meja. Disantap saat hangat-hangat, ya!”tibatiba Ibu Yodha muncul diantara keenam sahabat yang sedang asyik diskusi. Mereka saling pandang. “Tak baik makanan menunggu, Kawan!”kali ini Amer berkomentar. “Gayamu, Mer! Soal makan, mana ada yang bisa imbangi kau, Kawan!”kata Fateh pada Amer. “Ha…ha..ha, betul itu! Tapi, tak apalah, ada baiknya kita pending dulu diskusi ini. Lima belas menit, kita nikmati Pisang Goreng Hangat dan Secangkir Teh yang telah disiapkan oleh Ibu Yodha itu. Bagaimana Kawan?”usul Sabiq. “Setuju. Kita rehat sepuluh menit,”ucap Ayra dan Ikram hampir bersamaan. …∑πχαΩ…
“Memang apapun makanan yang disajikan seorang Ibu, terasa nikmat,”komentar Sabiq saat mencicipi Pisang Gooreng a la Ibu Yodha. “Tapi, kenapa begitu Sastrawan?”sindir Amer. | 130
“Karena sajian yang dihidangkan itu dimasak dengan ramuan cinta kasih dan ketulusan,”balas Sabiq. “Setuju! Dan itu adalah cinta kasih yang tulus dari seorang Ibu kepada anaknya,”tegas Fateh. “Tapi, rasa pisang goreng ini memang enak, Kawan,”kata Amer. “Kalau soal makanan dan lapar, mana ada yang tak enak di mulut Amer,”kali ini Ikram ikut komentar. “Ha…ha..ha..., soal yang itu aku sepakat dengan kau Ikram. Amer memang ahlinya mencicipi makanan. Bahkan bukan hanya itu, melahap makanan lebih tepatnya tepatnya,”tegas Fateh. Tampak keenam sahabat itu begitu ceria. Setelah hampir 20 menit, 10 menit molor dari yang disepakati, keenam sahabat itu kembali ke meja belajar mereka. “Sekarang siapa yang akan jadi sukarelawan?”tanya Ayra. “Aku rasa kau yang paling tepat kali ini, Ayra!”balas Yodha. “Wah, itu namanya senjata makan tuan,”ucap Ayra. “Bkan,
tapi
senjata
makna
nona,
karena
kau
perempuan,”sindir Amer “Tidak juga begitu. Kita sedang belajar bersama, toh! Dan Kau juga bagian dari tim yang punya talenta khusus dalam matematika, Kawanku Ayra,”tegas Yodha. “Baiklah aku coba,”kata Ayra singkat. Ayra memandangi soal di hadapannya. Ia berusaha merangkai pengetahuannya terkait dengan persoalan yang dihadapinya saat ini.
| 131
Cukup lama, setidaknya hampir 1,5 menit Ayra terpaku padangannya ke arah surat kiriman itu. “Seperti kita ketahui bahwa ciri dari bilangan yang habis dibagi oleh 4 adalah bahwa dua angka terakhirnya harus bisa dibagi 4. Sementara ciri bilangan habis dibagi oleh 9 adalah jika angka-angka penyusunnya dijumlahkan bisa dibagi oleh 9,”papar Ayra lagi. “Karenanya, 123X4Y bisa dibagi oleh 4, jika dan hanya jika 4Y bisa dibagi oleh 4. Untuk itu nilai Y bisa 0, 4 atau 8. Karenanya kemungkinannya adalah pada 2 angka terakhir: 40; 44; dan 48. Jika, Y = 0, maka
1+2+3+X+4+0 = 10+X …………….(10.4)
Y = 4, maka
1+2+3+X+4+4 = 14+X …………….(10.5)
Y = 8, maka
1+2+3+X+4+8 = 18+X …………….(10.6)
Sehingga, agar masing-masing: Persamaan (10.4) agar (10+X) menjadi kelipatan 9, haruslah X = 8. Persamaan (10.5) agar (14+X) menjadi kelipatan 9, haruslah X = 4. Persamaan (10.6) agar (18+X) menjadi kelipatan 9, haruslah X = 0 atau X = 9. Jadi, pada akhirnya, seluruh nilai pasangan (X,Y) yang mungkin adalah: (8,0); (4,4); (0,8), dan (9,8). Itu yang aku pahami kawan,”ucap Ayra. “Bukan main, hebat kau Ayra! Sekali jalan langsung selesai,”puji Fateh. “Alhamdulillah, aku punya Guru Matematika dan Kawankawan yang hebat-hebat, kan!”balas Ayra. “Selanjutnya, bagaimana?”tanya Fateh. | 132
“Selanjutnya tugas Yodha menemukan jalan penyelesaian bagi pertanyaan soal nomor 5 dari RRHS itu?”kali ini Yodha tidak bereaksi. Datar saja. Setelah kelima berhenti saling usul, semuanya nyaris saling tunjuk.Yodha pun bangkit. “Baik, aku akan coba selesaikan soal nomor 5, Kawankawan,”tegas Yodha. “Nah, itu baru Yodha,”kembali Amer komentar. “Pertama, bila kedua persamaan dalam persoalan kita tulis kembali, maka akan didapatkan persamaan sebagai berikut: ab + bc = 518
………………… (10.7)
ab – ac = 360
………………… (10.8)
Selanjutnya, bila Persamaan (10.7) dikurangi oleh Persamaan (10.8), akan diperoleh persamaan baru: bc + ac = 158 c(a +b) = 2 x 79
………………… (10.9)
Sehingga, berdasaran Persamaan (10.9) di atas, nilai yang sudah dapat ditentukan adalah nilai c, di mana nilai itu haruslah c = 1, atau c = 2, atau c = 79. Variasi pada nilai c ini memungkinkan terjadinya beberapa kemungkinan jawaban, yaitu: Kemungkinan 1 Jika c = 79, maka (a+b) = 2
a = b = 1.
Kondisi ini tidak memenuhi baik persamaan (10.7) maupun persamaan (10.8). Kemungkinan 2 Jika c = 2, maka (a+b) = 79. Bila nilai ini disubstitusikan ke persamaan (10.8) di mana: | 133
ab – ac = 360; a(79- a) – 2a = 360
b = 79 – a, maka:
a2 –77a+360 = 0 …(10.10)
Bila persamaan (10.10) difaktorkan, diperoleh: a2 – 77a + 360 = 0 (a – 72)(a – 5) = 0 nilai a = 5 atau a = 72. Selanjutnya kita lakukan analisa, yaitu: Saat a = 5,
b = 79 – 5 = 74, dan c = 2; maka nilai abc = 740.
Saat a = 72,
b = 79 – 72 = 7, dan c = 2; maka nilai abc = 1008.
Kemungkinan 3 Jika c = 1, maka (a+b) = 158. Bila nilai ini disubstitusikan ke Persamaan (10.8) di mana: ab – ac = 360; a(158 – a) –a = 360
b = 158 – a, maka:
a2–157a+360 = 0 … (10.11)
Bila Persamaan (10.11) difaktorkan akan diperoleh: a2 – 157a + 360 = 0
………………..… (10.12)
Persamaan (10.12) di atas tidak memiliki penyelesaian untuk bilangan bulat. Sehingga kemungkinan terbesar nilai perkalian abc adalah
1008!
Begitu
penjelasanku,
Kawan-kawan,”Yodha
mengakhiri penjelasannya. “Subhanallah, Yodha! Indah betul solusi itu, Kawan”puji Ayra. “Terima kasih, Ayra. Kau terlalu berlebihan,”balas Yodha. Merasa puas dengan penjelasan Yodha, kelima sahabat lainnya seperti serentak memberi tepukan hangat. Mereka begitu saling memahami kelebihan masing-masing. Dan bukan hanya itu, keenam sahabat itu pun tahu bagaimana menghargai kelebihan kawannya. | 134
Memang, mengapresiasi atas kelebihan orang lain, adalah langkah awal melapangkan hati. Lebih jauh, terhindar dari penyakit hati. Iri! “Oke kawan-kawan, selanjutnya siapa yang akan mengerjakan soal berikutnya?”tanya Ayra. “Biarkan aku mencoba, Ayra!”Jawab Amer bersemangat. “Silahkan, Kawan!”balas Ayra. Amer melangkah mantap ke depan. Dia berdiri posisi sempurna di sebelah papan tulis. Dia memulai dengan menarik nafas dalam-dalam. “Bismillah!”terdengar kata pertama, keluar dari mulut Amer. “Terhadap persoalan nomor 6 ini, bila persamaannya kita tulis ulang akan diperoleh bentuk sebagai berikut: a + b + c = 20 2
2
2
a +b +c = 318
………………… (10.13) ………………… (10.14)
Selanjutnya bila Persaman (10.11) dikuadratkan kedua ruasnya, ruas kanan dan ruas kiri, akan diperoleh: (a + b + c)2
= 202
a2+b2+c2+2ab+2ac+2bc = 400 ………………… (10.15) lantas, kita substitusikan Persamaan (10.14), khususnya bagian a2+b2+c2 ke Persamaan (10.15), sehingga: (318) + 2(ab+ac+bc)
= 400
2(ab+ac+bc)
= 400 – 318
2(ab+ac+bc)
= 82
Dan pada akhirnya, nilai yang dicari, adalah: 𝑎𝑎𝑎𝑎 + 𝑏𝑏𝑏𝑏 + 𝑎𝑎𝑎𝑎 = 41 | 135
Demikian apa yang bisa kupahami dari persoalan nomor 6 itu, Kawan-kawan,”terang Amer mengakhiri paparannya. “Singkat, padat, dan jelas, Kawan!”puji Sabiq. “Ternyata kecepatan makanmu sebanding dengan kecepatan menyelesaikan persoalan Matematika, Kawan!”puji sekaligus sindir Fateh pada Amer. “Terima
kasih,
Kawan-kawanku,”balas
Amer
sambil
memasang ekspresi wajah tersenyum. Senyuman yang khas! “Baiklah, selanjutnya siapa yang akan mencoba selesaikan persoalan nomor 7?”terdengar suara Ayra dari samping. Tak ada yang mengangkat tangannya tanda setuju. Saling pandang. Tapi, sebagian diantara mereka melihat ke arah Sabiq. “Aku..?”tanya Sabiq. Serentak kelima sahabatnya mengangguk. “Baiklah. Aku upayakan semampuku, Kawan,”kata Sabiq lagi. Ayra tenang dalam posisi memerhatikan papan tulis, Yodha memandang ke depan sambil memutar-mutar sepotong pisang goreng yang tinggal setengahnya. “Dari informasi yang tersedia pada persoalan di atas, maka dapat disimpulkan bahwa terdapat 3 (tiga) kombinasi pilihan topi yang memenuhi syarat untuk itu, yaitu: 1. 2 topi katun, 1 topi kulit dan 2 topi wol seharga: 2(Rp.30.000)+1(Rp.50.000)+ 2(Rp.10.000)=Rp. 130.000,2. 1 topi katun, 1 topi kulit dan 3 topi wol seharga: 1(Rp.30.000) +1(Rp.50.000)+3(Rp.10.000)=Rp. 110.000,3. 0 topi katun, 2 topi kulit dan 3 topi wol seharga: | 136
0(Rp.30.000)+2(Rp.50.000)+3(Rp.10.000)=Rp. 130.000,Karena setiap topi berbeda bahan, maka banyaknya cara untuk masing-masing kombinasi yang mungkin adalah: Kombinasi (1) adalah 1 x 5 x 3
= 15 cara
Kombiansi (2) adalah 2 x 5 x 1
= 10 cara
Kombinasi (3) adalah 1 x 10 x 1
= 10 cara
Sehingga, banyaknya cara untuk memberli 5 buah topi dengan persayaratan di atas adalah = (15 + 10 + 10) cara = 35 cara. Itu saja sejauh ini yang bisa aku temukan jalan penyelesaiannya, Kawan-kawan,”pungkas Sabiq. “Alhamdulillah, 7 dari 9 persoalan sudah selasai kita tuntaskan. Belum begitu larut dan kita masih punya 2 persoalan lagi. Siapa selanjutnya?”terang Ayra. “Persoalan nomor 8 tentang apa yang disebut sebagai polindrome, salah satu bilangan dengan bentuk simetris,”kata Yodha. “Simetris, maksudmu apa, Biq?”tanya Fateh dan Ayra bersamaan. “Ya, simetris. Kalau kita ibaratkan bisa dibagi dua bagian, akan terlihat bagian pertama dan bagian kedua sama, kiri dan kanan sama,”terang Sabiq. “Nah, kalau begitu bukan siapa yang harus menyelesaiakan. Aku pikir, kau yang paling tepat, kawanku Sabiq,”pinta Fateh. “He….he…he, pandai memang kau kawan,”balas Sabiq.
| 137
Tanpa diminta lebih jauh, Sabiq maju ke depan dan dengan langkah mantap dia berdiri tegak di depan papan tulis. “Seperti tadi aku sampaikan bahwa prinsip dasar bagi Palindrome adalah kesimetrisan ujung-ujung terhadap bagian tengah. Kata kunci dari persoalan di atas terletak pada frasa, “…pasangan palindrome
4-angka
yang
jika
dijumlahkan
menghasilkan
palindrome 5-angka?” Ingat bahwa kombinasi 4-angka terbesar adalah 9999 yang bila dijumlahkan sesamanya akan menghasilkan angka: = 9999 + 9999 = 19998 Bahwa 5-angka yang terbentuk, baik membentuk palindrome atau pun tidak, berawal pada angka 1 sebagai angka depannya! Sehingga dapat ditebak bahwa palindrome 5-angka yang terbentuk dari penjumlahan palindrome 4-angka, angka awalnya (puluhan ribu) dan juga harus angka akhirnya (satuan) adalah angka 1,”jelas Sabiq. Berhenti sejenak, Sabiq meneguk Teh Manis yang disediakan oleh Ibu Yodha dan terletak di meja. “Karenanya, akan terlihat dan terbentuk persamaan berikut: abba cddc+ 1 efe 1 Sehingga, kombinasi antar penjumlahan a dan c yang mungkin menghasilkan angka 1 adalah sebagai berikut: a
2
3
4
5
c
9
8
7
6 | 138
Karenanya, ada 2 kemungkinan, yaitu: Kemungkinan 1: Akan diperoleh bilangan palindrome jika b = d = 0, yaitu (a, c)= (2002, 9009); (3003, 8008); (4004, 7007); (5005, 6006). Sehingga akan terdapat 4 kemungkinan,”terang Sabiq bersemangat. “Kemungkinan 2: Akan diperoleh bilangan palindrome jika (b+d)= 11, yaitu pasangan (b,d) = (2,9); (9,2); (3,8); (8,3); (4,7); (7,4); (5,6); (6,5). Sehingga, dari kemungkinan 2 yang bisa terjadi, untuk menghasilkan palindrome 5-angka dari penjumlahan palindrome 4-angka adalah sebanyak: = (4 x 8) kemungkinan = 32 kemungkinan Akhirnya, total kemungkinan untuk menghasilkan palindrome 5angka dari penjumlahan palindrome 4-angka adalah: = 4 kemungkinan + 32 kemungkinan = 36 kemungkinan. Itu penjelasanku, Kawan-kawan,”tutup Sabiq. “Wah, bersemangat sekali, anak-anak Ibu ini. Sudah pukul 21.45, lho!”tiba-tiba Ibu Yodha masuk dan memperingatkan keenam sahabat yang sedang asyik dengan persoalan matematikanya. “Iya, Bu. Sudah hampir selesai, kok. Tinggal 1 soal lagi,”balas Ikram santun. “Baik, kawan-kawan. tinggal 1 soal terakhir, aku pikir ini bagian Tuan Rumah sajalah. Sepakat, kan?”tanya Ayra. “Sepakat!”terdengar suara bersahutan keenam sahabat itu. | 139
“Yodha, kehormatan untukmu. Silahkan, Kawan!”pinta Fateh. “Baik, Kawan-kawan, kita selesaikan persoalan kiriman dari RRHS ini. Dan tentang soal nomor 9 ini hanya tentang cara menyusun kembali dengan memanfaatkan sifat perkalian dan perpangkatan,”kata Yodha. “Bila kita tulis ulang, persamaan pada persoalan di atas adalah sebagai berikut: P = 32008 + 3-2008 Q = 32008 – 3-2008 Dengan mengingat sifat perkalian: (a + b) (a – b)
= a2 – b2
Maka nilai P2 – Q2 pun dapat dituliskan dalam bentuk: P2 – Q2 = (P + Q) (P – Q) = (32008 + 3-2008 + 32008 – 3-2008) (32008 + 3-2008 – 32008 – 3-2008) = (2 x 32008) (2 x 3-2008) = (4. 30) =4 Demikian,
semoga
memuaskan
rasa
dahaga
keingintahuan
kita,”pungkas Yodha. “Alhamdulillah!
Ternyata
dengan
kebersamaan
dan
kesungguhan kita bisa memecahkan seperti apa yang diminta oleh RRHS dalam surat misteriusnya, Kawan-kawan. Siapa pun RRHS itu, ucapan terima kasih sangat perlu kita sampaikan. Semoga surat yang dikirim ini menjadi tambahan amal shalehnya,”tutup Sabiq. “Amiiin!”terdengar ucapan penutup doa dari keenam sahabat.
| 140
Waktu telah menunjukkan pukul 22.05 malam. Keenam sahabat itu sepakat untuk mengakhiri diskusi mereka. Setelah berpamitan kepada Ibu Yodha, tampak lima anak dengan langkah pasti meninggalkan rumah Yodha, yang malam itu bulan bersinar sangat terang. Seterang hati masing-masing dari keenam sahabat yang menggenggam niat dan cita-cita mulia itu. …∑πχαΩ…
| 141
11
Kesebelasan Genk OSN “Hari ini adalah Hari Jum’at, Kawan. Sayyidul ayyam! Penghulunya hari dalam sepekan. Nabi kita mengajarkan untuk banyak
melakukan
hal
yang
bermanfaat.
Bahkan
beliau
menganjurkan untuk memotong kuku, berpakaian terbaik dan menggunakan wewangian. Karenanya, menjadi satu kebaikan pula bagi kita semua untuk menambah Genk OSN kita. Bagaimana Fateh?”tanya Sabiq setelah mendengar penjelasan Saver. “Aku setuju dan kita semua akan menjadi kesebelasan yang tangguh, Kawan!”balas Fateh. “Alhamdulillah, akhirnya kami lega karena diterima dalam Genk OSN ini. Mudah-mudahan ada manfaatnya,”kata Saver yang ditunjuk teman-temannya menjadi Juru Bicara. “Kesebelasan katamu, Kawan?”tanya Amer. “Ya, kesebelasan. Bukankah dengan hadirnya Iman, Denan, Saver, Bintang, dan Billy, ditambah kau, Yodha, Ayra, Ikram, Sabiq, dan aku sendiri kita berjumlah 11 orang, Kawan?”jelas Fateh. “Betul sekali kau, Kawan!”jawab Amer bersemangat “Kawan-kawan,
sebelumnya
kami
minta
maaf.
Tapi,
sebenarnya sebelum kami putuskan menemui kalian, Pak Kepala
| 142
Sekolah memberi catatan dan persyaratan,..”tiba-tiba Saver memecah kegirangan kesebelas sahabat menjelang siang itu. “Persyaratan apa itu, Kawan?”tanya Ayra penuh selidik. Saver meminta Bintang maju dan menjelaskan. “Bintang, rasanya kau yang paling tepat menceritakannya, Kawan!”kata Saver pada Bintang. Laki-laki kurus berkulit cokelat yang dipanggil Bintang itu maju sambil membawa secarik kertas di tangannya. “Kawan-kawan. Sebenarnya setelah kami bertemu Kepala Sekolah dan mendapat persetujuan beliau untuk bergabung dengan kalian, Pak Kepsek memberi satu syarat tambahan. Dan syarat itu adalah perintah yang tertulis pada sepucuk surat ini,”terang Bintang. “Apa pula isi surat itu, Bintang?”tanya Yodha heran. “Bukan apa-apa, hanya 5 persoalan Matematika, yang harus kami ...”kata Bintang terputus karena tiba-tiba terdengar suara khas Amer. “Jadi, Pak Kepsek memberi persyaratan tambahan bahwa untuk bisa bergabung dalam tim ini kalian harus mengerjakan masing-masing 1 soal, begitu!”potong Amer. “Tepat,
Mer.
Bukan
hanya
itu,
kami
pun
harus
menyelesaikannya serta menyampaikannya kepada kalian, di hadapan kita semua. Kalau menurut kalian apa yang kami sampaikan nanti sudah pas dan bisa memenuhi standar, kami layak bergabung. Kalau ternyata tidak, terserah kalian. Begitu pesan Kepala Sekolah,”terang Bintang.
| 143
“Ada-ada saja, Pak Kepsek kita itu. Dasar Guru Matematika, sih! Bagaimana menurutmu, Biq?”tanya Amer. Sabiq hanya tersenyum. Senyum yang sangat dikenal oleh kelima sahabatnya. “Sabiq? Ada hal yang aneh, kah?”kali ini Ayra bertanya. Sabiq masih mengembangkan senyumnya. Tidak kali ini justru senyum itu lebih lebar. Hampir saja tertawa. “Menurutku Pak Kepsek hanya menguji kesungguhan kalian, Kawan-kawan. Soal diterima atau tidak, hal itu tidak ada hubungannya dengan bisa atau tidak kalian mengerjakan kelima soal itu dan berkenan atau tidaknya kami. Bukankah tadi sudah kami sampaikan bahwa kami sangat senang kalian bisa bergabung,”terang Sabiq. “Kami paham itu, Kawan. Tapi, sebagai sebuah perintah dari Pak Kepsek, kami pun punya kewajiban untuk menyampaikannya kepada kalian,”tiba-tiba Denan yang sedari tadi duduk di sudut ruangan bersuara. “Ya, dan insyaallah kami sudah siap untuk memaparkannya kepada kalian,”tambah Bily yang juga tampak semangat. “Kapan kami bisa menjelaskannya, Kawan?”kali ini Iman angkat bicara. “Subhanallah! Kesebelasan ini memang unik, Kawan-kawan. Dan aku beruntung bisa menjadi bagian dari squad-nya,”tiba-tiba Ikram pun angkat bicara.
| 144
“Nanti saja, selepas sholat Jum’at dan makan siang, kita kembali ke ruang ini untuk sama-sama membahas apa yang telah diperintahkan oleh Pak Kepsek pada kalian. Oke!”balas Fateh. “Ya, setengah jam lagi masuk waktu Sholat Jum’at. Bersiapsiap dan berjalan ke Masjid terdekat rasanya 20 menit cukup. Jadi kita punya waktu untuk tahiyyatul masjid sebelum adzan, Kawankawan,”kata Ayra. “Perempuan kan tak wajib sholat Jum’at, Ayra!”sela Yodha. “Ya, tapi juga tidak dilarang ikut sholat Jum’at, Yodha,”balas Ayra. Tak lama kemudian kesebelas sahabat itu bubar. Memang, meskipun tak ada kewajiban bagi peremuan untuk sholat Jum’at di Masjid, tapi tak ada larangan untuk mengikutinya. Dan kali ini, Kesebelasan Genk OSN, yang satu diantaranya adalah perempuan, berjalan menuju Masjid untuk mengerjakan sholat Jum’at. …∑πχαΩ…
Tiga hari sebelumnya, sehari setelah keenam sahabat itu selesai mengerjakan persoalan yang mereka dapat dari Surat Misterius RRHS, sore-sore selepas Ashar, terjadi perdebatan sengit. Perdebatan tentang permintaan Kepala Sekolah yang disampaikan melalui Sabiq tentang memperbesar jumlah Genk OSN. “Kita sudah berjalan sejauh ini, tiba-tiba masuk anggota baru, apakah
justru
tidak
mungkin
terjadi
kemunduran
kawan-
kawan?”Amer memulai perdebatan hangat itu.
| 145
“Dan belum tentu mereka bisa beradaptasi dengan pola kita yang seperti ini, Kawan,”timpal Fateh. “Meskipun itu saran dari Kepala Sekolah, kita bisa menolaknya, kan? Ini soal efektifitas dan kelancaran kerja Genk OSN sendiri, Kawan”pendapat Ikram “Bagaimana menurutmu, Ayra?”tanya Sabiq. Ayra tak langsung menjawab. Perempuan satu-satunya dalam Genk OSN ini memang penuh kejutan. Optimisme yang melekat dalam dirinya jauh meninggalkan kelima kawannya. “Aku pikir tentu ada manfaatnya, dibalik segala kemungkinan terburuk yang kita pikirkan ini. Hanya saja kita harus atur ritme yang pas seandainya teman-teman yang diusulkan oleh Kepsek itu akhirnya bergabung, Kawan,”kata Ayra. “Bukankah kau selalu mengutip frasa, ‘Insyaallah ada jalan’, Biq! Pendapatku pada posisi itu. Toh, pada akhirnya hanya beberapa dari kita yang akan dikirim menjadi utusan resmi sekolah dan banyak event yang tersedia dan kita bisa berbagi! Karenanya, tidak ada salahnya kalau kita perbesar Genk OSN, Kawan-kawan,”kata Yodha. Sabiq menarik nafas panjang. Sebagai leader yang ditunjuk dalam diskusi kali ini, dia harus bisa menemukan jalan tengah yang diterima semua kawannya. Dan jalan tengah itu bukanlah hal yang mudah. “Aku tak ingin memengaruhi pendapat kalian, Kawan. Tapi mengajak berfikir lebih jauh dan mendasar. Pertama, bukanlah memecahkan satu persoalan dengan banyak kepala akan lebih mudah daripada memecahkan banyak persoalan dengan satu kepala? Kedua, | 146
ada ungkapan yang sudah lama kita kenal bahwa “bagai Aur dengan Tebing” dan “berat sama dipikul, ringan sama dijinjing,”Sabiq memulai aksinya. “Maksudmu, apa sesungguhnya, Kawan?”kali ini Yodha bertanya serius “Peribahasa tadi bermakna tolong menolong. Setiap orang memerlukan orang lain dalam kehidupannya. Bekerjasama dan saling membantu akan memudahkan dan mempercepat kerja. Tolongmenolong di antara satu sama lain. Seperti serumpun aur di tebing. Aur menumpang pada tebing untuk mendapatkan air, di saat yang sama tebing pun memerlukan aur. Setiap orang memerlukan orang lain dalam kehidupannya. Begitu pun Genk OSN kita ini, Kawan. dan insyaallah
semua
akan
baik-baik
saja.
Itu
kata
akhir
pendapatku,”kata Sabiq “Konkretnya, apa Biq?”desak Amer. “Kita temui dan dengar apa yang akan mereka sampaikan. Setelah itu kita baru bisa tentukan sikap. Meskipun, kita sepakat terlebih dahulu untuk cenderung pada membesarkan Genk OSN yang berarti ‘menerima’ mereka dalam kelompok ini,”pungkas Sabiq. Keenam sahabat itu tampaknya puas. Penjelasan Sabiq, Sastrawan-Penerjemah yang Dukun itu, tampaknya dapat mewakili sikap mereka terhadap persoalan yang diajukan oleh Kepala Sekolah beberapa hari lalu. …∑πχαΩ…
| 147
“Baiklah, demi memenuhi amanat dari Kepala Sekolah dan kami pun harus memberikan persyaratan yang pantas kepada kawankawan yang lebih dulu membentuk Genk OSN ini, izinkan kami membuka sepotong kertas yang disampaikan oleh Kepala sekolah beberapa hari lalu,”Saver memulai diskusi selepas sholat Jum’at dan makan siang. Selembar kertas itu sederhana saja. Sebelah folio bergaris yang dilipat menjadi empat bagian. Iman memberikannya kepada kepada Saver, selanjutnya Saver mulai membaca.
“Anak-anakku yang tercinta. Mohon maaf sebelumnya, Bapak titipkan beberapa persoalan Matematika yang Bapak himpun dari beberapa event dan perlombaan serta buku catatan Bapak sendiri. Mudah-mudahan soal ini bisa menambah semangat anak-anakku sekalian untuk terus meningkatkan kemampuan dalam memelajari Matematika dan memersiapkan diri. Harapan Bapak, stimulus ini semakin merekatkan kalian dengan teman-teman kalian yang lebih dahulu berinisiatif. Berikan yang terbaik buat mereka. Bapak percaya kalian mampu, insyaallah!” Wassalam, Kepala Sekolah Soal 1 Dalam suatu kelas, 3/5 bagian siswanya adalah perempuan. Ke dalam kelas tersebut ditambahkan 5 siswa pria dan 5 siswa perempuan. Sekarang 3/7 bagian siswanya adalah pria. Berapakah banyak siswa dalam kelas mula-mula? Soal 2 Bilangan A adalah bilangan asli terkecil yang merupakan hasil kali dari 3 bilangan prima pertama. Dua buah bilangan antara 200 dan 300 yang memiliki faktor prima tepat sama dengan bilangan A adalah …. | 148
Soal 3 Find all pairs (a; b) of digits such that ��� 𝑏𝑏 𝑎𝑎𝑎𝑎 =2− ����� 𝑎𝑎 𝑏𝑏汴
Soal 4 Sembilan lingkaran kongruen terletak di dalam persegi seperti terlihat pada gambar. Jika keliling sebuah lingkaran 62,8 cm dengan π = 3,14, maka luas daerah yang diarsir adalah =…. cm2 (A) 344 (B) 364 (C) 484 (D) 688 (E) 728
Soal 5 Misalkan n adalah suatu bilangan asli dan x adalah bilangan riil positif. Jika 3 2𝑥𝑥 𝑛𝑛 + −𝑛𝑛/2 − 2 = 0 𝑥𝑥 Maka nilai dari 2 1 𝑥𝑥 𝑛𝑛 + 4 sama dengan …..
Soal 6 A telephone pole is supported by a steel cable which extends from the top of the pole to a point on the ground 3 meters from its base. When Leah walks 2.5 meters from the base of the pole toward the point where the cable is attached to the ground, her head just touches the cable. Leah is 1.5 meters tall. How many meters tall is the pole?” | 149
“Itu isi selembar kertas dari Kepala Sekolah. Dan atas 6 persoalan yang diajukan, masing-masing kami akan mencoba memaparkan penjelasannya kepada kita semua, khususnya para pendahulu inisiator lahirnya Genk OSN sekolah kita,”papar Saver. “Jangan terlalu formil, Kawan. Santai saja. Kita kan bersekolah di tempat yang sama,”balas Ikram datar. “Baik. Soal nomor 1 akan diselesaikan oleh….,”Saver menghentikan kalimatnya. “Izinkan aku memulai Kawan!”Iman maju ke depan. “Silahkan, Man!”jawab Saver. “Untuk menyelesaikan persoalan seperti ini. Langkah pertama yang mungkin adalah, kita bisa membuat permisalan atau perumpamaan. Misalkan: Banyaknya siswa pria
= x orang,
Banyaknya siswa perempuan
= y orang
Karena 3/5 bagian siswanya adalah perempuan, maka bagian siswa laki-laki adalah 2/5, sehingga dapat dituliskan sebagai: 𝑥𝑥 2�5 = 𝑦𝑦 3� 5 𝑥𝑥 2 3 = ⇔ 𝑦𝑦 = 𝑥𝑥 … … … … . (11.1) 𝑦𝑦 3 2
Selanjutnya setelah ditambahkan 5 pria dan 5 perempuan sehingga 3/7 bagian siswanya adalah pria, maka: 𝑥𝑥 + 5 3�7 = 𝑦𝑦 + 5 4� 7 | 150
+ 5 3 = ↔ 3𝑦𝑦 + 15 = 4𝑥𝑥 + 20 𝑦𝑦 + 5 4 3𝑦𝑦 + 15 = 4𝑥𝑥 + 20
3𝑦𝑦 − 4𝑥𝑥 = 5 … … … … … … … . … . (11.2)
Kalau kita substitusikan persamaan (11.1) ke persamaan (11.2) akan diperoleh: 3 3 � 𝑥𝑥� − 4𝑥𝑥 = 5 2 9 𝑥𝑥 − 4𝑥𝑥 = 5 2 1 � 𝑥𝑥� = 5 2 𝑥𝑥 = 10
Karena nilai x = 10, maka 3 𝑦𝑦 = (10) 2
𝑦𝑦 = 15
Sehingga, banyaknya siswa mula-mula adalah: = x+y = 10 +15 = 25 orang Demikian
penjelasanku
atas
persoalan
Nomor
1,
Kawan-
kawan,”terang Iman mengakhiri presentasinya. “Aih, mantap Bro!”komentar Amer lugas. “Terima kasih, Mer,”balas Iman. “Selanjutnya, siapa yang akan maju?”tanya Saver.
| 151
“Aku, Ver, Bily!”terdengar jawaban singkat seorang anak lelaki bertubuh sedang agak tambun dan berkacamata, sambil mengangkat tangan kanannya. “Silahkan, Bil!”balas Saver. Anak laki-laki berkacamata itu maju ke depan, menerima alat tulis dan berdiri tepat di sebelah papan tulis. “Aku berusaha menyelesaikan soal nomor 3, tentu semampu daya tangkap yang aku bisa, Kawan-kawan,’jelas Bily lagi. “Kita di sini sama-sama belajar, Kawan. prinsipnya take and give. Begitu prinsip yang kami bangun. Dan baiknya menjadi prinsip kita juga,”Yodha mengingatkan. “Baiklah. Terhadap soal nomor 3 ini lebih pada keberanian kita melihat kemungkinan-kemungkinan jawaban atas persoalan. Bila kita tulis ulang persoalannya, maka dikatakan: ��� 𝑎𝑎𝑎𝑎 𝑏𝑏 = 2 − … … … … … … … … . . … . . (11.3) ��� 𝑎𝑎 𝑏𝑏𝑏𝑏
Hal ini berarti pula bahwa persamaan (11.3) di atas dapat ditulis dalam bentuk: 10𝑎𝑎 + 𝑏𝑏 𝑏𝑏 − 1 = 1 − … … … … . . … . . (11.4) 10𝑏𝑏 + 𝑎𝑎 𝑎𝑎
Selanjutnya, persamaan (11.4) di atas dapat diubah dalam bentuk a dan b sebagai berikut: 10𝑎𝑎 + 𝑏𝑏 − (10𝑏𝑏 + 𝑎𝑎) 𝑎𝑎 − 𝑏𝑏 = … … … (11.5) 10𝑏𝑏 + 𝑎𝑎 𝑎𝑎
Bila persamaan (11.5) disederhanakan akan diperoleh bentuk sebagai berikut: 9(𝑎𝑎 − 𝑏𝑏) 𝑎𝑎 − 𝑏𝑏 = … … … … … … … … (11.6) 10𝑏𝑏 + 𝑎𝑎 𝑎𝑎
| 152
Bentuk persamaan (11.6) di atas menunjukan bahwa kondisi seperti itu tercapai bila:
atau,
𝑎𝑎 = 𝑏𝑏 … … … … … … … … … … … … . . . (11.7) 9𝑎𝑎 = 10𝑏𝑏 + 𝑎𝑎
4𝑎𝑎 = 5𝑏𝑏 … … … … … … … … … … … … . . (11.8)
Sehingga seluruh pasangan (a,b) yang memungkinkan adalah: 1. Dari
a = b;
(1,1), (2,2), (3,3),…., (9,9), dan
2. Dari
4a = 5b
(5,4)
Akhirnya pasangan yang memenuhi adalah himpunan dari: {(1,1), (2,2), (3,3),(4,4), (5,4), (5,5), (6,6), (7,7), (8,8, (9,9)}. Itu saja bentuk penyelesaian yang bisa aku kerjakan dari persoalan Nomor 3, Kawan-kawan,”pungkas Bily. “Plok, plok, plok,”terdengar applause dari Sabiq dan Ikram. “Terima kasih, Kawan,”balas Bily pelan. Suasana tampak cair. Tampak wajah Ayra seperti tersenyum puas. Fateh pun seperti mengangguk-angguk. Justru wajah kaget dan kagum terpancar jelas dari ekspresi Amer. Yodha pun seperti mendapat semangat baru. “Baik, selanjutnya aku tunjuk saja. Bintang, giliranmu Kawan!”pinta Saver pada Bintang. Laki-laki yang dipanggil Bintang itu maju ke depan. Sopan dan hati-hati sekali ia menerima kapur dan mendekat ke papan tulis. “Sebelumnya aku mohon maaf kalau nanti apa yang aku smapaikan tak memuaskan. Aku akan mencoba menyelesaikan soal Nomor 5, Kawan-kawan,”kalimat pembuka Bintang. | 153
“Silahkan, Kawan. Santai saja, Bro!”potong Ayra. “Mari kita perhatikan lebih seksama apa yang dimaksud oleh persoalan. Disebutkan bahwa n adalah suatu bilangan asli dan x adalah bilangan riil positif. Dan jika diketahui bahwa persamaan berikut ini terpenuhi dalam bentuk 2𝑥𝑥 𝑛𝑛 +
Maka nilai dari 2
3
𝑥𝑥 −𝑛𝑛/2
−2=0
1 𝑥𝑥 𝑛𝑛 + 4 adalah? Sampai di sini kita perlu pahamkan beberapa hal. Pertama sifat perpangkatan. Bahwa: 1 = 𝑋𝑋 −𝑎𝑎 𝑋𝑋 𝑎𝑎 Sebaliknya
1 = 𝑋𝑋 𝑎𝑎 … … … … … … … … … . … (10.16) 𝑋𝑋 −𝑎𝑎 (𝑋𝑋 𝑎𝑎 )𝑏𝑏 = 𝑋𝑋 𝑎𝑎𝑎𝑎 … … … … … … … … … (10.17)
Sehingga,
2𝑥𝑥 𝑛𝑛 +
3
− 2 = 0 … … … . … . . (10.18) 𝑥𝑥 −𝑛𝑛/2 Dengan sifat persamaan (10.16) maka persamaan (10.18) dapat ditulis sebagai 2𝑥𝑥 𝑛𝑛 + 3𝑥𝑥 𝑛𝑛/2 − 2 = 0 … … … … . (10.19)
Selanjutnya dengan memanfaatkan sifat persamaan (10.17), maka persamaan (10.19) dapat ditulis sebagai: | 154
2(𝑥𝑥 𝑛𝑛/2 )2 + 3𝑥𝑥 𝑛𝑛/2 − 2 = 0 … … … … (10.20)
Dengan menganggap bahwa 𝑥𝑥 𝑛𝑛/2 = A, maka persamaan (10.20)
dapat ditulis ulang menjadi: 2A2 + 3A – 2 = 0
(A + 2)(2A – 1) = 0 Sehingga dengan memfaktorkan persamaan dalam A tersebut akan diperoleh A = - 2 atau A = ½ untuk A = - 2
⇔
𝑥𝑥 𝑛𝑛/2 = −2,
⇔
𝑥𝑥 𝑛𝑛/2 = 1/2,
tidak terpenuhi karena x harus bilangan riil positif untuk A = ½
memenuhi, dan bila dikuadratkan kedua sisinya akan diperoleh: (𝑥𝑥 𝑛𝑛/2 )2 = (1/2)2 𝑥𝑥 𝑛𝑛 = 1/4
Sehingga, nilai dari: 2
1 𝑥𝑥 𝑛𝑛 + 4
=
2
1 1 �4 + � 4
=4
Itu saja yang mampu aku sajikan dalam pembahasan terhadap soal nomor 5 ini, Kawan-kawan!”pungkas Bintang. “Ck…ck…ck. Bukan main. Sempurna, Bro!”tiba-tiba Amer berkomentar sambil mengangkat ibu jari tangan kanannya. “Terima kasih, Kawan,”balas Bintang. Tampak jelas kepuasan dan kebanggaan pada wajah-wajah yang hadir di ruang diskusi itu. Sabiq, Ayra, dan Ikram saling pandang. Mereka seperti menyimpan perasan yang sama tentang | 155
kemampuan rekan baru mereka. Seulas senyum terpancar di wajah Ayra. Anggukan kecil pada bahasa tubuh Ikram, dan tarikan nafas yang dalam oleh Sabiq. “Selanjutnya, giliran Denan. Silahkan pilih bagian soal yang belum dikerjakan, Kawan!”tawar Saver. “Aku pilih mengerjakan nomor 2. Langsung saja, ya Ver?”pinta Denan. “Silahkan,
Kawan!
alat
tulis
dan
waktu
sepenuhnya
milikmu,”balas Saver. “Seperti disebutkan dalam soal bahwa, ‘A adalah bilangan asli terkecil yang merupakan hasil kali dari 3 bilangan prima pertama. Selanjutnya ditanyakan tentang dua buah bilangan antara 200 dan 300 yang memiliki faktor prima tepat sama dengan bilangan A’. Untuk persoalan ini kita perlu pisahkan secara hati-hati, indikator yang diberikan oleh persoalan,”kata Denan. Kalimat pembuka Denan yang cukup panjang. Tapi semua kawannya tampaknya serius mendengarkan. “Berdasarkan data soal bahwa A yang merupakan hasil kali dari 3 bilangan prima pertama berarti: A = 2 x 3 x 5 = 30 ……………….(10.21) Dengan demikian dua buah bilangan –antara 200 dan 300, yang memiliki faktor prima tepat sama dengan bilangan A’, adalah bilangan kelipatan 30 seperti ditunjukan oleh persamaan (10.21) di atas. Sehingga, dua bilangan tersebut dapat dituliskan menjadi:
| 156
X = k 30, dimana k merupakan bilangan bulat positif nonPrima. Sementara itu berlaku pula pembatasan bahwa nilai X terletak antara 200 dan 300. Dengan kata lain harus dipenuhi: = 200 < x < 300 ……………. (10.22) Nilai k pada persamaan (2) yang memenuhi adalah: ⇔
k=8
X = 30 (8) = 240
⇔
k=9
X = 30 (9) = 270
Akhirnya, dua buah bilangan itu adalah 240 dan 270. Sekian,”papar Denan mengakhiri penjelasannya. “Selamat,
Kawan!
Penjelasanmu
sangat
menarik
dan
clear!”puji Ikram pada Denan. “Terima kasih, Kawanku. Itu pun aku buat tadi malam dengan mengundurkan waktu tidurku,”balas Denan. “Wah, sampai segitunya, Kawan!”tanya Amer. “Ya, karena aku takut membuat malu diriku sendiri dan kawan-kawan saat diminta presentasi. Meskipun akhirnya selesai juga,”terang Denan lagi. “Ha…ha…ha. Tidak perlu sejauh itulah, Kawan. Tapi aku harus katakan padamu, kau hebat Kawan!”puji Ayra sambil memberi tanda “V” pada 2 jari tangan kanannya, Victory! “Berikutnya tinggal 2 soal. Dan ini menjadi bagianku. Mudahmudahan bisa diterima apa yang akan aku paparkan,”kata Saver tegas. “Silahkan Pak Juru Bicara Tim, Pak Jubir!”kali ini ungkapan tulus muncul dari Fateh yang sedari tadi banyak memilih diam dan memerhatikan kawan-kawannya. | 157
“Soal ini lebih dekat dengan persoalan Geometri. Seperti tertera pada soal disebutkan bahwa ‘sembilan lingkaran kongruen terletak di dalam persegi seperti terlihat pada gambar. Jika keliling sebuah lingkaran 62,8 cm dengan π = 3,14, maka luas daerah yang diarsir adalah =…. cm2 Sebelum lebih jauh membahas persoalan ini, perlu kita perhatikan baik-baik penggambaran ulang soalnya dalam bentuk yang lebih spesifik sebagai berikut:
Bahwa disebutkan keliling sebuah lingkaran adalah 62,8 cm dengan memandang nilai π = 3,14. Selanjutnya, jika kita uraikan akan terlihat bahwa: Keliling Lingkaran 2πr
= 62,8 cm = 62,8 (dengan π = 3,14)
Sehingga nilai r kecil dapat dicari sebagai berikut: 2πr
= 62,8
r
= (62,8/(2x3,14))
r
= 10 cm
Karenanya, panjang rusuk 1 persegi kecil adalah: = 2 x 10 cm = 20 cm Selanjutnya perhatikan ilustrasi lingkaran dan persegi di atas. Terlihat bahwa: | 158
Luas Arsiran
= 4 x (Lpersegi – Llingkaran) = 4 x (s2 – πr2) = 4 x (202 – 3,14x102) = 4 x (400 – 314) = 4 x 86 cm2 = 344 cm2
Sehingga nilai yang dicari adalah luas arsiran tadi, sebesar 344 cm2. Berarti ada pada pilihan A,”terang Saver. “Tunggu dulu, Kawan! penjelasan atas angka 4 untuk masingmasing segiempat dan lingkaran, bagaimana?”tiba-tiba Amer menyela sebelum Saver menyelesaikan penjelasannya. “Baik. seperti tadi aku katakan, mohon perhatian baik-baik perubahan gambar di bawah ini:
Terlihat bahwa keempat warna hitam yang berada di ujung-ujung lingkaran sesungguhnya menggambarkan bagian dari segiempat yang tidak masuk daerah lingkaran. Dan 4 buah warna hitam tersebut ada sebanyak 4 kali diapit oleh 4 lingkaran,”terang Saver. “Oke, clear, Bro! Thanks atas penjelasannya,”puji Amer. “Jadi kawan-kawan, itu jalan penyelesaianku untuk soal nomor 4. Mudah-mudahan bisa sama-sama kita pahami,”pungkas Saver. Tampak Yodha dan Ikram, dan Ayra bersamaan memandang ke arah Sabiq. Ketiganya seakan ingin menyampaikan pesan rahasia. Dan tampak Sabiq pun senyum-senyum. | 159
“Kita beruntung, dengan tambahan tim ini,”kira-kira begitu pernyataan dan arti pandangan ketiga sahabat itu. “Ya, mudah-mudahan harapan kita bisa tercapai, Kawankawan,”kira-kira begitu jawaban dan arti senyum-senyumnya Sabiq. Saver masih berdiri tegak dengan kapur tulisnya. Ia masih memandang lurus ke depan. “Masih ada satu persoalan yang tersisa, Kawan. Dan untuk soal ini, sebelumnya ada yang perlu aku sampaikan,”kata Saver. Tampak sepuluh orang sahabatnya begitu serius memandang perubahan sikap dan ucapan Saver. “Pertama, aku tak begitu paham Bahasa Inggeris, sehingga kemungkinan bias memahami maksud dari pertanyaan ini sangat besar.
Kedua,
persoalan
yang
diajukan
pun
bagiku
agak
membingungkan. Karenanya, aku berupaya melukiskan semampuku apa kira-kira yang dimaksud oleh soal itu,”jelas Saver. “Santai saja, Bro! kita punya penerjemah handal saat ini,”ujar Amer sambil melirik ke arah Sabiq. “Baik. setelah berupaya memahami makna dari kalimat soal, aku melukis gambar seperti di bawah ini!
| 160
(1) Pertama bahwa untuk menerjemahkan ‘a steel cable which extends from the top of the pole to a point on the ground 3 meters from its base’, adalah jarak titik b ke titik c (base); (2) Kedua makna dari ‘When Leah walks 2.5 meters from the base of the pole toward the point where the cable is attached to the ground’, adalah jarak antara titik b ke titik L atau posisi Leah berdiri; dan (3) Ketiga, arti dari her head just touches the cable. Leah is 1.5 meters tall, kulukis dengan posisi Leah berdiri yang menyentuh garis (kabel) a ke c. Sehingga, pertanyaan ‘how many meters tall is the pole?’ aku maknai sebagai mencari tinggi h yang berupa jarak antara titik a dan titik b,”Saver berhenti sejenak. “Tunggu dulu, Kawan. Bagaimana menurutmu Biq?”tiba-tiba Ayra menyela dan bertanya pada Sabiq. “Fantastic! Sebuah pemaknaan dan penggambaran yang sempurna, Kawan!”balas Sabiq singkat. “Terima kasih, Biq. Selanjutnya bila kita sederhanakan penggambaran di atas, maka akan terbentuk bangun dengan masingmasing ukurannya seperti di bawah ini:
Sehingga, akan terbentuk dua buah segitiga yang sebangun, di mana: | 161
Δ BAD ~ Δ L’AL Karenanya, dengan prinsip kesebangunan dan perbandingan sudut, bisa kita peroleh persamaan: 𝐵𝐵𝐵𝐵 𝐿𝐿′𝐿𝐿 = 𝐷𝐷𝐷𝐷 𝐿𝐿𝐿𝐿
⇔ (h/3) = (1,5/0,5)
⇔ h = [(3x1,5)/0,5] meter ⇔ h = 9 meter.
Demikian penjelasan dan cara penyelesaian yang bisa aku sampaikan kepada Kawan-kawan sekalian. Dan untungnya, Pak Kepsek memberikan petunjuk berupa jawaban akhir untuk setiap pertanyaan. Dan alhamdulillah, setelah kami cross check, ternyata kunci jawabannya sama dengan hasil yang dikerjakan oleh Kawankawan,”pungkas Saver. “Congratulations!
Kawan-kawan,
kalian
sudah
meluluhlantakkan benteng pertahanan Ayra, Ikram, Yodha, dan Sabiq. Mereka tim penilai yang aku yakin tak punya pilihan lain, selain menerima kalian bergabung menjadi satu tim!”tiba-tiba terdengar suara Amer. “Bukankah begitu, Kawanku Sabiq?”tanya Amer lagi. Sabiq hanya tersenyum. Begitu pun Ayra tersenyum. Ikram bahkan sampai tampak meneteskan air mata. “Ada apa denganmu, Kawan?”tanya Ayra. “Aku sangat terharu. Sesungguhnya Allah telah memberi banyak kemudahan kepada kita. Dan hari ini kita saksikan bagaimana kawan-kawan yang mau bergabung, justru memberi | 162
pelajaran berharga, sangat berharga, bagi kita. Selamat bergabung, Kawan-kawan!”tegas Ikram. Terdengar tawa lepas dari kesebelas sahabat yang kini bergabung dalam satu tim, Genk OSN itu. Kegembiraan yang lahir dari kerelaan menerima kebersamaan dan berbagi. Ya, bukankah amal jama’i, mengerjakan satu pekerjaan bersama-sama jauh lebih terasa ringan daripada dikerjakan sendiri. Dan perasaan itu, saat ini sedang dirasakan kenikmatannya oleh Kesebelasan Genk OSN itu! …∑πχαΩ…
| 163
12
l’histoire se répète Udara di kawasan itu sejuk sepanjang hari. Bahkan menjelang senja, seiring condongnya Sang Mentari ke peraduan dan turunnya kabut putih yang semakin lama semakin tebal, dingin adalah kata yang tepat untuk menggambarkan udara di sana. Namun, udara dingin itu nyaris tak terasa dalam ruang diskusi itu. Sebelas anak-anak muda terlibat dalam sebuah perdebatan seru. Entah apa yang sedang mereka perdebatkan. Di deretan kanan tampak Ayra, Ikram, Amer, Bily, dan Saver, sementara di seberangnya, tampak Denan, Bintang, Fateh, Yodha, dan Iman. “Kita harus lebih fokus dalam melakukan pembahasan dan menemukan pola-pola persoalan menjelang pelaksanaan OSN,”kata Ayra. “Ya, tapi bukan berarti kita membatasi diri pada kemungkinan persoalan-persoalan sejenis dan variasinya,”balas Iman. “Waktu kita terbatas untuk melakukan sebanyak-banyaknya eksplorasi. Kita dibatasi oleh itu, Kawan,”tambah Bily. “It’s Ok! Tapi bukankah kita mengalami proses yang semakin hari semakin positif kemajuannya. Kau rasakan sendiri, kan Bil?”balas Fateh di seberangnya.
| 164
“Kita tidak bisa terus begini, Kawan-kawan. Aku setuju fokus. Tapi, aku pun sependapat dengan variasi. Kita harus temukan titik singgungnya, Kawan-kawan!”tanggapan Ikram. “Jadi apa usul konkretmu, Kram?”balas Yodha. Yang ditanya diam. Dia tak mengemukakan kalimat lanjutannya. Suasana semakin tak menentu. Semua tampak ingin bicara dan mengemukakan pendapatnya. Sementara di depan tampak Sabiq mengamati dengan tenang, kesepuluh kawannya yang sedang beradu argumentasi. Sampai saatnya,… “Kawan-kawan, mohon perhatian, please!”tiba-tiba Sabiq bersuara agak keras sambil mengetuk meja dengan keempat jari kanannya yang dikepalkan. Aneh, tiba-tiba suasana hening seketika. Angin terasa berhenti. Kesepuluh sahabat yang sedari tadi berdebat keras menghentikan perdebatan mereka. Dan serentak mereka menoleh ke arah asal suara. “Kita paham bahwa kebebasan adalah hak dasar yang diberikan Allah kepada setiap hamba-Nya. Tapi, kebebasan itu selalu dibatasi oleh kebebasan orang lain dan kepentingan bersama. Dan kepentingan bersama itu, dalam hal ini, adalah kepentingan kita bersama, kepentingan untuk memersiapkan diri menghadapi OSN yang tinggal beberapa bulan lagi,”kata-kata tegas itu meluncur deras dari mulut Sabiq. Sepuluh sahabat itu tetap terpaku. Tak seorang pun berucap sepatah kata yang berusaha memotong pembicaraan. Mereka masih memandang serius ke arah Sabiq. | 165
“Oke, begini saja. Masing-masing kelompok mengutus seorang perwakilan ke depan, sampaikan apa yang dianggap penting dan harus kita kerjakan selanjutnya. Waktunya lima menit, aku tunggu!”tiba-tiba Sabiq memberi perintah. Seperti tersihir, kedua kelompok itu cepat berembug. Tampak dua kelompok begitu serius berembug. Tak sampai dalam hitungan 3 menit, terlihat masing-masing perwakilan keluar dari kelompoknya, menuju ke depan. Ikram mewakili sebelah kanan dan Fateh dari sebelah kiri. Keduanya terus berjalan, mendekat ke aras Sabiq. “Bagaimana, Kawan? Sudah disepakatikah?”tanya Sabiq. “Alhamdulillah, sudah Kawan. Aku dulu, atau…?”tanya Ikram. “Silahkan kau saja dulu, Kram. Aku berikutnya,”potong Fateh cepat. “Baik. Begini, Biq. Pada prinsipnya kami merasa bahwa kita kurang fokus, kurang langsung ke subtansi persoalan, yaitu materi OSN. Kita terlalu asyik dengan persoalan yang beragam. Intinya, kelompok kami berharap agar kita kembali ke persoalan utama, membahas soal-soal OSN saja. Itu yang kami harapkan,”terang Ikram. “Bagaimana menurutmu, Fateh?”tanya Sabiq. “Kami berbeda pendapat dengan kawan-kawan kelompok Ikram. Seperti sejak awal sama-sama kita tahu, bahwa OSN bukan satu-satunya tujuan dari pembentukan kelompok. Meskipun namanya Genk OSN. Tapi lebih jauh dari itu. Karenanya, selain membahas persoalan terkait materi OSN, Matematika khususnya sebaiknya, kita | 166
juga memperkaya dengan hal-hal seperti yang selama ini telah kita kerjakan. Tentu kita punya skala prioritas pengerjaan. Karena menurut kami, kita sedang dalam proses belajar,”terang Fateh. Sabiq tampak berfikir keras. “Oke, aku sudah mendengar apa yang menjadi harapan masing-masing kelompok. Sekarang kembalilah ke kelompok masing-masing, beri aku waktu 3 menit untuk berfikir dan merumuskan jalan keluarnya!”jawab Sabiq singkat. Kedua sahabat itu pun mundur. Mereka kembali pada masingmasing kelompoknya. Selanjutnya mereka menunggu apa yang akan diputuskan oleh Sabiq. Tampak Sabiq sedang menuliskan sesuatu pada secarik kertas. 3 menit hampir terlampaui. Dan sejurus kemudian, ia mengangkat kepala, lurus menghadap ke arah sepuluh kawannya yang sedari tadi was-was menunggu apa yang akan diputuskannya. “Kawan-kawan, seperti telah menjadi kebiasaan dan adat yang baik di negeri ini, adat yang sangat dipengaruhi oleh hukum dalam agama kita, Islam, bahwa musyawarah untuk memutuskan sesuatu hal yang menjadi kepentingan bersama adalah cara terbaik mempertemukan semua kepentingan. Dan sore menjelang malam ini, proses itu telah dan sedang kita lakukan. Dan Alhamdulillah, setelah mendengar dari masing-masing perwakilan kelompok tentang apa langkah kita selanjutnya, maka aku mengambil keputusan yang insyaallah baik untuk kita semua,”kata Sabiq.
| 167
Dia berhenti sejenak. Terlihat pandangannya mengarah ke masing-masing
kelompok.
Dia
menarik
nafas
dalam-dalam,
mengeluarkannya perlahan. “Dan jalan tengah yang aku usulkan adalah. Pertama, kita akan melakukan 2 kali lagi pertemuan dan pembahasan intensif untuk memperkaya perbendaharaan kita tentang persoalan seputar Olimpiade Matematika baik nasional maupun internasional. Kedua, pertemuan yang 2 kali itu adalah pertemuan yang membahas tentang persoalan OSN tiga tahun terakhir, dan pertemuan yang membahas materi keseluruhan tema yang lazim diajukan pada olimpiade secara global dengan persoalan internasional. Ketiga, bahwa malam ini kita akan mencari dan menyinergikan bahan-bahan untuk pertemuan pertama sekaligus membagi kelompok kerja. Dan terakhir, kita kembali ke ruangan ini selepas sholat Isya,”terang Sabiq tegas. “Alhamdulillah, kelompok kami setuju dengan usulmu itu, Kawan,”balas Ikram “Kami juga setuju atas usulanmu itu, Kawan,”tandas Fateh mewakili kelompoknya. Tepuk tangan meriah terdengar memecah keheningan sore. Tiba-tiba terdengar suara dari anggota kelompok. “Meski sudah diputus dan disepakat, aku sebagai anggota kelompok patuh pada putusan itu. Ada yang ingin aku tanyakan padamu kawan. Tentu bila aku dizinkan menyampaikannya,”tiba-tiba Ayra bertanya. Sabiq tersenyum tipis.
| 168
“Silahkan, Kawan. Ini forum kita, tak ada yang istimewa di antara kita,”balas Sabiq “Baik. Apa dasar pertimbangan sampai kau putuskan perlunya kita
memelajari
dan
membahas
soal-soal
OSN
3
tahun
belakangan?”tanya Ayra. “Setuju! Aku pun mau tanya hal yang sama padamu, Kawan?”tiba-tiba Amer berseru cukup keras. “Aku juga”sahut Yodha. “Dan aku pun seperti itu, Kawan,”komentar Saver dan Denan bersamaan. Selebihnya tampak mengangguk-anggukkan kepala mereka. Sabiq masih tersenyum. Kali ini senyum khas yang sangat dipahami oleh kawan-kawannya. “Kau memasang wajah seperti itu lagi, Kawan. Kami tahu, pastilah ada sesautu yang mendasar yang ingin kau sampaikan,”kali ini Fateh angkat bicara. “Sesaat setelah kalian datang menyampaikan sikap masingmasing kelompok, tiba-tiba aku teringat pada sepenggal pepatah Perancis yang menyebut l’histoire se répète, yang berarti bahwa sejarah akan berulang. Setidaknya beberapa unsur dari sejarah menjadi penanda akan perulangan itu,”jelas Sabiq. “Lantas apa hubungannya dengan OSN, Kawan?”tanya Yodha. “Kawan-kawan, bukankah OSN itu dibuat dalam bentuk standar. Kata standar ini bermakna bahwa ada hal-hal yang dipahami dengan kriteria tertentu dan setiap tahunnya menjadi acuan dalam | 169
penyusunan soal-soal OSN. Untuk itu, memahami persoalan yang diajukan setiap tahunnya, dalam hal ini 3 tahun seperti yang kita sepakati, menjadi penting. Setidak-tidaknya kita tahu pola dan cakupan pertanyaan yang diajukan. Dari pola dan cakupan itu kita bisa menebak arah dan kecenderungan serta cakupan persoalan OSN yang akan diajukan tahun-tahun berikutnya,”papar Sabiq. “Gila, kau Boy! Tak keliru kami angkat kau jadi pimpinan kelompok ini. Aku setuju sepenuhnya dengan pendapatmu soal se... répè…,apa tadi itu?”tanya Amer. “l’histoire se répète, Kawan! Sejarah akan berulang. Begitu pun OSN,”balas Sabiq. “Ya, se répète itu maksudku!”tambah Amer. “Baiklah, kita akhiri saja pertemuan sore ini. Nanti malam, selepas Isya’ kita berkumpul lagi untuk melakukan persiapan buat besok. Oke!”pungkas Sabiq. Diskusi sore menjelang Maghrib itu pun berakhir. Kesebelas sahabat itu menuju ke kamar masing-masing. Mereka memang beruntung, mendapat pinjaman 4 kamar tidur dan 1 ruang pertemuan dari ayah salah seorang teman mereka yang tergolong kaya di sekolah. Meski anaknya tak termasuk dalam Kesebelasan Genk OSN, orang tua mereka bangga dengan tekad dan kesungguhan kawankawan anaknya itu. Karenanya, saat kesebelas sahabat anaknya itu merencanakan bermalam untuk konsenterasi menghadapi OSN, ia meminjamkan tempat yang tergolong nyaman di kawasan wisata nan asri di pedesaan daerah pegunungan. Bahkan, kesebelas anak jenius kampong itu mendapat bantuan bis. | 170
Suara adzan Maghrib sayup-sayup terdengar dari kampung sekitar kawasan wisata itu. Udara pun semakin bertambah dingin. Tapi, kehangatan tetap bersemayam di kesebelas dada anggota Genk OSN itu. …∑πχαΩ…
Udara dingin, penerangan minim, bunyi suara Jangkrik dan sesekali burung malam adalah ciri melekat alam pedesaan. Sebuah situasi yang hari ini begitu susah ditemukan di beberapa kawasan yang bergerak menuju kota, kosmopolit. Dan dalam situasi seperti itu, tampak sebelas anak muda; sepuluh laki-laki dan seorang perempuan tengah asyik merencanakan langkah menghadapi event-event nasional dan internasional dalam bidang matematika. “Alhamdulillah, jadi kita sudah bisa menyepakati untuk membagi ke dalam lima kelompok kecil. Masing-masing terdiri atas 5 orang. Sementara aku lebih bertugas memandu sekaligus bantubantu,”jelas Sabiq. “Bantu-bantu itu perlu dipertegas berarti menerjemahkan soal dalam bahasa Inggris, sehingga jelas maksudnya, dan….”kata Amer menghentikan kalimatnya. “Dan apa, Mer?”tanya Sabiq “Dan sekaligus menjadi Dukun, Kawan!”kelakar Amer. “Ha….ha…ha..,”terdengar tawa lepas kesebelas anak muda dalam ruangan itu. “Baik. sekarang kita membahas hal teknisnya. Karena kita sudah sepakat untuk memelajari dan membahas soal-soal OSN 3 | 171
tahun belakangan, berarti OSN tahun kemarin, OSN dua dan tiga tahun yang lalu, maka pembagiannya adalah seperti ini,”terang Sabiq sambil berdiri menuju whiteboard yang tersedia di ruangan. “Karena setiap tahun jumlah soal yang diajukan dalam OSN tingkat nasional adalah 10, lima soal pada hari pertama dan lima soal pada hari kedua, maka jumlah soal yang tersedia sebanyak 3x10=30 soal. Selanjutnya karena kita telah sepakat untuk membentuk 5 kelompok,
maka
masing-masing
kelompok
mendapat
jatah
pembagian sebanyak 6 soal. Demi keadilan, maka dari 6 soal yang menjadi jatah setiap kelompok, kita bagi ke dalam 3 tahun. Sehingga setiap kelompok akan mendapatkan: Pertama, 2 soal dari soal OSN Tingkat Nasional tiga tahun lalu. Kedua, 2 soal dari OSN Tingkat Nasional dua tahun lalu; dan Ketiga, 2 soal dari OSN Tingkat Nasional tahun kemarin. Selanjutnya, dari 2 soal untuk masingmasing tahun itu, kita bagi lagi menjadi: 1 soal berasal dari hari pertama, dan 1 soal berasal dari hari kedua. Bagaimana, Kawankawan?”terang Sabiq “Setuju a hundred percent, Bro!”teriak Amer dari belakang. “Tunggu dulu! Menurutku sebaiknya 3 tahun itu berasal dari tingkatan yang berbeda pula,”saran Ayra. “Maksudmu seperti apa konkretnya, Ayra?”tanya Saver “Ya, selain berasal dari tahun yang berbeda, juga dari level yang berbeda, yaitu: OSN Tingkat Kabupaten-Kota, OSN Tingkat Provinsi, dan OSN Tingkat Nasional. Untungnya, kita bisa lebih memahami tipologi dan kecenderungan, syukur-syukur pola soal OSN itu sendiri,”terang Ayra. | 172
“Aku sependapat dengan usulan Ayra,”balas Bintang dari seberang. “Jadi, bagaimana kawan-kawan?”tanya Sabiq kembali. “Sebaiknya kita ikuti saran Ayra. Keragaman tahun dan tingkatan akan lebih membuat kita bisa melihat lebih luas persoalanpersoalan terkait, Kawan-kawan,”jelas Yodha. “Kita
setujui
usul
Ayra
atau
tetap
OSN
Nasional
seluruhnya?”tanya Sabiq “Setuju usul Ayra!”terdengar suara bersamaan dari 10 orang anak-anak muda itu. “Baik. kita sepakati seperti itu. Kita akan menyelesaikan persoalan OSN dari 3 tahun terakhir dengan tingkatan yang berbeda. Selanjutnya kita beranjak ke hal berikut, rencana kegiatan besok. Kita berharap soal OSN 3 tahun berturut-turut dari 3 level berbeda itu selesai kita kerjakan sebelum Maghrib. Karenanya aku mengusulkan 3 tahapan pembahasan. Pertama, selepas Dhuha sampai Dzuhur. Kedua antara Dzuhur dan Ashar, tentu setelah makan siang. Ketiga, selepas Ashar hingga menjelang Maghrib. Bagaimana kawan-kawan?”tanya Sabiq. “Setuju
saja.
Prinsipnya
besok
harus
bisa
kita
selesaikan,”jawab Saver. Tampak kawan-kawan mereka yang lain pun dapat menerima usulan pola pembahasan itu. “Alhamdulillah, artinya malam ini kita sudah selesaikan satu step
dari
acara utama
kita.
Insyaallah besok
kita
mulai
action,”pungkas Sabiq. | 173
Suasan menjadi tenang. Tiap anak merasa memili tanggung jawab dna kewajiban yang sepadan, adil. Terpancar kepuasan di wajah-wajah mereka. “Tapi,…,”tiba-tiba
Sabiq
berkata
agak
keras
dan
menghentikan kalimatnya. “Tapi, apa, Kawan?”tanya Fateh. “Tapi, tunggu dulu! Coba perhatikan baik-baik tulisan pada dinding itu. Tulisan yang berada di dekat lukisan beberapa orang petani yang sedang memanen padi di ladangnya itu!”balas Sabiq. “Ayra, coba lihat dan bacakan pada kita semua!”kata Ikram. Ayra melangkah mendekati lukisan, dan matanya terbelalak saat memerhatikan seksama kata-kata yang tersusun dan terangkai dalam sebuah kalimat. “Bacakan agak keras Ayra, agar kita semua dengar!”pinta Denan. “Baik. Dengar baik-baik, ya!”kata Ayra. Ayra memastikan ia tak salah membaca, ia lebih mendekatkan kepalanya, dan memandang serius ke arah tulisan itu. “Kawan-kawan, mohon perhatian! Ini bunyi kalimatnya, ‘Leluhurku adalah seorang laki-laki yang terlahir pada abad delapanbelas.
Suatu saat dia berusia x tahun pada tahun x2.
Berapakah usia leluhurku pada tahun 1776?’” bacaan Ayra terdengar jelas bergema ke segala penjuru ruangan. “Haa! tulisan seperti itu tertempel di dinding ruang ini?”tanya Amer heran.
| 174
“Kawan-kawan, coba lihat lagi ke dekat pintu keluar itu! Ada coretan mirip tulisan dalam beberapa paragraf. Bil, tolong kau bacakan buat kita,”kata Sabiq lagi. “Baik Kawan, aku segera ke sana,”jawab Bily tegas. “Wow, disini pun ada tulisan unik. Begini tertulis, the number N2 is 25% more than number N1, the number N3 is 20% more than N2, and the number N4 is x% less than N3. For what value of x is N4=N1?,”kata Bily yang terdengar jelas oleh semua kawankawannya. Tiba-tiba semua pasang mata di ruangan itu tertuju ke arah Sabiq. “Biq, apakah kau….,”Ikram dan Fateh membuka mulutnya bersamaan. “Tidak, tidak, tidak! Jangan pernah kalian berpikir bahwa aku yang menempel tulisan itu, Kawan! Tidak mungkin. Aku pun heran. Sebenarnya, sejak tadi aku masuk ke ruangan ini, untuk pertama kali, aku sudah tahu. Tapi, aku sengaja tak menceritakannya pada kalian, karena awalnya kupikir hanya kerjaan orang iseng. Ternyata tulisan itu memang sudah lama tertempel di sana,”terang Sabiq. “So… what do you think, Guys!”Saver memotong pembicaraan dan mengarahkan pandangannya pada Sabiq. “Kalau kalian berkenan, aku minta waktu kalian semua. Setelah rehat ngopi, kita tidak langsung bubar. Tapi, kita selesaikan makna dari tulisan ini. Aku pikir kita masih punya dua kelompok. Fateh, Ikram, silahkan dibahas dalam kelompok, dan setelah itu
| 175
masing-masing kelompok mengajukan seorang Juru Penjelas ke depan untuk kita semua. Oke!”saran Sabiq. Semua anak yang ada dalam ruangan itu saling pandang. Setelah beberapa saat hening. “Setuju!”sahut Fateh dan Ikram hampir bersamaan. Malam itu sebuah kejutan terjadi. Rasa kantuk yang tadi telah menyerang semua anak muda dalam ruangan itu seakan menguap, lenyap dengan temuan tulisan aneh yang berisi teka-teki matematika. Kopi dan snack berupa penganan kecil yang terdiri atas Ubi Goreng, Talas Rebus dan Pisang Rebus telah tersedia di meja, menjadi santapan di sela-sela diskusi internal kelompok atas dua persoalan yang tertempel di dinding ruangan itu. …∑πχαΩ…
Lima belas menit rehat kopi dan asupan makanan khas kampung telah berlalu dalam santapan sebelas anak muda itu. Dan tampaknya masing-masing kelompok pun sudah menemukan jalan penyelesaian atas makna dari tulisan yang terdapat dalam ruangan itu. “Baik. kelompok mana dan siapa yang akan maju terlebih dahulu,”tanya Sabiq pada kawan-kawannya. “Biarkan kelompok kami dulu, Biq. Dan kami mengajukan Ayra sebagai Juru Penjelas,”balas Ikram. “Silahkan Ayra!”pinta Sabiq Ayra maju ke depan. Ia menerima alat tulis dan menuju whiteboard.
| 176
“Kawan-kawan, setelah kami amati dan pelajari maknadari tulisan yang pertama, kami berkesimpulan bahwa tulisan itu dibuat oleh orang yang gemar Matematika, khususnya soal Aljabar. Disebutkan dalam tulisan itu bahwa leluhur orang itu adalah seorang laki-laki yang terlahir pada abad kedelapanbelas, dan suatu saat leluhurnya berusia x tahun pada tahun x2. Ditanyakan usia leluhur orang itu pada tahun 1776. Karenanya, ada beberapa hal yang harus dipahami terlebih dahulu, yaitu: Pertama, frasa pada abad kedelapanbelas bermakna bahwa tahun itu berkisar antara 1700 – 1800; Kedua, berusia x tahun pada tahun x2 berarti bahwa nilai x2 adalah menunjukkan tahun yang berada pada rentang 1700 - 1800. Selanjutnya, karena nilai dari: 412 = 1681 (𝐴𝐴𝐴𝐴𝐴𝐴𝐴𝐴 17)
432 = 1849 (𝐴𝐴𝐴𝐴𝐴𝐴𝐴𝐴 19)
maka dapat dipastikan bahwa tahun yang dimaksud (x2) adalah diantara kedua nilai kuadrat di atas, yaitu: 412 < 𝑥𝑥 2 < 432
Sehingga nilainya adalah: 𝑥𝑥 2 = 422
422 = 1764 (𝐴𝐴𝐴𝐴𝐴𝐴𝐴𝐴 𝑘𝑘𝑘𝑘𝑘𝑘𝑘𝑘𝑘𝑘𝑘𝑘𝑘𝑘𝑘𝑘𝑘𝑘𝑘𝑘𝑘𝑘𝑘𝑘𝑘𝑘𝑘𝑘)
Hal itu berarti pula bahwa leluhur orang itu pada tahun 1764 berusia: 𝑥𝑥 = �𝑥𝑥 2 𝑥𝑥 = 42
Sehingga leluhur orang itu dilahirkan pada: 1764 − 42 = 1722
| 177
Karenanya, pada tahun 1776 usia leluhur orang itu adalah: = 1776 − 1722 = 54 𝑡𝑡𝑡𝑡ℎ𝑢𝑢𝑢𝑢
Dengan demikian, maka pada tahun 1776, leluhur orang itu berusia 54 tahun. Demikian penjelasan kelompok kami,”papar Ayra. “Plok… plok…. plok,”terdengar applause dari kawankawannya. “Subhanallah! Kalian berhasil kawan-kawan,”puji Sabiq “Bukan kami, Biq. Tapi kita yang berhasil,”balas Ikram. “Oke, selanjutnya kelompokmu Fateh!”kata Sabiq. “Kami ajukan Yodha sebagai Juru Penjelas. Silahkan kawanku Yodha!”kata Fateh. “Baik. langsung saja. Persoalan ini bisa diselesaikan dengan backward method! Kita mulai dari belakang. Dan kami pun sependapat persoalan ini masih dalam ranah Aljabar,”terang Yodha. Yodha maju dengan langkah mantap. Ia menerima alat tulis dari Ayra. “Dari penggalan kalimat, the number N4 is x% less than N3 berarti dapat dituliskan sebagai: 𝑥𝑥 𝑁𝑁4 = 𝑁𝑁3 �1 − � … … … … … … … … . . . . (12.1) 100 Selanjutnya dari penggalan kalimat, the number N3 is 20% more than N2 berarti dapat dituliskan sebagai: 𝑁𝑁3 = 𝑁𝑁2 �1 +
20 � … … … … … … … … … (12.2) 100
Dan, dari penggalan kalimat N2 is 25% more than number N1
berarti dapat pula dituliskan sebagai: | 178
𝑁𝑁2 = 𝑁𝑁1 �1 +
25 � … … … … … … … … … . . . . (12.3) 100
Sehingga pada akhirnya makna dari penggalan kalimat for what value of x is N4=N1? Dapat dicari dengan menggabungkan persamaan (12.1), persamaan (12.2) dan persamaan (12.3) sebagai berikut: 𝑁𝑁4 = �1 −
𝑥𝑥 20 25 � �1 + � �1 + � 𝑁𝑁 … … . . (12.4) 100 100 100 1
Untuk kondisi di mana N4 = N1, maka persamaan (12.4) =1. Sehingga, 𝑥𝑥 6 5 �� �� � 100 5 4 3 𝑥𝑥 �� � 1 = �1 − 100 2 𝑥𝑥 2 � � � = �1 − 100 3 300 − 200 𝑥𝑥 = 3 1 𝑥𝑥 = 33 3 1 = �1 −
Demikian penjelasan kelompok kami, Kawan-kawan. Bahwa nilai x yang memenuhi adalah seperti tertera di atas, yaitu 33 1/3,”terang Yodha. “Alhamdulillah, akhirnya kita bisa istirahat dengan tenang, Kawan-kawan,”tiba-tiba Amer berseru. “Huss, memangnya kita semua sudah sakratul maut!”potong Bintang.
| 179
“Ha…ha…ha…,”terdengar tawa lepas sebelas anak muda dari ruang diskusi di tengah kawasan pedesaan yang udaranya semakin dingin dan menusuk tulang itu. Dari kejauhan, beberapa ratus meter dari ruangan tempatr sebelas anak itu berdiskusi, tampak sepasang mata terus mengamati apapun yang dikerjakan oleh anak-anak jenius kampung itu. “Ternyata kalian bisa menebak teka-teki pemanasan itu. Semoga bisa beratahn dan tersu semangat, anak-anak,”kata pemilik sepasang mata itu pada dirinya sendiri dalam kegelapan. …∑πχαΩ…
| 180
13
Kombinasi yang Indah
Sinar
Matahari
belum
sepenuhnya
mampu
menghangatkan
permukaan bumi. Bahkan di beberapa tempat, kabut putih tipis masih menghalangi sinar sang surya. Di sana sini masih terlihat tetesan embun jatuh ke tanah. Kicauan burung begitu ramai terdengar. Hamparan padi yang menghijau biaskan sebagian cahaya Matahari yang berhasil menerobos kabut dan melewati pegunungan yang berjejer rapi membungkus kawasan pedesaan yang asri itu. Suasana begitu hening dan damai. Tapi, semua keadaan itu berbeda seratus delapan puluh derajat dengan situasi di ruangan diskusi itu. Sebelas anak muda tampak terlibat intens membahas persoalan yang tadi malam telah mereka rencanakan. “Langsung saja. Tadi pagi selepas sholat subuh, Ikram, Fateh, Saver dan Bily menemui aku. Kawan kita ini menceritakan bahwa hasil penelusuran melalui media daring, kita temukan soal dan pembahasan OSN Matematika Tingkat Nasional yang dipublikasikan oleh Bapak Tutur Widodo dan Mohammad Tohir. Setelah aku lihat memang rasanya pembahasan keduanya representatif untuk kita telaah,”Sabiq memulai diskusi pagi selepas Dhuha itu. | 181
“Tapi, bagaimana komunikasi dengan penulisnya. Apakah sudah dilakukan?”tanya Bintang. “Ya, Bintang. Aku sudah berkirim surel ke Pak Tutur dan Pak Tohir. Dan allhamdulillah, Pak Tutur pun sudah membalas dan memberi izin kita untuk memanfaatkan pembahasan yang beliau tuliskan. Hanya saja dari Pak Mohammad Tohir, belum ada balasan suratnya,”terang Sabiq. “Tapi, bisa juga sambil menunggu izin dari beliau, kita bisa meneruskan apa yang tadi malam telah kita sepakati, kan!”ujar Denan bersmeangat. “Baik, untuk kelompok 1 saya ulangi kembali, terdiri atas Amer dan Saver. Selanjutnya, kelompok 2, terdiri atas Yodha dan Bily Kelompok 3, terdiri atas Ayra dan Bintang. Berikutnya kelompok 4 yang terdiri atas Ikram dan Denan. Terakhir adalah kelompok 5 yang terdiri atas Fateh dan Iman. Prisipnya masingmasing kelompok mengerjakan 1 soal pada hari pertama dan 1 soal pada hari kedua di setiap tahun OSN. Teknisnya, kita berbagi saja,”atur Sabiq. “Dan kau, Biq?”sindir Fateh. “Seperti kalian lihat, aku hanyalah Penerjemah dan Dukun. Aku memilih pada posisi pengatur jalannya diskusi, dan Ban Serep saja,”kata Sabiq “Ban Serep apa maksudnya, Biq?”tanya Bily. “Ya, kalau pecah ban, sehingga mogok atau macet, baru digunakan,”balas Sabiq.
| 182
“Dasar, kau Biq!”kata Amer, Denan, dan Yodha hampir bersamaan. “Oke selanjutnya kita terus bergerak. Kita mulai dari Amer dan Saver. Silahkan kawan!”pinta Sabiq. “Terima kasih, Biq. Kami akan maju bergantian. Untuk pertamaa kali, Amer mendaulat aku untuk maju,”terang Saver. Saver membuka catatan pada berupa selembar kertas folio bergaris. “Persoalan yang menjadi tugas kami adalah soal nomor 1 hari pertama dan Nomor 1 hari kedua OSN Matematika tingkat Nasional untuk 3 tahun berbeda. Karenanya, untuk memudahkan penjelasan atas penyelesaiannya seluruh soal akan ditulis terlebih dahulu, untuk selanjutnya kami akan lakukan telaah,”terang Saver. “Kawan, pola itu bisa kita sepakati disini. Aku pikir kelompok berikutnya pun boleh melakukan seperti itu,”tambah Yodha. “Setuju. Pola itu akan lebih membuat lebih fokus atas persoalan yang diajukan,”timpal Ikram. Saver masih menunggu komentar lain. “Aku pikir usul itu baik. Tapi, pembagian waktu yang telah kita sepakati bisa tak terpenuhi. Justru agar setiap kelompok mendapat keutamaan waktu dalam sehari ini, yaitu waktu Dhuha, waktu tergelincirnya matahari, dan waktu lepas ashar, maka sebaiknya kita selesaikan persoalan setiap tahun di setiap waktu dengan setiap kelompok,”tanggap Sabiq. “Ada benarnya juga pendapatmu itu, Biq,”tegas Bintang
| 183
“Dan kita akan menemukan pola persoalan setiap tahun bila kita telaah secara utuh dan tidak tercerai dalam rentang waktu berbeda,”tegas Sabiq lagi. “Baiklah, aku setuju saja,”ujar Saver, ia pun melanjutkan. “Setelah kami pisahkan dan kelompokkan kembali, maka persoalan yang diajukan kepada kami berdua adalah sebagai berikut: (1) Jika diketahui himpunan H = f(x, y)|(x – y)2 + x2 – 15x + 50 = 0, dengan x dan y bilangan asli}, tentukan banyak himpunan bagian dari H. (2) Pada suatu hari, seorang peneliti menempatkan dua kelompok spesies yang berbeda yakni amoeba dan bakteri pada suatu media yang sama, masing-masing dalam jumlah tertentu (dalam satuan sel). Peneliti tersebut mengamati bahwa pada hari berikutnya, yakni hari kedua, ternyata setiap sel masing-masing spesies membelah diri menjadi dua sel. Pada hari yang sama setiap sel amoeba memangsa tepat satu sel bakteri. Pengamatan selanjutnya yang dilakukan setiap hari menunjukkan pola yang sama, yakni setiap sel masing-masing spesies membelah diri menjadi dua sel dan kemudian setiap sel amoeba memangsa tepat satu sel bakteri. Pengamatan pada hari ke-100 menunjukkan bahwa setelah masing-masing spesies membelah diri dan kemudian setiap sel amoeba memangsa tepat satu sel bakteri, ternyata membuat bakteri punah. Tentukan perbandingan jumlah amoeba dengan jumlah bakteri pada hari pertama. Selanjutnya, Amer yang akan memberi penjelasan atas persoalan nomor 1. Silahkan, Kawan!”pinta Saver pada Amer. Amer melangkah maju dengan langkah tegap. Gayanya memang sebanding dengan suaranya yang khas itu. “Oke, pada persoalan nomor 1 ini ada beberapa hal yang harus dipahami bersama: Pertama, arti dari tanda garis tegak yang memisahkan dalam sebuah persamaan, seperti pada persoalan | 184
misalnya, f(x, y)|(x – y)2 + x2 – 15x + 50 = 0, terdapat bentuk berupa garis tegak (|) antara f(x, y) dan (x – y)2 + x2 – 15x + 50 = 0, adalah bahwa f(x, y) dapat membagi persamaan (x – y)2 + x2 – 15x + 50 = 0, dan hal itu berarti pula bahwa f(x, y) adalah faktor-faktor dari (x – y)2 + x2 – 15x + 50 = 0. Karenanya, bila nilai x dan nilai y pada f(x, y) disubstitusikan ke dalam persamaan (x – y)2 + x2 – 15x + 50 akan menghasilkan nol. Kedua, persamaan (x – y)2 + x2 – 15x + 50 = 0 dapat dianalogikan dalam bentuk sederhana menjadi: A2 + B = 0, di mana: A2 = (x – y)2 dan B = x2 – 15x + 50 Sehingga, dalam bentuk yang sederhana dapat ditulis ulang sebagai: H = f(x, y)|(x – y)2 + x2 – 15x + 50 = 0 2
H = f(x, y)|A + B = 0
……………(13.1) ……………(13.2)
Dari persamaan (13.2) terlihat bahwa: A2 selalu ≥ 0
↔ (x – y)2 pastilah ≥ 0
Karenanya agar persamaan (13.1) terpenuhi, agar nilai f(x, y) = 0 Maka, haruslah x2 – 15x + 50 ≤ 0. x2 – 15x + 50 ≤ 0 (x – 5)(x – 10) ≤ 0 5 ≤ x ≤ 10 Dengan mengingat bahwa x adalah bilangan asli, maka nilai yang mungkin untuk x adalah 5, 6, 7, 8, 9, 10. Ketiga, langkah berikutnya kita lakukan pemeriksaan satu persatu berdasarkan nilai yang memungkinkan, sebagai berikut : i. Jika x = 5 atau x = 10, maka diperoleh: (5)2 – 15(5) + 50 = 0; dan (10)2 – 15(10) + 50 = 0. | 185
Oleh karena, itu agar f(x, y) = 0 maka (x – y)2 = 0. Akibatnya x = y. Jadi, diperoleh dua jawaban (5, 5) dan (10, 10); ii. Jika x = 6 maka diperoleh: f(6, y) = (6 – y)2 + 62 – 15(6) + 50 = (6 – y)2 – 4. Oleh karena itu, agar f(6, y) = 0 maka haruslah (6 – y)2 = 4 , Akibatnya 6–y = 2 atau 6–y = –2, y = 4 atau y = 8. Jadi, diperoleh dua jawaban yaitu (6, 4) dan (6, 8) iii. Jika x = 7 maka diperoleh f(7, y) = (7 – y)2 + 72 – 15(7) + 50 = (6 – y)2 – 6. Karena y bilangan asli maka tidak ada nilai y yang memenuhi sehingga diperoleh P(7, y) = 0. Jadi, untuk kasus ini tidak ada jawaban yang memenuhi. iv. Jika x = 8 maka diperoleh f(8, y) = (8 – y)2 + 82 – 15(8) + 50 = (8 – y)2 – 6. Karena y bilangan asli maka tidak ada nilai y yang memenuhi sehingga diperoleh P(8, y) = 0. Jadi, untuk kasus ini tidak ada jawaban yang memenuhi. v. Jika x = 9 maka diperoleh f(9, y) = (9 – y)2 + 92 – 15(9) + 50 = (9 – y)2 – 4. Oleh karena itu, agar f(9, y) = 0 maka haruslah (9 – y)2 = 4, Akibatnya, 9 – y = 2 atau 9 – y = –2, y = 7 atau y = 11. Jadi, diperoleh dua jawaban yaitu (9, 7) dan (9, 11). Akhirnya, himpunan H memiliki 6 anggota berpasangan, yaitu: H = {(5, 5), (6, 4), (6, 8), (9, 7), (9, 11), (10, 10)}. Sehingga, banyaknya himpunan bagian dari H memenuhi persamaan: | 186
𝐵𝐵𝑎𝑎𝑛𝑛𝑦𝑦𝑦𝑦𝑦𝑦𝑦𝑦𝑦𝑦𝑦𝑦 ℎ𝑖𝑖𝑚𝑚𝑝𝑝𝑢𝑢𝑢𝑢𝑢𝑢𝑢𝑢 𝑏𝑏𝑎𝑎𝑔𝑔𝑖𝑖𝑖𝑖𝑖𝑖 ℎ𝑖𝑖𝑚𝑚𝑝𝑝𝑢𝑢𝑢𝑢𝑢𝑢𝑢𝑢 𝐻𝐻 = 2𝑛𝑛
Untuk n = 6, maka,
𝐵𝐵𝑎𝑎𝑛𝑛𝑦𝑦𝑦𝑦𝑦𝑦𝑦𝑦𝑦𝑦𝑦𝑦 ℎ𝑖𝑖𝑚𝑚𝑝𝑝𝑢𝑢𝑢𝑢𝑢𝑢𝑢𝑢 𝑏𝑏𝑎𝑎𝑔𝑔𝑖𝑖𝑖𝑖𝑖𝑖 ℎ𝑖𝑖𝑚𝑚𝑝𝑝𝑢𝑢𝑢𝑢𝑢𝑢𝑢𝑢 𝐻𝐻 = 26
𝐵𝐵𝑎𝑎𝑛𝑛𝑦𝑦𝑦𝑦𝑦𝑦𝑦𝑦𝑦𝑦𝑦𝑦 ℎ𝑖𝑖𝑚𝑚𝑝𝑝𝑢𝑢𝑢𝑢𝑢𝑢𝑢𝑢 𝑏𝑏𝑎𝑎𝑔𝑔𝑖𝑖𝑖𝑖𝑖𝑖 ℎ𝑖𝑖𝑚𝑚𝑝𝑝𝑢𝑢𝑢𝑢𝑢𝑢𝑢𝑢 𝐻𝐻 = 64
Demikian penjelasan kelompok kami atas persolaan nomor 1. Penyelesaian atas perrsoalan berikutnya akan disampaikan oleh Saver. Silahkan Kawan!”terang Amer mengakhiri penjelasannya. “Plok, plok, plok, plok,”terdengar tepukan tangan meriah kawan-kawan Amer. “Terima kasih, Kawan-kawan,”balas Amer. Amer
beranjak
mundur,
kini
giliran
Saver
bertugas
menerangkan kepada kawan-kawannya tentang persoalan berikutnya. “Terhadap persoalan nomor 2 ini, yang merupakan Soal Nomor 1 pada hari kedua, ada beberapa hal yang menjadi kata kunci,”kalimat pembuka Saver. Bintang, Fateh, Ikram, Ayra, Bily, Yodha, dan Denan dan Iman serius memerhatikan. Sementara Sabiq lebih memilih memejamkan kedua matanya. Itu biasa yang dia lakukan bila sedang serius mendengar dan menyimak suatu keterangan penting. “Kata kunci yang aku maksud adalah penggalan frasa, ‘pola yang sama, yakni setiap sel masing-masing spesies membelah diri menjadi dua sel dan kemudian setiap sel amoeba memangsa tepat satu sel bakteri’. Dan ‘pada hari ke-100 ternyata bakteri punah’,”terang Saver. Kesepuluh temannya tampak makin serius. Sabiq masih saja memejamkan kedua matanya. | 187
“Mari kita berandai-andai untuk persoalan ini! Jumlah amoeba pada hari pertama
= a;
Jumlah bakteri pada hari pertama
= b;
Jumlah amoeba pada hari ke-i
= Ai; dan
Jumlah bakteri pada hari ke-i
= Bi
Maka dari informasi persoalan yang diajukan kita bisa menuliskan untuk keadaan amoeba sebagai berikut: = 20a
A1
=a
A2
= 2A1 = 21a
A3
= 2A2 = 22a
= 4a
A4
= 2 A3 = 23a
= 8a
.
=.
=.
=.
.
=.
=.
=.
.
=.
=.
=.
A100
= 2 A99 = 299a
……………… (13.3)
Dan kita bisa menuliskan untuk keadaan bakteri sebagai berikut: B1= b
= 20b
B2= 2B1 – 2a
= 21b – 21a
= 21b – 1.21a
B3= 2B2 – 4a
= 22b – 4a – 4a
= 22b – 2.22a
B4= 2B3 – 8a
= 23b – 8a – 8a – 8a
= 23b – 3.23a
.= .
=.
.= .
=.
B100= 299b – 99.299a
………… (13.4)
Selanjutnya, dari informasi soal dinyatakan bahwa ‘pada hari ke-100 ternyata bakteri punah’, yang dalam bentuk persamaan dapat dinyatakan sebagai berikut: | 188
Bakteri punah ⇔ B100 = 0
……………… (13.5)
Sehingga, dari persamaan (13.4) dan persamaan (13.5) dapat dibuat persamaan baru yang menyatakan kepunahan bakteri sebagai:: 299b – 99.299a 99
=0
99
2 b = 99.2 a b = 99 a 1 𝑎𝑎 = 𝑏𝑏 99 Oleh karena itu, perbandingan antara jumlah amoeba dengan jumlah bakteri pada hari pertama adalah: a:b
= 1 : 99
Demikian bagian pertama dari tugas yang dibebankan kepada kelompok kami, Kawan-kawan,”pungkas Saver. “Subhanallah! Membutuhkan kejelian dan kesabaran tinggi, Kawan,”komentar Bily. “Ya, sampai ditemukan polanya!”timpal Ayra. Kali ini tampak Sabiq tersenyum dan telah membuka kedua matanya. Dan di wajahnya, terpampang seulas senyum. Senyum kepuasan. “Satu tahap pembuka telah kita lalui dengan baik. Berikutnya, kelompok siapakah?”tanya Sabiq pada kawan-kawannya. “Biar kelompok kami, aku Ikram, dan mencoba soal berikutnya, Kawan!”tiba-tiba Denan menyambut tawaran Sabiq. “Silahkan
Kawanku,
waktu
dan
tempat
menjadi
milikmu!”balas Sabiq. Denan melangkah pasti. Diraihnya alat tulis dari tangan Saver. Dan dia memulai aksinya. | 189
“Pada prinsipnya, tugas kami adalah soal nomor 2 baik pada hari pertama maupun hari kedua. Kedua soal itu adalah sebagai berikut: Pertama. Seorang pesulap menyatakan dirinya ahli menebak pikiran dengan pertunjukkan berikut. Salah seorang penonton awalnya diminta secara tersembunyi menuliskan sebuah bilangan lima angka, lalu menguranginya dengan jumlah angka-angka penyusun bilangan tersebut, kemudian menyebutkan empat dari lima angka penyusun bilangan hasil (dengan urutan sebarang). Selanjutnya pesulap tersebut dapat menebak angka yang masih disembunyikan. Sebagai contoh, jika penonton menyebutkan empat bilangan hasil: 0, 1, 2, 3, maka pesulap akan tahu bahwa angka yang disembunyikan adalah 3. (a) Berilah suatu contoh Anda sendiri dari proses di atas; (b) Jelaskan secara matematis bentuk umum dari proses tersebut! Kedua. Diketahui n adalah bilangan bulat positif. Jika 𝑓𝑓(𝑛𝑛) =
4𝑛𝑛 + √4𝑛𝑛2 − 1
√2𝑛𝑛 + 1 + √2𝑛𝑛 − 1
Tentukan f(13) + f(14) + f(15) + … + f(112). Begitu redaksi kedua soal yang akan kami selesaikan. Untuk yang pertama, aku sendiri yang akan mengerjakan, sementara persoalan kedua, Kawan kita Ikram yang akan menjelaskannya pada kita semua,”Denan berkata dengan jelas. Dengan sifat dan karakter Denan yang periang dan sering bercanda serta jarang tampak serius, tampaknya kawan-kawannya was-was dan memasang mata penasaran.
| 190
“Bagaimana kawan kami ini akan menjelaskan persoalan OSN ini?”kira-kira begitu pikir mereka. Hanya Sabiq dan Yodha yang tampak tenang dan tak menunjukkan ekspresi heran. “Kami sepakat langsung menjawab bagian pertanyaan b, karena bagian a adalah konsekuensi logis dari pemahaman kita atas inti persoalan yang sebenarnya terekam dalam pertanyaan b,”papar Denan. “Wah, agak unik juga cara kalian ini, Kawan,”komentar Amer. “Sabar dulu, Mer. Kita beri kesempatan Denan menjelaskan apa maksud dari kata-katanya itu,”potong Yodha cepat. Denan menarik nafas dalam-dalam. Udara tipis yang mulai hangat karena mentari mulai tinggi, ditarik dalam-dalam olehnya. “Penjelasan matematis aksi pesulap di atas sesungguhnya merupakan operasi aljabar keterbagian atau dalam bahasan matematikanya sering disebut sebagai divisibility,”terang Denan. “Andai kita misalkan bilangan awal sebelum proses adalah AWAL dan bilangan hasil dari setelah proses adalah AKHIR, maka dari informasi persoalan akan diperoleh bahwa AWAL AKHIR
�������� = 1000a + 1000b + 100c + 10d + e = 𝑎𝑎𝑏𝑏𝑐𝑐𝑑𝑑𝑑𝑑
= 1000a+ 1000b + 100c + 10d + e – (a+b+c+d+e) = 9999a + 999b + 99c + 9d
………….. (13.4)
Dari persamaan (1) di atas terlihat bahwa bilangan AKHIR merupakan bilangan kelipatan 9. Berdasarkan sifat dari bilangan kelipatan 9 dimana “jumlah digit-digit dari penyusunnya harus habis dibagi 9”, maka contoh bilangan lima angka yang lain, dapat
| 191
disebutkan adalah: “Bila penonton menyebut angka 7, 9, 2, dan 4, maka bilangan yang disembunyikan adalah angka 5, karena: = 5 + (7 + 9 + 2 + 4) = 27 Karena, 2 + 7 = 9. Itu penjelasan yang mampu kami sajikan. Sekaligus menjawab pertanyaan a dan b,’terang Denan. “Jadi konkretnya, apa Nan?”tanya Amer. Denan tercengang. Dia agak bingung dengan pertanyaan yang diajukan oleh Amer. “Ya, konkretnya adalah: atas pertanyaan bagian a yang menyebutkan, ‘berilah suatu contoh Anda sendiri dari proses di atas,?’ jawaban kami adalah, ‘bila penonton menyebut angka 7, 9, 2, dan 4, maka bilangan yang disembunyikan adalah angka 5’. Dan terhadap pertanyaan bagian b yang menyatakan, ‘jelaskan secara matematis bentuk umum dari proses tersebut?’ jawaban kami adalah, ‘bahwa bilangan AKHIR merupakan bilangan kelipatan 9, yang berdasarkan sifat dari bilangan kelipatan 9 itu maka ‘jumlah digitdigit dari penyusunnya harus habis dibagi 9’. Itu adalah penjelasan matematisnya, Kawan-kawan sekalian,”terang Amer. “Ooo, begitu. Pahamlah aku. Nah, kalau aku sebut 4 angka dari bilangan lima angkanya adalah 5183, berapa yang aku sembunyikan, Nan?”tanya Amer lagi penasaran. “Karena 5+1+8+3 = 17, maka angka yang kau sembunyikan adalah 1! Karena 17 + 1 = 18. Dan 8 +1 = 9. Angka itu adalah kelipatan 9, Kawan!”terang Denan bersemangat.
| 192
“Plok, plok, plok, plok, plok!”terdengar tepuk tangan meriah dari kawan-kawan Denan dan Amer yang ada di ruangan itu. Sementara Yodha dan Sabiq senyum-senyum dengan peristiwa Amer–Denan barusan. “Selanjutnya,
persoalan
berikut
akan
dipaparkan
oleh
Ikram!”terang Denan. Anak laki-laki yang dipanggil Ikram itu maju. Ia membetulkan latak kacamata minusnya, memandang lurus ke depan. Dan mantap menerima alat tulis dari Denan. ‘Kawan-kawan, untuk persoalan ini, tidak ada kata yang tepat selain, coba dulu dan jangan putus asa serta khawatir dengan yang terlihat. Lupakan bentuknya, fokus pada substansinya,”terang Ikram membuka pembahasan. “Kau tampaknya mau menapaki jalur Sastrawan juga, Kawan!”sindir Amer. Ikram tetap tenang. Ia menarik nafas dalam-dalam. “Terhadap fungsi yang menyatakan bahwa: 𝑓𝑓(𝑛𝑛) =
4𝑛𝑛 + √4𝑛𝑛2 − 1
√2𝑛𝑛 + 1 + √2𝑛𝑛 − 1
Perlu terlebih dahulu dipahami apa yang disebut sebagai Sekawan atau Conjugate dalam operasi bilangan akar. Fungsi di atas perlu disederhanakan terlebih dahulu dengan cara mengalikan dengan Sekawannya, yaitu: 𝑓𝑓(𝑛𝑛) =
4𝑛𝑛 + √4𝑛𝑛2 − 1
√2𝑛𝑛 + 1 + √2𝑛𝑛 − 1
𝑥𝑥
√2𝑛𝑛 + 1 − √2𝑛𝑛 − 1 √2𝑛𝑛 + 1 − √2𝑛𝑛 − 1
Dengan memerhatikan sifat perkalian:
… … . . (13.5) | 193
(a + b) (a – b)
= a2 – b2
Maka bagian penyebut perkalian pada persamaan (13.5) dapat disederhankan menjadi: = �√2𝑛𝑛 + 1 + √2𝑛𝑛 − 1�𝑥𝑥(√2𝑛𝑛 + 1 − �2𝑛𝑛 − 1) = �(√2𝑛𝑛 + 1)2 − (√2𝑛𝑛 − 1)2 � = (2𝑛𝑛 + 1) − (2𝑛𝑛 − 1)
=2
Sehingga bentuk persamaan (13.5) dapat ditulis menjadi: 4𝑛𝑛 + √4𝑛𝑛2 − 1(√2𝑛𝑛 + 1 − √2𝑛𝑛 − 1) … … (13.6) 2 Perhatikan bahwa bentuk berikut adalah sama! 𝑓𝑓(𝑛𝑛) =
= �4𝑛𝑛2 − 1 = �4𝑛𝑛2 − 12
= �4𝑛𝑛2 − 1 = �(2𝑛𝑛 + 1)(2𝑛𝑛 − 1)
Karenanya, persamaan (2) dapat diuraikan menjadi: =
4𝑛𝑛(√2𝑛𝑛 + 1 − √2𝑛𝑛 − 1) + �(2𝑛𝑛 + 1)(2𝑛𝑛 − 1)(√2𝑛𝑛 + 1 − √2𝑛𝑛 − 1) 2
=
2𝑛𝑛√2𝑛𝑛 + 1 − 2𝑛𝑛√2𝑛𝑛 − 1 + √2𝑛𝑛 − 1 + √2𝑛𝑛 + 1 2
=
4𝑛𝑛�√2𝑛𝑛 + 1 − √2𝑛𝑛 − 1� + (2𝑛𝑛 + 1)√2𝑛𝑛 − 1 − (2𝑛𝑛 − 1)√2𝑛𝑛 + 1 2
(2𝑛𝑛 + 1)√2𝑛𝑛 + 1 − (2𝑛𝑛 − 1)√2𝑛𝑛 − 1 … … … . . … … (13.7) 2 Bentuk akhir persamaan fungsi pada persoalan adalah sebagaimana
=
tertuang dalam persamaan (13.7),”terang Ikram, dan dia berhenti. Ikram beranjak ke arah samping, ia mengambil segelas air mineral dan meneguknya. Kemudian dia kembali pada posisi semua, melanjutkan penjelasannya. | 194
“Selanjutnya adalah tahap substitusi besaran-besaran yang ditanyakan dan penyederhanaan bentuk akhir persamaan, sehingga: 𝑓𝑓(13) =
27√27 − 25√25 2
𝑓𝑓(15) =
31√31 − 29√29 2
𝑓𝑓(14) = .
29√29 − 27√27 2
𝑓𝑓(111) =
𝑓𝑓(112) =
223√223 − 221√221 2
225√225 − 223√223 2
Sehingga bentuk akhir dari:
𝑓𝑓(13) + 𝑓𝑓(14) + 𝑓𝑓(15)+. . +𝑓𝑓(111) + 𝑓𝑓(112) =
225√25 − 25√25 2
𝑓𝑓(13) + 𝑓𝑓(14) + 𝑓𝑓(15)+. . . +𝑓𝑓(111) + 𝑓𝑓(112) = 1.625
Demikian penjelasan kelompok kami, Kawan-kawan,”pungkas Ikram. “Indah, solusi yang indah. Ketelitian dan ketekunan yang
berbuah kesederhanaan yang indah, Kawan-kawan,”komentar Sabiq. “Kali ini aku sepakat dengan komentar Sastrawan kita,”timpal Fateh. Semua yang ada di ruangan itu tampaknya setuju dengan komentar Sabiq dan persetujuan Fateh. Wajah-wajah mereka menampilkan kepuasaan berbalut kekaguman. | 195
“Baiklah, semakin terasa hangat udara dalam ruangan ini. Giliran siapa berikutnya?”tanya Sabiq. “Izinkan kami menyelesaikan bagian berikutnya. Hanya saja, kami baru yakin untuk 1 persolan, sementara soal yang kedua, masih kami telaah lebih teliti,”tiba-tiba Yodha angkat bicara. “Maksudmu apa, kawan?”tanya Ayra heran. “Ya, dua persoalan yang menjadi bagian kami untuk menyelesaikannya itu adalah soal geometri dan kombinatorik. Untuk yang pertama kami berkeyaninan bisa memecahkan dan memahami panduan yang diberikan oleh Pak Tutur. Tapi, untuk persoalan kedua, kami masih mendalaminya,”terang Yodha. “Kalau begitu, yang sudah selesai saja ajukan dan sajikan kepada kita, Kawan! agar efektif dan waktu kita cukup,”usul Ikram. “Aku setuju dengan usul Ikram,”balas Bintang. “Oke, kalau forum setuju, silahkan Kawan!”Sabiq meminta Yodha ke depan. “Terima
kasih.
Kawan-kawan,
Mohon
maaf
atas
kekurangsiapan kami,”Yodha memulai. Yodha bergerak maju, ia meraih alat tulis yang diberikan Ikram. Menarik nafas dalam-dalam, dan… “Persoalan yang harus kami selesaikan adalah soal nomor 3 pada hari pertama dan soal nomor yang sama pula di hari kedua. Kedua soal itu adalah: (1) Pada suatu keranjang buah terdapat 20 Apel, 18 Jeruk, 16 Mangga, 10 Nenas dan 6 Pepaya. Jika seseorang ingin mengambil 10 buah dari keranjang tersebut, ada berapa banyak komposisi buah terambil yang mungkin? | 196
(2) Budi menyusun empat belas buah bola masing-masing berjari-jari 10 cm. Sembilan buah bola pertama diletakkan di atas meja sedemikian sehingga membentuk persegi dan saling bersinggungan. Empat buah bola berikutnya diletakkan di atas sembilan bola pertama sehingga saling bersinggungan. Bola keempat belas ditaruh di atas empat bola tadi, sehingga menyinggung empat bola tersebut. Jika Bambang mempunyai lima puluh lima buah bola yang masing-masing juga berjarijari 10 cm dan semua bola tersebut disusun mengikuti pola yang dilakukan Budi, hitungah ketinggian pusat bola yang paling atas diukur dari permukaan meja pada susunan bola yang dilakukan Bambang,”jelas Yodha Yodha diam, terpaku beberapa saat. Tak lama. Dalam hitungan detik, dia kembali menggerakkan tangannya. Dia menggambar. “Terhadap persoalan nomor 2 di atas, hal pertama yang harus kita lakukan adalah berusaha menggambarkannya dalam bentuk yang sederhana. Dan gambaran yang paling tepat untuk itu adalah gambaran tentang susunan bola Billiard, bukankah begitu, Mer?”seru Yodha tiba-tiba. “Lho, kok tanya aku?”jawab Amer heran. Semua kawan Amer hanya senyum-senyum, mereka paham betul apa maksud dari pertanyaan Yodha. “Bukankah diantara kita kau ahlinya dalam soal bermain bola Billiard, Kawan?”kata Yodha lagi. “Ya, itu sungguh-sungguh aku akui. Tapi, apa maksudmu dengan ‘susunan bola Billiard’, Kawan?”tanya Amer serius. “Ya, saat permainan bola Billiard dimulai, biasanya dilakukan dengan menyusun kelima belas bola sedemikian rupa, sehingga membentuk susunan sedemikian rupa. Dan yang terpenting dari | 197
susunan-susunan bola itu, bahwa semua bola bersinggungan dan tidak terdapat sela pada bola-bola yang terletak di pinggir. Tapi...!”terang Yodha sambil menggambar sesuatu di papan tulis. “Tapi apa, Kawan?”tanya Amer. “Tapi pada persinggungan empat bola itu, terdapat ruang sedemikian rupa. Betulkan, Mer!”tanya Yodha balik. “Ah, ternyata kau tahu banyak soal formasi bola Billiard, Kawan. Apa yang kau jelaskan itu benar adanya,”balas Amer. “Nah, Kawan-kawan. Disinilah kata kuncinya. Ruang yang terbentuk berupa sela-sela antara 4 bola yang disusun bersinggungan itu selanjutnya akan diisi bola-bola pada tingkatan berikutnya, sampai jumlahnya 55 bola, bola yang disusun oleh Bambang itu,”terang Yodha lagi. Tampak kesepuluh kawan Yodha serius memerhatikan. Mereka memang menaruh tingkat kepercayaan yang tinggi pada Yodha bila terkait soal-soal geometri. Yodha kembali serius ke papan tulis. “Perhatikan gambar-gambar berikut ini. Kelima susunan bola dengan formasi itu adalah:
Bola-bola itu kita susun sedemikian rupa, dimulai mulai dari formasi 5x5 bola, berturut-turut adalah: 5x5 susunan dasar; 4x4 susunan | 198
kedua; 3x3 susunan ketiga; 2x2 susunan keempat; dan formasi 1bola sebagai susunan puncak,”terang Yodha lagi. “Selanjutnya, bila kita perhatikan persoalannya bahwa, ‘Bambang mempunyai lima puluh lima buah bola’, dan ‘semua bola tersebut disusun mengikuti pola yang dilakukan Budi’, maka dari kedua keterangan ini dapat dipastikan bahwa: (1) Pola susunan ke-55 bola mengikuti pola bilangan kuadrat sempurna (perfect square number), yaitu: 12+22+32+42+52 = 55; (2) Bentuk akhir susunan ke-55 bola itu adalah, sebagai berikut:
Terlihat bahwa susunan itu membuat limas dengan alas berbentuk bujur sangkar. Sehingga bila kita lihat bentuk kerangka dasar dan susunan di atas dari samping, akan terlihat sebagai berikut:
Sehingga, dari gambar kerangka dasar bahwa bentuk dasarnya adalah segi empat berupa bukur sangkar, karena merupakan susunan dari 5 | 199
bola di pada masing-masing sisinya yang saling bersinggungan. Sementara TM adalah tinggi Limas, yang merupakan garis tegak lurus terhadap diagonal sisi alas limas,”papar Yodha. Kawan-kawannya masih tampak sangat serius. “Hanya saja, ada satu hal yang harus diperhatikan, yaitu bahwa tinggi limas pada bentuk kerangka dasar, garis TM, tidaklah sama dengan ketinggian pusat bola yang paling atas diukur dari permukaan meja. Untuk mencapai permukaan meja maka panjang TM pada susunan bola yang dilihat dari samping, haruslah ditambahkan panjang ME, jari-jari 1 lingkaran lagi. Sehingga, secara matematis akan bisa kita hitung sebagai berikut: Tinggi pusat bola diukur dari permukaan meja = TE TE = TM + ME Dari ΔABC pada kerangka dasar, dengan dalil Phytagoras: 𝐴𝐴𝐶𝐶 2 = 𝐴𝐴𝐵𝐵2 + 𝐵𝐵𝐶𝐶 2
𝐴𝐴𝐶𝐶 = √𝐴𝐴𝐵𝐵2 + 𝐵𝐵𝐶𝐶 2
𝐴𝐴𝐶𝐶 = √802 + 802 𝐴𝐴𝐶𝐶 = 80√2 cm 1
𝐶𝐶𝑀𝑀 = �80√2� 2 𝐶𝐶𝑀𝑀 = 40√2 cm
Dari ΔCMT pada tampak samping, dengan dalil Phytagoras: 𝑇𝑇𝑀𝑀2 = 𝑇𝑇𝐶𝐶 2 − 𝐶𝐶𝑀𝑀2
2
𝑇𝑇𝑀𝑀2 = 802 − �40√2�
𝑇𝑇𝑀𝑀2 = 6400 − 3200 𝑇𝑇𝑀𝑀2 = 3200
| 200
𝑇𝑇𝑀𝑀 = 40√2 cm
Akhirnya, tinggi pusat bola diukur dari permukaan meja adalah: TE = TM + ME TE = (40√2+10) cm Demikian penjelasan dan gambaranku ,”terang Yodha pada kawankawannya. “Subhanallah!
Aku
sampai
berkeringat
mengikuti
penjelasanmu, Yodha!”komentar Amer. “Aku pun harus memasang telinga dan mataku seserius mungkin, Kawan!”timpal Saver. Fateh, Ayra, Denan, Bintang pun mengangguk. Mereka tampaknya sepakat butuh konsenterasi dan perhatian lebih dalam soal geometri yang dipresentasikan oleh Yodha kali ini. “Terima kasih, Kawan-kawan. Itu pulalah alasan mengapa kami tak punya cukup waktu menyelesaikan soal tentang Kombinatorik.
Kami
berharap
kita
sama-sama
menyelesaikannya,”terang Yodha lagi. “Oke, aku pikir hal itu sudah kita bicarakan, dan kita telah pula sepakat tentang itu. Tapi, satu hal yang pasti, penjelasan kelompok ini terhadap soal geometri dalam soal yang diajukan, sungguh luar biasa dan pantas mendapat applause kita bersama!”kata Sabiq “Plok, plok, plok, plok, plok,”terdengar tepukan meriah dari teman-teman Yodha. “Terima kasih, Kawan-kawan,”balas Yodha dan Bily. Waktu terus bergerak, matahari semakin meninggi. Kesebelas anak itu masih serius dan bersemangat. Kicauan burung dan sesekali | 201
suara rumpun dan daun bambu diterpa angin berpadu dengan rasa dingin yang semakin hilang dan berganti hangat di kawasan itu. “Berikutnya, giliran siapa yang akan menyelesaikan persoalan nomor 4?”tanya Sabiq. “Kami,
kelompok
kami
insyaallah
akan
mencoba
menguraikan persoalan itu. Dan ini murni persoalan Geometri baik pada hari pertama, maupun hari kedua. Karenanya kami mohon maaf bila tak bisa memenuhi harapan. Apalagi kepada kawan kita Yodha yang ahlinya,”ucap Ayra. “Tak ada masalah, Ayra. Seperti selalu aku sampaikan bahwa kita sama-sama belajar di sini. Aku akan sangat berterima kasih bila kita semua memiliki pemahaman yang lebih baik lagi tentang apa saja, Geometri khususnya,”balas Yodha datar. “Selanjutnya, Bintang akan terlbih dahulu memaparkan kita. Silahkan, Bintang!”pinta Ayra “Baik, terima kasih Ayra. Kawan-kawan. kelompok kami mendapat kehormatan menyelesaikan persoalan terkait geometri. Kedua persoalan itu adalah sebagai berikut: (1) Di dalam Taman Khatulistiwa akan dibuat bangunan berbentuk limas dengan alas segitiga sama sisi berbahan tembus pandang dengan panjang sisi alas 8√3 m dan tinggi 8 m. Sebuah bola dunia akan ditempatkan di dalam limas tersebut. Dengan mengabaikan ketebalan bahan pembuat limas, tentukan panjang terbesar jari-jari bola dunia yang mungkin dapat dibuat; (2) Diketahui sebuah segitiga ABC dengan panjang sisisisinya adalah 5 cm, 8 cm dan √41 cm. Tentukan luas maksimum persegi panjang yang mungkin dapat dibuat di dalam segitiga ABC tersebut,”kata Bintang | 202
Bintang membetulkan letak alat tulis di tangannya. Kali ini dia memutar-mutar alat tulis itu disela-sela jari-jarinya. “Setelah kami telaah, prioritas pengerjaannya adalah soal kedua, baru setelah itu soal pertama. Hal ini untuk memudahkan, karena tampaknya terdapat hubungan yang agak mirip antara kedua soal ini,”terang Bintang lagi. “Kalau kita perhatikan sungguh-sungguh, maka kata kunci pada “maksimum”. Sebuah persegi panjang atau apapun yang terbentuk dalam suatu bangun apapun, segitiga misalnya, akan memiliki luas maksimum jika ukurannya maksimum. Dan ukuran maksimum itu terjadi bila ukurannya terletak pada sisi bangun induknya,”terang Bintang “Penjelasanmu berbelit-belit, Kawan!”potong Amer. “Maaf, nanti akan jelas dengan gambar, Mer. Dari soal di atas Intinya adalah, bahwa luas maksimum persegi panjang yang mungkin dapat dibuat di dalam segitiga, bila salah satu sisi persegi panjangnya berhimpit dengan salah satu sisi segitiga, dan titik sudut di sisi yang berlawanan dengan sisi yang berhimpit tersebut menyinggung dua sisi segitiga yang lain. Untuk lebih jelasnya perhatikan gambar di bawah ini!”terang Bintang lagi.
| 203
“Segitiga yang dimaksud oleh soal di atas adalah ΔABC dengan AB = 5 cm, BC = 8 cm, dan AD = √41 cm. Garis AD merupakan garis tinggi dapat ditentukan dengan memanfaatkan sifatnya sendiri, yaitu sebagai garis yang melalui sebuah titik sudut dan tegaklurus pada sisi di hadapan titik sudut tersebut,”terang Bintang lagi. “Karenanya garis AD tegaklurus terhadap garis BC, sehingga dengan dalil Phytagoras dapat ditentukan dari ΔABD dan ΔACD: AB2 – BD2 = AC2 – DC2 52 - BD2 = (√41)2 – (BC – BD)2 25 – BD2 = 41 – (8 – BD)2 25 – BD2 = 41 – 64 + 16BD – BD2 16 BD = (25+64) – 41 BD = (48)/16 BD = 3 cm. Selanjutnya dapat ditentukan panjang garis AD, dimana dengan memanfaatkan segitiga siku-siku ABD, maka: AD2 = AB2 – BD2 AD2 = 52 – 32 AD = 4 cm Sehingga tinggi ΔABC adalah 4 cm,”papar Bintang. Sesaat Bintang menarik nafas dalam-dalam, mengeluarkannya, dan kembali ke depan papan tulis. | 204
“Selanjutnya, kita ke pokok persoalan yang utama, yaitu, ‘menentukan luas maksimum persegi panjang yang mungkin dapat dibuat di dalam segitiga ABC’. Misalkan persegi panjang itu adalah EFQR sepeti pada gambar sebelumnya, dan panjang dan lebar dari persegi panjang yang dicari berturut-turut adalah p dan l. Dimana, berdasarkan gambar terlihat: QR = EF = panjang = p EQ = FR = lebar = l Dengan dalil kesebangunan pada segitiga, karena EF sejajar BC maka mudah dibuktikan bahwa ΔAEP ~ ΔEBQ dan ΔAPF ~ ΔFRC. Oleh karena itu diperoleh: 𝐸𝐸𝑃𝑃 𝐴𝐴𝑃𝑃 4 − 𝑙𝑙 = = … … … … … … … … … … . . … … (13.8) 𝐵𝐵𝑄𝑄 𝐸𝐸𝑄𝑄 𝑙𝑙
𝑃𝑃𝐹𝐹 𝐴𝐴𝑃𝑃 4 − 𝑙𝑙 = = … … … … … … … … … … … … … (13.9) 𝑙𝑙 𝑅𝑅𝐶𝐶 𝐹𝐹𝑅𝑅 Sehingga dari persamaan (13.8) dan persamaan (13.9) akan diperoleh persamaan: 4 − 𝑙𝑙 𝐸𝐸𝑃𝑃 𝑃𝑃𝐹𝐹 𝐸𝐸𝑃𝑃 + 𝑃𝑃𝐹𝐹 𝑝𝑝 = = = = … … … (13.10) 𝑙𝑙 𝐵𝐵𝑄𝑄 𝑅𝑅𝐶𝐶 𝐵𝐵𝑄𝑄 + 𝑅𝑅𝐶𝐶 8 − 𝑝𝑝
Dan selanjutnya, …”tiba-tiba Bintang hentikan penjelasannya. “Tunggu dulu kawan! bagaimana kau menjelaskan persamaan (13.10) khususnya pada bagian persamaan ini! 𝐸𝐸𝑃𝑃 𝑃𝑃𝐹𝐹 𝐸𝐸𝑃𝑃 + 𝑃𝑃𝐹𝐹 = = 𝐵𝐵𝑄𝑄 𝑅𝑅𝐶𝐶 𝐵𝐵𝑄𝑄 + 𝑅𝑅𝐶𝐶
Kenapa pembilang ditambah pembilang dibagi dengan penyebut ditambah penyebut, untuk nilai perbandingan yang sama, hasilnya
| 205
tetap sama? Sebaiknya jelaskan dengan contoh, Kawan!”potong Amer penasaran. Bintang melirik ke arah Ayra. Ayra tersenyum dan memberi anggukan pelan. “Baik, mari kita mulai dengan contoh. Kita berandai andai! Misalkan,
Dan
𝐸𝐸𝑃𝑃 2 = 𝐵𝐵𝑄𝑄 3
𝑃𝑃𝐹𝐹 6 = 𝑅𝑅𝐶𝐶 9
Maka nilai dari penjumlahan pembilang-tambah-pembilang dan penyebut-tambah-penyebut, dengan perbandingan yang sama ini adalah: 𝐸𝐸𝑃𝑃 + 𝑃𝑃𝐹𝐹 2 + 6 8 2 = = = 𝐵𝐵𝑄𝑄 + 𝑅𝑅𝐶𝐶 3 + 9 12 3
Dan nilai akhir dari penjumlahan itu adalah sama dengan nilai awal dari masing-masing pecahan yang dijumlahkan, yaitu 2/3!”terang Bintang “Bisa contoh lain?”tanya Amer masih penasaran. “Silahkan, Kawan!”balas Bintang. “Bagaimana kalau penyebut dan pembilang dari tujuh per empat belas ditambah sembilan per delapan belas?”tanya Amer. “Dengan prinsip pembilang-tambah-pembilang dan penyebuttambah-penyebut, karena nilai perbandingannya adalah senilai, maka jawabnya pasti tetap seperti nilai perbandingan pertama 1/2 kawan. Lihat saja!”tantang Bintang. | 206
Bintang menulis penjumlahan yang dimaksud oleh Amer. “Mari, sama-sama kita perhatikan! 16 7+9 = 14 + 18 32
Bukankah nilai dari penjumlahan di atas adalah: 7 9 1 16 = = = 32 14 18 2
Atau bisa ambil nilai lain selain ½. Misalnya: 9 3 𝑑𝑑𝑎𝑎𝑛𝑛 12 4 Sehingga,
12 3 3+9 = = 4 + 12 16 4
Asalkan saja dengan perbandingan yang sama, maka pembilangtambah-pembilang dan penyebut-tambah-penyebut, akan tetap hasilnya! Ini sisi lain dari kesebangunan,”tegas Bintang. “Baik, aku sudah bisa memahaminya, Kawan. Terima kasih,”balas Amer. Terlihat kawan-kawan mereka hanya tersenyum melihat rasa penasaran Amer. “Baik kita kembali ke inti persoalan. Dari persamaan (13.10), akhirnya kita dapat menemukan persamaan dalam bentuk panjang (p) dan lebar (l), yaitu: (4 − 𝑙𝑙)(8 − 𝑝𝑝) = 𝑝𝑝𝑙𝑙
32 − 4𝑝𝑝 − 8𝑙𝑙 + 𝑝𝑝𝑙𝑙 = 𝑝𝑝𝑙𝑙
4𝑝𝑝 = 32 − 8𝑙𝑙
𝑝𝑝 = 8 − 2𝑙𝑙 … … … … … … . … … . . (13.11)
| 207
Selanjutnya kita tentukan inti pertanyaannya, yaitu ‘tentukan luas maksimum persegi panjang yang mungkin’, dengan cara mengalikan panjang (p) dan lebar (l), sehingga diperoleh persamaan: 𝐿𝐿𝑢𝑢𝑎𝑎𝑠𝑠 = 𝑝𝑝𝑥𝑥𝑙𝑙
𝐿𝐿𝑢𝑢𝑎𝑎𝑠𝑠 = (8 − 2𝑙𝑙)𝑙𝑙
𝐿𝐿 = 8𝑙𝑙 − 2𝑙𝑙 2 … … … … … … … . … . . (13.12)
Terlihat bahwa persamaan luas di atas (13.12), merupakan fungsi kuadrat dalam l, yaitu memenuhi
Untuk dapat
𝑓𝑓(𝐿𝐿) = 8𝑙𝑙 − 2𝑙𝑙 2
menemukan nilai ekstrim, maksimum atau minimumnya,
dilakukan
dengan
cara
menemukan
titik
puncak
persamaan,”tegas BIntang. “Selanjutnya, untuk fungsi kuadrat y(x)=ax² +bx+ c, diketahui bahwa memiliki koordinat titik puncak: −𝑏𝑏 −𝐷𝐷 ; 2𝑎𝑎 4𝑎𝑎 Dimana D adalah Diskriminan, dengan: 𝑇𝑇𝑖𝑖𝑡𝑡𝑖𝑖𝑖𝑖 𝑃𝑃𝑢𝑢𝑛𝑛𝑐𝑐𝑐𝑐𝑐𝑐 (𝑃𝑃) =
𝐷𝐷 = ꒄ 2 − 4𝑎𝑎𝑐𝑐
Dalam persoalan ini, di mana f(l) = 8l – 2l2 memenuhi fungsi kuadrat dalam l, dan diketahui bahwa nilai masing-masing besarannya adalah a = -2; b = 8, dan c = 0. Sehingga: −𝐷𝐷 −(82 − 4(−2)(0)) −64 = = =8 4(−1) 4𝑎𝑎 −4
−𝑏𝑏 −8 8 = = =2 2𝑎𝑎 2(−2) 4
Jadi, luas maksimum persegi panjang yang mungkin dibuat sebesar 8 cm2, yang terjadi pada saat lebarnya l = 2 cm, dan panjang p = 4 cm. | 208
Demikian penjelasan kelompok kami atas pertanyaan nomor 4 hari kedua. Selanjutnya Ayra akan menerangkan tentang penyelesaian atas soal nomor 4 hari pertama,”pungkas Bintang. “Wow, keren! Penyelesaian yang sederhana tapi jelas dan mudah dipahami, Kawan!”komentar Denan. “Terima kasih, Kawan!”balas Bintang datar. Ayra maju. Ia melangkah mantap. Dia melepaskan pandangan ke segala penjuru, ke arah kawan-kawannya yang memerhatikannya dengan serius. “Baik, terhaadap persoalan hari pertama nomor 4 yang menyatakan bahwa akan dibuat bangunan berbentuk limas dengan alas segitiga sama sisi berbahan tembus pandang dengan panjang sisi alas 8√3 m dan tinggi 8 m. Sebuah bola dunia akan ditempatkan di dalam limas tersebut. Dengan mengabaikan ketebalan bahan pembuat limas, tentukan panjang terbesar jari-jari bola dunia yang mungkin dapat dibuat’ adalah mirip dengan penyelesaian yang tadi dikerjakan oleh Bintang,”tegas Ayra. “Hanya saja, kali ini bentuknya adalah Lingkaran di dalam Limas, tapi intinya tetap sama yaitu, ‘menentukan panjang terbesar jari-jari bola dunia yang mungkin dapat dibuat’ sedemikian rupa,”terang Ayra lagi. “Karenanya, agar terpenuhi persyaratan panjang terbesar jarijari bola dunia yang mungkin dapat dibuat, maka harus dilukis sebuah bola yang jari-jari bola dunia akan maksimum jika bola menyinggung keempat bidang sisi limas. Sehingga, bila kita buat
| 209
sketsa sederhana Limas dengan alas dan tingginya, akan terlihat seperti pada gambar di bawah ini:
Titik D adalah proyeksi titik P terhadap bidang ABC sehingga PD tegak lurus dengan ΔABC. Dengan kata lain PD adalah tinggi Limas PABC. Dan dari soal diketahui bahwa PD = 8 m. Karena AD = BD = CD maka D adalah pusat lingkaran luar ΔABC. Karena ΔABC merupakan segitiga samasisi maka AE adalah median sekaligus altitude (tinggi) segitiga, dan D adalah pusat segitiga sehingga: 2 𝐴𝐴𝐷𝐷 = 𝐴𝐴𝐸𝐸 3
Dan karena ΔAEB merupakan segitiga yang memenuhi proporsi segitiga 300 – 600 – 900, maka kita peroleh:
dan
𝐴𝐴𝐸𝐸 =
√3 𝐴𝐴𝐵𝐵 2 3
𝐴𝐴𝐸𝐸 = �2 𝐴𝐴𝐵𝐵 … … … … … … … … … . (13.13)
Sehingga,
| 210
2 3 𝐴𝐴𝐷𝐷 = � 𝐴𝐴𝐵𝐵 3 2
𝐴𝐴𝐷𝐷 =
𝐴𝐴𝐷𝐷 =
√3 𝐴𝐴𝐵𝐵 3
√3 8√3 3
𝐴𝐴𝐷𝐷 = 8 𝑚𝑚
Langkah selanjutnya adalah menentukan jari-jari bola dunia yang maksimum panjangnya. Dan prinsip yang tadi berlaku untuk persoalan sebelumnya, bahwa agar maksimum harus menyentuh semua limas, berlaku pula dalam persoalam ini. Perhatikan gambar di bawah!
Misalkan jari-jari bola dunia adalah r, maka r = TD = TF. Untuk mencari panjang AE bisa menggunakan dalil Phytagoras pada ΔABE yaitu seperti pada persamaan (13.13), sehingga: 𝐴𝐴𝐸𝐸 =
𝐴𝐴𝐸𝐸 =
√3 𝐴𝐴𝐵𝐵 2
√3 8√3 2
𝐴𝐴𝐸𝐸 = 12 𝑚𝑚,
Oleh karena itu: DE = EF = 4 m,”terang Ayra. | 211
Ayra berhenti sejenak. Ia kembali mengatur posisi tubuhnya. Kembali ditariknya nafas dalam-dalam. Sejurus kemudian, ia siap melanjutkan. “Dari ΔPDE, dengan bantu Phytagoras, kita peroleh: PE2 = PD2 + DE2 PE2 = 82 + 42 PE2 = √80 PE = 4√5 m. Sehingga, PF = PE – EF PF = (4√5 – 4) m, dan PT = 8 - 𝑟𝑟
Akhirnya, dengan dalil Phytagoras kembali pada ΔPTF diperoleh: PT2 = PF2 + TF2 ⇔ (8 - r)2 = (4√5 - 4)2 + r2
⇔ 64 -16r + r2 = 16 (√5 – 1)2 + r2
⇔ 64 -16r + r2 = 16 (√5 – 1)2 + r2 ⇔ 64 -16r = 16 (√5 – 1)2
⇔ 16r = 64 -16 (√5 – 1)2 ⇔ r = 4 - (√5 – 1)2
Sehingga,
⇔ r = 4 - (5 – 2√5+1)
r = 4 - (5 – 2√5+1) r = (2√5 – 2) m | 212
Jadi, panjang jari-jari bola dunia maksimum adalah (2√5–2) meter,”terang Ayra mengakhiri keterangannya. Semua pasang mata yang hadir di ruang itu terbelalak! Ternyata Ayra dan Bintang pun mampu menyelesaikan persoalan Geometri seperti kawan mereka Yodha. “Wah, kau sudah punya pesaing dalam hal geometri, Yodha?”sindir Amer. “Alhamdulillah, semakin banyak yang tahu lebih tentang Geometri akan semakin baik, Kawan!”balas Yodha enteng. Tepukan meriah kembali terdengar dalam ruang itu. Tepukan berupa apresiasi tulus dari beberapa kawan kepada kawannya, untuk mereka semua, Kesebelasan Genk OSN! “Kita telah menyelesaikan hampir 80% soal OSN Matematika tingkat Nasional dua tahun lalu, Kawan. Dua soal berikutnya, giliran siapa?”tanya Sabiq. Tanpa menunggu lama, Fateh mengangkat tangan kanannya. Begitu pun Iman. Ya, mereka memang satu kelompok. “Kelompok kami, Biq. Aku dan Iman mendapat bagian soal terakhir hari pertama dan soal terakhir hari kedua. Tapi, kami menghadapi persoalan yang sama dengan kelompok Bily dan Yodha, kombinatorika. Sehingga hanya 1 dari 2 soal yang mampu kami serap dan presentasikan semaksimal yang kami ketahui untuk kawankawan saat ini,”terang Fateh. “It’s clear, Bro! kami menunggu aksimu!”teriak Amer dari belakang.
| 213
Fateh melangkah ke depan. Didampingi oleh Iman yang tampak berjalan begitu bersemangat di samping Fateh. “Persoalan yang diajukan kepada kelompok kami, adalah: ‘Berapakah sisa dari 20122012 + 20142012 dibagi oleh 20132?’ Terhadap persoalan ini, Iman akan menjelaskan kepada kita semua, soal yang kami yakin bisa memahami dan menjelaskannya kepada Kawan-kawan. Silahkan, Man!”kata Fateh. “Baik terima kasih telah memberi kepercayaan padaku, Fateh,”jawab Iman. “Sebelum sampai pada penjelasan tentang substansi persoalan, ada baiknya sama-sama kita dipahami terlebih dahulu apa yang dimaksud dan kegunaan Teorema Binomial. Atau sering dikenal sebagai Binomial Newton itu,”terang Iman dengan aksi yang membuat kawan-kawannya menahan rasa penasaran. “Dalam aljabar elementer,”lanjut Iman, “’Teorema Binomial’ adalah teorema yang digunakan untuk menjelaskan mengenai pengembangan eksponen dari penjumlahan antara
dua
variabel.
Karenanya
disebut
dengan
kata
‘binomial’,”Iman semakin semangat. Berdasarkan
teorema
ini,
dimungkinkan
untuk
n
mengembangkan eksponen (x+ y) menjadi sebuah penjumlahan dari suku-suku dengan bentuk axbyc, dimana eksponen b dan c adalah bilangan bulat non-negatif dengan (b + c) = n, dan koefisien a dari setiap suku adalah bilangan bulat positif tertentu tergantung pada n dan b,”terang Iman lagi.
| 214
Iman menyeka sedikit keringat yang tampaknya mulai membasahi bagian atas keningnya. Tindakan sesaat ini tak luput dari perhatian Sabiq dan Ikram yang sedari tadi memang serius memerhatikan apa yang akan disampaikan oleh Iman. “Karenanya, secara umum teorema binomial itu ditulis sebagai berikut: 𝑛𝑛
𝑛𝑛 (𝑥𝑥 + 𝑦𝑦) = � � � 𝑥𝑥 𝑘𝑘 𝑦𝑦 𝑛𝑛−𝑘𝑘 … … … … … … . . (13.14) 𝑘𝑘 𝑛𝑛
𝑘𝑘=0
Dari persamaan (13.14) di atas, terlihat bahwa ada hubungan antara kombinasi dengan teorema binomial. Hubungan itu terletak pada bentuk �𝑛𝑛𝑘𝑘 �, yang tidak lain adalah tanda untuk menyatakan bentuk
dari kombinasi”terang Iman.
Kali ini, kawan-kawan Iman tak bisa lagi menahan rasa, justru sebaliknya hampir menelan ludah!
Ternyata, penjelasan Iman
membuat mereka begitu terpesona. Sementara Sabiq dan Ikram serta Yodha terlihat senyum tipis saja. “Mari kita perhatikan ilustrasi berikut. Dalam aljabar, kita tahu bahwa (a + b)3 = a3 + 3a2b +3ab2 + b3. Apa artinya angka-angka di depan huruf yang bervariasi pangkatnya itu? Secara sederhana dapat kita sebutkan bahwa penjabaran dari (a + b)3 merupakan perkalian dari 3 faktor, yaitu:
(a + b)3 = (a + b) (a + b) (a + b),”terang
Iman bersemangat. “Lalu, kita pilih bagian yang ingin kita kalikan dari ketiga faktor itu. Misalnya, jika kita memilih a dari setiap faktor dan mengalikannya, maka kita peroleh aaa. Jika kita memilih a dari faktor pertama, a dari faktor kedua dan b dari faktor ketiga kemudian | 215
mengalikannya, maka kita peroleh aab, dan seterusnya. Sehingga semua kemungkinan pemilihan baik a maupun b dari masing-masing faktor adalah: aaa; aab; aba; abb; baa; bab; bba; bbb, yang jika dikalikan, akan ditulis dalam bentuk sebaga berikut: a3; a2b; a2b; ab2; a2b; ab2; ab2; dan b3 dan jika semua suku-suku diatas dijumlahkan dan dikelompokkan berdasarkan kesamaannya, maka hasilnya adalah: a3 + 3a2b + 3ab2 + b3,”terang Iman. Kali ini Iman kembali menyeka keringat yang terus membasahi keningnya itu. “Selanjutnya, kalau kita perhatikan bahwa angka 3 yang merupakan koefisien dari a2b muncul dari pemilihan a dari 2 faktor dan b dari 1 faktor sisanya. Hal ini bisa dilakukan melalui proses kombinasi dalam 𝐶𝐶23 atau 𝐶𝐶13 cara. Cara yang sama bisa dilakukan
untuk memperoleh koefisien b3 yang dalam hal ini merupakan pemilihan a dari 0 faktor dan b dari 3 faktor lainnya yang dapat dilakukan dalam 𝐶𝐶03 atau 𝐶𝐶33 cara, dan seterusnya,”tegas Iman lagi.
“Karenanya, melalui hubungan kombinasi dengan teorema
binomial, maka dapat merumuskan ulang rumus teorema binomial pada persamaan (1) di atas sebagai berikut:
Atau,
𝑛𝑛
(𝑥𝑥 + 𝑦𝑦) = � 𝐶𝐶𝑘𝑘𝑛𝑛 𝑥𝑥 𝑛𝑛−𝑘𝑘 𝑦𝑦 𝑘𝑘 … … … . . (13.15) 𝑛𝑛
𝑘𝑘=0 𝑛𝑛
𝑛𝑛 (𝑥𝑥 + 𝑦𝑦) = � � � 𝑥𝑥 𝑛𝑛−𝑘𝑘 𝑦𝑦 𝑘𝑘 … … … . . (13.16) 𝑘𝑘 𝑛𝑛
𝑘𝑘=0
| 216
Selanjutnya, …”tiba-tiba Iman menghentikan kalimatnya karena terdengar suara menyela. “Aih panjang dan buaanyaak kali teorinya, Man. Apa bisa kau berikan satu contoh sederhananya?”terdengar suara Amer yang khas itu. Semua kawan mereka hanya tersenyum. “Baik, berikut adalah contoh hubungan antara teorema binomial dan hubungannya dengan kombinasi itu. Misalnya kita ingin
mengetahui
koefisien
dari
masing-masing
suku
dari
6
perpangkatan dari (a + b) . Bila mendasarkan pada konsep perpangkatan dengan sifat segitiga Pascal, maka koefisien masingmasing sukunya akan memenuhi: Baris Pertama
1
= 20
Baris Kedua
1+1
= 21
Baris Ketiga
1+2+1
= 22
Baris Keempat
1+3+3+1
= 23
Baris Kelima
1+4+6+4+1
= 24
Baris Keenam
1+5+10+10+5+1
= 25
Baris Ketujuh
1+6+15+20+15+6+1
= 26
Sehingga hasil akhirnya adalah menggunakan koefisien pada Baris Ketujuh, yaitu: (𝑎𝑎 + 𝑏𝑏)6 =
= 𝑎𝑎6 + 6𝑎𝑎5 𝑏𝑏 + 15𝑎𝑎4 𝑏𝑏 2 + 20𝑎𝑎3 𝑏𝑏 3 + 15𝑎𝑎2 𝑏𝑏 4 + 6𝑎𝑎𝑏𝑏 5 + 𝑏𝑏 6
Dengan menggunakan sifat kombinasi, maka masing-masing keofisien akan bisa diketahui sebagai berikut (𝑎𝑎 + 𝑏𝑏)6 :
= 𝑪𝑪𝟔𝟔𝟎𝟎 𝑎𝑎6 + 𝑪𝑪𝟔𝟔𝟏𝟏 𝑎𝑎5 𝑏𝑏 + 𝑪𝑪𝟔𝟔𝟐𝟐 𝑎𝑎4 𝑏𝑏2 + 𝑪𝑪𝟔𝟔𝟑𝟑 𝑎𝑎3 𝑏𝑏3 + 𝑪𝑪𝟔𝟔𝟒𝟒 𝑎𝑎2 𝑏𝑏4 + 𝑪𝑪𝟔𝟔𝟓𝟓 𝑎𝑎𝑏𝑏 5 + 𝑪𝑪𝟔𝟔𝟔𝟔 𝑏𝑏6
| 217
Dengan memerhatikan bahwa: 𝑛𝑛 𝑛𝑛! 𝐶𝐶𝑘𝑘𝑛𝑛 = � � = (𝑛𝑛 − 𝑘𝑘)! 𝑘𝑘! 𝑘𝑘
Maka nilai dari masing-masing kombinasi di atas adalah: 6 1𝑥𝑥2𝑥𝑥3𝑥𝑥4𝑥𝑥5𝑥𝑥6 𝑪𝑪𝟔𝟔𝟎𝟎 = � � = = 𝟏𝟏 0 1𝑥𝑥2𝑥𝑥3𝑥𝑥4𝑥𝑥5𝑥𝑥6! 0! 6 1𝑥𝑥2𝑥𝑥3𝑥𝑥4𝑥𝑥5𝑥𝑥6 𝑪𝑪𝟔𝟔𝟏𝟏 = � � = = 𝟔𝟔 1 1𝑥𝑥2𝑥𝑥3𝑥𝑥4𝑥𝑥5𝑥𝑥1! 6 1𝑥𝑥2𝑥𝑥3𝑥𝑥4𝑥𝑥5𝑥𝑥6 𝑪𝑪𝟔𝟔𝟐𝟐 = � � = = 𝟏𝟏𝟓𝟓 2 1𝑥𝑥2𝑥𝑥3𝑥𝑥4𝑥𝑥1𝑥𝑥2 6 1𝑥𝑥2𝑥𝑥3𝑥𝑥4𝑥𝑥5𝑥𝑥6 𝑪𝑪𝟔𝟔𝟑𝟑 = � � = = 𝟐𝟐𝟎𝟎 3 1𝑥𝑥2𝑥𝑥3𝑥𝑥1𝑥𝑥2𝑥𝑥3 6 1𝑥𝑥2𝑥𝑥3𝑥𝑥4𝑥𝑥5𝑥𝑥6 𝑪𝑪𝟔𝟔𝟒𝟒 = � � = = 𝟏𝟏𝟓𝟓 4 1𝑥𝑥2𝑥𝑥1𝑥𝑥2𝑥𝑥3𝑥𝑥4 6 1𝑥𝑥2𝑥𝑥3𝑥𝑥4𝑥𝑥5𝑥𝑥6 𝑪𝑪𝟔𝟔𝟓𝟓 = � � = = 𝟔𝟔 5 1! 1𝑥𝑥2𝑥𝑥3𝑥𝑥4𝑥𝑥5 6 1𝑥𝑥2𝑥𝑥3𝑥𝑥4𝑥𝑥5𝑥𝑥6 𝑪𝑪𝟔𝟔𝟔𝟔 = � � = = 𝟏𝟏 6 01𝑥𝑥2𝑥𝑥3𝑥𝑥4𝑥𝑥5𝑥𝑥6
Terlihat bahwa baik menggunakan sifat segitiga Pascal maupun prinsip kombinasi, nilai koefisien untuk masing-masing suku yang sama adalah sama pula. Hal ini menjadi jawaban atas keresahan Kawan kita Amer,”terang Iman lagi. Kali ini bukan hanya ingin menelan ludah, tanpa sadar beberapa suara halus terdengar mendesis dari mulut kawan-kawan Iman. “Ck.., ck.., ck..,ck…,ck,”suara pelan itu terdengar. Suasan masih hening. Iman masih dengan gayanya. “Setelah kita bisa memahami hubungan antara Teorema Binomial dan Kombinasi kaitannya dengan ekspansi bilangan | 218
berpangkat, kita sudah siap masuk ke persoalan inti, yaitu persoalan yang ditanyakan oleh soal!”tegas Iman. “Nah, untuk bagian ini menjadi kewenangan sepenuhnya Kawan kita Fateh. Silahkan, Kawan!”pinta Iman sambil memberi tanda jempol dan digerakkan ke kanan badannya. Meniru gaya mempersilahkan para Abdi Dalem Kraton Solo dan Jogja. “Terima kasih, Kawan. Pembukaan yang indah,”puji Fateh. Fateh berjalan biasa saja, tapi bagi kawan-kawannya terasa lambat. Mereka serasa ingin tahu, apalagi kejutan yang akan disampaikan Fateh nanti. Fateh masih berjalan biasa. Sampai akhirnya dia menerima alat tulis dari Iman dan memandang lurus ke depan, ke hadapan kawan-kawannya. ”Kawan-kawan pertanyaannya pendek dan sederhana saja, ‘berapakah sisa dari 20122012 + 20142012 dibagi oleh 20132?’ tapi untuk pertanyaan yang pendek ini kita perlu cermat dan sedikit modifikasi,”ujar Fateh membuka penjelasannya. “Untuk dapat menyelesaikan persamaan di atas. Pertama, kita andaikan misalnya:
Hal ini berarti pula, bahwa:
Dan
2013 = 𝑝𝑝 2014 = 𝑝𝑝 + 1 2012 = 𝑝𝑝 − 1
Sehingga, persoalannya bisa ditulis ulang menjadi: Sisa dari [(p – 1)2012 + (p + 1) 2012] dibagi p2 adalah…..,”terang Fateh. | 219
“Dengan memanfaatkan Teorema Binomial, seperti disebutkan oleh Iman tadi, kita bisa mengekspansikan masing-masing suku menjadi di atas menjadi: Suku (p – 1)2012 sebagai: 2012
(𝑝𝑝 − 1)
2012
= � 𝐶𝐶𝑘𝑘2012 𝑝𝑝2012−𝑘𝑘 (−1)𝑘𝑘 𝑘𝑘=0
Bila persamaan di atas diekspansi dan dikeluarkan 2 suku terakhir, akan didapati bentuk yang baru sebagai: 2012
(𝑝𝑝 − 1)
2010
2012 = � � 𝐶𝐶𝑘𝑘2012 𝑝𝑝2012−𝑘𝑘 (−1)𝑘𝑘 � − 𝐶𝐶2011 𝑝𝑝 + 1. . (13.17) 𝑘𝑘=0
Perhatikan persamaan di atas, terlihat bahwa nilai pada sigma yang bergerak dari 0 ke 2012 telah diubah menjadi bergerak dari 0 ke 2010. Karenanya, 2012
� 𝐶𝐶𝑘𝑘2012 𝑝𝑝2012−𝑘𝑘 (−1)𝑘𝑘
𝑘𝑘=0
berubah menjadi: 2010
� 𝐶𝐶𝑘𝑘2012 𝑝𝑝2012−𝑘𝑘 (−1)𝑘𝑘
𝑘𝑘=0
hal ini karena 2 suku terakhirny telah dikeluarkan menjadi: 1 dan 2012 𝐶𝐶2011 𝑝𝑝, di mana:
2012 2012! 2012 𝐶𝐶2011 𝑝𝑝 = � � 𝑝𝑝 = 𝑝𝑝 (1)! 2011! 2011
2012 𝑝𝑝 = 2012𝑝𝑝 𝐶𝐶2011
2012 0 𝐶𝐶2012 𝑝𝑝 = �
Selanjutnya, suku,
2012 0 2012! (1) = 1 � 𝑝𝑝 = (0)! 2012! 2011
| 220
2010
� � 𝐶𝐶𝑘𝑘2012 𝑝𝑝2012−𝑘𝑘 (−1)𝑘𝑘 � 𝑘𝑘=0
pada persamaan (13.17) dapat dimodifikasi dengan mengeluarkan nilai p2, sehingga: 2010
��
𝑘𝑘=0
𝐶𝐶𝑘𝑘2012 𝑝𝑝2012−𝑘𝑘 (−1)𝑘𝑘 �
2010
= 𝑝𝑝 � � 𝐶𝐶𝑘𝑘2012 𝑝𝑝2010−𝑘𝑘 (−1)𝑘𝑘 � 2
𝑘𝑘=0
dan akhirnya persamaan (13.17) dapat dituliskan menjadi: 2012
(𝑝𝑝 − 1)
2010
= 𝑝𝑝 � � 𝐶𝐶𝑘𝑘2012 𝑝𝑝2010−𝑘𝑘 (−1)𝑘𝑘 � − 2012𝑝𝑝 + 1 2
𝑘𝑘=0
Atau dapat pula ditulis dalam bentuk:
(𝑝𝑝 − 1)2012 = 𝑚𝑚𝑝𝑝2 − 2012𝑝𝑝 + 1 … … … … … … … . (13.18)
Dengan,
2010
𝑚𝑚 = � � 𝐶𝐶𝑘𝑘2012 𝑝𝑝2010−𝑘𝑘 (−1)𝑘𝑘 � 𝑘𝑘=0
Sehingga persamaan yang tadinya rumit dengan symbol-simbol menjadi tampak lebih sederhana,”terang Fateh. Fateh menghentikan penjelasannya. Ia memandang coretan yang ia buat sendiri di hadapannnya. Kemudian, “Kawan-kawan, dengan cara pengerjaan yang sama di atas pula, maka suku (p + 1)2012 dapat kita ekspansikan sebagai berikut: 2012
(𝑝𝑝 + 1)2012 = � 𝐶𝐶𝑘𝑘2012 𝑝𝑝2012−𝑘𝑘 (+1)𝑘𝑘 𝑘𝑘=0
2012
(𝑝𝑝 + 1)2012 = � 𝐶𝐶𝑘𝑘2012 𝑝𝑝2012−𝑘𝑘 𝑘𝑘=0
| 221
2012
(𝑝𝑝 + 1)
2010
= 𝑝𝑝 � � 𝐶𝐶𝑘𝑘2012 𝑝𝑝2010−𝑘𝑘 � + 2012𝑝𝑝 + 1 2
𝑘𝑘=0
Atau dapat ditulis sebagai :
(𝑝𝑝 + 1)2012 = 𝑛𝑛𝑝𝑝2 + 2012𝑝𝑝 + 1 … … … … … … … . (13.19)
Dengan,
2010
𝑛𝑛 = � � 𝐶𝐶𝑘𝑘2012 𝑝𝑝2010−𝑘𝑘 � 𝑘𝑘=0
Sehingga, bentuk akhir persamaan keseluruhannya adalah sebagai berikut: (𝑝𝑝 − 1)2012 + (𝑝𝑝 + 1)2012 =
= 𝑚𝑚𝑝𝑝2 − 2012𝑝𝑝 + 1 + 𝑛𝑛𝑝𝑝2 + 2012𝑝𝑝 + 1
atau menjadi lebih sederhana sebagai:
(𝑝𝑝 − 1)2012 + (𝑝𝑝 + 1)2012 = 𝑝𝑝2 (𝑚𝑚 + 𝑛𝑛) + 2 … … . . (13.20)
Dari bentuk akhir persamaan (13.20) terlihat bahwa (m + n) merupakan bilangan bulat yang menjadi kelipatan dari p2, dan pastilah habis dibagi oleh p2. Sehingga jawaban atas pertanyaan ‘berapakah sisa dari 20122012 + 20142012 dibagi oleh 20132’adalah 2!”pungkas Fateh. “Subhanallah! Excellent, Kawanku Fateh!”teriak Amer dan Saver bersamaan. Sementara di barisan depan, tak jauh dari Fateh berdiri, Sabiq tersenyum sangat lebar. Dan,…. setitik air bening menetes dari sudut kiri mata kanannya. “Terima kasih, Kawan-kawan. Ini semua adalah proses kita bersama. Namun, seperti kami sebutkan di depan. Untuk soal berikutnya, soal kombinatorika, kami tangguhkan,”terang Fateh. | 222
Serempak terdengar applause sangat panjang dan lama sekali. Sepuluh anak berdiri memberi tepuk tangan meriah kepada kawan mereka yang menjelaskan penyelesaian soal OSN Matematika tentang pembagian berpangkat tinggi dengan pendekatan Teorema Binomial Newton dan Teori Kombinasi. “Jadi, bagaimana dengan 2 soal Kombinatoriknya, Kawankawan?”tiba-tiba Sabiq bertanya serius. “Nah, bukankah kau sebut dirimu sebagai Ban Serep, di awal kita diskusi, Kawan?”tanya Bintang balik pada Sabiq “Ya. Itu benar,”jawab Sabiq singkat. “Saat ini, ban itu sedang kita perlukan agar kenderaan ini tetap bisa berjalan. Kau yang harus selesaikan, Kawan!”pinta Bintang. Seperti
dikomando,
Sembilan
kawan
mereka
lainnya
mengangguk tanda setuju! “Baiklah. Kebetulan, tadi aku cek ada surel dari Pak Tutur yang berisi penjelasan tentang Soal Nomor 10 OSN Matematika dua tahun lalu itu. Tapi, sebelum sampai ke soal itu. Kita selesaikan soal kombinatorika dari kelompok Yodha yang tertinggal, Kawankawan,”terang Sabiq. “Alhamdulillah, akhirnya bisa juga kita selesaikan persoalan ini,”teriak Denan. Sabiq tak ingin berlama-lama. Ia langsung ke depan. “Terhadap pertanyaan tentang komposisi buah yang terambil, dimana disebutkan bahwa pada suatu keranjang buah terdapat 20 Apel, 18 Jeruk, 16 Mangga, 10 Nenas dan 6 Pepaya, dan jika seseorang ingin mengambil 10 buah dari keranjang tersebut, ada | 223
berapa banyak komposisi buah terambil yang mungkin, kita akan selesaikan melalui ‘Kombinasi dengan Pengulangan’,”terang Sabiq. “Pak Dukun, ada saja ilmumu itu. Apa pula yang kau maksud dengan Kombinasi dengan Perulangan itu? Terangkanlah!”tanya Amer penasaran. “Tentu, Kawanku Amer. Seperti tadi Iman dan Fateh telah singgung tentang Kombinasi. Maka soal ini adalah bagian khusus tentang Kombinasi dengan Perulangan itu. Misalkan ada n obyek yang akan diletakkan pada r tempat tanpa urutan dengan r ≤ n. Jika disyaratkan bahwa satu tempat hanya bisa menampung paling banyak 1 obyek maka banyaknya cara adalah nCr, seperti yang telah kita bahas sebelumnya. Tapi, andaikan terdapat n obyek identik dan disyaratkan bahwa seluruh obyek akan dibagikan ke r tempat dengan masing-masing tempat dapat tidak ditempati maupun ditempati satu atau lebih obyek. Pertanyaannya adalah, ada berapa banyak cara untuk menyusunnya?”pancing Sabiq. Tampak kesepuluh temannya serius. “Nah, ini kata kuncinya. Karena identik maka urutan dalam persoalan ini tidak diperhatikan. Taruh n obyek, misalkan bola, tersebut dalam satu baris. Tambakan (r –1) batas di antaranya sehingga kini seolah-olah ada (n + r – 1) ’tempat’. Akibat penambahan (r – 1) batas tersebut maka n bola tersebut akan terbagi dalam r bagian, yaitu di sebelah kiri batas ke-1, di antara batas ke-1 dan ke-2 sampai dengan di sebelah kanan batas ke-(r−1). Masingmasing bagian tersebut melambangkan banyaknya bola pada masingmasing tempat. Sehingga persoalannya sekarang adalah memilih (r − | 224
1) tempat dari (n + r – 1) tempat yang tersedia. Dengan rumusan kombinasi, kita bisa tentukan bahwa banyaknya cara adalah: �
(𝑛𝑛 + 𝑟𝑟 − 1)! 𝑛𝑛 + 𝑟𝑟 − 1 𝑛𝑛 + 𝑟𝑟 − 1 �=� � = 𝐶𝐶𝑛𝑛𝑛𝑛+𝑟𝑟−1 = 𝑛𝑛 𝑟𝑟 − 1 𝑛𝑛! (𝑟𝑟 − 1)!
Contoh sederhananya, misalnya, terdapat 4 buah yang bola akan dibagikan seluruhnya ke dalam 3 buah kantong. Ada berapa banyak cara menyusunnya?”tba-tiba Sabiq bertanya kepada semua. Sabiq berhenti sejenak. Dia perhatikan satu-satu wajah kawankawannya. Dia mengulurkan alat tulis kepada mereka. Semuanya menggeleng. “Oke. Terhadap persoalan seperti ini, maka bila rumusan di atas kita terapkan akan terlihat bahwa banyaknya cara adalah: 𝑛𝑛+𝑟𝑟−1
〱𝑛𝑛
,
di mana: 𝑛𝑛 = 4, 𝑟𝑟 = 3, dan 𝑛𝑛 + 𝑟𝑟 − 1 = 6. Maka,
𝐶𝐶𝑛𝑛𝑛𝑛+𝑟𝑟−1 = 𝐶𝐶46 𝐶𝐶46 =
1𝑥𝑥2𝑥𝑥3𝑥𝑥4𝑥𝑥5𝑥𝑥6 (1𝑥𝑥2)𝑥𝑥1𝑥𝑥2𝑥𝑥3𝑥𝑥4
𝐶𝐶46 = 15 𝑐𝑐𝑎𝑎𝑟𝑟𝑎𝑎
Selanjutnya, kelimabelas cara itu adalah pasangan dari: (4,0,0), (3,1,0), (3,0,1), (2,2,0), (2,0,2); (2,1,1), (1,3,0), (1,2,1), (1,1,2), (1,0,3); (0,4,0), (0,3,1), (0,2,2), (0,1,3), dan (0,0,4) dengan (a, b, c) menyatakan kantong pertama berisi a bola, kantong kedua berisi b bola dan kantong ketiga berisi c bola,”terang Sabiq. Sabiq berhenti lagi. Diteguknya sisa air putih yang tinggal setengah gelas itu. Kesepuluh sahabatnya tampak sangat serius | 225
mengikuti penjelasan Sabiq tentang cerita Kombinasi dengan Pengulangan itu. “Kegunaan lain dari Kombinasi dengan pengulangan adalah untuk menyelesaikan persoalan mengenai perhitungan banyaknya penyelesaian persamaan linier,”Sabiq melanjutkan penjelasannya. “Maksudmu, Biq!”tanya Saver. “Ya, persamaan linier. Misalkan saja terdapat persamaan: x1 + x2 + x3+⋅⋅ + xr = n. Jika xi merupakan bilangan bulat tak negatif, maka ada berapa banyak
penyelesaian
yang
memenuhi
persamaan
di
atas?
Sebenarnya, persoalan ini sama saja dengan membagi n obyek identik ke dalam r tempat di atas. Kita ketahui bahwa banyaknya penyelesaian adalah
n+r-1Cn.
Dengan cara yang sama, akan kita
peroleh banyaknya pasangan yang memenuhi bilangan bulat tak negatif , yaitu: 𝐶𝐶𝑛𝑛𝑛𝑛+𝑟𝑟−1 = 𝐶𝐶33+4−1 = 𝐶𝐶36
𝐶𝐶36 = 20
Dan pasangan-pasangan (x1, x2, x3, x4) yang memenuhi itu adalah: (0,0,0,3), (0,0,1,2), (0,0,2,1), (0,0,3,0), (0,1,0,2); (0,1,1,1), (0,1,2,0), (0,2,0,1), (0,2,1,0), (0,3,0,0); (1,0,0,2), (1,0,1,1), (1,0,2,0), (1,1,0,1), (1,1,1,0); (1,2,0,0), (2,0,0,1), (2,0,1,0), (2,1,0,0), dan (3,0,0,0),”papar Sabiq. “Lantas bagaimana hubungan keteranganmu itu dengan soal buah-buahan yang diajukan pada 2 tahun lalu itu, Biq?”tanya Ayra penasaran. “Nah, inilah inti dari penjelasanku tadi,”tegas Sabiq | 226
Sabiq kembali memandang lurus ke arah kawan-kawannya. Dia tampak serius. Keseriusan yang berbeda seperti biasanya. “Misalkan, x1=Apel, x2=Jeruk, x3=Mangga, x4=Nenas dan x5=Pepaya. Selanjutnya banyaknya komposisi buah yang terambil equivalen dengan banyaknya penyelesaian (x1; x2; x3; x4; x5) dari persamaan x1 + x2 + x3 + x4 + x5 = 10, dengan 0 ≤ xi ≤ 10. Seperti kita ketahui bahwa banyaknya penyelesaian dari x1 + x2 + x3 + x4 + x5 = 10 adalah: 𝐶𝐶𝑛𝑛𝑛𝑛+𝑟𝑟−1
Dengan memasukkan nilai n = 10 dan r = 5, maka 14 𝐶𝐶𝑛𝑛𝑛𝑛+𝑟𝑟−1 = 𝐶𝐶10 14 𝐶𝐶10 = 14 𝐶𝐶10 =
14! (14 − 10)! 𝑥𝑥10!
14𝑥𝑥13𝑥𝑥12𝑥𝑥11 4𝑥𝑥3𝑥𝑥2𝑥𝑥1
14 𝐶𝐶10 = 1001
akan tetapi karena x5 ≤ 6, atau jumlah Pepaya hanya 6 buah, maka nilai 1001 harus dikurangi dengan banyaknya penyelesaian jika x5= 7; 8; 9; 10. Selanjutnya kita hitung banyak kemungkinan tersebut : a. Jika x5 = 7, maka diperoleh x1 + x2 + x3 + x4 = 3. Oleh karena itu banyaknya penyelesaiannya adalah 6C3 = 20; b. Jika x5 = 8, maka diperoleh x1 + x2 + x3 + x4 = 2. Oleh karena itu banyaknya penyelesaiannya adalah 5C2 = 10; c. Jika x5 = 9, maka diperoleh x1 + x2 + x3 + x4 = 1. Oleh karena itu banyaknya penyelesaiannya adalah 4C1 = 4; d. Jika x5 = 10, maka diperoleh x1 + x2 + x3 + x4 = 0. Oleh karena itu banyaknya penyelesaiannya adalah 3C0 = 1 | 227
Oleh karena itu banyaknya komposisi buah terambil yang mungkin adalah: = 1001 – (20 + 10 + 4 + 1) = 1001 – 35 = 966 cara!”papar Sabiq. “Subhanallah! Kau memang Ban Serep yang serba guna, Biq! Ice breaker, Man!”komentar Amer. “Plok, plok, plok, plok,”terdengar tepuk tangan meriah dari sepuluh kawannya. Sabiq hanya tersenyum tipis. “Bagaimana dengan penjelasan soal Kombinatorika punya kelompok Fateh dan Iman, Biq?”tiba-tiba Ayra yang kini antusias bertanya. “Terhadap soal nomor ini, baiknya kita baca utuh saja apa yang disampaikan oleh Pak Tutur dulu, ya!”jawab Sabiq singkat. “Baik, tolong bacakan untuk kita semua, Kawan!”pinta Bily. “Mohon maaf sebelumnya, Kawan-kawan!”tiba-tiba Ikram memotong ucapan Sabiq. “Biq, aku pikir ada hal yang penting kita ketahui bersama,”kata Ikram. “Apakah hal penting itu, Kawan?’tanya Sabiq. “Terkait persoalan yang pernah ditanyakan pada OSN beberapa tahun lalu. Aku merasa ada yang aneh, karena bentuk ini baru
aku
temukan.
Mungkin
kau
pernah
tahu
dan
bisa
menjelaskannya di forum ini,”kata Ikram lagi. “Persoalan seperti apa itu, Kawan?”tanya Sabiq lagi.
| 228
“Baik, aku bacakan. Mohon kawan-kawan nilai apakah ini lebih penting dari surel itu. Begini redaksinya, ‘misalkan A, B dan P adalah paku-paku yang ditanam pada papan ABP. Panjang AP = a satuan dan BP = b satuan. Papan ABP diletakkan pada lintasan X1X2 dan Y1Y2 sehingga A hanya dapat bergerak bebas sepanjang lintasan X1X2 dan B hanya bergerak bebas sepanjang lintasan Y1Y2 seperti pada gambar berikut. Misalkan x adalah jarak titik P terhadap lintasan Y1Y2 dan y jarak titik P terhadap lintasan X1X2. Tunjukkan bahwa persamaan lintasan titik P adalah: 𝑥𝑥 2 𝑦𝑦 2 + =1 𝑏𝑏 2 𝑎𝑎2
Dan ilustrasi persoalan yang ditanyakan adalah sebagai berikut:
Aku pikir, karena bentuk persamaannya, agak unik. Bagaimana Kawan-kawan?”papar Ikram. Tiba-tiba suasana menjadi sibuk. Semua mengunakan alat tulis yang ada di meja-masing-masing. 1 menit berlalu, 2 menit, dan... “Ya, aku pikir persoalan bentuk bangunnya sederhana. Hanya saja bentuk persamaan yang terbentuk memang agak unik. Aku pun baru pertama kali menemukan bentuk persamaan seperti ini,”tibatiba Yodha memecah keheningan.
| 229
Sabiq tampak berpikir keras, Ayra pun begitu. Fateh tampak lebih serius. Saver masih asyik dengan alat tulis, pun dengan Bintang. “Ya, rasanya aku pernah membaca dan sedikit menganalisis persamaan seperti itu. Tapi agak lupa. Atau begini saja. Sementara aku mengingat-ingat tentang bentuk persamaan aneh itu, biarkan Yodha
menjelaskan
aspek
bangunnya.
Bagaimana,
Kawan-
kawan?”tawar Sabiq. “Tapi, bagaimana dengan surel itu?”sela Ayra. “Melihat bentuk soal yang diajukan Ikram ini, aku pikir bisa kita pending sementara dan kita ganti pembahasan dengan yang diajukan oleh Ikram. Bila ada waktu yang cukup, kita minta Sabiq meneruskan penjelasannya tentang isi surel itu. Meskipun begitu, sebaiknya kita baca utuh apa yang ada dalam surel itu. Sehingga kita bisa dapat manfaat dari keduanya,”usul Iman. “Aku setuju itu!”komentar Amer. “Itu pun baik buat kita,”timpal Denan. “Oke, kita sepakati kita bahas yang diajukan oleh Ikram, dan kita muat utuh surel Pak Tutur dengan penjelasan sekadar membuat agar lebih terang. Untuk itu, silahkan Yodha memaparkan dari sisi bentuk bangunnya, sisi Geometri!”Sabiq memutuskan. Yodha bangkit dengan alat tulis di tangan. Dia menuju papan tulis, menggambar, dan memberi beberapa tanda-tanda. “Baik, Kawan-kawan. Tadi, beberapa saat setelah Ikram membacakan persoalannya pada kita, aku coba mencoret-coret
| 230
gambar yang mungkin agak memberi pencerahan atas gambar yang ditampilkan. Hasil modifikasi gambar itu kira-kira seperti ini:
Setelah diperhatikan baik-baik, kata kuncinya terdapat pada frasa, “misalkan x adalah jarak titik P terhadap lintasan Y1Y2 dan y jarak titik P terhadap lintasan X1X2”. Kenapa? Karena konsekuensi jarak suatu titik terhadap lintasan haruslah membentuk garis yang tegak lurus terhadap lintasan itu sendiri. Karenanya: (1) Akan terbentuk segitiga siku-siku BDP di D (2) Akan terbentuk segitiga siku-siku ACP di C Selanjutnya, harus dicari hubungan antara variabel-variabel a, b, x, dan y. Dan untuk itu alat yang kita gunakan adalah sifat kesebangunan dalam geometri,”papar Yodha. Tampak semua kawannya serius memerhatikan apa yang dipaparkan oleh Yodha. Begitupun Sabiq. Bahkan tampaknya ia sedang berusaha mengingat-ingat sesuatu. “Dengan memerhatikan ΔBDP sebagai segitiga siku-siku dan bantuan perumusan Phytagoras, maka: 𝑃𝑃𝐷𝐷 2 + 𝐷𝐷𝐵𝐵2 = 𝐵𝐵𝑃𝑃2 𝑥𝑥 2 + 𝐷𝐷𝐵𝐵2 = 𝑏𝑏 2 𝐷𝐷𝐵𝐵2 = 𝑏𝑏 2 − 𝑥𝑥 2
| 231
𝐷𝐷𝐵𝐵 = �𝑏𝑏 2 − 𝑥𝑥 2 … … … … … … … … … … . . (13.21)
Berikutnya adalah dengan memerhatikan bahwa ΔBDP ~ ΔACP, maka:..,”tiba-tiba Yodha menghentikan penjelasannya karena ada suara menyela. “Aku belum begitu paham dengan konsep kesebangunan dalam Geometri itu, Yodha. Bisa kau jelaskan secara singkat bagaimana agar mudah memahaminya!”potong Amer. Yodha tampak berpikir keras. Ia bisa memahami dengan mudah soal kesebangunan ini, tapi Amer ada benarnya juga, bagaimana dengan kawan-kawannya? Setelah berpikir beberapa saat, Yodha kembali bersikap tenang. “Baiklah kawan-kawan, kita sedikit flash back. Belajar sedikit teori kesebangunan. Secara umum, dua buah segitiga dikatakan sebangun apabila sisi-sisinya memiliki perbandingan yang sama. Dan segitiga yang memiliki sisi-sisi yang sama dikatakan kongruen (sama dan sebangun). Selanjutnya, terdapat beberapa hal yang dipenuhi untuk disebut sebangun itu. Perhatikan gambar sederhana di bawah ini!”kata Yodha sambil menggambar 2 buah segitiga yang berbeda ukuran.
“Dua buah segitiga, seperti pada gambar di atas, ΔABC dan ΔDEF dikatakan sebangun jika memenuhi salah satu syarat berikut : | 232
(a) Ketiga sudutnya sama. Dengan kata lain ∠A = ∠D, ∠B =
∠E dan ∠C = ∠F. Jika diperhatikan syarat sebenarnya
hanyalah dua buah sudutnya sama sebab sudut ketiga akan sama jika dua sudut lainnya sama. (b) Sisi-sisinya memiliki perbandingan yang sama, karenanya 𝐴𝐴𝐵𝐵 𝐴𝐴𝐶𝐶 𝐵𝐵𝐶𝐶 = = 𝐷𝐷𝐸𝐸 𝐷𝐷𝐹𝐹 𝐸𝐸𝐹𝐹 (c) Dua sisi memiliki perbandingan yang sama serta sudut yang mengapit kedua sisi tersebut juga sama. 𝐴𝐴𝐵𝐵 𝐴𝐴𝐶𝐶 = 𝐷𝐷𝐸𝐸 𝐷𝐷𝐹𝐹 ∠A = ∠D
Itu teori dasarnya, Kawan!”tegas Yodha “Ternyata sederhana sekali, Kawan,”komentar Amer. “Ya, memang sederhana tapi besar konsekuensi dan manfaatnya untuk Geometri yang paling rumit sekalipun. Nah, kita kembali pada persoalan di atas. Dengan kata kunci dikatakan sebangun jika memenuhi salah satu syarat, kalau aku sebut bahwa pada ΔBDP dan ΔACP, ‘dua sisi memiliki perbandingan yang sama serta sudut yang mengapit kedua sisi tersebut juga sama’, maka sudut apa yang sama itu, Mer?”tiba-tiba Yodha bertanya. Amer tampak serius memandang ke arah papan tulis, ia memerhatikan dengan serius. Dan kurang dari 1 menit, Amer menurunkan kepalanya dan memalingkan wajahnya ke arah Yodha. “Karena ΔBDP dan ΔACP adalah sebangun, maka ∠DPB =
∠CAP. Ya, sudut P pada ΔDPB sama besarnya dengan sudut A pada
∠CAP, Kawan,”jawab Amer.
| 233
“Tepat! Karenanya pula, maka dapat dituliskan: 𝐶𝐶𝑃𝑃 𝐷𝐷𝐵𝐵 = … … … … … … … … … … … … … . … . (13.22) 𝐴𝐴𝑃𝑃 𝑃𝑃𝐵𝐵 Sehingga, berdasarkan persamaan (13.21) di atas, maka persamaan (13.22) dapat ditulis menjadi: 𝑦𝑦 √𝑏𝑏 2 − 𝑥𝑥 2 = 𝑎𝑎 𝑏𝑏
Dengan menguadratkan kedua sisi, akan diperoleh: 𝑦𝑦 2 𝑏𝑏 2 − 𝑥𝑥 2 = 𝑎𝑎2 𝑏𝑏 2
𝑦𝑦 2 𝑏𝑏 2 = 𝑎𝑎2 (𝑏𝑏 2 − 𝑥𝑥 2 )
𝑎𝑎2 𝑥𝑥 2 + 𝑏𝑏 2 𝑦𝑦 2 = 𝑎𝑎2 𝑏𝑏 2
Bila kedua ruas dibagi dengan (a2b2), akan diperoleh bentuk: 𝑥𝑥 2 𝑦𝑦 2 + =1 𝑏𝑏 2 𝑎𝑎2
Akhirnya, terbukti bahwa persamaan lintasan titik P adalah: 𝑥𝑥 2 𝑦𝑦 2 + =1 𝑏𝑏 2 𝑎𝑎2
Demikian penjelasanku tentang pembuktian persamaan dari sisi teori kesebangunan. Selanjutnya, urusan Sabiq menjelaskan maksud dari persamaan itu. Terima kasih,”pungkas Yodha. “Bukan main kau Yodha! Mantab betul! Soal geometri, dia memang
patut
diacungi
jempol!
Bagaimana
tambahanmu,
Biq?”komentar Ayra. Sabiq maju ke depan, tapi ia agak ragu. Ada yang mengganjal. “Sampaikan saja apa adanya, Kawan. Toh, yang ditanyakan oleh soal sudah kita jawab! Kita hanya ingin tahu lebih jauh tentang bentuk persamaan yang memang unik ini,”kata Ikram tenang. | 234
“Baiklah kawan-kawan, ingatanku ini pun sekadarnya saja. Dan kenapa aku ragu, karena materi ini belum diajarkan di tingkatan kita. Menurut buku yang aku baca, ia masuk dalam materi pelajaran di SMU, setingkat di atas kita. Tapi, sebagai sebuah pengetahuan, kita tak salah untuk tahu,”Sabiq memulai keterangannya. “Ini berawal dari apa yang disebut sebagai Irisan atau Potongan Kerucut. Ternyata, bila Kerucut kita potong sedemikian rupa, akan menghasilkan bentuk permukaan potongan yang memenuhi fungsi kuadrat. Dan fungsi kuadrat itu terdiri atas 4 bentuk variasi kemungkinan, yaitu: lingkaran, elips, hiperbola, atau parabola. Seperti ini kira-kira gambarnya,”terang Sabiq sambil menggambar Kerucut yang dipotong di sana-sini di papan tulis.
“Kita fokus ke elips saja. Seperti sama-sama kita ketahui, saat belajar tentang persamaan suatu lingkaran, bahwa lingkaran dengan titik pusat di titik (a, b) dan berjari-jari r akan memiliki persamaan: (𝑥𝑥 − 𝑎𝑎)2 + (𝑦𝑦 − 𝑏𝑏)2 = 𝑟𝑟 2
Persamaan di atas merupakan bentuk standar. Dengan membagi kedua ruas persamaan tersebut dengan r2, kita akan memeroleh bentuk persamaan:
| 235
(𝑥𝑥 − 𝑎𝑎)2 (𝑦𝑦 − 𝑏𝑏)2 𝑟𝑟 2 + = 2=1 𝑟𝑟 2 𝑟𝑟 2 𝑟𝑟 Persamaan di atas sederhananya menyatakan bahwa jarak vertikal dan horizontal ke titik pusat grafik adalah sama, yaitu sebesar r atau jari-jari
lingkaran,
karena
nilai
penyebutnya
sama.
Nah,
pertanyaannya adalah bagaimana jika nilai penyebutnya berbeda? Inilah yang disebut sebagai persamaan elips,”papar Sabiq masih datar. “Karena
penyebutnya
berbeda,
maka
bentuk
standar
persamaan elips adalah sebagai berikut: (𝑥𝑥 − 𝑎𝑎)2 (𝑦𝑦 − 𝑏𝑏)2 + =1 𝑝𝑝2 𝑞𝑞 2
Selanjutnya, terdapat beberapa hal tentang elips ini, yaitu: bahwa jarak terjauh antara dua titik pada elips disebut sebagai sumbu mayor, dengan titik-titik ujung sumbu mayor disebut titik-titik puncak elips. Ruas garis yang tegak lurus dan membagi sumbu mayor menjadi 2 bagian yang sama disebut sumbu minor. Perhatikan gambar berikut,”kata Sabiq sambil menggambar bentuk elips dengan bagian-bagiannya.
“Hal lain yang penting dari sifat dan bentuk elips, adalah nilai p (sumbu mayor) dan q (sumbu minor), yaitu: | 236
(1) Jika p > q, maka sumbu mayornya horizontal (sejajar dengan sumbu-x) dengan panjang 2p, dan sumbu minornya vertikal dengan panjang 2q; (2) Jika p < q, maka sumbu mayornya vertikal (sejajar dengan sumbu-y) dengan panjang 2q, dan sumbu minornya horizontal dengan panjang 2p,”terang Sabiq lagi. “Aih, sulit dan ribet kali teorinya. Bisa kau beri contoh, Biq!”potong Amer cepat. Sabiq agak lama berfikir. Dia sedang mencari sesuatu tampaknya. Dan…. “Baiklah, kita misalkan saja terdapat bentuk persamaan kuadrat, seperti sebagai berikut: 25𝑥𝑥 2 + 4𝑦𝑦 2 = 100
Yang pasti itu adalah bentuk persamaan elips. Bagaimanakah cara menulis persamaan tersebut ke dalam bentuk standar dan menentukan titik pusat, nilai p, dan q-nya, serta seperti apa grafiknya?”ujar Sabiq. “Nah, kalau ini lebih konkret, Kawan!”sela Amer lagi. Tampak
kawan-kawan
lainnya
senyum-senyum
dengan
tingkah Amer. Memang, Amer adalah siswa yang paling tak begitu suka dengan teori, apalagi teori yang panjang-panjang dan rumit. Dia lebih senang bekerja dan belajar dengan contoh. “Pertama, kita perhatikan bahwa koefisien-koefisien dari x2, y2 dan r2 tidak sama, yaitu 25, 4, dan 100, meskipun masing-masing memiliki tanda yang sama, positif. Sehingga, persamaan tersebut | 237
merupakan persamaan elips dengan pusat di (0, 0). Selanjutnya kita ubah persamaan tersebut ke dalam bentuk standar, dengan membagi kedua ruas dengann 100, sehingga: 25𝑥𝑥 2 + 4𝑦𝑦 2 = 100 25𝑥𝑥 2 4𝑦𝑦 2 + =1 100 100
𝑥𝑥 2 𝑦𝑦 2 + =1 4 25
(𝑥𝑥 − 0)2 (𝑦𝑦 − 0)2 + =1 22 52
Persamaan di atas menyatakan bahwa titik pusat adalah (0,0) dengan masing-masing nilai p = 2 dan q = 5. Hal ini berarti bahwa p < q. Karenanya, merujuk pada sifat kedua dari elips di atas, maka sumbu mayor elipsnya adalah vertikal, sejajar dengan sumbu-y dengan panjang 2q, sementara sumbu minornya horizontal dengan panjang 2p,”terang Sabiq Ia masih berdiri tegak di depan kawan-kawannya. Ia ingin meneruskan. “Hal lainnya adalah, dengan (0,0) sebagai titik pusat, maka perpotongan grafik terhadap sumbu-x ada di titik-titik (–2,0) dan (2,0), sedangkan titik-titik puncaknya, dan perpotongan grafik dengan sumbu-y, ada di titik-titik (0,–5) dan (0,5) dan gambarannya seperti ini, Kawan-kawan,”terang Sabiq sambil menggambar elips tegak di papan tulis
| 238
“Aku pikir cukuplah penjelasan tentang bentuk persamaan elips ini. Yang penting kita tahu sifat dasar dan cara membaca serta memahami bentuk dan grafiknya,”pungkas Sabiq. “Subhanallah! Pengetahuanmu tentang hal lain baik sekali, Kawan,”puji Saver. “Hanya kebetulan pernah membaca saat buka-buka buku sains di perpustakaan tentang alam semesta dan cerita tentang Keppler yang menyebut elips sebagai bentuk lintasan orbit planet di sekitar Matahari,”jawab Sabiq singkat. Serempak kesepuluh kawannya berdiri dan memberi applause kepada Sabiq, sebagai bentuk apresiasi atas pengetahuannya tentang persamaan elips yang memang belum mereka pelajari di sekolah. “Terima kasih, Kawan-kawan. Tapi sebaiknya kita kembali ke inti persoalan. Dan persoalan tadi yang kita tunda adalah tentang surel dari Pak Tutur perihal jawaban atas soal terakhir OSN itu,”Sabiq kembali mengingatkan kawan-kawannya. “Setuju, Kawan. Langsung saja kau bacakan untuk kita semua, setelah itu bila ada hal yang bisa kita diskusikan, ya kita bahas bersama,”komentar Iman pelan. | 239
“Tunggu dulu, Kawan! kalau saranku surat itu kita baca masing-masing saja. Aku lebih tertarik, jika waktu yang ada ini, kita pakai untuk menagih janji Sabiq!”tiba-tiba Amer mengajukan usul baru. “Janjiku? Janji yang mana, Mer?”tanya Sabiq heran. “Kalau Kawan-kawan masih ingat waktu kita bahas bagian Appe.., apa itu?”kalimat Amer terpotong. “Appetizer!”Ayra membetulkan. “Ya, Appetizer. Kau telah berjanji akan menjelaskan tentang prinsip kembalikan-pinjaman dan bayarlah-hutang. Menurutmu akan lebih baik bila waktu ini kita gunakan membahas hal itu, Kawan,”terang Amer. Entah kenapa, Amer selalu bisa menarik perhatian kawankawannya. Kepolosan dan keceriaan serta keterusterangan adalah sesuatu yang sangat dikenal oleh kawan-kawannya dalam diri Amer. Dan kali ini pun tampaknya mereka akan menyetujui usul Amer itu. “Baik juga usul itu,”komentar Fateh. “Aku pikir juga begitu,”ujar Saver. “Hitung-hitung bayar hutangmu, Biq!”celoteh Ikram. “Masuk akal dan sangat membantu. Tapi, dengan catatan ada soal konkret yang kita kerjakan sebagai pembanding,”pendapat Ayra. Kawan-kawan lainnya tak komentar, hanya saja ekspresi wajah mereka setuju dengan usul Amer dan dengan respons keempat kawan mereka yang mendukungnya.
| 240
“Baiklah, kalau begitu. Tapi, aku berharap, kita semua membaca sungguh-sungguh surel itu, karena beliau sengaja mengirimkannya untuk kita,”pinta Sabiq. “Kalau begitu, sekarang kau lunasi janjimu, Kawan!”pinta Amer sambil mengerdipkan mata kirinya kepada Sabiq. Sabiq berdiri dan dia sudah siap di dekat papan tulis. “Kawan-kawan, sebenarnya apa yang aku sebut sebagai prinsip kembalikan–pinjaman dan bayarlah-hutang itu hanya istilahku saja. Prinsip sebenarnya ialah apa yang sering disebut sebagai Simon's Favorite Factoring Trick. Sebuah teknik atau cara memecahkan persamaan linier. Teknik ini dipopulerkan oleh Simon Rubinstein-Salzedo saat berusaha memecahkan persamaan linear yang tampaknya sederhana, ternyata mengandung kerumitan. Ide dasarnya ialah membangun sebuah persamaan yang bisa difaktorkan sedemikian rupa, sehingga penyelesaiannya menjadi lebih mudah, bagaimana pun caranya!”jelas Sabiq “Nah, bagian kalimat
terakhir itu, ‘membangun sebuah
persamaan yang bisa difaktorkan sedemikian rupa, sehingga penyelesaiannya menjadi lebih mudah, bagaimana pun caranya’, yang aku suka. Kau jelaskan lebih detail, Kawan!”pinta Amer. “Tentu dengan contoh konkret pula!”timpal Ayra. Sabiq mulai menarik nafas panjang, dalam sekali. “Baik, pada salah satu soal OSN tingkat Propinsi terdapat persoalan seperti ini, ‘semua pasangan bilangan asli m dan n yang memenuhi persamaan: 2 3 + =1 m n
| 241
adalah.....”. Mungkin ada diantara kawan-kawan yang mau menyelesaikannya? Kau Mer? Maju!”tiba-tiba Sabiq bertanya. Amer maju. Dia raihnya alat tulis, dan dia mulai menulis. “Baik, persamaan itu, bila kita tuliskan dan kita uraikan akan menjadi: 2 3 + =1 𝑚𝑚 𝑛𝑛
2𝑛𝑛 + 3𝑚𝑚 = 𝑚𝑚𝑛𝑛
Dan selanjutnya….”kalimat Amer terhenti. Tangannya pun berhenti menulis. Ia hanya bisa memandang. Hampir 35 detik. “Aku menyerah, Kawan!”ujar Amer jujur. “Baik. terima kasih, Mer. Ada kawan-kawan yang mau mencoba? Ayra, Ikram, Saver, Denan, kau Yodha”tanya Sabiq. Semua nama yang disebut hanya diam dan menggeleng. “Inilah hasil usaha Simon yang pantas kita apresiasi. Ingat bahwa idenya adalah membangun sebuah persamaan yang bisa difaktorkan sedemikian rupa, sehingga penyelesaiannya menjadi lebih mudah, bagaimana pun caranya! Harus tetap kita pegang,”tegas Sabiq lagi. “Sebenarnya, bentuk umum dari Simon’s Trick itu digunakan untuk persamaan-persamaan linier yang bentuknya memenuhi: 𝑥𝑥𝑦𝑦 + 𝑎𝑎𝑥𝑥 + 𝑏𝑏𝑦𝑦 = 0.
Bila persamaan di atas disederhanakan, maka akan didapati: 𝑥𝑥(𝑦𝑦 + 㑣 ) + 𝑏𝑏𝑦𝑦 + 𝑏𝑏𝑎𝑎 = 𝑏𝑏𝑎𝑎
Lihat, kawan-kawan! Ada tambahan suku ba baik di ruas kiri maupun di ruas kanan. Itulah makna dari frasa bagaimana pun | 242
caranya, agar menjadi lebih mudah proses selanjutnya. Lihat saja! Persamaan yang telah ditambahi unsur ba itu, bila disusun ulang, akan diperoleh: 𝑥𝑥(𝑦𝑦 + 𝑎𝑎) + 𝑏𝑏(𝑦𝑦 + 𝑎𝑎) = 𝑏𝑏𝑎𝑎
Dan akhirnya menjadi bentuk paling sederhana, yaitu: (𝑦𝑦 + 𝑎𝑎)(𝑥𝑥 + 𝑏𝑏) = 𝑏𝑏𝑎𝑎
Dan bentuk sederhana itu adalah perkalian dua unsur yang menghasilkan nilai tertentu, yaitu unsur ba,”pungkas Sabiq Sabiq kembali menuliskan sesuatu di papan tulis. Tidak! Ternyata dia menulis ulang persamaan yang tadi ditulis oleh Amer. “Mer, aku yakin setelah mendapat penjelasan tentang Simon’s Trick, kau bisa selesaikan persamaan ini: 2 3 + = 1 ⇔ 2𝑛𝑛 + 3𝑚𝑚 = 𝑚𝑚𝑛𝑛 𝑚𝑚 𝑛𝑛
Persamaan, yang tadi kau berhenti saat menceritakannya,” pancing Sabiq. Amer kembali maju ke depan. “Baik dari bentuk persamaan terakhir, yaitu: 2𝑛𝑛 + 3𝑚𝑚 = 𝑚𝑚𝑛𝑛
dapat ditulis ulang dengan bantuan Simon’s Trick, menjadi: 𝑚𝑚𝑛𝑛 − 2𝑛𝑛 − 3𝑚𝑚 = 0
Dan langkah berikutnya, menambah faktor ba dalam hal ini mn di kedua ruasnya, sehingga: 𝑚𝑚𝑛𝑛 − 2𝑛𝑛 − 3𝑚𝑚 + (2𝑥𝑥3) = 0 + (2𝑥𝑥3) 𝑚𝑚𝑛𝑛 − 3𝑚𝑚 − 2𝑛𝑛 + 6 = 6 𝑚𝑚(𝑛𝑛 − 3) − 2𝑛𝑛 + 6 = 6
Sehingga, bila dikelompokkan akan diperoleh: | 243
𝑚𝑚(𝑛𝑛 − 3) − 2(𝑛𝑛 − 3) = 6
Akhirnya, terjadi persamaan: (𝑚𝑚 − 2)(𝑛𝑛 − 3) = 6
Wah, memang joss trik Si Simon’s itu, Biq! Kawan-kawan, ternyata aku pun bisa,”teriak Amer sekencang-kencangnya. “Jadi hutang janjiku sudah terpenuhi, Kawan?”sindir Sabiq. Amer hanya bisa mengangguk. “Baik sebenarnya Amer belum menjawab persoalan yang diajukan. Dia hanya sampai pada menemukan bentuk paling sederhana persamaan yang bisa difaktorkan. Kita kembali ke persoalan, bahwa diminta untuk menemukan ‘semua pasangan bilangan asli m dan n yang memenuhi persamaan: 2 3 + =1 𝑚𝑚 𝑛𝑛
Dari bentuk akhir yang ditulis oleh Amer terlihat bahwa pasangan yang memenuhi adalah faktor dari bilangan 6, dan itu merupakan hasil perkalian dua suku di sebelah kiri. Sehingga, pasangan yang memenuhi adalah sebagai berikut: (𝑚𝑚 − 2)(𝑛𝑛 − 3) = 6 ⇔ (1,6); (2,3); (3,2); (6,1)
(1,6) ⇔ 𝑚𝑚 − 2 = 1 ⇔ 𝑚𝑚 = 3 𝑑𝑑𝑎𝑎𝑛𝑛 𝑛𝑛 − 3 = 6 ⇔ 𝑛𝑛 = 9; (𝑚𝑚, 𝑛𝑛) = (3,9)
(2,3) ⇔ 𝑚𝑚 − 2 = 2 ⇔ 𝑚𝑚 = 4 𝑑𝑑𝑎𝑎𝑛𝑛 𝑛𝑛 − 3 = 3 ⇔ 𝑛𝑛 = 6; (𝑚𝑚, 𝑛𝑛) = (4,6)
(3,2) ⇔ 𝑚𝑚 − 2 = 3 ⇔ 𝑚𝑚 = 5 𝑑𝑑𝑎𝑎𝑛𝑛 𝑛𝑛 − 3 = 2 ⇔ 𝑛𝑛 = 5; (𝑚𝑚, 𝑛𝑛) = (5,5)
| 244
(6,1) ⇔ 𝑚𝑚 − 2 = 6 ⇔ 𝑚𝑚 = 8 𝑑𝑑𝑎𝑎𝑛𝑛 𝑛𝑛 − 3 = 1 ⇔ 𝑛𝑛 = 4; (𝑚𝑚, 𝑛𝑛) = (8,4)
Sehingga, seluruh pasangan m dan n yang memenuhi adalah: (3,9), (4,6), (5,5), dan (8,4), Kawan-kawan,”papar Sabiq. Terdengar tepuk tangan meriah memecah keseriusan kesebelas anak muda itu. Tapi, tidak dengan Ayra. Tampaknya masih ada yang mengganjal baginya. “Ada apa, Ayra? Ada yang kurang pas?”tanya Sabiq. “Jujur, aku harus lebih serius memerhatikan penjelasanmu. Mungkin dengan berlatih terus akan terbiasa,”jawab Ayra. “Begini saja, apa kau mau mengerjakan satu persoalan lagi buat kita semua, Ayra?”tantang Sabiq. Ayra mengangguk tanda setuju. “Baik,
mungkin kau bisa dari persoalan ini, Ayra!
‘Tentukanlah semua pasangan bilangan bulat positif (x,y) yang memenuhi persamaan xy + x – 2y = 9!”tantang Sabiq “Baik aku coba selesaikan, Kawan-kawan. Bila persamaan tersebut kita susun ulang akan didapatkan bentuk: 𝑥𝑥𝑦𝑦 + 𝑥𝑥 − 2𝑦𝑦 = 9
Karena nilai ba adalah = - 2, maka persamaan itu bisa kita modifikasi bentuknya menjadi: 𝑥𝑥𝑦𝑦 + 𝑥𝑥 − 2𝑦𝑦 + (−2) = 9 + (−2) 𝑥𝑥𝑦𝑦 + 𝑥𝑥 − 2𝑦𝑦 − 2 = 9 − 2
𝑥𝑥(𝑦𝑦 + 1) − 2(𝑦𝑦 + 1) = 7
(𝑥𝑥 − 2)(𝑦𝑦 + 1) = 7
| 245
Alhamdulillah, aku semakin paham, Biq. Terima kasih, Kawankawan!”seru Ayra. “Subhanallah, kau cepat sekali memahaminya, Ayra,”puji Fateh. “Dan langkahmu tinggal mencari hasil kali 2 bilangan yang nilanya adalah 7. Karena 7 adalah bilangan prima, maka kedua bilangan itu pastilah 1 dan 7 itu sendiri,”timpal Iman. “Ya, dan itu berarti pasangan (x,y) yang memenuhi adalah: (1,7) dan (7,1), sehingga: (1,7) ⇔ 𝑥𝑥 − 2 = 1 ⇔ 𝑥𝑥 = 3 𝑑𝑑𝑎𝑎𝑛𝑛 𝑦𝑦 + 1 = 7 ⇔ 𝑦𝑦 = 6; (𝑥𝑥, 𝑦𝑦) = (𝟑𝟑, 𝟔𝟔)
(7,1) ⇔ 𝑥𝑥 − 2 = 7 ⇔ 𝑥𝑥 = 9 𝑑𝑑𝑎𝑎𝑛𝑛 𝑦𝑦 + 1 = 1 ⇔ 𝑦𝑦 = 0; (𝑥𝑥, 𝑦𝑦) = (9,0)
Namun karena yang dicari adalah pasangan bilangan bulat positif, maka untuk nilai y = 0 tidak memenuhi,”tegas Ayra bersemangat. “Plok, plok, plok, plok, plok!”terdengar suara tepukan meriah “Kawan-kawan, bukankah kita sudah membuktikan bahwa ketekunan dan kesabaran yang dipadu dengan kerja keras dan kerja cerdas, insyaallah berbuah hasil yang terbaik!”sitir Sabiq saat semua kawannya tampak senang telah berhasil menyelesaikan persoalan yang sudah dipersiapkan sebelumnya. Matahari semakin tinggi, beberapa saat lagi ia akan berada tepat di atas kepala sebelas anak muda yang begitu bersemangat dan puas pagi menjelang siang itu. Mereka bubar dan siap-siap berwudhu’ untuk sholat Dzuhur. …∑πχαΩ… | 246
14
The Queen of the Problems “Mer, bukankah di antara kita, kau yang paling paham soal makan-memakan,
dimakan,
dan
makanan?”tiba-tiba
Sabiq
meninggikan suaranya dalam antrian makan siang selepas sholat Dzuhur sehingga semua kawannya jelas mendengar. “Maksudmu apa, Kawan?”jawab Amer. “Tapi, aku ragukan kalau kau juga paham soal makanan yang satu ini?”sindir Sabiq. Ayra, Fateh, Denan, Bintang, Bily, dan Ikram saling pandang. Sementara yang lain masih menunggu antrian mengambil makanan. “Ha....ha...ha..., asalkan kau taruh kata makan, atau makanan, dimakan, termakan, memakan, atau apapun yang ada kata makan-nya dalam soal itu, insyaallah aku bisa, Kawan!”balas Amer lantang. Kali ini bukan hanya Ayra, Fateh, Denan, Bintang, Bily, dan Ikram, tapi Yodha, Saver, dan Iman pun saling pandang. Mereka berusaha menebak apa yang akan dikatakan Sabiq. “Baik kalau begitu kawan. Kau lihat kan Mer, bentuk tempat makan siang kita ini! Di ujung sana, ada Tempat Nasi yang besar, di kanannya terdapat Tumpukan Piring, Tempat Sendok dan Garpu, Tempat Buah, dan Kertas Tisu. Sementara di sebelah kirinya terdapat, Tempat Sayur Lodeh, Sepiring Ikan Asin, Semangkuk | 247
Sambal, dan Sepiring Tahu dan Tempe Goreng. Sisi kanan dan sisi kiri, keduanya dipertemukan dengan tumpukan Seteko Besar Air Minum,”kata Sabiq. “Iya, benar, kau Sastrawan. Lantas apa masalahnya dengan susunan seperti itu, Kawan?”tanya Amer. Diantara sebelas anak muda yang mau makan siang itu, hanya Saver, Ikram, Ayra, Yodha, Bintang, dan Fateh tentunya, yang tampak senyum-senyum. “Dasar kau, Biq. Kita saat ini mau makan!”sela Fateh pelan. Sabiq hanya senyum-senyum simpul sambil mengedipkan mata kirinya pada Fateh. “Jadi, apa masalahnya, Kawan?”tanya Amer penasaran. “Oke. Andaikan Tempat Nasi yang di ujung itu tetap pada posisinya, sementara yang lain, Tumpukan Piring, Tempat Sendok dan Garpu, Tempat Buah, Kertas Tisu, Tempat Sayur Lodeh, Sepiring Ikan Asin, Semangkuk Sambal, dan Sepiring Tahu dan Tempe Goreng, serta Seteko Besar Air Minum, bisa dan boleh berpindah-pindah. Coba kau hitung ada berapa cara untuk menyajikan menu siang ini?”tanya Sabiq. “Aku langsung bisa pastikan jawaban Amer ‘tidak tahu’, Biq!”potong Ayra. Dan Amer hanya diam. Tanpa ekspresi. “Lho, kok?”Sabiq heran. “Pertanyaanmu salah dan kurang lengkap, Kawan?”sela Fateh. Sabiq bingung, dia coba mengingat apa kalimat pertanyaannya pada Amer tadi. | 248
“Ya, Biq. Pertanyaannmu harus kau revisi!”timpal Saver. Kali ini Sabiq senyum-senyum. Tampaknya dia sadar akan kekeliruan pertanyaannya. “Oke, aku ganti pertanyaannya, Kawan. ‘Bila Tempat Nasi yang di ujung itu tetap pada posisinya, sementara yang lain, Tumpukan Piring, Tempat Sendok dan Garpu, Tempat Buah, Kertas Tisu, Tempat Sayur Lodeh, Sepiring Ikan Asin, Semangkuk Sambal, dan Sepiring Tahu dan Tempe Goreng, serta Seteko Besar Air Minum, bisa dan boleh berpindah-pindah. Ada berapa carakah untuk menyajikan menu makan siang ini?”tanya Sabiq. “Ha....ha....ha. Santai Bro! Beri aku waktu nikmati makan siang ini dulu. Setelah itu aku jawab pertanyaanmu,”jawab Amer. “Baik, kawan-kawaan kita tunggu jawaban Amer selesai makan siang, oke?”kata Sabiq pada teman-temannya. “Setuju!”terdengar jawaban serempak. Satu per satu sebelas anak itu mengisi piring dan gelas mereka dengan menu makan siang hari itu. Tampak keceriaan menghiasi raut wajah-wajah mereka. Tampaknya perpaduan Nasi Putih, Sayur Lodeh, Ikan Asin, Sambal, Tahu dan Tempe Goreng, adalah menu yang paling pas untuk situasi pedesaan yang udaranya masih asri ini. Tidak lebih dari 25 menit, kesebelas sahabat itu telah menyelesaikan makan siang mereka. Dan kali ini adalah saat makan siang yang spesial. Mengapa? Karena Amer akan memberi penjelasan tentang kemungkinan susunan menu makan siang mereka hari ini.
| 249
“Biq, seperti tadi aku katakan bahwa apa pun persoalan, asalkan kau taruh kata makan, atau makanan, dimakan, termakan, memakan, atau apapun yang ada kata makan-nya dalam soal itu, insyaallah aku bisa, Kawan. Kaitan dengan persoalan yang kau ajukan tadi, kira-kira begini penyelesaiannya,”Amer membuka pembicaraan. Semua kawannya serius menunggu apa yang akan dijelaskan oleh Amer. Mereka menunggu. Dan tak perlu lama. “Untungnya dalam revisi pertanyaanmu tadi ada kata makannya. Karenanya, tiba-tiba saja otakku berputar cepat tentang apa yang disebut dengan Permutasi, Kawan!”jelas Amer memulai keterangannya. “Hanya saja, Permutasi jenis ini adalah Permutasi Khusus. Permutasi yang terjadi pada kondisi sirkular dengan salah satu komponennya pada posisi tetap. Dalam bahasa asing dikenal sebagai Circular Permutation, dan kita menyebutnya sebagai Permutasi Siklis, Kawan!”terang Amer lagi. Kali ini wajah Ayra, Fateh, Denan, Bintang, Bily, Ikram, Yodha, Saver, dan Iman, tampak sangat serius. Suasana begitu hening. Andai pada waktu itu, sebatang jarum jatuh dari meja ke lantai pun, tampaknya suaranya akan terdengar jelas. “Karenanya, bila kita andaikan masing-masing obyek adalah: n1 = Tempat Nasi, n2 = Tumpukan Piring, n3 = Tempat Sendok dan Garpu, n4 = Tempat Buah, | 250
n5 = Kertas Tisu, n6 = Tempat Sayur Lodeh, n7 = Sepiring Ikan Asin, n8 = Semangkuk Sambal, n9 = Sepiring Tahu dan Tempe Goreng, dan n 10 = Seteko Besar Air Minum, Dengan menganggap bahwa N1 adalah berada pada posisi tetap, maka akan terjadi permutasi siklis dengan 10 obyek. Sebelum sampai menghitung, andai kita gambarkan situasinya adalah seperti gambar di bawah ini,”tiba-tiba Amer menghentikan penjelasannya Amer melihat ke kiri dan ke kanan. Dan terdapat kertas bekas yang tergeletak di meja. Ia menggambar. “Perhatikan kawan-kawan! Berikut adalah ilustrasi singkat untuk jumlah obyek dari 1 sampai 4.
Terlihat bahwa: Untuk n = 1 obyek
(1-1)! = 0!
= 1 cara
n = 2 obyek
(2-1)! = 1!
= 1 cara
n = 3 obyek
(3-1)! = 2!
= 1x2
= 2 cara
n = 4 obyek
(4-1)! = 3!
= 1x2x3
= 6 cara
.
.
=.
=.
Sehingga pola umum untuk Permutasi Siklis adalah sebagai berikut: 𝒏𝒏 = 𝒏𝒏 𝒐𝒐𝒃𝒃𝒚𝒚𝒚𝒚𝒚𝒚 ⟺ (𝒏𝒏 − 𝟏𝟏)! = 𝟏𝟏𝒙𝒙𝟐𝟐𝒙𝒙𝟑𝟑𝒙𝒙𝟒𝟒𝒙𝒙𝟓𝟓𝒙𝒙 … 𝒙𝒙 (𝒏𝒏 − 𝟏𝟏) 𝒄𝒄𝒂𝒂𝒓𝒓𝒓𝒓
| 251
Karenanya, untuk 10 obyek dapat dihitung menjadi: (10 − 1)! = 1𝑥𝑥2𝑥𝑥3𝑥𝑥4𝑥𝑥5𝑥𝑥6𝑥𝑥7𝑥𝑥8𝑥𝑥9 cara
(10 − 1)! = 362.880 𝑐𝑐𝑎𝑎𝑟𝑟𝑎𝑎!
Begitu penjelasanku, Bro! Bagaimana Kawan?”terang Amer. “Subhanallah, memang betul bahwa segala persoalan yang ada hubungannya dengan makan, termakan, dimakan, memakan, makanan, atau apapun yang ada frasa makannya, kau jagonya, Kawan! Jawabanmu tepat. Maafkan aku sempat meragukanmu”balas Sabiq. Kali ini, Sabiq pun kembali mengedipkan matanya pada Fateh. Fateh hanya mengangguk dan tersenyum kecil. “Plok, plok, plok, plok, plok,”terdengar tepukan meriah dari kawan-kawan Amer. “Baik kawan-kawan, 15 menit setelah ini kita berkumpul di ruang bahas tadi malam. Kita mulai pembahasan untuk OSN Matematika SMP Tingkat Propinsi tepat pukul 13.30,”kata Ayra mengomando teman-temannya. Itulah sebelas sahabat yang selalau punya cerita. Dan Sabiq yang didaulat sebagai Satrawan, Penerjemah, sekaligus Dukun itu, selalu punya cara menghidupkan suasana. Dia tahu betul sifat kawankawannya. Begitupun sebaliknya, kawan-kawannya tahu betul sifat Sabiq. …∑πχαΩ…
Jarum panjang pada jam dinding di ruang itu berhimpit dengan angka 5, sementara jarum pendeknya hampir berada di tengah-tengah | 252
antara angka 1 dan 2. Meski masih lima menit dari tengat yang telah mereka sepakati untuk berkumpul, kesebelas kawan itu telah lebih dahulu masuk ke dalam ruangan. Hingga tiba saat yang ditentukan, Sabiq pun membuka acara itu resmi. “Baiklah kembali kita buka diskusi ini. Sebelumnya perlu dipahami bahwa OSN SMP Matematika Tingkat Propinsi tahun kemarin terdiri atas 2 bagian, Bagian A dan Bagian B. Bagian A terdiri atas 10 soal sementara Bagian B terdiri atas 5 soal. Karenanya, seperti telah kita sepakati aspek keadilan dan pengalaman sebagai dasar belajar kita, maka untuk Bagian A masing-masing dari kita menjelaskan 1 soal, sementara untuk Bagian B, kembali masing-masing kelompok menjelaskan 1 soal lagi,”terang Sabiq. “Nah, posisimu bagaimana, Kawan?”tanya Bily. “Seperti biasa, aku menjadi pengarah diskusi ini dan Ban Serep, Kawanku Bily!”jawab Sabiq. “Ha...ha...ha..,”terdengar tawa lepas dari ruang di mana sebelas sahabat itu sedang berdiskusi. “Baik, siapa yang bersedia menyelesaikan soal Nomor 1, soal tentang persamaan linier?”tanya Sabiq. “Biarkan aku mencoba Kawan!”jawab Bily spontan. “Silahkan, Bil!”balas Sabiq. Bily maju, dia menerima alat tulis, dan mulai menuliskan yang dia maksud. “Sebelumnya, sebaiknya kau bacakan buat kita semua soalnya, Bil!”terdengar Saver bersuara dari belakang. | 253
“Oh, ya. Begini persoalannya, Kawan. ‘diketahui x dan y adalah bilangan positif. Salah satu solusi dari persamaan 20x + 14y = 2014 adalah (x,y) = (100,1). Salah satu solusi yang lain adalah...,”kata Bily. Kembali Bily berkonsentrasi dengan maksudnya. “Ini merupakan persamaan linier dengan
2 variabel yang
untungnya salah satu dari penyelesaiannya sudah diketahui. Sehingga untuk mencari yang lain, cukup dengan mensubstitusikan nilai x dan y serta melihat pola persamaan yang dihasilkan. Karenanya: 20x +14y = 2014, atau 10x +7 y = 1007, atau 10𝑥𝑥 = 1007 − 7𝑦𝑦 … … … … … … … … … . . (14.1) 𝑥𝑥 =
1007 − 7𝑦𝑦 … … … … … … … … … … . (14.2) 10
Terlihat bahwa persamaan (14.1) dan persamaan (14.2) merupakan bentuk umum dari persoalan di atas. Sehingga, dapat kita katakan bahwa hasil kali 7y haruslah menghasilkan bilangan yang memiliki satuan 7, karena 10x minimal akan menghasilkan bilangan puluhan,”terang Bily. “Itu coba-coba dengan mancari-cari, namanya, Bil!”potong Amer. “Ya, dan sah-sah saja, Kawan! Bukankah trial and error adalah hal yang bisa diterima dalam Matematika dan sains pada umumnya! Bukankah begitu, Biq?”tanya Bily balik. Sabiq tersenyum dan mengangguk. “Ya, tapi apa hanya itu caranya?”tanya Denan pada Bily. | 254
“Setidaknya barus cara itu yang aku ketahui, Kawan!”balas Bily. “Oke, teruskan saja dulu Bil,”pinta Fateh. “Baik. Sekali lagi bahwa kata kuncinya 7y haruslah menghasilkan bilangan yang memiliki satuan 7, maka dapat dipastikan bahwa angka satuan dari bilangan yang akan dikalikan adalah satu. Selanjutnya dilakukan proses sehingga memenuhi persamaan dengan batasan bilangan bulat positif. Karenanya, dengan tebakan awal y = 11, misalnya, akan diperoleh: untuk y = 11
⟺ 10x = 1007 – 7(11) ⟺ x = (1007-77)/10 ⟺ x = 930/10 ⟺ x = 93
untuk y = 21
⟺ pasangan (x,y) = (93,11) ⟺ 10x = 1007 – 7(21) ⟺ x = (1007-147)/10 ⟺ x = 860/10 ⟺ x = 86
⟺ pasangan (x,y) = (86,21)
Dan seterusnya, sampai syarat yang ditentukan bisa dipenuhi. Sehingga, bila dibuat tabulasi dengan memecah masing-masing kolom berdasarkan atas besaran-besaran yang terdapat pada persamaan (14.2), yaitu: y; 7y; (1007-7y) dan x, akan dihasilkan nilai pasangan (x,y) untuk y = (11); (21); (31); (41); (51); (61); (71); (81); (91); (101); (111); (121); (131); (141); dan (151), yang merupakan jawaban atas pertanyaan di atas sebagai berikut: | 255
y
7y
1007-7y
x
(x,y)
11
77
930
(930)/10= 93
(93,11)
21
147
860
(860)/10= 86
(86,21)
31
217
790
(790)/10= 79
(79,31)
41
287
720
(720)/10= 72
(72,41)
51
357
650
(650)/10= 65
(65,51)
61
427
580
(580)/10= 58
(58,61)
71
497
510
(510)/10= 51
(51,71)
81
567
440
(440)/10= 44
(44,81)
91
637
370
(370)/10= 37
(37,91)
101
707
300
(300)/10= 30
(30,101)
111
777
230
(230)/10= 23
(23,111)
121
847
160
(160)/10= 16
(16,121)
131
917
90
(90)/10=9
(9,131)
141
987
20
(20)/10= 2
(2,141)
151
1057
-50
(-50)/10= -5
(-5,151)
Tampak bahwa pasangan (x,y) sebagai salah satu solusi yang lain adalah, salah satu diantara pasangan: {(2,141), (86,21), (79,31), (72,41), (65,51), (58,61), (51,71), (44,81), (37,91), (30,101), (23,111), (16,121), (9,131), (93,11)}. Sementara untuk pasangan (x,y) = (-5, 151) tidak memenuhi, karena bukan bilangan bulat positif, melainkan bilangan bulat negatif. Demikian penjelasanku, Kawan-kawan,”terang Bily. “Mantab, Bro! Sukses kau Kawan!”puji Fateh. “Terima kasih, Fateh,”balas Bily. “Selanjutnya, siapa yang akan menjelaskan penyelesaian Nomor 2?”tanya Sabiq.
| 256
“Beri aku kesempatan untuk menjawab persoalan itu, kawankawan!”Denan maju menyodorkan dirinya. “Silahkan, Nan!”balas Sabiq. Denan maju, sikapnya sederhana saja. Tapi kemantapan hatinya tampak dari optimisme yang tergambar di wajahnya. “Soal ini tentang bilangan dan operasinya. Lebih tepatnya operasi perkalian dengan syarat tertentu. Disebutkan bahwa ‘jika x dan y merupakan bilangan real yang memenuhi x2 + y2 = 1, maka nilai terbesar dari perkalian x dan y adalah ....’.,”terang Denan memulai penjelasannya. Seperti biasa semua kawannya serius menyimak apa yang disampaikan oleh Denan. “Untuk sampai ke persoalan, ada beberapa sifat perkalian dan perpangkatan yang harus kita ingat kembali, yaitu, bahwa: (𝑥𝑥 + 𝑦𝑦)2 = 𝑥𝑥 2 + 𝑦𝑦 2 + 2𝑥𝑥𝑦𝑦 … … … … . (14.3)
Dari persamaan (1) di atas kita dapat menemukan hasil perkalian x dan y, di mana: 2𝑥𝑥𝑦𝑦 = 𝑥𝑥 2 + 𝑦𝑦 2 − (𝑥𝑥 + 𝑦𝑦)2
Sementara dari informasi soal kita ketahui bahwa: 𝑥𝑥 2 + 𝑦𝑦 2 = 1 … … … … … … … … … … . … (14.4)
Dengan mensubstitusikan persamaan (14.4) ke persamaan (14.3) akan diperoleh persamaan yang baru: 2𝑥𝑥𝑦𝑦 = 1 − (𝑥𝑥 + 𝑦𝑦)2
1 (𝑥𝑥 + 𝑦𝑦)2 … … … … … … … … … … … … . (14.5) 𝑥𝑥𝑦𝑦 = − 2 2 | 257
Dari persamaan (14.5) terlihat bahwa xy merupakan bentuk dari perkalian x dan y seperti yang diminta oleh soal. Persoalannya, kapankah memberikan nilai terbesar?”papar Denan bersemangat. Denan berhenti sejenak, dia meneguk setengah gelas air putih. “Terlihat jelas bahwa bentuk perkalian xy pada persamaan (14.5) di atas mengandung dua suku, dapat diidentikkan dengan: xy = A + B di mana; A = ½; dan B = −
(𝑥𝑥+𝑦𝑦)2 2
Suku pertama bernilai positif sementara suku kedua bernilai negatif. Karenanya: Nilai perkalian xy akan menghasilkan nilai terbesar bila suku kedua, yang bernilai negatif, berubah menjadi positif. Masalahnya suku kedua (𝑥𝑥 + 𝑦𝑦)2 merupakan suku berpangkat 2
yang akan selalu bernilai positif atau nol untuk bilangan real. Hal ini sesuai dengan syaratkan oleh soal bahwa x dan y merupakan bilangan real. Sehingga, jawaban atas pertanyaan di atas adalah nilai terbesar dari perkalian x dan y adalah ½,”terang Denan dengan penuh semangat. “Tunggu dulu, Bro. Tadi kau sebutkan bahwa ‘masalahnya suku kedua (𝑥𝑥 + 𝑦𝑦)2 merupakan suku berpangkat 2 yang akan
selalu bernilai positif atau nol untuk semua bilangan real’. Apakah bisa
bilangan
berpangkat
2
atau
kuadrat
bernilai
negatif,
Kawan?”sela Bintang. “Bisa, Kawan! Kalau bilangan itu bukan real!”jawab Denan serius. | 258
Hampir semua yang ada di ruang itu heran. Hanya Sabiq dan Yodha yang tampak tenang-tenang saja. Ekspresi mereka datar. “Maksudmu ada bilangan yang bila dikuadratkan hasilnya negatif,
begitu
tepatnya
Kawan?”tanya
Amer
mempertegas
pertanyaan Bintang. “Ya, ada! Dan bilangan itu yang disebut sebagai bilangan tidak real alias bilangan bukan real, Kawan. Tampaknya Yodha dan Sabiq yang pantas menjelaskan persoalan bilangan bukan real ini kepada kita,”terang Denan. Yodha yang disebut namanya, melirik ke arah Sabiq. Sabiq mengangguk dengan senyum khas. “Ya, jawaban Denan benar adanya. Memang berdasarkan teori bahwa secara umum bilangan dapat dibagi menjadi dua yaitu bilangan real dan bilangan tidak real. Selanjutnya, bilangan real dibagi menjadi dua yaitu bilangan rasional dan bilangan tak rasional. Nah, dari sini agak lebih jelas karena bilangan rasional adalah bilangan yang penulisannya dapat diubah ke dalam bentuk (a/b) dengan a dan b keduanya bilangan bulat dan b ≠ 0, sedangkan bilangan tak rasional adalah bilangan yang tak dapat diubah dalam bentuk (a/b) dengan a dan b keduanya bilangan bulat dan b ≠ 0. Contoh paling sederhana bilangan tak rasional adalah √2, e, π, dan log a,”terang Yodha. “Tapi, semua yang kau jelaskan itu masih dalam wilayah bilangan real, Kawan. Kau belum jelaskan apa itu bilangan tidak real itu!”potong Ayra.
| 259
“Benar Ayra. Aku teruskan dulu sedikit tentang teori bilangan tadi. Selanjutnya, bilangan rasional dapat dibagi ke dalam dua kelompok yaitu bilangan bulat dan bilangan pecahan. Nah, terkait dengan inti persoalan kita yaitu tentang bilangan bukan real atau tidak real, bilangan jenis itu adalah bilangan imajiner (imaginary number) atau sering disebut juga sebagai bilangan kompleks (complex number). Bilangan jenis ini dilambangkan dengan huruf i atau j, dimana: 𝑖𝑖 = √−1,”terang Yodha.
“Bisa kau berikan contoh, Kawan?”tanya Fateh. “Ya, saat kita membahas tentang Diskriminan pada persamaan
kuadarat. Apa yang kau ingat tentang Diskriminan pada persamaan kuadrat ax2 +bx +c = 0, Fateh?”tanya Yodha balik. “Ada beberapa hal yang terkandung dalam persamaan kuadrat ax2 +bx +c = 0, diantaranya terdapat 2 akar x yang memenuhi persamaan di atas di mana: 𝑥𝑥1,2 =
−𝑏𝑏 ± √𝑏𝑏 2 − 4𝑎𝑎𝑐𝑐 2𝑎𝑎
Dan sesungguhnya merupakan pengembangan kreatif dari kuadrat sempurna, yaitu: 𝑎𝑎 �𝑥𝑥 +
𝑏𝑏 2 𝑏𝑏 2 𝑏𝑏 2 − 4𝑎𝑎𝑐𝑐 � = 𝑎𝑎𝑥𝑥 2 + 𝑏𝑏𝑥𝑥 + 𝑎𝑎 � 2 � = 𝑎𝑎𝑥𝑥 2 + 𝑏𝑏𝑥𝑥 + 𝑐𝑐 + 4𝑎𝑎 4𝑎𝑎 2댦
Selanjutnya dari suku 𝑏𝑏 2 − 4𝑎𝑎𝑐𝑐 dikenal apa yang disebut sebagai Diskriminan, dilambangkan huruf D dengan 3 kemungkinan, yaitu: (1) 𝑏𝑏 2 − 4𝑎𝑎𝑐𝑐 > 0; (2) 𝑏𝑏 2 − 4𝑎𝑎𝑐𝑐 = 0
(3) 𝑏𝑏 2 − 4𝑎𝑎𝑐𝑐 < 0,”terang Fateh lagi.
| 260
“Ada hal yang lain, Fateh?”tanya Yodha lagi. Sementara kedua kawan itu sedang berdiskusi, sembilan sahabat mereka serius menikmati diskusi itu, meskipun mereka sama-sama tahu bisa saja masuk dalam diskusi itu, kapan pun mereka mau. Itu adalah code of conduct yang telah mereka sepakati jauh-jauh hari sebelumnya. “Ya, terhadap kondisi pertama, dimana 𝑏𝑏 2 − 4𝑎𝑎𝑐𝑐 > 0, hal itu
berarti bahwa dari persamaan ax2 +bx +c = 0 terdapat 2 nilai x yang memenuhi dan merupakan bilang real, yaitu:
dan,
𝑥𝑥1 = 𝑥𝑥2 =
−𝑏𝑏 + √𝑏𝑏 2 − 4𝑎𝑎𝑐𝑐 2𝑎𝑎
−𝑏𝑏 ± √𝑏𝑏 2 − 4𝑎𝑎𝑐𝑐 2𝑎𝑎
Yang secara grafis dapat digambarkan sebagai berikut:
Dan terhadap kondisi kedua, dimana 𝑏𝑏 2 − 4𝑎𝑎𝑐𝑐 = 0, hal itu berarti
bahwa dari persamaan ax2 +bx +c = 0 terdapat hanya 1 nilai x yang memenuhi dan merupakan bilangan real. Satu nilai x ini sering disebut sebagai akar yang berulang, dan dari sisi grafik dapat disebut sebagai garis singgung terhadap sumbu-x, seperti ilustasi gambar di bawah ini: | 261
Sementara terhadap kondisi ketiga, dimana 𝑏𝑏 2 − 4𝑎𝑎𝑐𝑐 < 0, hal ini
berarti bahwa dari persamaan ax2 +bx +c = 0, tidak terdapat 1 titik pun dari grafik yang bersinggungan atau berpotongan dengan sumbu-x
kapan
dan
dimanapun,
sehingga
dikatakan
tidak
mempunyai akar real,”terang Fateh lagi. “Nah, dari keteranganmu tadi bahwa pada kondisi nilai Diskriminan yang lebih kecil dari nol itu, adakah grafik fungsi kuadratnya?”tanya Yodha. “Ada! Grafiknya seperti gambar di bawah ini salah satunya, pada keadaan nilai a > 0!”jelas Fateh.
Tampak semua kawan mereka semakin serius mengamati setiap kalimat dan gambar grafik yang disampaikan oleh Fateh dan Yodha. “Kondisi ketiga itulah konteks persoalan yang kita hadapi. Bahwa pada kondisi 3 itu dikenal istilah yang disebut sebagai akarakarnya tidak nyata atau bukan real tetapi memiliki akar-akar kompleks atau akar-akar imajiner. Konsekuensinya grafik yang | 262
dihasilkan tidak akan pernah berpotongan dengan sumbu-x. Dengan kata lain, fungsi y = ax2 +bx +c akan bernilai positif untuk semua nilai x bila a > 0, dan akan bernilai negatif untuk semua nilai x bila a < 0,”terang Yodha. “Alhamdulillah, kami paham penjelasanmu. Tapi, mana contoh konkretnya, Kawan!”terdengar suara Amer cukup keras memenuhi ruangan. “Baik. Mari sama-sama kita selesaikan dan cari himpunan penyelesaian dari persamaan kuadrat y = x2 – 2x + 6, misalnya!”balas Yodha. Tampak semua yang hadir di ruang itu sibuk mengambil alat tulis mereka masing-masing. “Yang jelas titik potong terhadap sumbu-y, saat x = 0, adalah pada koordinat (x,y) = (0,6), Bro!”jawab Amer paling awal. “Terhadap sumbu-x, bagaimana Kawan Amer?”tanya Yodha. “Berarti, saat y = 0, maka nilai x2 – 2x + 6 = 0, sehingga...”terdengar gumaman Saver dan terhenti pada titik-titik. “Tidak ditemukan untuk semua bilangan real, kan!”tanya Yodha. “Betul, kau kawan. Aku mencoba menyelesaikannya, seperti ini: 𝑥𝑥 2 − 2𝑥𝑥 + 6 = 0. Karena tak bisa difaktorkan, aku cari jalan
dengan memisahkan sehingga persamaan di atas menjadi: (𝑥𝑥 − 1)2 + 5 = 0 (𝑥𝑥 − 1)2 = −5
Dan penyelesaiannya adalah: 𝑥𝑥1 = 1 + √−5 dan 𝑥𝑥2 = 1 − √−5
| 263
Tapi, sampai di titik ini, aku berhenti,”tiba-tiba Ayra ikut bersemangat. “Nah, untuk kondisi x harus bilangan Real maka Himpunan Penyelesaiannya adalah ∅ alias Himpunan Kosong. Tapi jika kita
menggunakan pendekatan bilangan Tak Real, akan diperoleh penyelesaian akar-akarnya sebagai berikut : 𝑥𝑥1 = 1 + √5𝑖𝑖 dan 𝑥𝑥2 = 1 − √5𝑖𝑖,
Karena i merupakan bilangan imajiner dimana i = √−1. Akibatnya, Himpunan Penyelesaiannya pun merupakan bilangan kompleks, sehingga HP {𝑥𝑥1 = 1 + √5𝑖𝑖 ; 𝑥𝑥2 = 1 − √5𝑖𝑖},”pungkas Yodha.
“Mumtaz! Aku pikir penjelasan Yodha tentang Teori Bilangan
khususnya Bilangan Tak Real sudah cukup, dan kita beri applause buat Denan dan Yodha!”Sabiq mengakhiri diskusi yang hangat itu. Kembali ruangan itu dipenuhi suara tepukan hangat. Tepukan dari sahabat untuk sahabat. “Sekarang, saatnya menikmati secangkir teh hangat, Kawankawan!”kembali Amer berseru “Setuju....! Ha....ha...ha..,”terdengar sahutan yang serempak dari sebelas sahabat itu. …∑πχαΩ…
Suasana rehat tea break kesebelas sahabat itu begitu meriah. Meski sedang menikmati secangkir teh hangat, masih terdengar diskusi seputar materi matematika yang tadi mereka diskusikan. Dan, 500 meter arah barat laut dari ruangan itu, tampak sepasang mata dengan
| 264
sorot mata tajam dan jenggot yang hampir memutih seluruhnya, memerhatikan kegiatan anak-anak muda itu. “Akhirnya, kalian menemukan jalan sendiri. Alhamdulillah. Teruslah berusaha anak-anak!”pikir pemilik sorot mata tajam itu. “Baiklah Kawan-kawan, siapa yang akan mencoba soal nomor 3 ini?”tanya Sabiq kepada kawan-kawannya yang telah kembali pada posisi untuk berdiskusi. “Aku, biarkan aku mencoba kemampuanku, Biq!”jawab Saver mantap. “Silahkan, Kawanku!”balas Sabiq. Saver maju dan menerima alat tulis. Dia menarik nafas dalamdalam, menghembuskannya ke luar dengan sedikit sentakan. “Kawan-kawan, terhadap persoalan nomor 3 ini, kita perlu ketelitian dan kecermatan memisahkan bidang. Disebutkan bahwa ‘sebuah lingkaran berada dalam seperempat lingkaran besar, seperti pada gambar di samping. Jika jari-jari lingkaran besar = 8 satuan, maka luas daerah yang diarsir adalah …”. Nah, mari kita perhatikan terlebih dahulu gambarnya, sebagai berikut,”kata Saver.
Data yang diberikan hanyalah seperempat lingkaran besar dan jarijari lingkaran besar = 8 satuan. Untuk itu kita harus menentukan apa tepatnya yang ditanya,”terang Saver. “Sudah terang, kan Ver! Daerah yang diarsir!”potong Fateh. | 265
“Betul, sudah terang. Tapi, yang aku maksud akan lebih jelas bila kita lengkapi dengan ilustrasi,”terang Saver. Saver mendekat ke papan tulis, ia mulai menggambar. “Mohon sama-sama kita perhatikan tambahan ilustrasi di bawah ini!”pinta Saver.
“Kawan-kawan, apa yang ada dalam pikiran Kawan-kawan setelah melihat ilustrasi ini sekarang?”tanya Saver. “Daerah yang diarsir dipersempit cakupannya,”jawab Bily. “Ada yang bisa menyebutkan lebih spesifik?”pancing Saver. “Luasan daerah yang diarsir adalah Luas Seperdelapan Juring Lingkaran Besar dikurangi Luas Segitiga dikurangi lagi oleh Luas Tiga-per-delapan Juring Lingkaran Kecil,”jawab Ayra. “Nah, ini yang aku maksud sebagai menentukan apa tepatnya yang ditanya itu, Kawan. Dan selanjutnya kita akan menghitung tahap demi tahap,”terang Saver. Tampak Sabiq, Fateh, Yodha, Ikram, dan Ayra menganggukangguk. Mereka merasa seperti berada pada tiap goresan Saver. “Kita mulai dari yang sederhana dan jelas bisa ditentukan, yaitu besar sudut Tiga-per-delapan Juring Lingkaran Kecil. Dari informasi soal kita ketahui bahwa: 𝐴𝐴𝐵𝐵 = 𝐴𝐴𝐷𝐷 = 𝐴𝐴𝐶𝐶 = 𝑅𝑅 = 8 satuan
| 266
Karena ΔBAD siku-siku, maka ∠𝐵𝐵𝐴𝐴𝐶𝐶 =900/2= 450 ∠A𝑃𝑃𝑂𝑂 = ∠𝑃𝑃𝑂𝑂𝑄𝑄 = 900 Maka ∠𝐴𝐴𝑂𝑂𝑃𝑃 = (1800 – ∠𝐴𝐴𝑃𝑃𝑂𝑂 – ∠𝐵𝐵𝐴𝐴𝐶𝐶) = (1800 – 900 – 450) = 450 Karena ∠𝐶𝐶𝑂𝑂𝑃𝑃 + ∠𝐴𝐴𝑂𝑂𝑃𝑃 = 1800 Maka ∠𝐶𝐶𝑂𝑂𝑃𝑃 = 1800 − ∠𝐴𝐴𝑂𝑂𝑃𝑃 = 1800 − 450 = 1350 Selanjutnya kita perlu mengetahui panjang jari-jari lingkaran kecil, yaitu: 𝑂𝑂𝐶𝐶 = 𝑂𝑂𝑃𝑃 = 𝑂𝑂𝑄𝑄
Misalkan 𝑂𝑂𝐶𝐶 = 𝑂𝑂𝑃𝑃 = 𝑂𝑂𝑄𝑄 = 𝑟𝑟
dan AP = OQ ; OP = AQ Maka 𝑂𝑂𝐴𝐴
= 𝐴𝐴𝐶𝐶 − 𝑂𝑂𝐶𝐶 = 8 – 𝑟𝑟
Dari 𝑠𝑠𝑒𝑒𝑔𝑔𝑖𝑖𝑡𝑡𝑖𝑖𝑔𝑔𝑎𝑎 𝑠𝑠𝑖𝑖𝑘𝑘𝑢𝑢−𝑠𝑠𝑖𝑖𝑘𝑘𝑢𝑢 𝐴𝐴𝑃𝑃𝑂𝑂 ∶
𝐴𝐴𝑃𝑃2 + 𝑂𝑂𝑃𝑃2 = 𝑂𝑂𝐴𝐴2 𝑟𝑟2 + 𝑟𝑟2 = (8 – 𝑟𝑟)2 2𝑟𝑟2 = 64 − 16𝑟𝑟 + 𝑟𝑟2 2𝑟𝑟2 − 𝑟𝑟2 + 16𝑟𝑟 − 64 = 0 𝑟𝑟2 + 16𝑟𝑟 − 64 = 0 ………………………..(14.6)
Persamaan (14.6) di atas memenuhi bentuk kuadrat ax2 +bx +c = 0, dengan a = 1; b = 16, dan c = -64. Sehingga nilai masing-masing akarnya dapat ditentukan sebagai berikut: 𝑥𝑥1,2 =
−𝑏𝑏 ± √𝑏𝑏 2 − 4𝑎𝑎𝑐𝑐 2𝑎𝑎
Sehingga identik dengan persamaan di atas, nilai r dapat ditentukan: 𝑟𝑟1,2 =
−16 ± �162 − 4(1)(−64) 2(1)
| 267
𝑟𝑟1 =
𝑟𝑟1 = dan
−16 + √256 + 256 2 −16 + 16√2 2
𝑟𝑟1 = −8 + 8√2
𝑟𝑟2 =
−16 − √256 + 256 2
𝑟𝑟2 =
−16 − 16√2 2
𝑟𝑟2 =
−16 − √256 + 256 2
𝑟𝑟2 = −8 − 8√2 (𝑡𝑡𝑖𝑖𝑑𝑑𝑑𝑑𝑑𝑑 𝑚𝑚𝑒𝑒𝑚𝑚𝑚𝑚𝑚𝑚𝑚𝑚ℎ𝑖𝑖)
Setelah diketahui nilai r pada lingkaran kecil, dapat dihitung luas daerah yang diarsir, dengan memerhatikan ketiga luas utama, yaitu bahwa Luasan daerah yang diarsir adalah Luas Seperdelapan Juring Lingkaran Besar dikurangi Luas Segitiga dikurangi lagi oleh Luas Tiga-per-delapan Juring Lingkaran Kecil,”terang Saver. Berhenti sejenak, Saver kembali menarik nafas dalam-dalam. “Selanjutnya kita hitung satu per satu, yaitu: Pertama, luas Seperdelapan Juring Lingkaran Besar adalah: 𝐿𝐿𝑢𝑢𝑎𝑎𝑎𝑎 𝑠𝑠𝑒𝑒𝑝𝑝𝑝𝑝𝑝𝑝𝑝𝑝𝑝𝑝𝑝𝑝𝑝𝑝𝑝𝑝𝑝𝑝𝑝𝑝 𝑗𝑗𝑢𝑢𝑟𝑟𝑟𝑟𝑟𝑟𝑟𝑟 𝑏𝑏𝑒𝑒𝑠𝑠𝑠𝑠𝑠𝑠 = 1 = 𝜋𝜋82 8
450 𝑥𝑥𝜋𝜋82 3600
= 8𝜋𝜋 satuan luas
1350 2 𝑥𝑥𝜋𝜋�−8 + 8√2� 𝐿𝐿𝑢𝑢𝑎𝑎𝑎𝑎 𝑡𝑡𝑖𝑖𝑔𝑔𝑔𝑔 𝑝𝑝𝑒𝑒𝑟𝑟 𝑑𝑑𝑒𝑒𝑙𝑙𝑙𝑙𝑙𝑙𝑙𝑙𝑙𝑙 𝑗𝑗𝑢𝑢𝑟𝑟𝑟𝑟𝑟𝑟𝑟𝑟 𝑘𝑘𝑒𝑒𝑐𝑐𝑐𝑐𝑐𝑐 = 3600
| 268
3 = 𝜋𝜋 (128 − 128√2 + 64) 8 3 = 𝜋𝜋 (192 − 128√2) 8
= 72𝜋𝜋 − 48𝜋𝜋√2 satuan luas
1 𝐿𝐿𝑢𝑢𝑎𝑎𝑎𝑎 𝑠𝑠𝑒𝑒𝑔𝑔𝑔𝑔𝑔𝑔𝑔𝑔𝑔𝑔𝑔𝑔 𝐴𝐴𝑃𝑃𝑂𝑂 = 𝒓𝒓𝒙𝒙𝒓𝒓 2 1 = �−8 + 8√2�(−8 + 8√2) 2 1 = (192 − 128√2) 2 = 96 − 64√2 satuan luas
Dan akhirnya, luasan daerah yang diarsir dapat ditentukan sebagai: 𝐿𝐿𝑢𝑢𝑎𝑎𝑎𝑎 𝐷𝐷𝑎𝑎𝑒𝑒𝑒𝑒𝑒𝑒ℎ 𝐴𝐴𝑟𝑟𝑠𝑠𝑖𝑖𝑟𝑟 = 8𝜋𝜋 − �72𝜋𝜋 − 48𝜋𝜋√2� − (96 − 64√2) = 8𝜋𝜋 − 72𝜋𝜋 + 48𝜋𝜋√2 − 96 + 64√2 = 48𝜋𝜋√2 − 64𝜋𝜋 − 96 + 64√2
= (48√2 − 64)𝜋𝜋 + 64√2 − 96
= (48√2 − 64)𝜋𝜋 + 32(2√2 − 3) satuan luas,
adalah bentuk akhir dari luas daerah yang diarsir, Kawankawan!”pungkas Saver. “Subhanallah! Semakin mantap saja kau, Ver!”puji Amer sambil memberikan tepuk tangan sebagai apresiasi atas penjelasan kawannya itu. “Meski tidak sulit perhitungannya, tapi sungguh membutuhkan ketelitian dan kecermatan memisahkan persoalan atas bangunbangunnya,”komentar Ayra.
| 269
Tampak Fateh, Sabiq, Ikram, dan Yodha pun setuju dengan pendapat Ayra. “Alhamdulillah, kita sudah menyelesaikan 4 soal yang menarik. Selanjutnya, siapa yang akan mencoba menjelaskan soal berikutnya, Kawan?”tanya Sabiq kepada teman-temannya. Bintang mengangkat tangan kanannya. Tampaknya dia mau mencoba kemampuannya, tentu di hadapan kawan-kawannya. “Biar aku coba, Kawan!”jawab Bintang. “Silahkan, Kawan!”balas Sabiq. “Baiklah, langsung saja. Disebutkan bahwa ‘jumlah 1007 bilangan bulat positif berbeda adalah 1023076. Dimana tidak ada satupun dari bilangan-bilangan tersebut yang lebih besar dari 2014. Minimal banyaknya bilangan ganjil pada deret bilangan tersebut adalah …’. Terhadap persoalan ini, kita perlu menggaris-bawahi beberapa hal, yaitu. Pertama, 1007 bilangan bulat positif berbeda adalah jumlahnya 1023076. Kedua, tidak ada satupun dari bilangan tersebut lebih besar dari 2014,
dan ketiga, minimal banyaknya
bilangan ganjil pada deret,”terang Bintang memulai penjelasannya. Bintang memutar-mutar alat tulis di sela-sela jarinya. Di bergerak ke arah papan tulis. “Kita mulai dari belakang, Kawan-kawan! Untuk itu,...”tibatiba Bintang menghentikan penjelasannya karena ada suara menyela. “Wah, main belakang pula kau, Kawan!”potong Amer. “Mer, aku serius, soal ‘main belakang’ ini ‘bukan main-main’. Nanti Ikram akan menjelaskannya setelah penjelasanku. Oke! Biarkan aku teruskan dulu penjelasanku,”pinta Bintang. | 270
Ikram yang disebut namanya agak sedikit menarik kepalanya ke belakang, sementara kawan-kawan yang lainnya hanya senyumsenyum. Mereka tahu betul sifat kawan mereka Amer. “Sorry, Bro! Silahkan, Kawan!”balas Amer. “Karena minimal banyaknya bilangan ganjil pada deret bilangan yang ditanya, maka haruslah digunakan dalam deret bilangan itu bilangan genap sebanyak mungkin. Kekurangannya, ditambahkan dari bilangan ganjil yang tersedia. Sehingga: Jumlah deretan bilangan bulat yang genap, misalnya JBGP, yaitu: 𝐽𝐽𝐵𝐵𝐺𝐺𝐺𝐺 = 2 + 4 + 6 + ⋯ … + 2012
Tampak bahwa JBGP merupakan deret Aritmetika dengan suku pertama 2 dan bedanya adalah (4-2) = (6-4) = 2. Dan harus berakhir di angka 2012, karena berdasarkan keadaan pertama bahwa ‘tidak ada satupun dari bilangan-bilangan tersebut yang lebih besar dari 2014’. Sementara 2013 adalah bilangan bulat ganjil. Sehingga: 𝑈𝑈𝑛𝑛 = 𝑎𝑎 + (𝑛𝑛 − 1)𝑏𝑏
2012 = 2 + (𝑛𝑛 − 1)2
𝑛𝑛 =
2012 − 2 +1 2
𝑛𝑛 = 1006 𝑠𝑠𝑢𝑢𝑘𝑘𝑘𝑘
Untuk itu, bila dihitung jumlah suku dengan U1 = 2; b = 2 dan Un=2012; 𝑛𝑛 (𝑈𝑈 + 𝑈𝑈𝑛𝑛 ) 2 1 1006 (2 + 2012) 𝑆𝑆𝑛𝑛 = 2
𝑆𝑆𝑛𝑛 =
𝑆𝑆𝑛𝑛 = 1.013.042
| 271
Selanjutnya dari data soal diketahui bahwa ‘jumlah 1007 bilangan bulat positif berbeda adalah 1.023.076’, untuk itu terjadi kekurangan atau selisih (Δ1), sebesar: ∆1 = 1.023.076 − 1.013.042
∆1 = 10.034
Kekurangan atau selisih ini diambil dari sejumlah bilangan bulat ganjil, sedemikian rupa, yang akan menggantikan posisi bilanganbilangan bulat genap terkecil sebelumnya, yaitu bilangan-bilangan bulat ganjil terbesar yang kurang dari 2014: ∆2 = 2013 − 2 + 2011 − 4 + 2009 − 6 + 2007 − 8 + 2005 − 10 ∆2 = 10.015 Dari
kekurangan
pada
∆1 = 10.034
dan
tambahan
pada
∆2 = 10.015, yang diambil dari 5 suku bilangan ganjil masih terdapat
selisih sebesar:
∆= ∆1 − ∆2
∆= 10.034 − 10.015 ∆= 𝟏𝟏𝟗𝟗
Akhirnya,
ditambahkan
bilangan
bulat
ganjil
19
untuk
menggenapkan 1007 bilangan bulat positif berbeda seperti yang diminta oleh soal. Sehingga, ‘minimal banyaknya bilangan ganjil pada deret bilangan tersebut adalah’ sebanyak 6 bilangan, di mana 5 bilangan yang menggantikan posisi bilangan genap terkecil dan 1 dari kekurangan, yang terdiri atas: 2013; 2011; 2009; 2007; 2005; dan 19,”pungkas Bintang. “Bukan main, semakin canggih saja kau, Kawan!”puji Amer.
| 272
“Terima kasih, Kawan. Aku hanya berusaha memberi yang terbaik buat Genk kita ini,”balas Bintang bersahaja. “Subhanallah. Laa ilmaa lanaa illaa maa ‘allamtanaa! Aku harus memberi ucapan khusus pula buatmu, Kawan!”ujar Sabiq. Dan kawan-kawan lainnya pun serempak memberi applause pada kawan mereka itu. “Oke, kita sudah cukup jauh menyelesaikan 4 soal. Berikutnya, aku pikir yang paling tepat menjelaskan persoalan ini adalah Fateh. Ini ada kaitannya dengan ‘keahlianmu’, Kawan!”ujar Sabiq sambil melirik ke arah Fateh. “Terima kasih kawan. Tapi, bila ada kawan-kawan lain yang sudah mencoba menyelesaikannya, silahkan saja!”balas Fateh. Semua tampak diam. Masing-masing saling pandang. “Sudahlah Kawan, soal beginian itu memang bidangmu! Kami tunggu,”potong Amer cepat. “Baik kalau begitu. Izinkan aku memulai saja. Disebutkan dalam pertanyaan bahwa ‘terdapat bilangan ribuan dengan jumlah angka-angkanya 8. Contoh bilangan ini adalah 1232. Bilangan yang memenuhi sifat ini ada sebanyak …’. Soal ini soal tentang kombinasi kaitannya dengan letak angka-angka yang sedemikian rupa sehingga tetap merupakan angka ribuan. Hanya saja berapa banyak kemungkinan tata letak itu bisa disusun,”terang Fateh memulai. “Wah mirip soal sebelum makan tadi, dong!”potong Amer. “Ya, tapi ini merupakan kombinasi yang lebih kompleks. Ini mirip dengan 2 persoalan tadi malam yang kita bahas, yaitu tentang cara mengambil 10 buah dengan kombinasi 5 buah dan antrian karcis | 273
kaitannya dengan jarak terpendek yang mungkin dicapai dari satu titik ke titik yang lain,”balas Fateh lugas. “Pokoknya,
kalau
soal
beginian,
aku
percaya
kau,
Kawan!”kembali Amer memuji Fateh, disertai acungan jempol. “Terima kasih, Kawan. Tapi, simpan dulu pujianmu, kita konsenterasi pada persoalan. Oke!”balas Fateh. Fateh memfokuskan perhatiannya pada kawan-kawannya. Dia seperti memikirkan sesuatu yang berat. “Terhadap persoalan ini, kita bisa pecahkan dengan 2 jalan. Pertama dengan perumusan, seperti saat kita pecahkan soal kombinasi buah dan antrian karcis. Kedua dengan bantuan tabel konkret. Untuk yang pertama, mohon kawan-kawan mengingat kembali tentang penjelasan terhadap 2 soal kombinasi tadi malam,”kata Fateh. Tampak seperti menerima perintah komandan dalam sebuah pasukan, semua kawan Fateh yang hadir di ruangan itu membuka catatan mereka tentang hasil diskusi tadi malam. Beberapa tampak serius memandang, sementara lainnya tampak sedikit memicingkan mata, sementara yang lain lagi tampak mengangguk-angguk. Tentu, masing-masing merekalah yang tahu dari arti bahasa tubuh itu. “Sebelumnya, mari sama-sama kita perhatikan gambar kotakkotak di bawah ini,”pinta Fateh
| 274
Fateh melengkapi kotak-kotak yang digambarnya dengan beberapa angka dan huruf. Dia siap melanjutkan. “Karena menurut data soal, bahwa terdapat bilangan ribuan dengan jumlah dari angka-angkanya adalah 8, maka dapat disebutkan bahwa terdapat 4 angka tersusun sedemikian rupa sehingga jumlah dari keempat angkanya adalah 8. Kita misalkan angka-angka itu adalah x1, x2, x3, dan x4, dan angka 0 sampai dengan 8 adalah kemungkinan angka-angka penyusun dari empat angka itu, seperti tertera pada gambar di atas. Dengan mengingat kembali apa yang didefinisikan
sebagai
Kombinasi
dengan
Pengulangan
pada
persoalan buah sebelumnya, maka kombinasinya dirumuskan sebagai: �
(𝑛𝑛 + 𝑟𝑟 − 1)! 𝑛𝑛 + 𝑟𝑟 − 1 𝑛𝑛 + 𝑟𝑟 − 1 �=� � = 𝐶𝐶𝑛𝑛𝑛𝑛+𝑟𝑟−1 = 𝑟𝑟 − 1 𝑛𝑛 𝑛𝑛! (𝑟𝑟 − 1)!
Selanjutnya, banyaknya solusi bilangan bulat tidak negatif dari persamaan x1 + x2 + x3 + x4 = 8. Hal ini berarti: �
𝑛𝑛 + 𝑟𝑟 − 1 8+4−1 �=� � = 𝐶𝐶311 𝑟𝑟 − 1 4−3
Atau sama dengan banyaknya cara terpendek untuk mencapai ujung kanan bawah kotak pada gambar di atas (titik angka 8) dari ujung kiri atas kotak (titik x4). Hal ini berarti sebanyak 8 jarak ke kanan dan 3 jarak ke bawah: 𝐶𝐶311 =
𝐶𝐶311 =
11! 3! (11 − 3)!
9𝑥𝑥10𝑥𝑥11 = 11𝑥𝑥5𝑥𝑥3 3𝑥𝑥2𝑥𝑥
𝐶𝐶311 = 165 𝑐𝑐𝑎𝑎𝑟𝑟𝑟𝑟
| 275
Akan tetapi dalam kombinasi di atas masih mengandung nilai x1 = 0, padahal dalam keadaan ribuan x1 ≠ 0, atau dengan kata lain, angka pertama tidak boleh sama dengan nol!,”terang Fateh. Fateh berhenti agak lama. Hampir 1 menit dia memandang gambar yang ia buat sendiri di papan tulis. Sepuluh kawannya masih sangat serius memerhatikan dan menunggu penjelasan berikutnya. “Lantas, kalau masih mengandung nilai 0, bagaimana kombinasi akhirnya, Teh?”tiba-tiba Ikram menyela. “Tepat sekali! Karenanya, harus dikurangi oleh banyaknya nilai x1= 0, yaitu sebanyak 8 jarak ke kanan dan sebanyak 2 jarak ke arah bawah, sehingga jumlah jaraknya adalah: 𝐶𝐶210 =
𝐶𝐶210 =
10! 2! (10 − 2)!
9𝑥𝑥10𝑥𝑥8! 9𝑥𝑥10 = 2! 8! 2
𝐶𝐶210 = 45 𝑐𝑐𝑎𝑎𝑟𝑟𝑟𝑟
Dengan demikian, banyaknya solusi yang dimaksud adalah: 𝐶𝐶311 −𝐶𝐶210 = 165 − 45 𝐶𝐶311 −𝐶𝐶210 = 120 𝑐𝑐𝑎𝑎𝑟𝑟𝑟𝑟
Itu penjelasan dengan jalan pertama, memanfaatkan Kombinasi dengan Pengulangan dan koreksi atas posisinya,”terang Fateh. Kawan-kawannya masih serius. Mereka masih menunggu jalan kedua, jalan tabel. “Jalan berikutnya, adalah jalan yang kita tempuh andaikan kita lupa dengan perumusan kombinasi, yaitu jalan manual. Jalan pembuatan
tabel
kemungkinan
kombinasi
angka-angka
penyusunnya,”terang Fateh. | 276
“Maksudmu, kita bisa membuat tabel terhadap persoalan yang ditanyakan, Teh?”tanya Ayra. “Ya, Ayra. Kita bisa membuat tabel sehingga tampak jelas bilangan-bilangan 4 angka yang menyusun ribuan dengan jumah 8 setiap angka penyusun bilangan 4 angka itu,”terang Fateh lagi. “Konkretnya, Kawan!”tanya Iman. Fateh kembali mengambil alat tulis. Dia membuat tabel sedemikian rupa. Hampir lima menit, dan akhirnya dia berbalik, kembali menghadap ke kawan-kawannya. “Perhatikan, Kawan-kawan! inilah tabel kombinasi 4 angka yang berjumlah 8 dan membentuk angka ribuan itu!”ujar Fateh.
Angka 8000 7100 6200 6110 5300 5210 5111 4400 4310 4220 4211 3320 3311 3221 2222
Kombinasi Letak Angka 8000 7100, 7010, 7001, 1007, 1070, 1700 6200, 6020, 6002, 2006, 2060, 2600 6110, 6101, 6011, 1610, 1601, 1160, 1106, 1061, 1016 5300, 5030, 5003, 3005, 3050, 3500 5210, 5201, 5120, 5102, 5021, 5012, 2510, 2501, 2150, 2105, 2051, 2015, 1520, 1502, 1250, 1205, 1052, 1025 5111, 1511, 1151, 1115 4400, 4040, 4004 4310, 4301, 4130, 4103, 4031, 4013, 3410, 3401, 3140, 3104, 3041, 3014, 1430, 1403, 1340, 1304, 1043, 1034 4220, 4202, 4022, 2420, 2402, 2240, 2204, 2042, 2024 4211, 4121, 4112, 2411, 2141, 2114, 1421, 1412, 1241, 1214, 1142, 1124 3320, 3302, 3230, 3203, 3032, 3023, 2330, 2303, 2033 3311, 3131, 3113, 1331, 1313, 1133 3221, 3212, 3122, 2213, 2231, 2123, 22132, 1223, 1232, 1322, 2321, 2312 2222 Total
Jlh 1 6 6 9 6 18 4 3 18 9 12 9 6 12 1 120
| 277
“Wah,
indah
sekali
Kawan!
Ternyata
perhitungan
kombinasimu sesuaian dengan tabel manual,”komentar Amer. “Terima kasih, dan sampai di sini saja yang mampu aku jelaskan,”pungkas Fateh. “Plok, plok, plok, plok, plok,”terdengar tepukan meriah dari kawan-kawan Fateh. “Untuk Bagian A OSN Provinsi ini kita masih punya 5 soal lagi, Kawan. Siapa yang akan menjelaskan penyelesaian soal nomor 6?”tanya Sabiq kepada kawan-kawannya. Tampak sepuluh sahabat saling pandang. “Bismillah, aku mau coba, Biq,”kata Ayra sambil mengangkat tangannya. “Silahkan Ayra. Kami menunggu penjelasanmu,”balas Sabiq. Ayra melangkah maju, diraihnya alat tulis, dia langsug menuju ke papan tulis. “Kawan-kawan, terhadap soal nomor 6 ini, mau tidak mau kita harus bisa menggambarkannya. Disebutkan bahwa, ‘misalkan ABCD adalah suatu daerah trapesium sedemikian sehingga perpanjangan sisi AD dan perpanjangan sisi BC berpotongan di titik E. Diketahui panjang AB = 18, CD = 30 dan tinggi trapesium tersebut adalah 8. Jika F dan G masing-masing adalah titik tengah AD dan BC, maka luas segitiga EFG adalah…’. Seperti tadi aku katakan, kita harus menggambarkannya!”terang Ayra. Ayra berhenti sejenak, tampaknya ada hal lain yang ingin disampaikan.
| 278
“Tapi, karena soal tidak menjelaskan jenis trapesiumnya, apakah trapesium siku-siku, trapesium sama kaki, atau trapesium sebarang, tetapi hanya mempersoalkan luasan yang terbentuk, maka kita misalkan saja ia adalah sebuah trapesium siku-siku, sehingga gambarannya kira-kira seperti ini:
Selanjutnya, dari penggambaran di atas, kita mendapatkan beberapa konsekuensi, yaitu: Karena F dan G adalah titik tengah dan HJ adalah tinggi, maka: HI = HJ = 4;
AF = FD;
BG = GC
Untuk menghitung luas ΔEFG, maka 2 besaran yang harus diketahui, yaitu panjang EF (EA + AF) dan FG. Untuk itu: Misalkan EA = x, Karena ΔECD ~ ΔEBA, maka: 𝐸𝐸𝐷𝐷 𝐸𝐸𝐴𝐴 = 𝐷𝐷𝐶𝐶 𝐴𝐴𝐵𝐵 (𝐴𝐴𝐷𝐷 + 𝐴𝐴𝐸𝐸) 𝐸𝐸𝐴𝐴 = 𝐴𝐴𝐵𝐵 𝐷𝐷𝐶𝐶 8 + 𝑥𝑥 𝑥𝑥 = 30 18 30𝑥𝑥 = 18(8 + 𝑥𝑥)
30𝑥𝑥 = 144 + 18𝑥𝑥
(30 − 18)𝑥𝑥 = 144
| 279
𝑥𝑥 =
144 12
𝑥𝑥 = 12
Karenanya,
EF = EA + AF EF = 16 Selain itu, karena ΔECD ~ ΔEGF, maka: 𝐸𝐸𝐹𝐹 𝐸𝐸𝐴𝐴 = 𝐹𝐹𝐺𝐺 𝐴𝐴𝐵𝐵 (𝐸𝐸𝐴𝐴 + 𝐴𝐴𝐹𝐹) 𝐸𝐸𝐴𝐴 = 𝐴𝐴𝐵𝐵 𝐹𝐹𝐺𝐺 12 + 4 12 = 18 𝐹𝐹𝐺𝐺 12𝐹𝐹𝐺𝐺 = 16𝑥𝑥18 𝐹𝐹𝐺𝐺 =
16𝑥𝑥18 12
𝐹𝐹𝐺𝐺 = 24
Sehingga dengan demikian: 1 𝐿𝐿𝑢𝑢𝑎𝑎𝑎𝑎 ∆𝐸𝐸𝐹𝐹𝐺𝐺 = 𝑥𝑥𝑎𝑎𝑙𝑙𝑎𝑎𝑎𝑎𝑎𝑎𝑎𝑎𝑎𝑎𝑛𝑛𝑔𝑔𝑔𝑔𝑖𝑖 2 1 𝐿𝐿𝑢𝑢𝑎𝑎𝑎𝑎 ∆𝐸𝐸𝐹𝐹𝐺𝐺 = 𝑥𝑥𝐹𝐹𝐺𝐺𝐺𝐺𝐺𝐺𝐺𝐺 2 1 𝐿𝐿𝑢𝑢𝑎𝑎𝑎𝑎 ∆𝐸𝐸𝐹𝐹𝐺𝐺 = 𝑥𝑥24𝑥𝑥16 2
𝐿𝐿𝑢𝑢𝑎𝑎𝑎𝑎 ∆𝐸𝐸𝐹𝐹𝐺𝐺 = 192 𝑠𝑠𝑎𝑎𝑡𝑡𝑡𝑡𝑡𝑡𝑡𝑡 𝑙𝑙𝑢𝑢𝑎𝑎𝑎𝑎,
Itu penjelasan soal nomor 6 yang mampu kusampaikan, Kawankawan,”terang Ayra. “Kawan-kawan, applause buat Ayra. Dia selesaikan soal ini kurang dari 7 menit. Tepatnya 6 menit 12 detik!”terang Sabiq. | 280
“Congratulations, Ayra!”teriak kawan-kawannya. “Terima kasih, khususnya buat Yodha yang sebelum pertemuan
ini
bersedia memberitahuku
beberapa
hal
dasar
kesebangunan segitiga,”balas Ayra sambil tersenyum pada Yodha. Kembali apresiasi berupa tepuk tangan meriah diberikan oleh kawan kepada kawannya. “Tinggal 4 soal tersisa, Kawan-kawan. Siapa berikutnya yang akan mencoba soal nomor 7 ini?”tanya Sabiq “Aku. Aku akan mencoba kemampuanku, Biq”jawab Amer tegas sambil mengacungkan jari telunjuk tangan kanannya ke udara. “Oke, silahkan kawan. Waktu dan tempat menjadi hakmu saat ini,”ucap Sabiq sambil mengantarkan alat tulis ke arah Amer. Amer menerima alat tulis dengan tangan kanannya, dan tibatiba dia membungkukkan kepalanya, bak aksi seorang Dirijen sebuah Orkestra. “Plok, plok, plok, plok, plok,”terdengar sambutan hangat berupa appalause meriah dari kawan-kawan Amer. Bukan Amer kalau tak punya sensasi. Dan aksi barusan pun adalah bagian sensasi yang sangat dia kuasai itu. “Baiklah kawan-kawan, lansung saja. Dari soal ini sama-sama kita baca, disebutkan bahwa: ‘Diberikan dua persamaan berikut: 2 6 + = 2 … … … … … … . . (14.7) 𝑥𝑥 + 𝑦𝑦 𝑥𝑥 − 𝑦𝑦
9 4 − = −1 … … … … … … (14.8) 𝑥𝑥 + 𝑦𝑦 𝑥𝑥 − 𝑟𝑟
Dan ditanyakan, ‘nilai
𝑥𝑥 𝑦𝑦
yang memenuhi kedua persamaan tersebut
adalah berapa? Terhadap soal jenis ini. Aku mengerjakan dengan | 281
prinsip perkalian dan persamaan biasa,”kata Amer membuka penjelasannya. “Kenapa aku sebut biasa, karena sesungguhnya kedua persamaan di atas memenuhi sistem perkalian sederhana, yaitu: 𝐴𝐴 𝐶𝐶 + = 𝑘𝑘1 𝐵𝐵 𝐷𝐷 𝐸𝐸 𝐺𝐺 − = 𝑘𝑘2 𝐵𝐵 𝐷𝐷 Dengan A = 2, C = 6, E = 4, G = 9, k1= 2, k2 = -1, dan B = (x+y), serta D = (x-y). Karenanya, dengan perkalian silang terhadap persamaan (14.7) kita akan peroleh: 2( − 𝑦𝑦) + 6(𝑥𝑥 + 𝑦𝑦) =2 (𝑥𝑥 + 𝑦𝑦)(𝑥𝑥 − 𝑦𝑦) 2𝑥𝑥 − 2𝑦𝑦 + 6𝑥𝑥 + 6𝑦𝑦) =2 𝑥𝑥 2 − 𝑦𝑦 2 8𝑥𝑥 + 4𝑦𝑦 =2 𝑥𝑥 2 − 𝑦𝑦 2
8𝑥𝑥 + 4𝑦𝑦 = 2(𝑥𝑥 2 − 𝑦𝑦 2 )
4𝑥𝑥 + 2𝑦𝑦 = 𝑥𝑥 2 − 𝑦𝑦 2 … … … … … … . (14.9)
Dengan cara yang sama, terhadap persamaan (14.8) akan diperoleh: 4(𝑥𝑥 − 𝑦𝑦) − 9(𝑥𝑥 + 𝑦𝑦) = −1 (𝑥𝑥 + 𝑦𝑦)(𝑥𝑥 − 𝑦𝑦)
4𝑥𝑥 − 4𝑦𝑦 − 9𝑥𝑥 − 9𝑦𝑦) = −1 𝑥𝑥 2 − 𝑦𝑦 2 −5𝑥𝑥 − 13𝑦𝑦 = −1 𝑥𝑥 2 − 𝑦𝑦 2
−5𝑥𝑥 − 13𝑦𝑦 = −1(𝑥𝑥 2 − 𝑦𝑦 2 )
−5𝑥𝑥 − 13𝑦𝑦 = −𝑥𝑥 2 + 𝑦𝑦 2 … … … … … … (14.10)
| 282
Selanjutnya, dengan menjumlahkan persamaan (14.9) dan persamaan (14.10), akan diperoleh: 4𝑥𝑥 + 2𝑦𝑦 = 𝑥𝑥 2 − 𝑦𝑦 2
−5𝑥𝑥 − 13𝑦𝑦 = −𝑥𝑥 2 + 𝑦𝑦 2 + −𝑥𝑥 − 11𝑦𝑦 = 0 −𝑥𝑥 = 11𝑦𝑦 𝑥𝑥 − = 11 𝑦𝑦
Sehingga, akan diperoleh: 𝑥𝑥 = −11 𝑦𝑦
Itu saja yang bisa aku jelaskan tentang soal nomor ini, Kawankawan,”pungkas Amer sambil mengulangi aksi membungkukkan badan seperti di awal tadi. “Subhanallah! Amer justru menyelesaikan soal nomor 7 ini kurang dari 5 menit. Tepatnya 4 menit 6 detik, Kawan-kawan!”puji Sabiq sambil berdiri dan memberi tepuk tangan yang panjang. Tindakan Sabiq itu pun diikuti oleh kawan-kawannya. Dan Amer tersenyum puas dengan apresiasi dari kawan-kawannya itu. “Ternyata dalam soal kecepatan menyelesaikan persamaan Matematika, kemampuanmu berbanding terbalik dengan kecepatan makanmu, Bro!”sindir Ikram. “Maksudmu apa, Kawan?”tanya Amer. “Tadi kau selesaikan makan siangmu kuhitung-hitung hampir 18 menit, sementara soal persamaan itu hanya kau selesaikan dalam waktu 4 menit beberapa detik!”jawab Ikram.
| 283
“Kalau begitu kau salah, Kawan! Bukan berbanding terbalik, lebih tepatnya berbanding terbalik dengan kuadratnya. Betulkan, Biq?”balas Amer lagi. “Betul, betul, betul!”jawab Sabiq singkat. Semua kawan mereka senang dengan jawaban Amer dan candaan Ikram. Tawa ceria terpancar dari wajah-wajah mereka “Baik,
kawan-kawan,
siapa
yang akan
menyelesaikan
persoalan berikutnya?”tanya Sabiq memecah keceriaan kawankawannya. Ikram maju dengan wajah tenang. “Izinkan aku untuk mencobanya, Kawan!”pinta Ikram Sabiq tersenyum dengan jawaban singkat Ikram. Diantara semua kawannya, dia tahu betul kemampuan Ikram yang tersembunyi. Tapi, kerendahan hati dan kecenderungan memberi kesempatan kepada kawan-kawannya, sebagai sifat yang menonjol Ikram, membuat tak banyak orang yang mengetahi kemampuannya yang sebenarnya. Tapi, hal itu tak luput dari pengamatan Sabiq. Dan Ikram pun sadar akan hal itu. “Silahkan, Kawan!”balas Sabiq. “Kawan-kawan, soal nomor 8 ini sederhana saja dan singkat, tetapi sesungguhnya membutuhkan kecermatan dalam membuat pola umum dari sebuah sifat bilangan. Disebutkan bahwa, ‘jika a dan b bilangan bulat ganjil serta a > b maka banyak bilangan bulat diantara 2a dan b adalah …’. Sederhana saja. Apa yang kita lakukan?”tanya Ikram.
| 284
“Tentu harus menyelesaikan dengan menjawab pertanyaan yang diajukan itu, Kawan!”canda Amer. Terdengar tawa rendah kawan-kawan mereka mendengar candaan Amer. Ikram tetap tersenyum. “Tepat kita harus menyelesaikan dengan menjawab pertanyaan yang diajukan itu, pertanyaannya, bagaimana caranya dan darimana? Untuk itu, aku masih ingat diskusi awal kita dulu, bahwa memulai dari hal yang sederhana terus dan terus menuju ke hal yang lebih rumit,”terang Ikram. Sabiq senyum mendengar jawaban Ikram. “Dia menerjemahkan pesan Orang Bijak dari Timur,”pikir Sabiq. “Karenanya, membuat beberapa ilustrasi yang sederhana akan sangat membantu, Kawan-kawan!”terang Ikram lagi. “Ilustrasi katamu, Kawan?”tanya Denan. “Ya, ilustrasi atau bahasa sederhananya coba-coba atas informasi yang disajikan. Karenanya, untuk mengetahui jawaban dari berapa banyak bilangan bulat di antara 2a dan b, di mana a dan b merupakan bilangan bulat ganjil serta a > b, perhatikan ilustrasi berikut: (1) Misalkan, 𝑎𝑎1 = 11 𝑑𝑑𝑎𝑎𝑛𝑛 𝑏𝑏1 = 9 maka 2𝑎𝑎1 = 22 𝑑𝑑𝑎𝑎𝑛𝑛 𝑏𝑏1 =
9, Sehingga bilangan antara 22 dan 9 adalah: 10, 11, 12,13,
14, 15, 16, 17, 18, 19, 20, 21. Terdapat 12 bilangan; memenuhi pola 2 (11) – 9 – 1 = 12 bilangan;
(2) Misalkan, 𝑎𝑎2 = 9 𝑑𝑑𝑎𝑎𝑛𝑛 𝑏𝑏2 = 7 maka 2𝑎𝑎2 = 18 𝑑𝑑𝑎𝑎𝑛𝑛 𝑏𝑏2 =
7, Sehingga bilangan antara 18 dan 7 adalah: 8, 9, 10, 11, | 285
12,13, 14, 15, 16, 17. Terdapat 10 bilangan; memenuhi pola 2 (9) – 7 – 1 = 10 bilangan; (3) Misalkan, 𝑎𝑎3 = 7 𝑑𝑑𝑎𝑎𝑛𝑛 𝑏𝑏3 = 5 maka 2𝑎𝑎 = 14 𝑑𝑑𝑎𝑎𝑛𝑛 𝑏𝑏3 = 5,
Sehingga bilangan antara 14 dan 5 adalah: 6, 7, 8, 9, 10, 11, 12,13. Terdapat 8 bilangan; memenuhi pola 2 (7) – 5 – 1 = 8
bilangan; Karenanya, jika kita perhatikan pola-pola dari ketiga contoh sederhana di atas, akan terlihat sebagai berikut: 2 (11) – 9 – 1 = 12 2 (9) – 7 – 1 = 10 2 (7) – 5 – 1 = 8 .
⟺ 2a1 – b1 – 1 =k1
⟺ 2a2 – b2 – 1 = k2 ⟺ 2a3 – b3 – 1 = k3 ⟺ 2an – bn – 1 =kn
Sehingga, bila pola ke-n dianggap membentuk pola umum, maka dapat disimpulkan bahwa jika a dan b merupakan bilangan bulat ganjil serta a > b maka banyak bilangan bulat diantara 2a dan b adalah 2a – b – 1. Begitu penjelasanku kawan-kawan,”pungkas Ikram. “Ternyata kau benar kawan, bahwa memulai dari hal yang sederhana terus dan terus menuju ke hal yang lebih rumit, kita akan menemukan pola,”komentar Denan. “Dan ilustrasimu pun adalah ilustrasi yang sederhana pula, Kawan,”puji Saver. “Terima kasih, Kawan-kawan. Yang penting kita paham maksud pertanyaan dan tahu jalan pemecahannya. Bagaimana cara kita memahami, tergantung apa yang kita ketahui. Dan aku hanya tahu hal-hal yang sederhana saja,”balas Ikram bersahaja. | 286
Tampak Sabiq hanya senyum tipis. Dia tahu betul kemampuan kawannya yang satu ini. “Alhamdulillah, kita sudah hampir menyelesaikan Bagian A. Selanjutnya siapa yang akan menyelesaikan soal nomor 9, Kawankawan?”tanya Sabiq pada kawan-kawannya. “Kan tinggal 2 orang yang belum, Yodha dan Iman, di antara mereka saja berbagi, Biq”balas Ayra. “Silahkan kau dulu, Kawan!”tawar Yodha pada Iman “Baik, aku mencoba kemampuan dasarku ini,”balas Iman enteng. Di antara sebelas sahabat ini, Iman adalah yang paling kalem. Ia pendiam, tidak banyak bicara, lebih serius dalam mendengar dan memerhatikan.
Hingga
nyaris
banyak
yang
kurang
paham
kemampuan sejati dari teman mereka yang badannya agak subur ini. Tapi, hal itu tampaknya tak luput dari pengamatan 2 kawannya yang lain, Yodha dan Sabiq. “Baik Kawan-kawan, soal nomor 9 sesungguhnya tentang fungsi dan relasi, lebih tepatnya apa yang disebut sebagai Korespondensi Satu-Satu,”ujar Iman memulai penjelasannya. “Persoalan yang dihadapkan pada kita menyebut bahwa, ‘fungsi 𝑔𝑔 dari himpunan 𝑋𝑋 dikatakan satu-satu jika untuk setiap
dengan 𝑥𝑥1, 𝑥𝑥2 ∈ 𝑋𝑋 dengan 𝑔𝑔(𝑥𝑥1) = 𝑔𝑔(𝑥𝑥2) berlaku 𝑥𝑥1 = 𝑥𝑥2 . Jika 𝑋𝑋 =
{9, 6, 3, 2, 1} dan 𝑌𝑌 = {1, 2, 3, 4, 5, 6}, maka fungsi berbeda dari 𝑋𝑋 ke 𝑌𝑌 yang merupakan satu-satu dan setiap bilangan anggota 𝑋𝑋 tidak
dikaitkan dengan faktornya di 𝑌𝑌 ada sebanyak …’. Bagaimana
menjelaskannya? Untuk itu perlu rasanya kita mengulang sedikit | 287
tentang relasi dan fungsi serta korespondensi satu-satu itu,”terang Iman lagi. Entah kenapa suasana tiba-tiba hening. Suara yang jernih dan jelas dari penjelasan Iman telah menarik perhatian kawan-kawannya untuk lebih serius dan fokus. Sabiq dan Yodha hanya senyumsenyum dengan anggukan kecil. Iman kembali fokus. Kali ini menarik nafas cukup dalam, mengeluarkannya perlahan, dan mulai menatap ke arah kawankawannya. “Bila terdapat sekurang-kurangnya 2 himpunan, katakanlah A dan B, di mana masing-masing himpunan memiliki anggota, misalkan (a1,a2,a3,a4,a5) adalah anggota Himpunan A dan (b1, b2, b3) adalah anggota Himpunan B, maka sering dikatakan bahwa relasi dari himpunan A ke himpunan B adalah hubungan yang memasangkan anggota-anggota himpunan A dengan anggotaanggota himpunan B. Seperti apa hubungannya, dan berapa banyak anggota yang berhubungan itu hal lain,”tiba-tiba penjelasan Iman terpotong. “Bisa kau berikan contoh sederhana, Man?”potong Amer. Iman tersenyum dingin. “Baik. Mohon, masing-masing kita bersedia menyebutkan hobinya? Boleh dua atau lebih. Dimulai dari kau Mer?”tanya Iman. “Makan dan Diskusi!”jawab Amer “Bercerita dan Mengarang!”kata Saver “Menari, Menulis!”jawab Ayra “Membaca dan Puisi!”kata Sabiq | 288
“Olahraga dan Membaca!”balas Fateh. “Games dan Makan!”balas Denan “Naik Gunung dan Fotografi!”jawab Ikram. “Olahraga dan Matematika!”jawab Bintang. “Musik dan Olahraga!”jawab Bily. “Matematika dan Bersepeda!”ujar Yodha. “Ditambah aku sendiri yang suka melukis dan film, maka kita sesungguhnya
sudah
bisa
membuat
sebuah
relasi,
Kawan-
Kawan!”terang Iman. “Konkretnya, apa Kawan?”tanya Amer lagi. “Baik. andaikan Himpunan A adalah himpunan yang anggotaanggotanya adalah {Amer, Yodha, Bily, Bintang, Ikram, Denan, Fateh, Sabiq, Ayra, Saver, Iman}, dan Himpunan B adalah himpunan yang anggota-anggotanya adalah {Matematika, Bersepeda, Musik, Olahraga, Naik Gunung, Fotografi, Games, Membaca, Puisi, Menari, Menulis, Bercerita, Mengarang, Diskusi, Makan, Melukis, Film}, dan dibuat garis yang menghubungkan antara anggotaanggota Himpunan A dengan hobinya yang menjadi anggota-anggota Himpunan B, maka sesunggunya kita telah membuat sebuah fungsi relasi antara Himpunan A dan Himpunan B, di mana jenis relasinya adalah ‘Hobi’,”terang Iman lagi. “Clear, jelas sudah, Bro. Thanks!’jawab Amer. Tampak anggukan Yodha dan Sabiq semakin sering, dan senyumnya pun semakin lebar. “Baik, aku teruskan dulu sedikit teori sebelum sampai pada penjelasan tentang soal nomor 9 di atas,”kali ini Iman berbalik ke | 289
tempat duduknya, ia mengambil gelas tempat air putihnya, diteguk sisanya, dan kembali menarik nafas. “Santai saja, Bro. kita sabar mendengar penjelasanmu, Kok!”kata Denan. “Tapi, bila diperhatikan baik-baik, contoh Himpunan A dan Himpunan B tadi, terlihat bahwa ada sekurang-kurangnya satu orang anggota Himpunan A yang memiliki hubungan dengan 2 atau lebih anggota Himpuan B. Nah, di sinilah dikenal apa yang disebut dengan Korespondensi Satu-Satu, yaitu, fungsi yang memetakan anggota dari himpunan A dan B, dimana semua anggota A dan B dapat dipasangkan sedemikian sehingga setiap anggota A berpasangan dengan tepat satu anggota B dan setiap anggota B berpasangan dengan tepat satu anggota A,”terang Iman. “Jadi maksudmu…?”sela Ayra. “Ya, salah satu yang menjadi syarat suatu fungsi atau pemetaan dikatakan sebagai Korespondensi Satu-Satu jika banyak anggota himpunan A dan B sama atau sering ditulis sebagai n(A) = n(B),”terang Iman lagi. Suasana semakin serius. Kawan-kawan Iman tampaknya telah menikmati suara yang jelas dan jernih terdengar, yang keluar dari mulut laki-laki yang punya hobi melukis dan nonton film itu. “Selanjutnya perlu diketahui apa yang disebut sebagai himpunan asal yang sering dikenal dengan domain dan himpunan kawan atau kodomain, juga istilah range unutk menyatakan anggota kodomain yang dikenai relasi oleh domain, atau sering juga disebut sebagai
daerah
hasil.
Nah,
pertanyaan
selanjutnya
adalah, | 290
‘bagaimana cara mencari banyak korespondensi satu-satu yang mungkin antara himpunan A dan B?”pancing Iman. “Ya tentu dibuat daftarnya sedemikian rupa 1 per satu kemungkinannya, Kawan!”jawab Fateh. “Tepat sekali! Namun, ada kata “tapi-nya,”balas Iman. “Maksud kata ‘Tapi’ itu, apa Kawan?”kejar Fateh penasaran. “Memang, untuk jumlah anggota yang kecil, mudah membuat tabel kemungkinannya. Bagaimana dengan himpunan yang jumlah anggota himpunan relatif besar? Pasti akan sangat tidak praktis dan membingungkan!”terang Iman. “Berarti, kau mau katakan ada cara yang memungkinkan untuk menghitungnya tanpa harus membuat daftarnya, Kawan?”kali ini Ikram dan Bintang hampir bersamaan menanyakan hal yang sama. Iman hanya tersenyum. Sabiq dan Yodha masih terus dengan anggukannya, dan senyuman itu tentunya. “Bisa kau berikan contoh lagi, Kawan!”pinta Bily. “Baik. Bil, coba kau hitung berapa banyak korespondensi satusatu yang dapat dibuat dari himpunan M yang anggotanya Ayam dan Itik dan himpunan N yang anggotanya adalah Jagung dan Padi!”tanya Iman. “Dua!”jawab Bily. “Tepat! Dalam waktu kurang dari 3 detik, kau sudah bisa menjawab. Bisa kau sebutkan kemungkinannya, Kawan?”tanya Iman. “Insyaallah bisa, yaitu: Pertama Ayam Padi, dan Itik Jagung. Kedua, Ayam Jagung, dan Itik Padi, Kawan. Kira-kira | 291
seperti ini gambarnya, Kawan!”Bily maju dan menggambarkan sesuatu di papan tulis.
“Tepat sekali kawan. Nah kalau begini soalnya, “Berapa banyak korespondensi satu-satu yang dapat dibuat dari himpunan K terdiri atas huruf-huruf vokal, dan himpunan L yang merupakan bilangan cacah antara 0 dan 6?”tanya Iman lagi. Tampak Bily, dan teman yang lain sibuk menulis dan mengambil kertas. Lima detik berlalu, 30 detik berlalu, 1 menit berlalu. Tidak ada yang menjawab. Waktu terus berjalan. 2 menit, 3 menit, dan… “Ternyata sampai hitungan hampir menit keempat, tak satu pun dari kita yang bisa langsung menjawab. Ingat kawan itu baru dari himpunan K yang terdiri atas huruf vokal, yaitu {a, i, u, e, o}, dan himpunan L yang merupakan bilangan cacah antara 0 dan 6, yaitu {1, 2, 3, 4, 5}. Baru dari 5 anggota himpunannya. Bagaimana kalau lebih dari itu. Itu arti dari ‘Tapi..’ yang tadi kumaksudkan, Kawan!”terang Iman. “Aih, mantap kali ternyata kau, Kawan. Mohon maaf aku sudah salah sangka sama kau, ya!”tiba-tiba Amer berujar agak keras dengan bahasanya yang khas itu. Entah kenapa, beberapa kawannya pun tampak setuju dengan ucapan Amer itu. Sementara Yodha dan | 292
Sabiq tetap mengangguk-anggukkan kepala, mereka berdua tetap tersenyum. “Santai saja, Kawan. Kita ini satu tim, Genk OSN!”tegas Iman. “Jadi, bagaimana cara mengetahui jumlah kemungkinannya, Kawan?”tanya Denan “Aku teringat dengan diskusi kita sebelumnya, memulai dari yang sederhana menuju yang lebih kompleks dan menemukan pola. Mari kita berandai-andai, misalkan Himpunan A dan Himpunan B, dan perhatikan tabel berikut!”uajar Iman sambil menuju ke papan tulis. Ia menggambar sesuatu di sana, Tabel! “Andikan anggota dari masing-masing adalah sebagai berikut: Himpunan A
Himpunan B
Jumlah Korespondensi
1 (1)
1(a)
(1,a)
=1
2 (1,2)
2(a,b)
(1,a; 2,b) (1,b; 2,a)
=2
3 (1,2,3)
3(a,b,c)
(1,a; 2,b; 3;c) (1,a; 2,c; 3,b) (1,b; 2,a; 3,c) (1,b; 2,c; 3,b)
… Dari
… hubungan
antara
(1,c; 2,a; 3,b) (1,c; 2,b; 3,a)
=6
…
= ..
jumlah
… anggota
himpunan
yang
berkorespondensi satu-satu dengan jumlah korespondensi satusatunya atas, kita dapat membuat penyederhanaan berupa pola sebagai berikut: Untuk n(A) = n(B)
Banyaknya korespondensi
1
= 1x1
=1
2
= 1x2
=2
3
= 1x2x3
=6 | 293
4
= 1x2x3x4
= 24
5
= 1x2x3x4x5
= 120
..
=…
=…
n
=1x2x3x…xn
= n!
sehingga, menjawab contoh di atas, karena n(K) = n(L) = 5 maka banyak korespondensi satu-satu yang mungkin dapat dibuat dari himpunan K = {huruf vokal} dan L = {bilangan cacah antara 0 dan 6} adalah: 5! = 5 × 4 × 3 × 2 × 1 = 120 buah,”terang Iman. Tampak kawan-kawannya mencatat serius apa yang ditulis oleh Iman di papan tulis. “Karena itu, kita sudah siap untuk menyelesaikan persoalan yang diajukan oleh nomor 9 sebelumnya, Kawan-kawan. Mari kita kembali dan perhatikan. Aku ulangi kalimat pertanyaannya, ‘‘fungsi
𝑔𝑔 dari himpunan 𝑋𝑋 dikatakan satu-satu jika untuk setiap dengan 𝑥𝑥1, 𝑥𝑥2 ∈ 𝑋𝑋 dengan 𝑔𝑔(𝑥𝑥1) = 𝑔𝑔(𝑥𝑥2) berlaku 𝑥𝑥1 = 𝑥𝑥2 . Jika 𝑋𝑋 = {9, 6, 3, 2, 1}
dan 煬 = {1, 2, 3, 4, 5, 6}, maka fungsi berbeda dari 𝑋𝑋 ke 𝑌𝑌 yang
merupakan satu-satu dan setiap bilangan anggota 𝑋𝑋 tidak dikaitkan
dengan faktornya di 𝑌𝑌 ada sebanyak …’,”Iman menghentikan
ucapannya, tampaknya ia menunggu rekasi.
“Tidak ada, Kawan!”tiba-tiba Amer menjawab dengan polosnya. “Lho, kenapa, Mer?”tanya Iman serius. “Karena jumlah anggota himpunan X ada 5 sementara jumlah anggota
himpunan
Y
ada
6.
Bagaimana
mungkin
terjadi
korespondensi satu-satu?”tanya Amer balik.
| 294
“Nah, bagian frasa ini kau tak baca serius, Kawan. Bagian yang ini, ‘fungsi berbeda dari X ke 𝑌𝑌 yang merupakan satu-satu dan
setiap bilangan anggota 𝑋𝑋 tidak dikaitkan dengan faktornya di
𝑌𝑌’!”jelas Iman.
“Oh, iya. Artinya pada himpunan Y tidak termasuk angka 1
karena 1 merupakan faktor untuk seluruh bilangan bulat, dan itu berarti menjadi faktor untuk seluruh himpunan X. Jadi…”balas Amer. “Ya, jadi jumlah anggota himpunan Y tinggal 5 saja, yaitu: {2,3,4,5,6},”terang Iman lagi. “Kalau begitu, banyaknya kemungkinan korespondensi satusatu yang mungkin antara Himpunan X dan Himpunan Y adalah 5! Kawan, alias 120 kemungkinan fungsi yang terbentuk,”kali ini Bintang memberanikan diri menjawab persoalan. “Keliru lagi, Kawan! Kau pun masih kurang teliti membaca persoalan yang diajukan, Kawan!”balas “Ya, aku setuju denganmu, Kawan. Frasa setiap bilangan anggota 𝑋𝑋 tidak dikaitkan dengan faktornya di 𝑌𝑌’ bukan hanya untuk angka 1. Tapi setiap angka yang menjadi anggota himpunan X , yaitu
{9,6,3,2,1}, tidak dikaitkan dengan dengan faktornya pada himpunan Y, yaitu {2,3,4,5,6}. Karenanya, tidak boleh terjadi interaksi antara{9} dan {3}; {6} dan {2,3,6}; {3}dan {3}; {2} dan {2},”tibatiba Yodha angkat bicara. “Tepat, Kawan. Apa yang disampaikan Yodha tepat sekali. Nah, hal itu juga menjadi koreksi atas formulasi umum yang merupakan cara untuk menghitung berapa banyak korespondensi | 295
satu-satu yang dapat dibuat dari himpunan X dan himpunan Y. Dan interaksi yang dimungkinkan itu seperti tertera pada Tabel di bawah ini,”tegas Iman.
Iman memperlihatkan gambar tentang interaksi yang mungkin antara Anggota Himpunan X dan Anggota Himpunan Y yang membentuk korespondensi satu-satu dengan syarat seperti diminta. “Selanjutnya,
kita
memang
harus
kembali
ke
cara
konvensional alias manual. Setelah dikoreksi dengan syarat tidak dikaitkan dengan faktornya, maka lankah selanjutnya adalah membuat semua kemungkinan yang bisa terjadi. Untuk ini mohon sedikit waktu Kaan-kawan bersabar menunggu,”ucap Iman. Hampir 5 menit Iman menggambar, ia berbalik. “Aku sudah melukiskan semua kemungkinan korespondensi satu-satu dari Himpunan X ke Himpunan Y dengan syarat di atas. Hal terpenting yang harus mendapat perhatian adalah tentang range atau daerah hasil. Karena 1 pada Himpunan Y bukan merupakan range, maka kita akan mendapatkan: (1) Range dari g(9) yang mungkin adalah {2, 4, 5, 6} (2) Range dari g(6) yang mungkin adalah {4, 5} (3) Range dari g(3) yang mungkin adalah {2, 4, 5, 6} (4) Range dari g(2) yang mungkin adalah {3, 4, 5, 6} | 296
(5) Range dari g(1) yang mungkin adalah {2, 3, 4, 5, 6} Sehingga, berangkat dari akan terdapat 4 kondisi yang mungkin terjadi: Pertama, kondisi g(9) = 6, g(6) = 5, dan g(6) = 4, yaitu: (i) Untuk g(9) = 6, g(6) = 5 berarti hanya mencari range yang mungkin dari 3, 2, dan 1. Perhatikan gambar ilustrasi:
Terlihat bahwa terdapat 3 fungsi yang terbentuk; (ii) Untuk g(9) = 6, g(6) = 4, berarti hanya mencari range yang mungkin dari 3, 2, dan 1. Perhatikan gambar ilustrasi:
Terlihat bahwa terdapat 3 fungsi terbentuk. Sehingga untuk kondisi g(9) = 6, jumlah fungsi yang terbentuk 3 + 3 = 6; Kedua, kondisi g(9) = 5, g(6) = 4, mencari range yang mungkin dari 3, 2, dan 1. Perhatikan gambar ilustrasi berikut:
| 297
Terlihat bahwa terdapat 3 fungsi terbentuk. Sehingga untuk kondisi g(9) = 5, jumlah fungsi yang terbentuk 3. Ketiga, kondisi g(9) = 4, g(6) = 5, mencari range yang mungkin dari 3, 2, dan 1. Perhatikan gambar ilustrasi di bawah ini:
Terlihat bahwa terdapat 3 fungsi yang terbentuk. Sehingga untuk kondisi g(9) = 4, jumlah fungsi yang terbentuk 3; Keempat, kondisi (1) di mana g(9) = 2, g(6) = 5, tinggal mencari range yang mungkin dari 3, 2, dan 1. Perhatikan ilustrasi di bawah:
Terlihat bahwa terdapat 4 fungsi yang terbentuk. Sehingga untuk kondisi g(9) = 2, g(6) = 5, jumlah fungsi yang terbentuk sebanyak 3. Kondisi (2) di mana g(9) = 2, g(6) = 4, tinggal mencari range yang mungkin dari 3, 2, dan 1. Dan ilustrasinya adalah:
| 298
Terlihat bahwa 4 fungsi yang terbentuk. Sehingga untuk kondisi g(9) = 2, g(6) = 4, jumlah yang terbentuk sebanyak 4 fungsi. Karenanya untuk kondisi g(9) = 2, , jumlah fungsi yang terbentuk 4+4=
8.
Akhirnya,
jumlah
total
kemungkinan
terjadinya
korespondensi satu-satu fungsi dari himpunan 𝑋𝑋 = {9, 6, 3, 2, 1} ke
himpunan 𝑌𝑌 = {1, 2, 3, 4, 5, 6}, dimana setiap bilangan anggota 𝑋𝑋 tidak dikaitkan dengan faktornya di 𝑌𝑌 ada sebanyak 6 + 3 + 3 + 8 =
20 fungsi. Itu yang bisa aku sampaikan untuk menyelesaikan persoalan nomor 9, Kawan-kawan!”pungkas Iman. “Plok, plok, plok, plok, plok, plok, plok, plok, plok, plok, plok, plok,”terdengar tepuk tangan yang meriah dan panjang semua kawan Ikram. Bahkan semua yang bertepuk tangan sambil berdiri. Dan Sabiq serta Yodha pun kali ini tak lagi hanya senyum-senyum dan mengangguk-angguk. Mereka berdua ikut bertepuk tangan dan berdiri buat Iman. “Terima kasih, Kawan-kawan. Aku merasa sangat terhormat menerima standing applause ini,”balas Iman datar. “Baik-kawan-kawan, tinggal nomor terakhir Bagian A. giliranmu, Yodha!”ujar Sabiq Sesaat Yodha akan beranjak, terdengar suara dari belakang. “Biq, mohon maaf sebelumbnya. Waktu kita menjelang ashar sudah semakin sempit. Sementara Yodha dan kau belum memberi penjelasan pada kita,”tiba-tiba Ayra berbicara agak serius. “Apa maksudmu, Ayra?”tanya Sabiq. “Setelah aku perhatikan, soal nomor 10 ini masih seputar pelaung dan bagian dari kombinatorik. Sementara pada Bagian B ada | 299
persoalan baru yang belum pernah kita sentuh. Aku khawatir waktu kita tak cukup, sementara soal itu tak tersentuh,”kata Ayra lagi. “Jadi, maksudmu?”tanya Sabiq lagi. “Kalau aku usulkan sebaiknya kita tinggalkan sementara soal nomor 10 itu, sementara itu kau dan Yodha selesaikan beberapa bagian dari Bagian B,”usul Ayra. Beberapa
kawan
mereka
saling
pandang.
Ada
yang
mengangguk tanda setuju. “Boleh juga usul Ayra kita pertimbangkan. Bukankah soal Kombinatorik sudah beberapa kali kita selesaikan, sementara yang baru itu belum pernah kita singgung. Aku sepakat dengan usul Ayra,”komentar Fateh. “Kalau itu pertimbangannya, aku juga mendukung pendapat Ayra, Biq,”kata Saver. Denan mengangguk, Bily juga. Iman mengangkat jempol tangan kanannya, Bintang juga. Sementara Amer tampak senang dengan usulan Ayra itu. “Iyalah, Kawan. Kita juga ingin lihat atraksi Sang Dukun, Sastrawan dan Penerjemah ini memainkan jari-jemarinya di depan,”komentar Amer. “Baiklah, karena sudah lebih dari separuh yang usul, dan tak ada yang membantah, kita sepakati saja usul Ayra. Apakah kau sudah siap, Yodha?”tanya Sabiq. “Aku belum cek persoalannya, kau dululah selesaikan bagian yang mungkin dikerjakan. Atau tanya Ayra, apa persoalan pada Bagian B yang belum pernah kita sentuh.itu,”usul Yodha. | 300
Sabiq melirik ke arah Ayra. Yang dilirik paham maksud dari lirikan itu. “Aku penasaran dengan soal Nomor 1 dan Nomor 5, Kawan!”kata Ayra. Serempak semua kawannya pun melirik ke arah soal yang ditunjuk Ayra. Beberapa diantara mereka membenarkan apa yang disebut oleh Ayra. “Kalau Nomor 1 ya, memang kita belum pernah sentuh persoalan seperti itu. Tapi, soal nomor 5 rasanya pernah, yaitu saat awal-awal kita membentuk Genk OSN ini, Kawan,”terang Sabiq. “Ya, tapi belum bagi sebagian kawan yang lain,”tegas Ayra. Sabiq tersadar. Ia sempat lupa bahwa saat ini Genk OSN telah bertambah squad-nya menjadi Kesebelasan yang lebih tangguh. “Kau benar, Ayra. Baiklah kita selesaikan nomor yang kau maksud. Bagian yang tepat untuk Yodha, sesuai keahliannya aku pikir adalah soal Nomor 3, perihal Geometri. Bagaimana, Kawankawan?”tanya Sabiq “Setuju!” terdengar jawaban serempak dari kawan-kawannya. “Kau paling bisa kalau mengarahkan perhatian dan pendapat kawan-kawan, Ayra!”puji Ikram. Ayra hanya tersenyum. “Baik, aku mulai dengan soal nor 1 selanjutnya Yodha bersiap dengan soal nomor 3. Apakah ada yang ingin jadi volunteer untuk soal nomor 5?”tanya Sabiq. Suasana hening. Tak ada suara. Tak ada jawaban.
| 301
“Sudahlah, kau selesaikan saja, Kawan. Kami menikmati sajianmu,”komentar Amer. Sabiq mendekat kea rah papan tulis, diambilnya alat tulis, dan dia pun memulai aksinya. “Soal
ini
adalah
soal
tentang
pertidaksamaan.
Dan
penyelesaiannya sama seperti persamaan. Hanya saja tandanya bukan sama dengan, tetapi lebih besar atau lebih kecil. Sehingga kemungkinan hasilnya bukanlah angka atau bilangan tertentu biasa rentang
nilai
atau
interval
tertentu,”papar
Sabiq
memulai
penjelasannya. “Kalimat dan untai penjelasan seperti ini yang aku tunggu darimu, Kawan!’sela Amer. “Simpan dulu komentar dan pujianmu, Kawan. kita fokus, Oke!”balas Sabiq. “Soal nomor satu menyebutkan bahwa kita diminta untuk menemukan semua bilangan real 𝑥𝑥 yang memenuhi persamaan √2 − x > 2. Terhadap persoalan ini ada syarat yang harus
dipenuhi, yaitu bahwa: bilangan yang berada di bawah tanda akar harus lebih besar atau sama dengan nol. Karenanya: 2 − 𝑥𝑥 ≥ 0 2 ≥ 𝑥𝑥
𝑥𝑥 ≤ 2 … … … … … … … . . (14.11)
Persamaan (14.11) dalam bentuk garis bilangan digambarkan sebagai berikut:
| 302
Berikutnya adalah menyelesaikan persamaannya sendiri, yaitu dengan mengudratkan kedua sisi untuk menghilangkan tanda akar, sehingga: 2
�√2 − 𝑥𝑥� > 22
2 − 𝑥𝑥 > 4 2 − 4 > 𝑥𝑥 −2 > 𝑥𝑥
𝑥𝑥 < −2 … … … … … … . . (14.12)
Persamaan (14.12) dalam bentuk garis bilangan sebagai:
Selanjutnya,
menentukan
himpunan
penyelesaian,
yaitu
menggabungkan hasil dari syarat dan penyelesaian persamaan itu sendiri, yakni dengan memotongkan kedua bentuk garis bilangan sehingga hasil akhirnya adalah sebagai berikut:
Sehingga himpunan penyelesaiannya adalah 𝑥𝑥│𝑥𝑥 < −2, 𝑥𝑥 ∈ real. Demikian penjelasan atas soal nomor 1,”pungkas Sabiq.
Tepuk tangan meriah dan ucapan selamat dari kawankawannya tertuju pada Sabiq. “Bagaimana Yodha, sudah kau temukan penyelesaian atas soal nomor 3 itu?”tanya Sabiq. “Insyaallah!
Aku
sudah
membuat
gambar
dan
jalan
penyelesaiannya,”jawab Yodha.
| 303
“Kalau begitu, sekarang giliranmu, sementara aku coba menyelesaikan soal nomor 5,”tegas Sabiq. Yodha maju ke depan. Tak ada keraguan dalam langkah Yodha. Dan kawan-kawannya pun berkeyakinan penuh atas kemampuan Yodha dalam soal-soal Geometri. “Soal in merupakan gabungan dua bangun ruang yang disebut saling bersinggungan. Kedua bangun itu adalah limas dan bola. Dinyatakan bahwa, ‘diberikan kerangka limas 𝐴𝐴𝐵𝐵𝐶𝐶𝐷𝐷 dengan
alasnya adalah daerah segitiga siku-siku 𝐴𝐴𝐵𝐵𝐶𝐶. Diketahui sisi siku-
sikunya adalah 𝐴𝐴𝐵𝐵 dan 𝐴𝐴𝐶𝐶 dengan panjang 𝐴𝐴𝐵𝐵 = a√3 dan panjang
𝐴𝐴𝐶𝐶 = 4𝑎𝑎 , rusuk 𝐵𝐵𝐷𝐷 tegak lurus dengan bidang 𝐴𝐴𝐵𝐵𝐶𝐶 , dan panjang
𝐵𝐵𝐷𝐷 = 6𝑎𝑎 . Jika pada rusuk 𝐶𝐶𝐷𝐷 terdapat titik 𝑃𝑃 sehingga sebuah bola dengan 𝐷𝐷𝑃𝑃 sebagai diameternya menyinggung bidang alas 𝐴𝐴𝐵𝐵𝐶𝐶, hitung jari-jari bola tersebut’. Terhadap soal ini dan soal-soal
Geometri pada umumnya langkah yang paling dianjurkan adalah sebisa
mungkin
menggambarkan
atau
melukiskannya,”Yodha
memulai paparannya. Tampak, seperti biasa kawan-kawannya menikmati sajian Geometri a la Yodha. Kali ini pun tampaknya mereka akan mendapat sajian khusus. “Dari informasi soal yang diberikan bahwa, maka kira-kira gambarannya yang mungkin adalah sebagai berikut:
| 304
Lukisan
pertama
adalah
lukisan
seolah-olah
sesungguhnya,
sementara lukisan kedua merupakan penyederhanaan dari titik singgung kedua bangun ruang. Sementara data-data besaran yang diberikan adalah sebagai berikut: 𝐴𝐴𝐵𝐵 = 𝑎𝑎√3; 𝐴𝐴𝐶𝐶 = 4𝑎𝑎; dan 𝐵𝐵𝐷𝐷 = 6𝑎𝑎.
Sehingga, kita bisa menentukan bahwa: OD = OP = OQ adalah jarijari bola = Rb. Dari lukisan pertama, perhatikan segitiga siku-siku BAC, dengan Phytagoras diketahui: 𝐴𝐴𝐵𝐵2 + 𝐴𝐴𝐶𝐶 2 = 𝐵𝐵𝐶𝐶 2
𝐵𝐵𝐶𝐶 = �〰 𝐵𝐵2 + 𝐴𝐴𝐶𝐶 2
𝐵𝐵𝐶𝐶 = �(a√3)2 + (4a)2 𝐵𝐵𝐶𝐶 = �3𝑎𝑎2 + 16𝑎𝑎2
𝐵𝐵𝐶𝐶 = 𝑎𝑎√19
Dari lukisan kedua, perhatikan segitiga siku-siku BCD, dengan Phytagoras diketahui: 𝐵𝐵𝐶𝐶 2 + 𝐵𝐵𝐷𝐷 2 = 𝐶𝐶𝐷𝐷 2
𝐶𝐶𝐷𝐷 = �𝐵𝐵𝐶𝐶 2 + 𝐵𝐵𝐷𝐷 2
𝐶𝐶𝐷𝐷 = �(a√19)2 + (6a)2
𝐶𝐶𝐷𝐷 = �19𝑎𝑎2 + 36𝑎𝑎2
𝐶𝐶𝐷𝐷 = 𝑎𝑎√55
Selanjutnya, dari frasa, ‘… sehingga sebuah bola dengan 𝐷𝐷𝑃𝑃 sebagai
diameternya menyinggung bidang alas 𝐴𝐴𝐵𝐵𝐶𝐶’ berarti bahwa ada titik
singgung atara bidang alas ABC pada limas dengan bola. Pada
lukisan kedua titik itu adalah titik Q. Sehingga, akan bisa dilukiskan sebagai lukisan ketiga sebagai berikut: | 305
Dimana garis OQ merupakan jari-jari bola, Rb. Perhatikan garis CD: 𝑂𝑂𝐶𝐶 = 𝐶𝐶𝐷𝐷 − 𝑂𝑂𝐷𝐷
𝑂𝑂𝐶𝐶 = 𝑎𝑎√55 − 𝑅𝑅𝑏𝑏
Di sini kembali kita gunakan prinsip dasar dalam geometri, kesebangunan, dalam hal ini segitiga sebangun,”terang Yodha. “Sebangun yang tadi malam kau terangkan itu, kan Yodha?”tanya Amer. “Tepat, kawan. Perhatikan lukisan ketiga, lebih tepanya perhatikan segitiga siku-siku CBD dan seitiga siku-siku CQO. Dengan prinsip kesebangunan itu, apa yang bisa kau ceritakan, Mer?”tiba-tiba Yodha bertanya pada Amer. “Mengacu kepada tiga prinsip kesebangunan, maka segitiga siku-siku CBD adalah sebangun dengan segitiga siku-siku CQO, sehingga akan berlaku perbandingan sebagai berikut: 𝑂𝑂𝑄𝑄 𝐶𝐶𝑂𝑂 = 𝐷𝐷𝐵𝐵 𝐶𝐶𝐷𝐷
dan keduanya berhimpit pada ∠C. Betulkan, Kawan!”jawab sekaligus tanya Amer. “Tepat
sekali,
Kawan!
Dan
selanjutnya,
berdasarkan
perbandingan yang disebutkan oleh Amer di atas, dengan mensubstitusikan nilai CD yang sudah diketahui akan diperoleh: | 306
𝑂𝑂𝑄𝑄 𝐶𝐶𝑂𝑂 = 𝐷𝐷𝐵𝐵 𝐶𝐶𝐷𝐷
𝑅𝑅𝑏𝑏 𝑎𝑎√55 − 𝑅𝑅𝑏𝑏 = 6𝑎𝑎 𝑎𝑎√55
𝑅𝑅𝑏𝑏 𝑎𝑎√55 = 6𝑎𝑎(𝑎𝑎√55 − 𝑅𝑅𝑏𝑏 )
𝑅𝑅𝑏𝑏 𝑎𝑎√55 = 6𝑎𝑎2 √55 − 6𝑎𝑎𝑅𝑅𝑏𝑏
𝑅𝑅𝑏𝑏 𝑎𝑎√55 + 6𝑎𝑎𝑅𝑅𝑏𝑏 = 6𝑎𝑎2 √55 𝑅𝑅𝑏𝑏 (𝑎𝑎√55 + 6𝑎𝑎) = 6𝑎𝑎2 √55 𝑅𝑅𝑏𝑏 = 𝑅𝑅𝑏𝑏 =
𝑅𝑅𝑏𝑏 =
6𝑎𝑎2 √55
𝑎𝑎√55 + 6𝑎𝑎
𝑎𝑎(6𝑎𝑎�55)
𝑎𝑎�(55 + 6) 6𝑎𝑎√55
√55 + 6
Ini adalah nilai jari-jari bola yang dicari. Namun, karena masih dalam bentuk yang tidak sederhana, maka hasus dirasionalkan dengan mengalikan ke conjugate-nya. Ada yang mau coba?”tawar Yodha pada kawan-kawannya. “Aku mau coba!”kata Denan “Silahkan, Kawan!”balas Yodha sambil memberikan alat tulis pada Denan. “Karena nilai jari-jari bola adalah 𝑅𝑅𝑏𝑏 =
6𝑎𝑎√55
√55+6
, belum
sederhana, maka harus dirasionalkan dengan cara mengalikan Rb dengan conjugate-nya, sehingga menjadi: 𝑅𝑅𝑏𝑏 =
6𝑎𝑎√55
𝑥𝑥
√55 − 6
√55 + 6 √55 − 6
| 307
Dengan mengingat prinsip bahwa: (a + b) (a – b) = a2 – b2, maka: 𝑅𝑅𝑏𝑏 = 𝑅𝑅𝑏𝑏 = 𝑅𝑅𝑏𝑏 = 𝑅𝑅𝑏𝑏 =
6𝑎𝑎√55(√55 − 6)
�√55 + 6�(√55 − 6) 2
6𝑎𝑎(�55) − 36𝑎𝑎√55 2
(�55) − 62
55𝑥𝑥6𝑎𝑎 − 36𝑎𝑎√55 55 − 36 330𝑎𝑎 − 36𝑎𝑎√55 19
(330 − 36√55)𝑎𝑎 19 Jadi, hasil akhir dari perhitungan jari-jari bola itu adalah sebesar 𝑅𝑅𝑏𝑏 =
𝑅𝑅𝑏𝑏 =
(330−36√55)𝑎𝑎 , 19
“Tepat
kawan-kawan!”terang Denan.
cara
perhitunganmu,
Kawan.
Memang
itu
jawabannya,”pungkas Yodha. “Plok, plok, plok, plok, plok,”terdengar tepuk tangan meriah dari kawan-kawan Yodha. Juga tepukan dari Sabiq. “Alhamdulillah, Yodha menunjukkan pada kita bagaimana indahnya kesederhanaan dalam kesebangunan geometri, selanjutnya, bagian berikutnya adalah bagianku untuk menyelesaikannya, dan bagian ini ....,”Sabiq menghentikan kalimatnya karena telinganya mendengar suara lain dari bagian samping . “Kawanku Sabiq, kalau masih ada waktu sekitar lima menit, izinkan aku selesaikan soal nomor 2 untuk kita semua,”terdengar kalimat datar Bintang memecah keseriusan itu.
| 308
“Kenapa tidak. Kita masih punya waktu sekitar 15 menit sebelum Ashar. Kita berbagi saja. Bagaimana Kawan-kawan?”tanya Sabiq balik pada kawan-kawannya. “Tawaran yag menguntungkan kita semua, kenapa harus ditolak?”komentar Fateh. “Justru akan sangat baik menambah variasi pemahaman kita, kan!”timpal Ayra. “Aku senang dengan ide seperti ini, beri dia kesempatan, Biq!”pinta Amer. Sementara Yodha, Ikram, Saver dan yang lainnya memberikan senyuman dan anggukan mereka. “Semua kawan sudah memberi tempat bagimu. Kawan. Silahkan maju, Bintang!”pinta Sabiq “Baik, terima kasih Kawan-kawan. Karenanya, langsung saja,”balas Bintang sambil berjalan menuju papan tulis. “Disebutkan bahwa ‘diketahui jumlah 𝑛𝑛 buah bilangan bulat
positif ganjil berurutan adalah 5929. Tentukan 𝑛𝑛 terkecil yang
mungkin’. Terus terang, sedari tadi sambil menikmati sajian pembahasan Sabiq dan Geometri Yodha aku gabungkan antara terapan coba-coba Ayra dan tabel kombinasi Fateh,”kalimat pembuka Bintang. Sabiq tampak heran, Fateh juga. Ayra terkejut namanya disebut, dan kawan-kawan lain pun tertegun dengan kalimat cobacoba. “Dan ternyata kau menemukan solusinya, Kawan?”tanya Amer penasaran. | 309
“Menurut nalarku, ya. Tapi, nanti kita lihat. Untuk itu, aku mohon tambahan penjelasan setelah itu,” jelas Bintang. “Tapi, apa dasar yang kau pakai untuk kombinasikan cobacoba dan tabel itu, Kawan?”tanya Denan pun heran. “Ya, ada dasarnya. Tiga prinsip dasar. Dan hal itu adalah salah satu pertanyaan kuncinya. Bukankah ada frasa dalam pertanyaan yang menyebutkan bahwa ‘jumlah 𝑛𝑛 buah bilangan’. Hal ini berari
bahwa pasti n lebih besar dari 1, n > 1. Kedua, bilangan itu pastilah habis dibagi oleh n. Dan ketiga, karena n membagi pasti ia berada di tengah. Dari situlah aku coba beberapa variasi dan kemungkinan hasil, dan ini tabelnya,”jawab Bintang sambil menuju papan tulis dan menggambar tabel yang dia maksud.
Hasil Pembagian
Keterangan
(5929/3) = 1976,33 Tidak mungkin, bersisa dan tak habis dibagi. (5929/5) = 1185,8
Tidak mungkin, bersisa dan tak habis dibagi.
(5929/7) = 847
Mungkin karena tak bersisa dan habis dibagi
Dari hasil ini, selanjutnya aku pakai prinsip ketiga bahwa nilai hasil bagi pasti berada di tengah, sehingga: karena terdapat n = 7 bilangan ganjil: (1) Pasti ada 3 bilangan bulat ganjil berada di sebelah kanan n, yaitu: 849, 851, 853 (2) Pasti ada 3 bilangan bulat ganjil lainnya berada di sebelah kiri n, yaitu: 845, 843, 841. | 310
Selanjutnya, aku uji dengan menjumlahkan kembali ketujuh bilangan bulat positif ganjil itu: 841 + 843 + 845 + 847 + 849 + 851 + 853 = 𝟓𝟓𝟗𝟗𝟐𝟐𝟐𝟐
Ternyata hasil akhirnya memenuhi seperti yang ditanyakan. Demikian penjelasanku, Kawan-kawan,”terang Bintang menyudahi. “Subhanallah! Nalarmu tepat sekali, Kawan!”komentar Sabiq. “Terima kasih, Biq. Itu hasil dari interaksi kita selama ini. Aku belajar dari apa yang di-sharing kawan-kawan dalam diskusi kita,”balas Bintang bersahaja. “Ada yang ingin kau tambahkan tampaknya, Kawan?”tanya Fateh pada Sabiq saat melihat memberi pujian pada Bintang dengan senyuman khas itu. “Tidak begitu prinsip, hanya hal teknis saja, Kawan,”balas Sabiq “Silahkan, Kawan Satrawanku!”ujar Fateh lagi. “Apa yang dikerjakan Bintang adalah nalar yang dipandu oleh prinsip yang benar. Hal utama dalam kasus ini yang bisa ditambahkan sebagai prinsip dan luput dari keterangan Bintang adalah, ‘karena 5929 adalah bilangan ganjil dan merupakan penjumlahan dari n bilangan bulat positif ganjil, pasti n merupakan bilangan ganjil. Sebenarnya dari paparan Bintang sudah tergambar, hal itu saat dia menetapkan 3, 5, dan 7 sebagai pembagi. Tapi terlepas dari semua itu, apa yang disampaikan oleh Bintang sebagai hasil dari penalarannya, merupakan sesuatu yang logis dan menjawab permasalahan. Dan untuk itu, Bintang berhak atas applause yang hangat dari kita semua,”terang Sabiq. | 311
Terdengar tepukan meriah dari kawan-kawan Bintang. Dan bintang pun membalasa dengan senyum yang hangat. “Terima kasih, Kawan-kawan,”ujar Bintang. “Nah selanjutya, kau teruskan yang menjadi bagianmu, Kawan! Waktu kita semakin sempit,”tiba-tiba Amer dengan suaranya yang khas memecah kegemberiaan itu. “Baik, kalau boleh aku sebut, soal nomor 5 ini merupakan ujian kesabaran dan ketelitian. The Queen of the Problems,”ujar Sabiq memulai penjelasannya. “Nah, kutipan-kutipan indah tak terduga ini yang aku tunggu darimu, Kawan. The Queen of the Problems, aku suka itu. Bagaimana
dengan
kalian,
Kawan-kawan?”pancing
Amer
memanaskan suasana. Kawan-kawannya tahu tabiat Amer. Meskipun sesungguhnya, mereka sebenarnya setuju dengan apa yang disampaikan oleh kawan mereka yang tampak tanpa beban dalam menyampaikan pendapatnya itu. “Dari informasi soal disebutkan bahwa, ‘Untuk 𝑥𝑥 bilangan
real, dirumuskan suatu fungsi 𝑓𝑓(𝑥𝑥) = penjumlahan
berikut:
2 . 2+4 𝑥𝑥
Maka hitunglah hasil
1 2 � + 𝑓𝑓 � �+ 2014 2014
𝑓𝑓 �
2013 �’. 2014
⋯ + 𝑓𝑓 �
Terhadap persoalan seperti ini, seperti tadi aku sebutkan merupakan
ujian kesabaran dan ketelitian, lebih tepatnya sabar dan teliti dalam menemukan pola dari persoalan yang ditanyakan,”tambah Sabiq. Sabiq merapat ke arah papan tulis, dia mulai menuliskan beberapa hal di sana. | 312
“Kawan-kawan, mohon perhatikan bentuk fungsi pertama dan terakhir, yaitu: 1 � 2014
Fungsi pertama : 𝑓𝑓 �
2013 �. 2014
dan fungsi terakhir: 𝑓𝑓 �
Langkah pertama adalah melihat dan membuat hubungan antar fungsi. Perhatikan bahwa bentuk lain dari fungsi terakhir dapat kita manipulasi atau ubah sehingga memiliki hubungan dengan fungsi pertama, yakni: 2013 2014 − 1 𝑓𝑓 � � = 𝑓𝑓 � � 2014 2014 20141 1 2014 − 1 � = 𝑓𝑓 � − � 𝑓𝑓 � 2014 2014 2014 1 2013 � = 𝑓𝑓 �1 − � 𝑓𝑓 � 2014 2014
Lagkah berikutnya adalah membuat pemisalan atas suku. 1
Misalkan 𝑝𝑝 = 2014 maka 1 � 2014
𝑓𝑓 �
Sehingga,
𝑓𝑓(𝑥𝑥) =
= 𝑓𝑓(𝑝𝑝) dan 𝑓𝑓 �1 −
1 � 2014
= 𝑓𝑓(1 − 𝑝𝑝)
2 2 + 4𝑥𝑥
jika diubah dalam bentuk f(p), akan berubah menjadi: 𝑓𝑓(𝑝𝑝) =
2 2 + 4𝑝𝑝
Selanjutnya jika f(p) dijumlahkan dengan f(1-p) adalah: 𝑓𝑓(𝑝𝑝) + 𝑓𝑓(1 − 𝑝𝑝) =
𝑓𝑓(𝑝𝑝) + 𝑓𝑓(1 − 𝑝𝑝) =
2 2 + 𝑝𝑝 2 + (4)1−𝑝𝑝 2 + (4)
2(2 + (4)1−𝑝𝑝 ) + 2(2 + (4)𝑝𝑝 ) (2 + (4)𝑝𝑝 )(2 + (4)1−𝑝𝑝 )
| 313
𝑓𝑓(𝑝𝑝) + 𝑓𝑓(1 − 𝑝𝑝) =
4 + 2(4)1−𝑝𝑝 + 4 + 2(4)𝑝𝑝 4 + 2(4)1−𝑝𝑝 + 2(4)𝑝𝑝 + (4)1−𝑝𝑝 (4)𝑝𝑝
𝑓𝑓(𝑝𝑝) + 𝑓𝑓(1 − 𝑝𝑝) =
8 + 2(4)1−𝑝𝑝 + 2(4)𝑝𝑝 8 + 2(4)1−𝑝𝑝 + 2(4)𝑝𝑝
8 + 2(4)1−𝑝𝑝 + 2(4)𝑝𝑝 𝑓𝑓(𝑝𝑝) + 𝑓𝑓(1 − 𝑝𝑝) = 4 + 2(4)1−𝑝𝑝 + 2(4)𝑝𝑝 + (4)1
Kawan-kawan, perhatikan bantuk akhir dari 𝑓𝑓(𝑝𝑝) + 𝑓𝑓(1 − 𝑝𝑝).
Ternyata merupakan betuk lain dari pecahan (a/b), dimana nilai a = nilai b. Dengan kata lain: 𝑓𝑓(𝑝𝑝) + 𝑓𝑓(1 − 𝑝𝑝) = 1
Dari bentuk penjumlahan di atas, bila kita kembalikan kepada bentuk asal penjumlahan 𝑓𝑓 �
1 �+ 2014
2 �+ 2014
𝑓𝑓 �
2013 � 2014
⋯ + 𝑓𝑓 �
dan masing-
masing didekatkan sehingga memenuhi bentuk akhir pesamaan, maka: 1 2013 𝑓𝑓 � � + 𝑓𝑓 � �=1 2014 2014 2013 1 � + 𝑓𝑓 � �=1 𝑓𝑓 � 2014 2014 1 2013 � + 𝑓𝑓 � �=1 濄 � 2014 2014 .
+
.
1008 1006 � + 𝑓𝑓 � �=1 𝑓𝑓 � 2014 2014
= 1(1006)
1007 � 2014
Sementara untuk satu suku, yaitu 𝑓𝑓 �
tidak memiliki pasangan,
dan karenanya harus dipisahkan, dan diselesaikan tersendiri, sehinga:
| 314
1007 𝑓𝑓 � �= 2014
1007 �= 𝑓𝑓 � 2014
Karena nilai dari 1
maka:
2
1007
2 + 4(2014) 2 1
2 + 4( 2 )
4(2) = √4 = 2
2 2 1007 �= = 𝑓𝑓 � 2+2 4 2014 1 1007 �= 𝑓𝑓 � 2 2014
Sehingga hasil akhir penjumlahannya adalah sebagai berikut: 2 2013 1 � + 𝑓𝑓 � � + ⋯ + 𝑓𝑓 � � 𝑓𝑓 � 2014 2014 2014 1007 � = 1(1006) + 𝑓𝑓 � 2014 2 2013 1 1 � + 𝑓𝑓 � � + ⋯ + 𝑓𝑓 � � = 1006 + � � 𝑓𝑓 � 2014 2014 2 2014 1 = 1006 2 2013 = 2
Itulah hasil akhir dari proses kesabaran dan ketelitian kita, Kawankawan. Dan terima kasih atas kesediaannya untuk sabar menunggu dan teliti menelaah setiap langkahnya,”pungkas Sabiq. “Kau memang Satrawan, Dukun, Penerjemah, yang Gila, Biq!”tiba-tiba Amer kembali berseru. Dan seruan itu menambah satu lagi gelar Sabiq, Gila! | 315
Tepuk tangan meriah dan decak kagum kawan-kawan diapresiasikan atas kerja-teliti dan kerja-sabar ini. Kerja mereka semua, kerja Kesebelasan Genk OSN. All for One, One for All. …∑πχαΩ…
| 316
15
Tabel, Grafik, dan Coba-Coba
‘Ashar adalah waktu istimewa! Begitu berartinya sampai Tuhan Seru Sekalian Alam, Allah Subhanahu wa Ta’ala pun mengabadikan sumpahNya demi waktu ‘Ashar itu. ‘Ashr adalah tanda peralihan, dari masa teriknya matahari siang kepada redup perlahan. ‘Ashar adalah perubahan, dari pertumbuhan dan perkembangan maksimal menuju kedewasaan dan penuaan. Dan ‘Ashr juga berarti estafeta, dari satu fase generasi ke fase generasi berikutnya. Itulah mengapa waktu ‘Ashar adalah waktu yang paling dianjurkan untuk tetap terjaga dan banyak beribadah kepada-Nya. Dan, dimulai pada waktu ‘Ashar itu pulalah, Kesebelasan Genk OSN, sebelas anak-anak jenis kampong, membagi cerita mereka, kali ini tahap akhir dari serial diskusi sejak tadi malam tentang persoalan-persoalan Matematika yang diujikan pada OSN di tingkat Kabupaten dan Kota. “Kawan-kawan, ada 30 soal yang dipertanyakan pada OSN tingkat Kabupaten dan Kota yang terbagi ke dalam dua bagian, yaitu Pilihan Ganda dan Isian Singkat. Bagian Pertama terdiri atas 20 soal memiih jawaban dan Bagian Kedua terdiri atas 10 soal uraian | 317
singkat. Bagaimana kita membaginya?”tanya Sabiq pada kawankawannya. “Formasi seperti biasa saja, Biq!”jawab Amer spontan. “Formasi apa itu, Mer?”tanya Denan penasaran. “Ya, Formasi 2-1 Plus! Dua orang satu Tim mengerjakan bagian proporsional dan 1 orang masing-masing menyelesaikan 1 soal bagian Isian Singkat,”jelas Amer. “Plus-nya?”tiba-tiba Bily dan Fateh bertanya serentak. “Giliran Sabiq sebagai Ban Serep! Soal-soal yang kita ragukan atau tak terbagi, menjadi kewajiban Sabiq menyelesaikannya sebagai pengarah diskusi,”terang Amer lagi. Kali ini hampir semua kawan Amer mengamini, hanya Sabiq yang tampak geleng-geleng kepala. “Nah, kalau begitu aku setuju denganmu,”timpal Iman datar. “Lama-lama, kau semakin cerdas dan arif saja, Kawan,”kata Yodha pelan sambil menepuk pundak kanan Amer. “Baiklah kita sepakati seperti Formasi 2-1 Plus itu,”tegas Sabiq. Kesebelas kawan itu tertawa bersama. Memang, selama mereka mengenal Amer, ada saja ulahnya. Setidaknya membuat kejutan-kejutan. Karenanya, bagi mereka yang tak terbiasa akan bingung. Tapi itulah Amer. Dan kawan-kawannya paham betul sifat Amer itu. “Baiklah kawan-kawan, kita hanya punya waktu 150 menit sejak sekarang sebelum Maghrib. Kita efektifkan waktu dan bagi menjadi dua bagian. Pertama, untuk membentuk kelompok dan | 318
memilih serta menganalisis soal 30 menit, dan Kedua untuk menyelesaiakan seluruh persoalan 120 menit. Bagaimana?”usul Sabiq. “Kami sepakat saja, Kawan,”balas Ayra. “Oke, silahkan masing-masing melihat berkas yang ada di meja dan 25 menit sejak saat ini, harus sudah kita mulai pembahasan,”ujar Sabiq. Semua anak tampak sibuk membuka beberapa lembar soal OSN Tingkat Kabupaten dan Kota setahun lalu yang sudah tersedia di atas masing-masing meja panjang itu. Hening sesaat. Tak perlu waktu
lama,
kembali
riuh,
mereka
bangkit,
berdiri
dan
mendiskusikan atas telaah mereka. Sabiq tampak tenang dan mengamati seluruh proses di hadapannya. Dia sendiri memang sejak lepas pembahasan session sebelumnya sudah membuka soal-soal itu. Bahkan jauh sebelum berangkat ke perkampungan untuk konsentrasi ini pun, ia sudah membaca ketiga kombinasi soal OSN itu. Tak sampai 15 menit, suasana kembali normal. Tampaknya masing-masing anak sudah menemukan siapa anggota tim-nya. Dan tampaknya pula telah menemukan penyelesaian yang paling tepat, setidaknya menurut mereka. “Baiklah kawan-kawan, tampaknya kita sudah menyelesaikan tahap pertama. Kita siap maju ke tahap kedua?”tanya Sabiq bersemangat. “Insyaallah!”terdengar jawaban serempak. “Baik, siapa dan siapa yang akan memulai?”tantang Sabiq. “Kami, Iman dan aku,”jawab Ayra tegas. | 319
“Silahkan, Kawan!”kata Sabiq lagi. Ayra dan Iman maju bersama. Mereka berdiri di depan menghadap kawan-kawannya. “Aku akan mengerjakan soal nomor 2 sementara Iman akan memulai pembukaannya. Silahkan, Kawanku!”kata Ayra. “Terima kasih, Ayra,”balas Iman sopan. Iman menerima alat tulis dari Ayra. Dia berkonsentrasi dan memulai gerakan tangannya. “Soal nomor satu ini singkat saja. Disebutkan bahwa ‘bentuk 4
x -1 mempunyai faktor sebanyak...’ berapa?”kalimat pembuka Iman. “Awalnya kami kira soal ini bercanda. Dan kami tegaskan bahwa faktornya sudah pasti 4 dan pilihannya adalah option B. Begini penjelasannya,”kata Iman lagi. “Kalau persamaan di atas kita uraikan menjadi bentuk yang lebih sederhana, sesungguhnya merupakan hasil dari pengurangan dari dua bilangan kuadrat, yaitu: 𝑥𝑥 4 − 1 = (𝑥𝑥 2 )2 − 12
Dengan mengingat bahwa: (x + a) (x – a) = x2 – a2, maka: (𝑥𝑥 2 )2 − 12 = (𝑥𝑥 2 − 1)(𝑥𝑥 2 + 1)
Selanjutnya perhatikan kedua suku hasil pemaktoran di atas, suku yang di sebelah kanan masih bisa kita uraikan lagi menjadi: (𝑥𝑥 2 − 1)(𝑥𝑥 2 + 1) = (𝑥𝑥 − 1)(𝑥𝑥 + 1)(𝑥𝑥 2 + 1)
Nah, sampai di sini kami justru menemukan bahwa ternyata hanya memiliki 2 faktor yang bisa diperoleh melalui pemaktoran, bukan 4 seperti tebakan kami semula. Kedua faktor yang diperoleh dari
| 320
pemaktoran yaitu (𝑥𝑥 − 1)(ኈ + 1). Sementara untuk suku (𝑥𝑥 2 + 1)
tak bisa difaktorkan. Tapi...,”Iman menghentikan kalimatnya. “Tapi, apa Kawan?”tanya Amer penasaran.
“Kalau kawan-kawan ingat saat kita membahas tentang Diskriminan? Apa yang dijelaskan oleh Yodha tentang bilangan tak real alias imajiner yang disimbulkan dengan huruf i, dan nilainya adalah sama dengan √-1 itu, maka kami kemudian mencoba memaktorkan kembali suku yang tampaknya tak bisa difaktorkan, yaitu x2+1, menjadi: (𝑥𝑥 2 + 1) = �𝑥𝑥 − √−1��𝑥𝑥 + √−1�
Sehingga, bentuk akhir pemaktorannya adalah: 𝑥𝑥 4 − 1 = (𝑥𝑥 − 1)(𝑥𝑥 + 1)�𝑥𝑥 − √−1��𝑥𝑥 + √−1�
𝑥𝑥 4 − 1 = (𝑥𝑥 − 1)(𝑥𝑥 + 1)(𝑥𝑥 − 𝑖𝑖)(𝑥𝑥 + 𝑖𝑖)
Karenanya, kami berkesimpulan bahwa jawaban awal kami terbukti, yakni memiliki 4 faktor dengan alasan faktornya merupakan bilangan real dan bilangan tak real,”pungkas Iman. “Kenapa bisa begitu?”tanya Amer lagi. “Pertama karena soalnya tidka membatasi jenis bilangan faktornya. Kedua, karena bilangan real itu ada dan bilangan tak real juga ada. Dan hal itu terbukti dalam persamaan dan gambar seperti sebelumnya telah kita diskusikan,”terang Ayra lagi. “Nah, kalau ini alasannya aku sependapat dengan kalian Ayra dan Iman,”tiba-tiba Sabiq angkat bicara memecah kebuntuan. “Tapi, kenapa bisa begitu, Biq?”tanya Ikram serius. “Karena sesuatu yang disebut sebagai bilangan imajiner itu dalam prosesnya menghasilkan persamaan yang terukur dan bisa | 321
membentuk sebuah gambar atau grafik yang nyata, seperti disebutkan oleh Ayra dan Iman tadi,”terang Sabiq. “Kau memang Sastrawan, Kawan. Tak ada lagi celahku membantah. Oke, aku bisa terima kebenaran dengan alasan,”tanggap Amer. “Selanjutnya, Ayra akan menjelaskan soal nomor 2. Silahkan Ayra!”kata Iman. Ayra maju dan menerima alat tulis dari Iman. “Soal nomor 2, seperti ini redaksinya, ‘jika a, b, c, dan d adalah bilangan bulat positif dibagi 13 berturut-turut bersisa 12, 9, 11, dan 7, maka 3a + 4b -3c +2d dibagi 13 akan bersisa..’. Terhadap soal ini, kami menyelesaikannya dengan 2 cara, yaitu : konsep habis terbagi dengan sisa dan model pemisalan dengan sisa,”terang Ayra membuka pembahasan. “Selanjutnya, untuk cara yang pertama, karena yang ditanyakan juga merupakan nilai sisa dari hasil bagi 13, maka untuk mencari sisanya kita cukup mensubstitusikan atau memasukkan nilai bilangan bulat positif tersebut ke dalam bentuk persamaan yang ditanyakan. Sehingga akan ditemukan persamaan sebagai berikut: 3𝑎𝑎 + 4𝑏𝑏 − 3𝑐𝑐 + 2𝑑𝑑 = 3(12) + 4(9) − 3(11) + 2(7)
3(12) + 4(9) − 3(11) + 2(7) = 53
Dan dengan membagi dengan 13, akan diperoleh hasil: 53 = 4 𝑠𝑠𝑖𝑖𝑠𝑠𝑠𝑠 1 13
Itu cara konsep habis terbagi dengan sisa. Selanjutnya dengan model pemisalan dengan sisa. Yaitu berdasarkan informasi: | 322
𝑎𝑎 = 𝑠𝑠𝑖𝑖𝑠𝑠𝑠𝑠 12 ↔ 𝑎𝑎 = 13𝑝𝑝 + 12 13 𝑏𝑏 = 𝑠𝑠𝑖𝑖𝑠𝑠𝑠𝑠 9 ↔ 𝑏𝑏 = 13𝑞𝑞 + 9 13 𝑐𝑐 = 𝑠𝑠𝑖𝑖𝑠𝑠𝑠𝑠 11 ↔ 𝑐𝑐 = 13𝑟𝑟 + 11 13 𝑑𝑑 = 𝑠𝑠𝑖𝑖𝑠𝑠𝑠𝑠 7 ↔ 𝑑𝑑 = 13𝑠𝑠 + 7 13
Dengan memasukkan nilai a, b, c, dan d ke persamaan awal didapat: 3𝑎𝑎 + 4𝑏𝑏 − 3𝑐𝑐 + 2𝑑𝑑 =
= 3(13𝑝𝑝 + 12) + 4(13𝑞𝑞 + 9) − 3(13𝑟𝑟 + 11) + 2(13𝑠𝑠 + 7)
= 13𝑝𝑝′ + 36 + 13𝑞𝑞 ′ + 36 − 13𝑟𝑟 ′ − 33 + 13𝑠𝑠 ′ + 14 = (13𝑝𝑝′ + 13𝑞𝑞 ′ − 13𝑟𝑟 ′ + 13𝑠𝑠 ′ ) + 53
Terlihat bahwa suku yang di dalam kurung dengan penanda p’, q’, r’, dan s’ merupakan bilangan yang habis dibagi oleh 13. Sementara suku 53 merupakan sisa, sehingga: 53 = 4(13) + 1
Jadi sisa pembagian 3𝑎𝑎 + 4𝑏𝑏 − 3𝑐𝑐 + 2𝑑𝑑 oleh 13 adalah 1.
Karenanya, baik menggunakan konsep habis terbagi dengan sisa maupun model pemisalan dengan sisa, tetap menghasilkan nilai akhir yang sama,”pungkas Ayra. Tepuk tangan meriah, diterima Iman dan Ayra. “Sebuah pembuka yang menarik,”komentar Yodha. “Baik, selanjutnya giliran tim siapa yang maju?”tanya Sabiq. “Kami, aku dan Bily,”jawab Ikram tenang. “Silahkan, Kawan!”balas Sabiq sambil menyerahkan alat tulis.
| 323
“Kami
sudah
mendiskusikan
soal
nomor
3
dan
4,
kesimpulannya kami berbagi, aku mengerjakan nomor 3 dan Bily mengerjakan nomor 4,”terang Ikram. Ikram kembali fokus. Ia mulai membaca persoalannya. “Terhadap soal nomor 3 yang menyebut bahwa ‘nilai ratarata kelas Aadalah 73, nilai rata-rata kelas B adalah 88. Jika jumlah kedua kelas tersebut adalah 75 dan nilai rata-rata kedua kelas adalah 80, maka banyak siswa kelas A adalah ... orang’ dapat dilakukan analisis berasarkan statistik sederhana. Kita buat permisalan sebagai berikut: 𝑋𝑋𝑎𝑎 = 𝑛𝑛𝑖𝑖𝑙𝑙𝑙𝑙𝑙𝑙 𝑟𝑟𝑎𝑎𝑡𝑡𝑡𝑡 − 𝑟𝑟𝑎𝑎𝑡𝑡𝑡𝑡 𝑘𝑘𝑒𝑒𝑙𝑙𝑙𝑙𝑙𝑙 𝐴𝐴 = 73
𝑋𝑋𝑏𝑏 = 𝑛𝑛𝑖𝑖𝑙𝑙𝑙𝑙𝑙𝑙 𝑟𝑟𝑎𝑎𝑡𝑡𝑡𝑡 − 𝑟𝑟𝑎𝑎𝑡𝑡𝑡𝑡 𝑘𝑘𝑒𝑒𝑙𝑙𝑙𝑙𝑙𝑙 𝐵𝐵 = 88
𝑋𝑋𝑔𝑔 = 𝑛𝑛𝑖𝑖𝑙𝑙𝑙𝑙𝑙𝑙 𝑟𝑟𝑎𝑎𝑡𝑡𝑡𝑡 − 𝑟𝑟𝑎𝑎𝑡𝑡𝑡𝑡 𝑘𝑘𝑒𝑒𝑑𝑑𝑑𝑑𝑑𝑑 𝑘𝑘𝑒𝑒𝑙𝑙𝑙𝑙𝑙𝑙 = 80
𝑛𝑛𝑎𝑎 = 𝑗𝑗𝑢𝑢𝑚𝑚𝑙𝑙𝑎𝑎ℎ 𝑠𝑠𝑖𝑖𝑠𝑠𝑠𝑠𝑠𝑠 𝑘𝑘𝑒𝑒𝑙𝑙𝑙𝑙𝑙𝑙 𝐴𝐴 = 𝑎𝑎
𝑛𝑛𝑏𝑏 = 𝑗𝑗𝑢𝑢𝑚𝑚𝑚𝑚𝑚𝑚ℎ 𝑠𝑠𝑖𝑖𝑠𝑠𝑠𝑠𝑠𝑠 𝑘𝑘扜𝑙𝑙𝑎𝑎𝑠𝑠 𝐵𝐵 = 𝑏𝑏
𝑛𝑛𝑔𝑔 = 𝑗𝑗𝑢𝑢𝑚𝑚𝑚𝑚𝑚𝑚ℎ 𝑠𝑠𝑖𝑖𝑠𝑠𝑠𝑠𝑠𝑠 𝑘𝑘𝑒𝑒𝑑𝑑𝑑𝑑𝑑𝑑 𝑘𝑘𝑒𝑒𝑙𝑙𝑙𝑙𝑙𝑙 = (𝑎𝑎 + 𝑏𝑏) = 75
Berdasarkan rumusan rata-rata gabungan, dapat dihitung: 𝑋𝑋𝑔𝑔 =
80 =
𝑛𝑛𝑎𝑎 𝑋𝑋𝑎𝑎 + 𝑛𝑛𝑏𝑏 𝑋𝑋𝑏𝑏 𝑛𝑛𝑔𝑔 𝑋𝑋𝑔𝑔
𝑎𝑎73 + (75 − 𝑎𝑎)88 75
80𝑥𝑥75 = 𝑎𝑎73 + (75 − 𝑎𝑎)88
(88 − 80)75 = (88 − 73)𝑎𝑎
𝑎𝑎 =
(88 − 80)75 (88 − 73)
𝑎𝑎 = 40
| 324
Sehingga banyaknya siswa kelas A dapat diketahui, yaitu 40 orang,”ujar Ikram mengakhiri penjelasannya. Tepuk tangan dan ucapan selamat serta terima kasih dari kawan-kawannya tertuju pada Ikram. “Untuk nomor selanjutnya, Bily yang akan menerangkan detailnya, silahkan, Bil!”terang Ikram. Bily menggantikan posisi Ikram di depan. Ia siap memulai aksinya. “Soal nomor 4 tentang perbandingan, lebih tepatnya perbandingan uang. Disebutkan bahwa ‘suatu hari perbandingan jumlah uang Netty dan Agit adalah 2:1. Sehari kemudian Netty memberikan uangnya sejumlah Rp. 100.000,00 kepada Agit. Sekarang perbandingan uang Netty dan Agit 1:3. Jumlah uang Netty sekarang adalah Rp....’. Terhadap jenis soal seperti ini, pemisalan adalah langkah yang terbaik. Karenanya pun kita akan membuat pemisalan itu,”terang Bily. Tampak Amer kali ini tak protes, ia menikmati penjelasan sejak Ikram menerangkan dan sekarang giliran Bily. Amer tetap serius dan menikmati penjelasan itu “Karenanya, kita misalkan: 𝑁𝑁𝑎𝑎 = 𝑗𝑗𝑢𝑢𝑚𝑚𝑚𝑚𝑚𝑚ℎ 𝑢𝑢𝑎𝑎𝑛𝑛𝑛𝑛 𝑁𝑁𝑒𝑒𝑡𝑡𝑡𝑡𝑡𝑡 𝑚𝑚𝑢𝑢𝑙𝑙𝑙𝑙 − 𝑚𝑚𝑢𝑢𝑙𝑙𝑙𝑙
𝐴𝐴𝑎𝑎 = 𝑗𝑗𝑢𝑢𝑚𝑚𝑚𝑚𝑚𝑚ℎ 𝑢𝑢𝑎𝑎𝑛𝑛𝑛𝑛 𝐴𝐴𝑔𝑔𝑖𝑖𝑡𝑡 𝑚𝑚𝑢𝑢𝑙𝑙𝑙𝑙 − 𝑚𝑚𝑢𝑢𝑙𝑙𝑙𝑙
Dari kalimat, ‘‘suatu hari perbandingan jumlah uang Netty dan Agit adalah 2:1”, kita dapat tuliskan persamaan: 𝑁𝑁𝑎𝑎 2 = 𝐴𝐴𝑎𝑎 1
| 325
1 2
𝑁𝑁𝑎𝑎 = 𝐴𝐴𝑎𝑎 atau 𝐴𝐴𝑎𝑎 = 𝑁𝑁𝑎𝑎 .............................(15.1)
Selanjutnya, dari kalimat, ‘Sehari kemudian Netty memberikan uangnya
sejumlah
Rp.
100.000,00
kepada
Agit.
Sekarang
perbandingan uang Netty dan Agit 1:3’, kita dapat menemukan perbandingan baru, karena adanya pemberian uang dari Netty ke Agit, yaitu: 𝑁𝑁 = 𝑁𝑁𝑎𝑎 − 100.000, dimana: (𝑁𝑁𝑎𝑎 − 100.000) 1 = (𝐴𝐴𝑎𝑎 + 100.000) 3
3(𝑁𝑁𝑎𝑎 − 100.000) = 𝐴𝐴𝑎𝑎 + 100.000 3𝑁𝑁𝑎𝑎 − 𝐴𝐴𝑎𝑎 = 100.000 + 300.000
3𝑁𝑁𝑎𝑎 − 𝐴𝐴𝑎𝑎 = 400.000 …………………..…..(15.2)
Sehingga,
dengan
memerhatikan
persamaan
(15.1),
dan
mensubstitusikannya ke persamaan (15.2) akan diperoleh: 1 3𝑁𝑁𝑎𝑎 − 𝑁𝑁𝑎𝑎 = 400.000 2 5 𝑁𝑁 = 400.000 2 𝑎𝑎 400.000𝑥𝑥2 𝑁𝑁𝑎𝑎 = 5 𝑁𝑁𝑎𝑎 = 160.000
Akhirnya, kita bisa menghitung jumlah uang Netty sekarang: 𝑁𝑁 = 〱𝑎𝑎 − 100.000
𝑁𝑁 = 160.000 − 100.000 𝑁𝑁 = 60.000
Dengan kata lain, setelah memberikan uangnya sebanyak Rp. 100.000,00 kepada Agit, uang Netty sekarang tinggal Rp. 60.000, 00.
| 326
Itu penjelasan tim kami atas soal nomor 4, Kawan-kawan,”pungkas Bily. “Tampak semakin ringan saja aku memahami persoalanpersoalan yang kau terangkan itu, Kawan!”komentar Amer sesaat setelah Bily usai menjelaskan. “Sombong,
kau
Kawan.
Tak
boleh
itu!
Segeralah
Istighfar!”Denan mengingatkan. “Sorry, Bro. Bercanda, Kawan. Astaghfirullah!”balas Amer. Tepuk tangan meriah mengiring undurnya Bily dan Ikram dari depan. “Selanjutnya, tim siapa?”kembali Sabiq bertanya. “Tim kami, Kawan! Aku dan
Saver,”jawab Bintang
bersemangat. “Silahkan, Kawan,”balas Sabiq. Saver dan Bintang bergerak ke depan, mereka duduk bersebelahan
di
meja
depan,
Saver
mendapat
kehormatan
mendahului memberi penjelasan tim mereka. “Ada hal menarik pada soal nomor 5 ini, Kawankawan!”Saver memulai kalimatnya. Dan terbukti, kalimat pendek itu berhasil menarik perhatian Kawan-kawannya. Ayra, menoleh serus ke arah Saver, Ikram pun begitu. Denan mendongakkan wajah ke arah depan, Bily juga. Iman menarik nafas dalam-dalam membuat punggungnya lebih condong ke belakang, Yodha pun sama. Amer sampai membuka sedikit mulutnya, Fateh pun menghentikan goresan-goresan di kertas yang sedari tadi dia lakukan. Sabiq, hanya tersenyum. Tampaknya dia tahu | 327
bahwa kawannya yang satu ini melakukan satu teknik klasik membangun pay attention kawan-kawannya. “Hal menarik apa yang bisa membuat kita semua harus fokus padamu, Kawan?”tanya Amer. Saver hanya tersenyum. Tampaknya dugaan Sabiq benar. “Memang ada, tapi mungkin tak fantastik, Kawan-Kawan. Mari kita perhatikan bersama pertanyaannya, ‘jika f adalah fungsi linier, f(1) = 2000, dan f(x+1) + 12 = f (x), maka nilai f(100) = ...’. Sepintas sederhana, namun dalam kesederhanaannya itu, kami menemukan pola. Karenanya, kami mencoba menyelesaikannya dengan 2 pendekatan,”terang Saver. Kawan-kawannya masih serius memerhatikan apa yang disampaikan oleh Saver. “Pola pertama, kami lakukan dengan menelusuri dengan kaidah fungsi liniernya dan menemukan pola, sehingga: 𝑓𝑓(1) = 2000
𝑓𝑓(𝑥𝑥 + 1) + 12 = 𝑓𝑓(𝑥𝑥) ⇔ 𝑓𝑓(𝑥𝑥 + 1) = 𝑓𝑓(𝑥𝑥) − 12
Sehingga dengan melakukan substitusi langsung ke fungsi baru itu akan diperoleh: 𝑓𝑓(𝑥𝑥 + 1) = 𝑓𝑓(𝑥𝑥) − 12
𝑓𝑓(2) = 𝑓𝑓(1) − 12 = 2000 − 12
𝑓𝑓(3) = 𝑓𝑓(2) − 12 = (2000 − 12) − 1. (12) = 2000 − 2. (12)
𝑓𝑓(4) = 𝑓𝑓(3) − 12 = �2000 − 2. (12)� − 12 = 2000 − 3(12)
Bila kita perhatikan seksama, akan terlihat pola yang berlanjut, sehingga terdapat hubungan: 𝑓𝑓(𝑥𝑥) = 2000 − (𝑥𝑥 − 1)12
| 328
Sehingga untuk x = 100, bisa ditentukan nilainya menjadi: 𝑓𝑓(100) = 2000 − (100 − 1)12 𝑓𝑓(100) = 2000 − 99𝑥𝑥12 𝑓𝑓(100) = 2000 − 1188 𝑓𝑓(100) = 812
Itu cara pertama, mengikuti fungsi dan menemkan pola. Selanjutnya adalah
cara
kedua,
yaitu
dengan
deret,
tepatnya
deret
aritmetika,”terang Saver lagi. “Nah, kalau begini sudah cukup syarat kau menarik perhatian kita semua, Kawan,”komentar Amer. Saver hanya tersenyum. “Kita mulai dari pola untuk nilai f(2), kawan-kawan: 𝑓𝑓(2) = 𝑓𝑓(1) − 12 = 2000 − 12 = 1988
𝑓𝑓(3) = 𝑓𝑓(2) − 12 = (2000 − 12) − 12 = 1976
𝑓𝑓(4) = 𝑓𝑓(3) − 13 = �2000 − 2. (12)� − 12 = 1964
Kalau kita perhatikan, maka akan terbentuk deret aritmetika dengan suku pertama 2000 sedemikian rupa, sebagai berikut:
Selanjutnya,
2000, 1988, 1976, 1964, . ..
karena
merupakan
deret
Aritmetika,
kita
bisa
menentukan bedanya (b) sebagai berikut: 𝑏𝑏 = 𝑈𝑈𝑛𝑛 − 𝑈𝑈𝑛𝑛−1
𝑏𝑏 = 1988 − 200 = 1976 − 1988 = 1964 − 1976 = −12
Sehingga dengan diketahui nilai suku pertama dan bedanya, dapat dihitung nilai suku ke-100, yaitu n = 100, f(100) sebagai berikut: 𝑈𝑈𝑛𝑛 = 𝑈𝑈1 + (𝑛𝑛 − 1)𝑏𝑏
𝑈𝑈100 = 2000 + (99)(−12)
| 329
𝑈𝑈100 = 2000 − 1188 𝑈𝑈100 = 812
Dan ternyata, baik menggunakan cara pertama maupun cara kedua, hasilnya untuk f(100) adalah sama, yaitu 812. Begitu penjelasan dari tim kami. Dan selanjutnya Bintang akan mencoba uraikan soal berikutnya, Kawan-kawan. Silahkan, Bintang!”pungkas Saver. “Plok, plok, plok, plok, plok,”terdengar tepukan meriah dari kawan-kawan Bintang. Bintang maju menerima alat tulis dari Saver. “Terima kasih, atas kesempatannya. Aku dan Saver telah mendiskusikan hal ini dan bagianku adalah pertanyaan yang menyebutkan bahwa ‘diketahui H = {k│x2 -1 < x2 + k < 2(x +1), dengan x dan k bilangan bulat}. Banyaknya himpunan bagian dari himpunan
H adalah…’.
Terhadap
persoalan
ini
kita
bisa
menyelesaikannya dengan dua cara, yaitu: Coba-coba dan Analisis Persamaan Kuadrat. Kita mulai dengan cara pertama. Tapi, terlebih dahulu bentuk persamaan di atas harus disederhanakan, sehingga: 𝑥𝑥 2 − 1 < 𝑥𝑥 2 + 𝑘𝑘 < 2(𝑥𝑥 + 1)
𝑥𝑥 2 − 1 − 𝑥𝑥 2 < 𝑘𝑘 < 2𝑥𝑥 + 2 − 𝑥𝑥 2
−1 < 𝑘𝑘 < −𝑥𝑥 2 + 2𝑥𝑥 + 2
Selanjutnya kita lakukan proses coba-coba alias trial and error, untuk beberapa nilai x, yaitu: 𝑥𝑥 = 0
−1 < 𝑘𝑘 < −02 + 2(0) + 2
⇒ −1 < 𝑘𝑘 < 2 ⇒ 𝑘𝑘 = 0,1 (𝑚𝑚𝑒𝑒𝑚𝑚𝑚𝑚𝑚𝑚𝑚𝑚ℎ𝑖𝑖) 𝑥𝑥 = 1
| 330
−1 < 𝑘𝑘 < −12 + 2(1) + 2
⇒ −1 < 𝑘𝑘 < 3 ⇒ 𝑘𝑘 = 0,1,2 (𝑚𝑚𝑒𝑒𝑚𝑚𝑚𝑚𝑚𝑚𝑚𝑚ℎ𝑖𝑖) 𝑥𝑥 = 2
−1 < 𝑘𝑘 < −22 + 2(2) + 2
⇒ −1 < 𝑘𝑘 < 3 ⇒ 𝑘𝑘 = 0,1,2 (𝑚𝑚𝑒𝑒𝑚𝑚𝑚𝑚𝑚𝑚𝑚𝑚ℎ𝑖𝑖) 𝑥𝑥 = 3
−1 < 𝑘𝑘 < −32 + 2(3) + 2
⇒ −1 < 𝑘𝑘 < −1 ⇒ 𝑘𝑘 (𝑡𝑡𝑖𝑖𝑑𝑑𝑑𝑑𝑑𝑑 𝑚𝑚𝑒𝑒𝑚𝑚𝑒𝑒𝑒𝑒𝑒𝑒ℎ𝑖𝑖)
𝑥𝑥 = 4
−1 < 𝑘𝑘 < −42 + 2(4) + 2
⇒ −1 < 𝑘𝑘 < −6 ⇒ 𝑘𝑘 (𝑡𝑡𝑖𝑖𝑑𝑑𝑑𝑑𝑑𝑑 𝑚𝑚𝑒𝑒𝑚𝑚𝑚𝑚𝑚𝑚𝑚𝑚ℎ𝑖𝑖)
Sehingga, dengan melakukan coba-coba terhadap beberapa nilai x akan terbukti bahwa untuk x = 0, x = 1, dan x = 2, nilai k dapat dipenuhi. Hal itu berarti bahwa terdapat 3 elemen bilangan bulat yang memenuhi, n(H) = 3. Selanjutnya, untuk mengetahui banyaknya himpunan bagian dari himpunan H, dapat dilakukan dengan memanfaatkan rumus: 𝐵𝐵𝑎𝑎𝑛𝑛𝑛𝑛𝑛𝑛𝑛𝑛𝑛𝑛𝑛𝑛𝑛𝑛 ℎ𝑖𝑖𝑚𝑚𝑝𝑝𝑝𝑝𝑝𝑝𝑝𝑝𝑝𝑝 𝑏𝑏𝑎𝑎𝑔𝑔𝑔𝑔𝑔𝑔𝑔𝑔 ℎ𝑖𝑖𝑚𝑚𝑝𝑝𝑝𝑝𝑝𝑝𝑝𝑝𝑝𝑝 𝐻𝐻 = 2𝑛𝑛(𝐻𝐻)
𝐵𝐵𝑎𝑎𝑛𝑛𝑦𝑦𝑎𝑎𝑘𝑘𝑛𝑛𝑦𝑦𝑎𝑎 ℎ𝑖𝑖𝑚𝑚𝑝𝑝𝑝𝑝𝑝𝑝𝑝𝑝𝑝𝑝 𝑏𝑏𝑎𝑎𝑔𝑔𝑔𝑔𝑔𝑔𝑔𝑔 ℎ𝑖𝑖𝑚𝑚𝑝𝑝𝑝𝑝𝑝𝑝𝑝𝑝𝑝𝑝 𝐻𝐻 = 23 = 8
Itu cara pertama, cara coba-coba. Menemukan kemungkinan nilainilai yang memenuhi seperti dipersyaratkan oleh persamaan,”terang Bintang. “Wah, menarik juga solusimu itu, Kawan. Cepatlah kau jelaskan cara berikutnya itu,”tiba-tiba Amer menyela dan memasang wajah sangat serius. | 331
Beberapa kawan yang tahu sifat Amer bingung. Mereka heran belum pernah Amer seserius ini sebelumnya. “Sabar kawan! Insyaallah kita akan sampai kesana,”balas Bintang tenang. “Selain dengan coba-coba, tadi kami sebutkan dapat ditentukan dengan melakukan Analisis Persamaan Kuadrat, yang berarti fokus pada bentuk persamaan kuadrat dan sifat-sifatnya. Karenanya, bila kita perhatikan bentuk sederhana dari persamaaan kuadrat itu, yaitu: −1 < 𝑘𝑘 < −𝑥𝑥 2 + 2𝑥𝑥 + 2
maka untuk mengetahui kemungkinan nilai k tergantung pada nilai x yang disubstitusikan. Selanjutnya bila kita mengingat pelajaran tentang bentuk dan sifat persamaan kuadrat, di mana salah satunya adalah memiliki Nilai Ekstrim, baik Ekstrim Maksimum maupun Ekstrim Minimum,”terang Bintang serius. Tampak wajah Sabiq senyum-senyum, Yodha juga. Wajah Fateh berseri, Ayra juga. Tapi tidak dengan Amer. Entah kenapa dia sangat serius pada session ini. “Seperti kita pahami bahwa pada bentuk persamaan kuadrat 𝑦𝑦 = 𝑎𝑎𝑥𝑥 2 + 𝑏𝑏𝑥𝑥 + 𝑐𝑐 dapat ditentukan Titik Ekstrim, Yekstrim dan Xekstrim,
yaitu titik yang nilainya membuat fungsi bernilai ekstrim, baik maksimum maupun minimum dan memenuhi persamaan: −𝑏𝑏 2𝑎𝑎 −𝐷𝐷 = 4𝑎𝑎
𝑋𝑋𝑒𝑒𝑘𝑘𝑠𝑠 =
𝑌𝑌𝑒𝑒𝑘𝑘𝑠𝑠
Dimana D merupakan Diskriminan yang didefinisikan sebagai: | 332
𝐷𝐷 = 𝑏𝑏 2 − 4𝑎𝑎𝑐𝑐
Sehingga, dari persamaan bentuk sederhana di atas, bila kita perhatikan suku −𝑥𝑥 2 + 2𝑥𝑥 + 2 terlihat bahwa nilai a = - 1, b = 2 dan
c = 2. Karenanya: 𝑋𝑋𝑒𝑒𝑘𝑘𝑠𝑠 =
−𝑏𝑏 −(2) = ⇔ 𝑋𝑋𝑒𝑒𝑘𝑘𝑠𝑠 = 1 2𝑎𝑎 2(−1)
Selanjutnya, dengan menyubstitusikan nilai Xeks = 1 ke persamaan asal akan diperoleh: −1 < 𝑘𝑘 < −12 + 2(1) + 2
−1 < 𝑘𝑘 < 3
Untuk k bilangan bulat, maka nilai yang memenuhi persamaan di atas adalah {0, 1, 2},”terang Saver lagi. “Akhirnya kita bisa menghitung banyaknya himpunan bagian dari himpunan H dengan rumus: 2𝑛𝑛
𝐵𝐵𝑎𝑎𝑛𝑛𝑛𝑛𝑛𝑛𝑛𝑛𝑛𝑛𝑛𝑛𝑛𝑛 ℎ𝑖𝑖𝑚𝑚𝑝𝑝𝑝𝑝𝑝𝑝𝑝𝑝𝑝𝑝 𝑏𝑏𝑎𝑎𝑔𝑔𝑔𝑔𝑔𝑔𝑔𝑔 ℎ𝑖𝑖𝑚𝑚𝑝𝑝𝑝𝑝𝑝𝑝𝑝𝑝𝑝𝑝 𝐻𝐻 = 2𝑛𝑛
𝐵𝐵𝑎𝑎𝑛𝑛𝑛𝑛𝑛𝑛𝑛𝑛𝑛𝑛𝑛𝑛𝑛𝑛 ℎ𝑖𝑖𝑚𝑚𝑝𝑝𝑝𝑝𝑝𝑝𝑝𝑝𝑝𝑝 𝑏𝑏𝑎𝑎𝑔𝑔𝑔𝑔𝑔𝑔𝑔𝑔 ℎ𝑖𝑖𝑚𝑚𝑝𝑝𝑝𝑝𝑝𝑝𝑝𝑝𝑝𝑝 𝐻𝐻 = 23 = 8
Kawan-kawan, terlihat bahwa baik menggunakan cara coba-coba maupun analisis persamaan kuadrat, kedua-duanya memberikan hasil yang sama, yaitu 8,”pungkas Bintang. “Akhirnya, aku paham juga. Terima kasih, Kawan!”Amer tiba-tiba bersuara cukup keras. Kawan-kawannya bingung. Bahkan kali ini pun Sabiq, Fateh, dan Yodha juga bingung! “Ada apa, Mer? Sejak tadi kulihat kau begitu serius, tidak seperti biasanya. Dalam session ini kau kehilangan lucumu,”sindir Sabiq. | 333
“Biq, sejak aku baca soal itu, sampai sebelum Saver jelaskan, aku bingung dan tak paham maksud pertanyaannya. Bagaimana aku bisa menjawabnya? Tapi, setelah dijelaskan dengan 2 cara itu, Alhamdulillah, aku paham, Kawan,”terang Amer. Sabiq, Yodha, Ikram, Fateh, dan Ayra mengangguk-anggukan kepala mereka. Kini mereka paham kenapa Amer sedari tadi begitu sangat serius, tidak seperti biasanya. “Baik, sudah 6 soal kita kerjakan. Dan kita sudah menghabiskan waktu tidak kurang dari 25 menit. Jadi rerata 4 menit kita selesaikan satu soalnya. Kita lanjutkan sampai 10 soal, setelah itu kita rehat untuk coffee break. Giliran siapa selanjutnya?”tanya Sabiq. Suasana kembali tenang, “Giliran kami, aku dan Denan,”terdengar jawaban mantap dari Fateh. “Silahkan, Kawan-kawan!”balas Sabiq. Denan melangkah ke depan lebih dulu, Fateh menyusul di belakangnya. Mereka berdua mengambil tempat duduk tepat di depan Kawan-kawannya. Kali ini, Fateh yang mula-mula memegang alat tulis. “Kawan-kawan, kami akan mencoba menjelaskan persoalan nomor 7 dan nomor 8. Aku sendiri akan fokus pada soal nomor 7 sementara Denan akan menerangkan soal nomor 8,”ujar Fateh membuka session-nya. Fateh masih fokus pada alat tulisnya. Tampaknya dia sedang menyusun kata-kata untuk disampaikan kepada kawan-kawannya. | 334
“Kami tadi berdiskusi tentang dua soal ini. Entah kenapa, kami menemukan dengan satu metoda yang sama. Kami coba berkali-kali, dan Alhamdulillah, hasilnya tetap sama. Metoda itu adalah Metoda Tabulasi alias Tabel,”terang Fateh. “Wah, metoda apa pula itu, Kawan? Baru kali ini aku dengar di Matematika ada rumus atau Metoda Tabel. Tapi, tak apalah, kau terangkan saja, aku penasaran,”potong Amer. Bukan hanya Amer. Tapi Ayra, Ikram, Saver, Yodha, Bintang, dan
Bily
pun
terheran-heran.
Bahkan
Sabiq
pun
sempat
mengernyitkan keningnya, meski tak lama setelah itu, dia kembali tersenyum. Senyuman itu. Dan senyum itu sempat terlirik oleh Amer, hingga menambah rasa penasarannya. “Baik kita mulai saja. Soal yang kami tangani, nomor 7 menyebutkan begini, ‘tiga orang A, B, dan C pinjam meminjam kelereng. Pada awalnya ketiga orang tersebut memiliki sejumlah kelereng tertentu dan selama pinjam meminjam mereka tidak melakukan penambahan kelereng selain melalui pinjam meminjam diantara ketiga orang tersebut. Pada suatu hari A meminjami sejumlah kelereng kepada B dan C sehingga jumlah kelereng B dan C masing-masing menjadi dua kali lipat jumlah kelereng sebelumnya. Hari berikutnya B meminjami sejumlah kelereng kepada A dan C sehingga jumlah kelereng A dan C masing-masing menjadi dua kali lipat jumlah kelereng sebelumnya. Hari terakhir C meminjami sejumlah kelereng kepada A dan B sehingga jumlah kelereng A dan B masing-masing menjadi dua kali lipat jumlah kelereng sebelumnya. Setelah dihitung akhirnya masing-masing | 335
memiliki 16 kelereng. Banyak kelereng A mula-mula adalah,”Fateh membaca perlahan-lahan namun jelas. Fateh mulai memainkan alat tulis di tangannya, ia mendekati papan tulis, dan ia mulai menggambar. Bukan! Tepatnya ia sedang membuat tabel. “Kawan-kawan, inilah yang kami maksud dengan Metoda Tabulasi atau Tabel itu. Dari informasi soal kita ketahui bahwa ada 3 orang dengan 4 keadaan, yaitu: Keadaan Awal, Keadaan Hari Pertama, Hari Kedua, dan Hari Ketiga. Dari informasi ini kita bisa membuat tabel 3 baris untuk keadaan orang dan 4 kolom untuk keterangan perubahan. Jika kita misalkan keadaan awal, jumlah masing-masing kelerang adalah: 𝐾𝐾𝑒𝑒𝑙𝑙𝑙𝑙𝑙𝑙𝑙𝑙𝑙𝑙𝑙𝑙 𝐴𝐴 = 𝑝𝑝
𝐾𝐾𝑒𝑒𝑙𝑙𝑙𝑙𝑙𝑙𝑙𝑙𝑙𝑙𝑙𝑙 𝐵𝐵 = 𝑞𝑞
𝐾𝐾𝑒𝑒𝑙𝑙𝑒𝑒𝑟𝑟𝑟𝑟𝑟𝑟𝑟𝑟 𝐶𝐶 = 𝑟𝑟
Maka bisa dibuat tabel, sebagai berikut: Hari 1 𝐴𝐴 = 𝑝𝑝 − 𝑞𝑞 − 𝑟𝑟 𝐵𝐵 = 2𝑞𝑞 𝑐𝑐 = 2𝑟𝑟
Hari 2 𝐴𝐴 = 2(𝑝𝑝 − 𝑞𝑞 − 𝑟𝑟) 𝐴𝐴 = 2𝑝𝑝 − 2𝑞𝑞 − 2𝑟𝑟 𝐵𝐵 = 2𝑞𝑞 − (𝑝𝑝 − 𝑞𝑞 − 𝑟𝑟) − 2𝑟𝑟 𝐵𝐵 = 2𝑞𝑞 − 𝑝𝑝 + 𝑞𝑞 + 𝑟𝑟 − 2𝑟𝑟 𝐵𝐵 = 3𝑞𝑞 − 𝑝𝑝 − 𝑟𝑟 𝐶𝐶 = 4𝑟𝑟
Hari 3 𝐴𝐴 = 2(2𝑝𝑝 − 2𝑞𝑞 − 2𝑟𝑟) 𝑨𝑨 = 𝟒𝟒𝒑𝒑 − 𝟒𝟒𝒒𝒒 − 𝟒𝟒𝒓𝒓 𝐵𝐵 = 2(3𝑞𝑞 − 𝑝𝑝 − 𝑟𝑟) 𝑩𝑩 = −𝟐𝟐𝒑𝒑 + 𝟔𝟔𝒒𝒒 − 𝟐𝟐𝒓𝒓
𝐶𝐶 = 4𝑟𝑟 − (2𝑝𝑝 − 2𝑞𝑞 − 2𝑟𝑟) − (3𝑞𝑞 − 𝑝𝑝 − 𝑟𝑟) 𝑪𝑪 = −𝒑𝒑 − 𝒒𝒒 + 𝟕𝟕𝒓𝒓
Dari tabel di atas, para hari ketiga, di mana masing-masing orang memiliki 16 kelerang, akan terdapat persamaan:
𝐴𝐴 ⇒ 4𝑝𝑝 − 4𝑞𝑞 − 4𝑟𝑟 = 16 ⇔ 𝑝𝑝 − 𝑞𝑞 − 𝑟𝑟 = 4 … … . . … . (15.3)
| 336
𝐵𝐵 ⇒ − 2𝑝𝑝 + 6𝑞𝑞 − 2𝑟𝑟 = 16 ⇔ −𝑝𝑝 + 3𝑞𝑞 − 𝑟𝑟 = 8 … … (15.4) 𝐶𝐶 ⇒ −𝑝𝑝 − 𝑞𝑞 + 7𝑟𝑟 = 16 ⇔ − 𝑝𝑝 − 𝑞𝑞 + 7𝑟𝑟 = 16 … . … (15.5)
Sehingga kita mendapatkan 3 persamaan linier dengan 3 variabel. Jika persamaan (15.3) dikurangi oleh persamaan (15.4): 𝑝𝑝 − 𝑞𝑞 − 𝑟𝑟 = 4
−𝑝𝑝 + 3𝑞𝑞 − 𝑟𝑟 = 8 –
2𝑝𝑝 − 4𝑞𝑞 = −4 ⇔ 𝑝𝑝 − 2𝑞𝑞 = −2 … … … … … . . (15.6)
Dan jika persamaan (15.3) dikali dengan 7 dan ditambahkan persamaan (15.5): 7𝑝𝑝 − 7𝑞𝑞 − 7𝑟𝑟 = 28
−𝑝𝑝 − 𝑞𝑞 + 7𝑟𝑟 = 16 +
6𝑝𝑝 − 8𝑞𝑞 = 44 ⇔ 3𝑝𝑝 − 4𝑞𝑞 = 22 … … … … … . . (15.7)
Selanjutnya, bila dua kali persamaan (15.6) dikurangi oleh persamaan (15.7), diperoleh: 2𝑝𝑝 − 4𝑞𝑞 = −4
3𝑝𝑝 − 4𝑞𝑞 = 22 – −𝑝𝑝 = −26
⇔ 𝑝𝑝 = 26
Jadi, banyak kelereng A mula-mula adalah p yaitu sebanyak 26 buah,”pungkas Fateh. Fateh undur diri, giliran Denan maju, mendekat ke papan tulis. Denan menggambar sesuatu, bukan! Denan membuat Tabel, persis seperti yang dilakukan oleh Fateh. “Baik Kawan-kawan, seperti tadi dijelaskan oleh Fateh, kami mencoba menyelesaikan persoalan nomor 8 ini, juga menggunakan bantuan tabel. Pertanyaan yang diajukan adalah sebagai berikut, ‘jika | 337
jumlah dua bilangan positif adalah 24, maka nilai terkecil dari jumlah kebalikan bilangan-bilangan tersebut adalah …’. Terhadap persoalan ini hal pertama yang kami lakukan adalah menemukan halhal yang diketahui, selanjutnya dibuatkan tabel atas hal-hal yang diketahui itu,”terang Denan datar. “Karenanya, dari frasa, dua bilangan positif, bisa dibuat pemisalan. Misalkan kedua bilangan itu adalah m dan n, sehingga: 𝑚𝑚 > 0 𝑑𝑑𝑎𝑎𝑛𝑛 𝑛𝑛 > 0
Berikutnya, frasa yang menyebutkan bahwa jumlah dua bilangan positif adalah 24, hal ini berarti bahwa: 𝑚𝑚 + 𝑛𝑛 = 24
Selanjutnya, dari frasa, jumlah kebalikan bilangan-bilangan tersebut adalah, berarti bahwa yang dimaksud adalah jumlah dari kebalikan m dan kebalikan n. Sebelumnya perlu sama-sama kita ingat bahwa makna dari kebalikan seuatu bilangan adalah sama dengan 1 dibagi oleh bilangan itu sendiri, sehingga dipahami bahwa: 1 𝑚𝑚 1 𝐾𝐾𝑒𝑒𝑏𝑏𝑏𝑏𝑏𝑏𝑏𝑏𝑏𝑏𝑏𝑏𝑏𝑏 𝑛𝑛 = 𝑛𝑛
𝐾𝐾𝑒𝑒𝑏𝑏𝑏𝑏𝑏𝑏𝑏𝑏𝑏𝑏𝑏𝑏𝑏𝑏 𝑚𝑚 =
Karenanya, makna dari frasa, jumlah kebalikan bilangan-bilangan tersebut, adalah sama dengan: 1 1 + 𝑚𝑚 𝑛𝑛 Berikutnya, frasa terakhir yaitu, nilai terkecil dari jumlah kebalikan. Sampai di sini, kami melakukan tabulasi kemungkinan-kemungkinan hasil dan mencari yang terkecil di antaranya,”terang Denan lagi. | 338
m 1
n 23
1/m 1
1/n 1/23
1/m + 1/n 24/23
2
22
½
1/22
6/11
3
21
1/3
1/21
8/21
4
20
¼
1/20
3/10
5
19
1/5
1/19
24/95
6
18
1/6
1/18
2/9
7
17
1/7
1/17
24/119
8
16
1/8
1/16
3/16
9
15
1/9
1/15
24/135
10
14
1/10
1/14
6/35
11
13
1/11
1/13
8/47
12 13
12 11
1/12 1/13
1/12 1/11
1/6 8/47
14
10
1/14
1/10
6/35
15
9
1/15
1/9
24/135
16
8
1/16
1/8
3/16
17
7
1/17
1/7
24/119
18
6
1/18
1/6
2/9
19
5
1/19
1/5
24/95
20
4
1/20
¼
3/10
21
3
1/21
1/3
8/21
Perbandingan 1 23 24 144 = ; = 6 138 23 138 1 6 6 36 = ; = 6 66 11 66 1 21 8 48 = ; = 6 126 21 126 1 5 3 9 = ; = 6 30 10 30 1 95 24 144 = ; = 6 570 95 570 1 9 2 12 = ; = 6 54 9 54 1 119 24 144 = ; = 6 714 119 714 1 16 3 18 = ; = 6 96 16 96 1 135 24 144 = ; = 6 810 135 810 1 35 6 36 = ; = 6 210 35 210 1 47 8 48 = ; = 6 282 47 282 Terkecil 1 47 8 48 = ; = 6 282 47 282 1 35 6 36 = ; = 6 210 35 210 1 135 24 144 = ; = 6 810 135 810 1 16 3 18 = ; = 6 96 16 96 1 119 24 144 = ; = 6 714 119 714 1 9 2 12 = ; = 6 54 9 54 1 95 24 144 = ; = 6 570 95 570 1 5 3 9 = ; = 6 30 10 30 1 23 24 144 = ; = 6 138 23 138
| 339
22
2
1/22
½
6/11
23
1
1/23
1/1
24/23
1 23 24 144 = ; = 6 138 23 138 1 23 24 144 = ; = 6 138 23 138
Dari penggambaran tabel kemungkinan nilai (1/m + 1/n) di atas, terlihat bahwa nilai terkecil dari jumlah kebalikan bilangan-bilangan tersebut adalah 1/6. Dan kondisi itu terjadi saat nila x sama dengan nilai y, yaitu 12. Demikian penjelasan tim kami tentang soal nomor 7 dan nomor 8 yang diselesaikan dengan Metoda Tabulasi alias Metoda Tabel,”tutup Denan dengan senyum yang ceria. Tiba-tiba, terdengar suara sayup namun jelas memecah kegembiraan itu. Suara itu muncul dan mampu menghentikan tepukan meriah kawan-kawannya. “Maaf, Kawan-kawan. Aku senang dengan ‘temuan’ Metoda Tabulasi dan atau Metoda Tabel ini, tapi jujur aku masih menyimpan sedikit keraguan tentang apakah bisa diterapkan secara umum dalam menyelesaikan persamaan matematika?”terdengar jelas nada gugatan dari Iman. Entah kenapa sesaat setelah mendengar gugatan itu, hampir semua pasang mata menoleh ke arah dua orang, Yodha dan Sabiq. Kedua anak yang diperhatikan itu jadi salah tingkah. Mereka juga saling pandang. “Kau tahu arti sikap kami ini, Biq?”desak Amer. Sabiq memberi isyarat pada Yodha, namun balasan anggukan dan uluran telapak tangan kanan terbuka ke atas, justru mempersilahkan Sabiq untuk menjelaskan.
| 340
“Baiklah, aku coba menengahi persoalan ini. Pertama dan yang utama, tidak ada suatu rumus yang bisa diterapkan secara umum. Setiap rumusan atau persamaan ataupun metoda pastilah dibangun dengan keterbatasan-keterbatasan, Kawan-kawan,”kalimat pertama Sabiq. “Nah, bagian meracau ini yang aku suka dari kau, Kawan. Teruskan kuliahmu Sastrawan!”sela Amer lagi. Sabiq hanya tersenyum tipis. “Kedua, semua rumusan adalah pendekatan-pendekatan untuk memecahkan masalah. Yang paling penting adalah, bahwa banyak jalan menuju Makkah,”tiba-tiba keterangan Sabiq terpotong. Ada suara kembali menyela. “Bukan begitu, Biq! Banyak jalan menuju Roma!”suara Amer. “Itu dulu dan kata peribahasa, ini kataku, butuh kesepatan kita, banyak jalan menuju Makkah, Kawan! Okelah. Karenanya, banyak pula
cara
menyebutnya
menyelesaikan sebagai
masalah.
problem
Orang
solving.
zaman
modern
Kaitannya
dengan
Matematika, selain rumus baku yang dibuat dengan kondisi tertentu, dua hal lain yang sering digunakan, yaitu Grafik dan Tabel, ya seperti dijelaskan oleh Fateh dan Denan tadi,”terang Sabiq. “Mungkin bisa kau berikan contoh sederhananya selain yang tadi dijelaskan oleh Fateh dan Denan, Biq?”tanya Ikram santun. “Insyaallah bisa! Dalam pelajaran fisika misalnya, seringkali untuk menentukan jarak tempuh suatu benda yang bergerak lurus, baik bergerak lurus beraturan (GLB) atau pun bergerak lurus berubah beraturan (GLBB), digunakan grafik. Luas daerah di bawah grafik | 341
adalah sama dengan jarak tempuh benda yang bergerak dengan kecepatan dan dalam waktu tertentu,”terang Sabiq. “Konkretnya, Kawan?”tanya Ayra. “Baik. Masih ingat rumusan untuk jarak pada Gerak Lurus Beraturan, Ayra?”tanya Sabiq. “Masih. Jika benda bergerak dengan kecepatan v meter per detik dalam waktu t detik, maka jarak tempuhnya adalah: 𝑆𝑆 = 𝑉𝑉. 𝑡𝑡,
dengan S adalah jarak tempuh yang diukur dalam satuan meter,”jawab Ayra. “Tepat! Dan apakah kau masih ingat rumusan jarak dalam
Gerak Lurus Berubah Beraturan, Ikram?”tanya Sabiq lagi. “Insyaallah masih, Biq. Untuk benda bergerak berubah beraturan dengan kecepatan awal vo meter per detik, dan percepatan a meter per detik kuadrat, dalam waktu t detik, maka ia akan 1 2
memenuhi persamaan jarak: 𝑆𝑆(𝑡𝑡) = 𝑉𝑉𝑜𝑜 + 𝑎𝑎𝑡𝑡 2 ,”jawab Ikram sigap.
“Nah, kalau begitu mohon perhatikan grafik di bawah ini,
Kawan-Kawan,”pinta Sabiq. Anak laki-laki yang dipanggil Sabiq itu kemudian bergerak menuju papan tulis, dia menggambara sesuatu, dan gambaran itu seperti ini!
| 342
“Perhatikan baik-baik gambar di atas, Kawan-kawan. Tapi untuk menjelaskan, kita agak sedikit sombong, aku pakai Bahasa Inggris, ya!”sindir Sabiq. “Ah, serius, kau Kawan?”tanya Amer mulai gelisah. Kawan-kawan lainya mulai senyum-senyum. Memang di antara mereka hanya Amer yang Bahasa Inggrisnya pas-pasan. “Ya, sekalian belajarlah. Bagaimana?”pancing Sabiq “Ya, silahkan saja Kawan!”balas Bily. Sabiq memasang gaya. Ia seperti seorang Dosen Bahasa Inggris yang sedang memberi kuliah di kelas. “All friends,”kalimat pembukanya cukup meyakinkan. “We are asked to calculate the distance and displacement of the car from this graph. For 0 - 5 seconds, the displacement is equal to the area of the triangle on the left: 1 𝐴𝐴𝑟𝑟𝑒𝑒𝑒𝑒 ∆= 𝑏𝑏𝑥𝑥ℎ 2 1 𝐴𝐴𝑟𝑟𝑒𝑒𝑒𝑒 ∆= 5𝑥𝑥4 = 10 𝑚𝑚 2 For 5 - 12 seconds: The displacement is equal to the area of the rectangle: 𝐴𝐴𝑟𝑟𝑒𝑒𝑒𝑒 ⧠ = 𝑙𝑙𝑥𝑥𝑏𝑏
𝐴𝐴𝑟𝑟𝑒𝑒𝑒𝑒 ⧠ = 7𝑥𝑥4 = 28 𝑚𝑚
For 12 - 14 seconds the displacement is equal to the area of the triangle above the time axis on the right: 1 𝐴𝐴𝑟𝑟〰𝑎𝑎 ∆= 𝑏𝑏𝑥𝑥ℎ 2 1 𝐴𝐴𝑟𝑟𝑒𝑒𝑒𝑒 ∆= 2𝑥𝑥4 = 4 𝑚𝑚 2
| 343
For 14 - 15 seconds the displacement is equal to the area of the triangle below the time axis: 1 𝐴𝐴𝑟𝑟𝑒𝑒𝑒𝑒 ∆= 𝑏𝑏𝑥𝑥ℎ 2 1 𝐴𝐴𝑟𝑟𝑒𝑒𝑒𝑒 ∆= 1𝑥𝑥2 = 1 𝑚𝑚 2
Now, the total distance of the car is the sum of all of these areas: ∆x = 10 + 28 + 4 + 1 = 43 m,”pungkas Sabiq. “Subhanallah! Ternyata bukan hanya Metode Tabel, juga ada Metode Grafik,”kali ini Fateh yang justru keheranan. “Ya, Kawanku Fateh. Grafik, Tabel, Rumus, Diagram, dan Gambar Bangun sering digunakan untuk menjelaskan persoalan Matematika,”terang Sabiq. “Ini yang aku suka dari kau, Kawan. Bacaanmu beragam, jangan-jangan buku pedukunan pun kau baca, Kawan!”sindir Amer. “Kembali ke persoalan Tabel, sebenarnya, di awal saat kita bahas tentang soal-soal Appetizer, Fateh sudah menerapkannya untuk menghitung jumlah koin satuan yang setara nilai tertentu. Bukankah begitu, Teh?”tanya Sabiq. “Apa yang luput dari pengamatanmu, Kawanku. Kau benar, Biq,”balas Fateh. “Biq, kali ini aku serius, andai kau ditanya tentang jawaban soal nomor 8 ini, dan kau tak menggunakan tabel sebagai jalan penyelesaiannya, apa jawabmu?”kali ini terdengar suara yang jarang sekali bertanya, suara Yodha!
| 344
Semua tersadar. Yodha bertanya hal yang fundamental. Dalam beberapa saat terjadi saling tatap antara Sabiq dan Yodha. Tatapan yang tulus, tatapan dari pertanyaan serius menunggu jawaban serius. “Baik, Yodha. Ini jalan umum Matematika yang dikenal. Misalkan dua bilangan positif tersebut adalah p dan q, maka nilai terkecil untuk p + q = 24 apabila (1/p) + (1/q). Sedangkan untuk mendapatkan nilai terkecil haruslah (24/2) = 12, yakni untuk nilai p = 12 dan q = 12. Sehingga 𝑝𝑝 + 𝑞𝑞 = 12 ⇔
1 1 + 𝑝𝑝 𝑞𝑞
1 1 1 1 1 + = + = 𝑝𝑝 𝑞𝑞 12 12 6
Jadi, nilai terkecil dari jumlah kebalikan bilangan-bilangan tersebut adalah 1/6. Dan nilai ini sama dengan nilai hasil Tabel yang disampaikan oleh Fateh dan Denan,”pungkas Sabiq. Tampak Yodha tersenyum dan mengangguk. “Terima kasih, Kawan,”ucapan yang keluar dari mulut Yodha. Terdengar tepuk tangan meriah dari kawan-kawan Sabiq. Bukan hanya itu terdengar suara suit-suitan yang dikumandangkan oleh Amer dengan membentuk bibirnya sedemikian rupa. “Baik Kawan-kawan, tinggal 2 soal lagi dan kita rehat coffee break. Giliran tim siapa kali ini?”tanya Sabiq Semua melihat ke arah Amer dan Yodha. Amer berjalan lebih cepat dari Yodha ke depan. Ia bersemangat sekali. Yodha santai saja. Amer mengambil inisiatif memulai pembicaraan. | 345
“Tim kami menyelesaikan persoalan bilangan. Pertama soal keterbagian dengan sisa, dan kedua tentang teknik operasi bilangan sedemikian rupa. Dan untuk menjelaskan bagian pertama, soal nomor 9, akan berdiri di depan, Kawan kita Yodha!”Amer berpidato dengan gaya sangat resmi seperti presenter televisi. Tampak kawan-kawannya menikmati aksi Amer di depan. “Baik, soal nomor 9 ini tentang pembagian yang sisanya tidak pernah selesai dan membentuk pola perulangan dari angka-angka di belakang koma. Begini bunyi soalnya, ‘jika (2013/7000) ditulis dalam bentuk desimal, maka angka ke-2013 di belakang koma adalah …’. Untuk itu, bila kita kerjakan tahapan pembagiannya langkah per langkah. Akan kita peroleh seperti ini,”terang Yodha Yodha melangkah ke papan tulis dan melakukan pembagian ke bawah dengan cara biasa. Cukup lama, setidaknya hampir 2,5 menit. Dan akhirnya, Yodha menunjukkan hasil pembagiannya! “Kawan-kawan mohon perhatian serius ke papan tulis. Lihatlah seksama proses pembagian yang aku tulis, mohon koreksi bila ada yang salah atau tak tepat!”pinta Fateh. Beberapa kawannya serius mengamati proses pembagian yang tampaknya tak pernah berakhir itu. Tapi di beberapa bagian, Fateh memberi garis pembatas di antara angka-angka hasil bagi itu. “Terlihat bahwa setelah 3 angka di belakang koma, yaitu angka 287, terdapat 6 angka yang terus berulang. Dan keenam angka itu adalah 571428, berulang, berulang, dan berulang. Sehingga, kita dapat membuat beberapa simpulan sederhana, sebagai berikut:
| 346
a. Jika sisa pembaginya 1, maka angka di belakang koma adalah angka 5; b. Jika sisa pembaginya 2, maka angka di belakang koma adalah angka 7; c. Jika sisa pembaginya 3, maka angka di belakang koma adalah angka 1; d. Jika sisa pembaginya 4, maka angka di belakang koma adalah angka 4; e. Jika sisa pembaginya 1, maka angka di belakang koma adalah angka 5; f.
Jika sisa pembaginya 5, maka angka di belakang koma adalah angka 2;
g. Jika sisa pembaginya 6 atau 0, maka angka di belakang koma adalah angka 8;
| 347
Hal ini berarti setelah sisa pembagi 6 atau nol terjadi perulangan sedemikian rupa. Selanjutnya, karena terdapat 3 angka di depan yang tidak berulang, sehingga angka ke-2013 di belakang koma yang akan ditemukan harus dikoreksi posisinya menjadi: = Angka ke − (2013 – 3)
= Angka ke − 2010
Karenanya angka ke-2013 pada pembagian di atas dapat dicari dengan menentukan sisa pembagi 2010 oleh 6. Sehingga: 2010 = 6𝑘𝑘 + 𝑠𝑠𝑖𝑖𝑠𝑠𝑠𝑠 𝑘𝑘 =
2010 + 𝑠𝑠𝑖𝑖𝑠𝑠𝑠𝑠 6
𝑘𝑘 = 335 + 0
Karena sisa pembagian 2010 oleh 6 adalah Nol (0), maka berdasarkan atas konsekuensi huruf g, angka ke-2013 yang dicari adalah 8. Demikian penjelasanku, Kawan-kawan. Dan selanjutnya, Amer akan menjelaskan soal nomor 10,”pungkas Yodha. Amer maju dengan langkah pasti. Kali ini terdengar tepuk tangan yang panjang, tepuk tangan atas penjelasan Yodha dan tampilnya Amer. “Terima kasih atas apresiasinya, Kawan-kawan,”ucap Amer begitu percaya diri. Amer menerima alat tulis dari tangan Yodha, dia berdiri tegak di sebelah papan tulis menghadap kawan-kawannya. “Persoalan nomor 10, seperti tadi aku sampaikan di muka tentang teknik operasi bilangan. Soalnya mengatakan bahwa, ‘diberikan angka disusun sebagai berikut: 987654321. Berapa banyak tanda operasi penjumlahan harus disisipkan di antara | 348
angka-angka tersebut agar menghasilkan jumlah 99?’ Terhadap persoalan ini, membutuhkan sedikit kreativitas memainkan angka dan operasi utama matematika, khususnya penjumlahan,”terang Amer begitu semangat. Amer menuliskan sesuatu di papan tulis. “Perhatikan, Kawan-kawan. Hal utama yang harus diingat baik-baik bahwa soal ini mengandung aturan yang tak bisa dilanggar. Aturan itu ada pada kata “disisipkan”. Artinya kita hanya diminta menaruh tanda-tanda penjumlahan di antara kesembilan angka yang telah disusun itu. Selanjutnya, kalau kesembilan angka itu kita jumlahkan, akan dihasilkan: 9 + 8 + 7 + 6 + 5 + 4 + 3 + 2 + 1 = 45
Hal ini berarti bahwa jika hanya angka-angka satuan yang dijumlahkan tidak cukup mencapai nilai 99. Karenanya, harus ada angka bernilai puluhan dalam proses penjumlahan itu! Misalnya angka itu adalah xy yang berarti = (10x+y). Sehingga dapat dituliskan persamaan penjumlahan yang baru sebagai: (9 + 8 + 7 + 6 + 5 + 4 + 3 + 2 + 1) + (10𝑥𝑥 + 𝑦𝑦) − (𝑥𝑥 + 𝑦𝑦) = 99
Karena (9 + 8 + 7 + 6 + 5 + 4 + 3 + 2 + 1) = 45, maka: 45 + (10𝑥𝑥 + 𝑦𝑦) − (𝑥𝑥 + 𝑦𝑦) = 99
9𝑥𝑥 = 99 − 45 𝑥𝑥 =
99 − 45 9
𝑥𝑥 = 6
Karena nilai puluhan pada angka 6, berarti angka satuan yang mengiringinya adalah di sebelah kanan dari angka enam pada deretan sembilan angka 987654321, yaitu angka 5. Hal ini berarti | 349
bahwa nilai y adalah 5. Langkah berikutnya kita uji nilai penjumlahannya: 9 + 8 + 7 + 65 + 4 + 3 + 2 + 1 = 99
Ternyata hasil penjumlahan menunjukan hasil yang sama dengan yang diminta oleh persoalan. Sehingga banyaknya tanda operasi penjumlahan
yang
harus
disisipkan
di
antara
angka-angka
987654321 agar menghasilkan jumlah 99 adalah sebanyak 7 buah. Seperti itu yang bisa tim kami sajikan kepada Kawan-kawan sekalian. Semoga bermanfaat,”kalimat penutup Amer. “Plok, plok, plok, plok, plok,”terdengar tepukan meriah dalam ruangan itu. “Terima
kasih,
terima
kasih,”balas
Amer
sambil
membungkukan kepalanya ke depan gaya orang Jepang berterima kasih. “Oke, seperti tadi aku sebutkan. Kita sudah menyelesaikan 10 soal. Dan sekarang saatnya kita coffee break. Aku lihat sudah tersedia seteko besar minuman yang berasap dan harum. Kita rehat selama 15 menit. Setelah itu kita kembali dengan persoalan berikutnya,”komando Sabiq pada teman-temannya. “Alhamdulillah…!”terdengar ucapan hamdalah bersamaan dari sebelas anak-anak Kesebelsan Genk OSN. …∑πχαΩ…
“Mudah-mudahan Pisang Rebus dan Bandrek hangat tadi mampu mengembalikan kesegaran dan kejernihan berfikir kita, Kawan-kawan,”ucap Sabiq memulai second session sore itu. | 350
“Selama aku kenal Bandrek, belum pernah sesegar yang tadi kita nikmati. Aroma dan kehangatan Jahe yang dicampur dengan Gula Arennya, top-markotop, Man!”komentar Amer. “Bagaimana aku bisa membantah pendapatmu tentang makanan, Mer?”sindir Ayra. “Ha...ha...ha...,”terdengar tawa lepas sebelas kawan itu menggema, terus merambat pelan sampai jauh ke bukit dan pinggir hutan. “Tapi, ....”tiba-tiba Ikram menghentikan kalimatnya. “Tapi, apa Kram?”tanya Yodha. “Aku ragukan aroma dan kehangatan Jahe yang dicampur dengan Gula Aren yang dirasakan Amer itu memengaruhi otaknya, tampaknya langsung ke perut, Kawan-kawan,”sindir Ikram lagi. “He...he...he. Biq segera saja kau buka sesi kedua ini dan aku mau tunjukkan pengaruh Aroma dan kehangatan Jahe yang dicampur dengan Gula Aren yang diragukan Ikram itu,”balas Amer sambil tersenyum. “Baiklah kawan-kawan, kita mulai saja. Dan siapa ....,”kalimat Sabiq terhenti karena terdengarc jawaban yang tegas. “Aku, Biq. Penikmat Aroma dan kehangatan Jahe yang dicampur dengan Gula Aren itu,”jawaban Amer mendahului pertanyaan Sabiq. “Silahkan, Kawan!”kata Sabiq. Amer menuju depan, diambilnya alat tulis. Dia siap dengan aksinya. Tampak Denan, Bintang, Ikram, Yodha, Ayra, dan Saver senyum-senyum. Tampaknya pancingan mereka mengenai sasaran. | 351
“Langsung saja, Kawan. Aku selesaikan soal nomor 11. Disebutkan bahwa, ‘jika barisan berikut adalah barisan bilangan bulat positif berurutan yang dihilangkan semua bilangan kelipatan tiga: 1, 2, 4, 5, 7, 8, 10, 11, 13, 14, ..., maka suku ke-67 barisan tersebut adalah ...’. aku tegaskan, soal ini hanyalah soal logika sederhana. Sekali lagi logika sederhana kawan-kawan,”Amer memulai penjelasannya. Kawan-kawan mereka mulai serius memerhatikan apa yang ditulis dan disampaikan Amer. “Perhatikan
gambar
sederhana
ini!”kata
Amer
sambil
menggambarkan sesuatu di papan tulis.
Gambar ini adalah gambar barisan bilangan bulat positif berurutan yang dicoret semua kelipatan tiga-nya. Terlihat bahwa untuk setiap satu coretan bilangan bulat positif kelipatan 3, terdapat 2 angka bilangan bulat positif yang mendahuluinya. Atau untuk setiap 2 bilangan bulat positif yang mendahului terdapat 1 bilangan bulat positif kelipatan 3 yang dicoret.Nah, kalau kata dicoret kita ganti dengan dihilangkan, maka kedua hal tadi pun berlaku. Sehingga, dari 67 suku barisan tersebut, yaitu barisan bilangan bulat positif berurutan yang dihilangkan semua bilangan kelipatan tiga-nya, terdapat: 67 = 33 𝑏𝑏𝑖𝑖𝑙𝑙𝑙𝑙𝑙𝑙𝑙𝑙𝑙𝑙𝑙𝑙 𝑏𝑏𝑢𝑢𝑙𝑙𝑙𝑙𝑙𝑙 𝑝𝑝𝑜𝑜𝑠𝑠𝑠𝑠𝑠𝑠𝑠𝑠𝑠𝑠 𝑘𝑘𝑒𝑒𝑙𝑙𝑙𝑙𝑙𝑙𝑙𝑙𝑙𝑙𝑙𝑙𝑙𝑙 3 𝑠𝑠𝑖𝑖𝑠𝑠𝑠𝑠 1 𝑏𝑏𝑖𝑖𝑙𝑙𝑎𝑎𝑛𝑛𝑔𝑔𝑎𝑎𝑎𝑎 2
Sehingga, suku ke-67 barisan tersebut adalah: = 𝑆𝑆𝑢𝑢𝑘𝑘𝑘𝑘 𝑘𝑘𝑒𝑒 − (67 + 33)
| 352
= 𝑆𝑆𝑢𝑢𝑘𝑘𝑘𝑘 𝑘𝑘𝑒𝑒 − 100
Karenanya, suku ke-67 barisan tersebut adalah sama dengan suku ke-100 dari barisan bilangan bulat positif berurutan, yaitu: 𝑈𝑈𝑛𝑛 = 1, 2, 3, 4, 5, , … … , 𝑈𝑈100 ⇔ 𝑈𝑈100 = 100
Dan jawabannya adalah 100. Begitu yang bisa aku pikirkan setelah menikmati aroma dan kehangatan Jahe yang dicampur dengan Gula Aren tadi,”pungkas Amer. “Subhanallah! Keren kau, Mer!”puji Denan. Tepukan meriah terdengar dari semua yang ada di ruang itu. Dan tampak bahwa Amer puas, sementara Denan, Bintang, Ikram, Yodha, Ayra, dan Saver juga Sabiq tetap saja senyum-senyum. “Baik, berikutnya, siapa dan nomor berapa?”tanya Sabiq. “Izinkan aku coba, Kawan,”jawab Ayra singkat. “Silahkan, Ayra,”balas Sabiq. Ayra maju, ia mengambil alat tulis. Lagi, perempuan ini menuju papan tulis dan menggambar sesuatu di sana. “Ini persoalan nomor 12. Disebutkan bahwa ‘jika rata-rata 51 bilangan berurutan adalah 10, maka bilangan terkecil dari semua bilangan tersebut adalah...’. Soal ini menghadapkan kita dengan apa yang disebut sebagai statistika dalam Matematika. Kata kunci dalam soal
ini
adalah:
Pertama,
rata-rata
dan
kedua
bilangan
berurutan,”Ayra memulai penjelasannya. “Bila kita ingat sedikit konsep tentang rerata dalam Statistika, maka secara sederhana dirumuskan sebagai, ‘suatu bilangan tunggal yang dipergunakan untuk mewakili nilai sentral dari sebuah | 353
distribusi. Dalam bahasa sederhananya adalah jumlah nilai data dibagi dengan banyaknya data’. Sehingga dapat dituliskan sebagai: 𝑅𝑅𝑎𝑎𝑡𝑡𝑡𝑡 − 𝑟𝑟𝑎𝑎𝑡𝑡𝑡𝑡 =
data ke − 1 + data ke − 2 + . . . + data ke − n 𝑛𝑛
Di mana n adalah banyaknya data. Atau dalam bentuk simbol yang lebih umum, dituliskan menjadi: 𝑋𝑋 =
∑ 𝑓𝑓𝑋𝑋 ∑ 𝑓𝑓
Dimana 𝑋𝑋 adalah rata-rata, f adalah frekuensi dan X adalah nilai
data. Sehingga bila persoalan di atas kita selesaikan, akan terlihat bahwa: 10 =
Data ke − 1 + Data ke − 2 + . . . + Data ke − 51 51
Data ke − 1 + Data ke − 2 + . . . + Data ke − 51 = 510
Dari pengertian bahwa merupakan bilangan berurutan, maka kita bisa lakukan pemisalan sebagai berikut: Misalkan, Data ke-1
=a
Data ke-2
= (a + 1)
Data ke-3
= (a + 2)
...
= ...
Data ke-51
= (a + 50)
Sehingga, 𝑎𝑎 + (𝑎𝑎 + 1) + (𝑎𝑎 + 2) + ⋯ + (𝑎𝑎 + 50) = 510
Barisan di atas terdiri atas 2 kelompok, yaitu:
𝑎𝑎1 + 𝑎𝑎2 + 𝑎𝑎3 + ⋯ + 𝑎𝑎51 = 51𝑎𝑎 … … … … … . (15.8)
dimana 𝑎𝑎1 = 𝑎𝑎2 = 𝑎𝑎3 = 𝑎𝑎;
1 + 2 + 3 + ⋯ + 50 = 𝑆𝑆50
| 354
Dimana 𝑆𝑆50 membentuk deret Aritmetika, yang jumlahnya: 𝑆𝑆50 =
50 (1 + 50) 2
𝑆𝑆50 = 25𝑥𝑥51
𝑆𝑆50 = 1275 … … … … … … . . … . . (15.9)
Dari persamaan (15.8) dan persamaan (15.9) diketahui bahwa: 51𝑎𝑎 + 1275 = 510
51𝑎𝑎 = 510 − 1275
510 − 1275 51 765 = −15 𝑎𝑎 = − 51 𝑎𝑎 =
Karenanya, bilangan terkecil dari semua bilangan tersebut adalah bilangan pertama dari deretan itu, yaitu a = -15. Demikian yang bisa kujelaskan, Kawan-kawan,”pungkas Ayra. Kembali tepukan meriah terdengar membahana di ruangan itu. Ayra, perempuan satu-satunya dalam tim memang memiliki kemampuan yang tak diragukan. Ketelitian dan kecermatan adalah dua hal yang paling menonjol dari Ayra. “Oke, siapa selanjutnya yang akan maju?”tanya Sabiq pada kawan-kawannya. “Aku, Biq!”jawab Bily mantap. “Silahkan,
Kawan.
Nomor
berapa
yang
akan
kau
terangkan?”tanya Sabiq lagi. “Berikutnya, nomor 13! Kawanku,’jawab Bily singkat. Bily berjalan menuju papan tulis. Kali ini ia tidak mengambar, hanya mencoret beberapa bentuk hurup dan angka. Dan... | 355
“Kawan-kawan, soal nomor 13 ini soal tentang kombinatorika standar, tepatnya Peluang. Seperti kita ketahui secara teori bahwa ‘jika A adalah suatu kejadian dengan A merupakan himpunan bagian dari S, maka peluang kejadian A sama dengan: 𝑃𝑃(𝐴𝐴) =
𝑛𝑛(𝐴𝐴) 𝑛𝑛(𝑆𝑆)
Dimana: n(A) menyatakan banyaknya anggota dalam himpunan A; dan n(S) menyatakan banyaknya anggota dalam himpunan ruang contoh S,”terang Bily. Bily merapat ke papan tulis. Di kembali menuliskan beberapa hal. “Dari persoalan yang diajukan disebutkan bahwa, ‘sebuah kantong berisi 15 bola merah, 12 bola biru, dan 3 bola hijau. Diambil sebuah bola secara acak sebanyak 2 kali tanpa pengembalian.
Peluang
bola
yang
terambil
merah
pada
pengambilan pertama dan hijau pada pengambilan kedua adalah...’. Soal ini ada kaitannya dengan teori peluang, karenanya ada beberapa hal yang perlu diperhatikan. Pertama, jumlah atau banyaknya anggota dalam himpunan baik masing-masing dan keseluruhan, yaitu: (1) Banyaknya himpunan bola merah, n(M) = 15 bola; (2) Banyaknya himpunan bola biru, n(B) = 12 bola; dan (3) Banyaknya himpunan bola hijau, n(H) = 3 bola. Sehingga, banyaknya himpunan ruang bola (total), n(S) adalah (15+12+3) =30 bola. Kedua, dilakukan pengambilan 2 kali secara acak dan tanpa pengembalian. Hal ini berarti setelah pengambilan pertama, banyaknya himpunan ruang bola, n(S) berkurang 1. Ketiga, | 356
kejadian pengambilan bola pertama saling berkaitan dengan kejadian bola kedua,”terang Bily. “Oke, bagus betul teori itu, Bil. Konkretnya?”sela Amer. “Ya, konkretnya adalah: Peluang terambilnya 1 bola merah pada pengambilan pertama adalah: 𝑃𝑃(𝑀𝑀) =
𝑃𝑃(𝑀𝑀) =
𝑛𝑛(𝑀𝑀) 𝑛𝑛(𝑆𝑆) 15 30
Peluang terambilnya 1 bola hijau pada pengambilan kedua adalah: 𝑃𝑃(𝐻𝐻) =
𝑃𝑃(𝐻𝐻) =
𝑛𝑛(𝐻𝐻) 𝑛𝑛(𝑆𝑆 − 1)
3 29
Karena kedua kejadian saling berkaitan, maka Peluang bola yang terambil merah pada pengambilan pertama dan hijau pada pengambilan kedua adalah: 15 3 𝑥𝑥 30 29 3 𝑃𝑃(𝑀𝑀)1 ∩ 𝑃𝑃(𝐻𝐻)2 = 58 Seperti itu yang bisa aku sajikan kepada kawan-kawan tentang soal 𝑃𝑃(𝑀𝑀)1 ∩ 𝑃𝑃(𝐻𝐻)2 =
nomor 13 ini,”tutur Bily mengakhiri penjelasannya. Sekali lagi, apresiasi dalam bentuk tepukan tangan terdengar di ruang yang semakin terasa dingin meski tanpa AC itu. “Alhamdulillah,
semakin
cepat
rasanya
waktu
kita
menyelesaikan setiap persoalannya. Selanjutnya?”tanya Sabiq. | 357
Seseorang mengangkat tangan kanannya. “Biar aku coba kemampuanku, Kawanku Sabiq,”terdengar jawaban dari sudut sebelah kiri, Denan! “Silahkan, Kawan!”balas Sabiq Denan segera beranjak dari kursinya, dia berjalan menuju ke arah apan tulis. “Soal nomor 14 ini masih agak mirip dengan soal nomor 13. Kalau tadi bicara kombinasi kaitannya dengan peluang pengambilan sesuatu dari kelompoknya, kali ini lebih pada apa yang kita sebut sebagai Permutasi,”tutur Denan membuka pembahasan. “Disebutkan bahwa ‘lima orang anak akan naik mobil dengan kapasitas enam tempat duduk, yakni dua di depan termasuk pengemudi (Sopir), dua di tengah, dan dua di belakang. Jika hanya ada dua orang yang bisa mengemudi, banyak cara mengatur tempat duduk mereka adalah …,”kata Denan. Ia berhenti membaca. Sambil memejamkan mata, sesaat dia menarik nafas dalam-dalam, tampak dari gerakan dadanya yang tertarik ke atas. “Terhadap persoalan ini perlu dipertimbangan melakukan penggambaran atas situasinya. Pertama, mobil dengan kapasitas enam tempat duduk, yakni dua di depan termasuk pengemudi (Sopir), dua di tengah, dan dua di belakang. Kira-kira seperti ini gambarannya:
| 358
Selanjutnya, dari frasa hanya ada dua orang yang bisa mengemudi kita bisa membuat pemisalan sebagai berikut: Penumpang
: V, W, X, Y, Z
Sopir
: V dan W
Karenanya banyak cara mengatur tempat duduk mereka dapat ditentukan dengan membuat iluistrasi sebagai berikut: Kemungkinan I
⇔
Sopir : V
Terlihat bahwa, karena Sopir tetap pada tempatnya sementara masih tersedia 5 tempat duduk yang bisa ditempati oleh 4 (w, x, y, z) orang penumpang. Sehingga hal ini merupakan bentuk lain dari permutasi 4 tempat duduk dari 5 tempat duduk yang tersedia: 𝑃𝑃(5,4) =
𝑃𝑃(5,4) =
Kemungkinan II
5! (5 − 4)!
5𝑥𝑥4𝑥𝑥3𝑥𝑥2𝑥𝑥1 = 120 1 ⇔
Sopir : W
Terlihat bahwa, karena Sopir tetap pada tempatnya sementara masih tersedia 5 tempat duduk yang bisa ditempati oleh 4 (v, x, y, z) orang
| 359
penumpang. Sehingga hal ini merupakan bentuk lain dari permutasi 4 tempat duduk dari 5 tempat duduk yang tersedia: 𝑃𝑃(5,4) =
5! (5 − 4)!
5𝑥𝑥4𝑎𝑎3𝑥𝑥2𝑥𝑥1 = 120 1 Sehingga, banyak cara mengatur tempat duduk mereka adalah 𝑃𝑃(5,4) =
adalah: 2𝑥𝑥𝑃𝑃(5,4) = 2𝑥𝑥120 2𝑥𝑥𝑃𝑃(5,4) = 240
Akhirnya, banyak cara mengatur tempat duduk di dalam mobil adalah
240
cara,
Kawan-kawan!”jelas
Denan
mengakhiri
keterangannya. “Hebat, kau kawan. Sudah sangat banyak kemajuanmu dalam soal kombinatorika,”puji Amer. “Terima kasih, Kawan. Ini semua karena kebersamaan kita dalam diskusi selama ini,”jawab Denan. Applause hangat buat Denan kembali terdengar dari kawankawannya. Waktu terus bergerak. Tapi terasa lambat dalam ruangan yang terdiri atas sebelas anak-anak jenius kampung yang sedang dipenuhi gelora
semangat
menggali
dan
meningkatkan
pemahaman
Matematika itu. “Berikutnya, soal tentang Geometri ruang. Apakah kau akan mencoba, Yodha?”tanya Sabiq “Biq, izinkan aku mencoba kemampuan geometri yang aku pelajari selama kita interaksi. Dan Kawan Yodha mohon diingatkan | 360
bila aku keliru dalam bidang yang kau kuasai ini,”tiba-tiba terdengar suara Fateh menyela ucapan Sabiq. Semua mata tertuju pada Fateh. Mereka heran. Entah apa yang ada dalam perasaan Fateh, tapi ucapannya berarti bahwa ia telah sebanyak mungkin belajar tentang Geometri selama interkasi mereka selama ini. “Ba…baik, silahkan, Fateh!”balas Sabiq yang sedikit heran dengan sikap kawannya itu. Yodha hanya tersenyum dan memberi anggukan. “Kawan-kawan, aku berusaha memahami persoalan Geometri ini dengan sungguh-sungguh. Sebagai pengantar, tadi saat kita coffee break, sambil menyeruput secangkir Bandrek aku sempatkan diskusi sedikit dengan Yodha tentang soal Geometri ini. Itulah alasannya kenapa aku beranikan diri untuk menguji pemahamanku yang sedikit itu,”terang Fateh memulai penjelasannya. Yodha tetap tersenyum, sementara kawan-kawannya ada yang mengangguk dan memberi tanda jempol kepada Fateh dan Yodha. “Dalam persoalan ini disebutkan bahwa ‘jika diketahui panjang rusuk kubus ABCD.EFGH adalah 1 satuan, maka jarak titik E ke bidang datar AFH adalah … satuan’. Kawan-kawan, tampaknya sudah menjadi hukum dasar dalam geometri untuk bisa menungkan persoalan dalam bentuk gambar. Karenanya, atas informasi soal di atas, aku pun berusaha menggambarkannya. Dan gambaranku itu seperti ini,”papar Fateh sambil menuju papan tulis dan menggambar bentuk kubus sedemikian rupa dengan tambahan ilustrasi. | 361
“Mohon sama-sama kita perhatikan gambar berikut:
Diberikan informasi bahwa panjang rusuk kubus ABCD.EFGH adalah 1 satuan, sehingga: AB = BC = CD = DA = AE = EF = FG = GH = HE = 1 satuan. Garis AF = FH = HA adalah Diagonal Bidang ABEF, EFGH, dan ADEH, yang panjangnya dapat dihitung dengan rumusan Phytagoras. Perhatikan ΔABF: 𝐴𝐴𝐹𝐹 2 = 𝐴𝐴𝐵𝐵2 + 𝐵𝐵𝐹𝐹 2 ⇔ 𝐴𝐴𝐹𝐹 = √𝐴𝐴𝐵𝐵2 + 𝐵𝐵𝐹𝐹 2 𝐴𝐴𝐹𝐹 = �12 + 12 = √2 𝑠𝑠𝑎𝑎𝑡𝑡𝑡𝑡𝑡𝑡𝑡𝑡
Selanjutnya, karena I merupakan titik tengah garis FH dan garis tengah dari perpanjangan EG, maka: 1 1 𝐹𝐹𝐼𝐼 = 𝐹𝐹𝐻𝐻 = 𝐸𝐸𝐼𝐼 = √2 𝑠𝑠𝑎𝑎𝑡𝑡𝑢𝑢𝑢𝑢𝑢𝑢 2 2 Perhatikan ΔAFI dapat dihitung panjang AI: 𝐴𝐴𝐼𝐼 2 = 𝐴𝐴𝐹𝐹 2 − 𝐹𝐹𝐼𝐼 2 ⇔ 𝐴𝐴𝐼𝐼 = �𝐴𝐴𝐹𝐹 2 − 𝐹𝐹𝐼𝐼 2 1 𝐴𝐴𝐼𝐼 = �(√2)2 − ( √2)2 2
𝐴𝐴𝐼𝐼 = �2 −
1 2
| 362
3 𝐴𝐴𝐼𝐼 = � 2 𝐴𝐴𝐼𝐼 =
√6 2
Karenanya, untuk menemukan jarak titik E ke bidang datar AFH adalah sama dengan menemukan panjang jarak antara titik E ke titik J, garis EJ. Perhatikan ΔAIE: Luas ΔAIE = ½ AI x EJ … … … … . (15.10)
Karena ΔAIE adalah segitiga siku-siku di E, maka: Luas ΔAIE = ½ AE x EI … … … … (15.11)
Dari persamaan (15.10) da persamaan (15.11) ½ AI x EJ = ½ AE x EI AE x EI AI 1 1 x 2 √2 EJ = √6 2
EJ =
1 √3 𝑠𝑠𝑎𝑎𝑡𝑡𝑡𝑡𝑡𝑡𝑡𝑡 EJ = � = 3 3
Sehingga, jarak titik E ke bidang AFH adalah: √3 𝑠𝑠𝑎𝑎𝑡𝑡𝑡𝑡𝑡𝑡𝑡𝑡 3
Itu pemahamanku terhadap soal nomor 15 tentang Geometri,”ucap Fateh menutup penjelasannya. “Wah, ternyata pemahaman Geometrimu mantap, Fateh. Bukankah begitu, Yodha?”tanya Denan. Yodha yang disebut namamya hanya tersenyum tipis. | 363
“Bukan hanya sekadar mantap, namun logis dan sistematis uraian tiap tahapnya, Kawan-kawan!”timpal Yodha memuji hasil perhitungan Fateh atas jarak suatu terhadap suatu bidang yang ditanyakan dalam persoalan. Fateh hanya bisa tersipu. Dia merasa pujian Yodha terhadap kemampuan geometrinya begitu dalam. Kali ini tepuk tangan yang diterima oleh Fateh cukup panjang. Lebih lama sedikit daripada tepuk tangan sebelum-sebelumnya. “Baik, kita sudah selesaikan persoalan nomor 15. Selanjutnya, siapa yang akan menjelaskan soal nomor 16, kawan-kawan?”tanya Sabiq. Iman mengangkat tangan kanannya. Matanya lurus tertuju pada Sabiq. Tak sepatah kata pun keluar dari mulutnya. Ia langsung saja ke depan dan menerima alat tulis dari Fateh. “Insyaallah aku coba selesaikan soal nomor 16 ini, Kawankawan,”ucap Iman singkat. Iman masih berdiri beberapa meter dari papan tulis. “Silahkan, Iman. Waktu dan tempat menjadi milikmu, Kawan!”balas Sabiq. Iman masih berdiri. Dia tampaknya mencari kata yang pas. “Kawan-kawan, soal nomor 16 ini menurutku bukan sekadar soal Statistika, tapi soal kecermatan logika berfikir,”Iman memulai dengan bahasa yang sangat pelan. Suara Iman nyaris tak jelas terdengar. Tapi, entah kenapa semua kawannya serius mendengarkan.
| 364
“Disebutkan bahwa, ‘diketahui sekelompok data memiliki sifat-sifat berikut: (i) Terdiri dari 5 data bilangan bulat positip dengan rataan = 7; (ii) Median = Modus = 9. Jika jangkauan didefinisikan sebagai selisih data terbesar dengan data terkecil, maka jangkauan terbesar yang mungkin adalah…’. Sepintas memang ini sepenuhnya persoalan Statistika. Tapi, sekali lagi aku tegaskan, “Tidak hanya Statistika, tapi juga soal soal kecermatan logika berfkir”. Perhatikan ilustrasi berikut!”pinta Iman kepada kawannya untuk memerhatikan apa yang dia tulis di papan tulis. “Frasa 5 data bilangan bulat positip dengan rataan = 7, berarti jika data-data itu adalah x1, x2, x3, x4, dan x5, maka: 𝑥𝑥1 + 𝑥𝑥2 + 𝑥𝑥3 + 𝑥𝑥4 + 𝑥𝑥5 =7 5
𝑥𝑥1 + 𝑥𝑥2 + 𝑥𝑥3 + 𝑥𝑥4 + 𝑥𝑥5 = 35 … … … … … . . . (15.12)
Dari frasa ‘Median = Modus = 9’, dapat dipastikan susunan dari x1, x2, x3, x4, dan x5 yang diurutkan dari terkecil hingga terbesar memenuhi bentuk: 𝑥𝑥1 + 𝑥𝑥2 + 9 + 𝑥𝑥4 + 𝑥𝑥5 … … … … … … . . … … (15.13)
Dan dari gabungan frasa bilangan bulat positif dan jangkauan terbesar yang mungkin adalah…, maka nilai x1 sebagai nilai data terendah setelah diurutkan adalah 1. Karena jangkauan terbesar hanya mungkin bila pengurangnya adalah bilangan terkecil, karena: 𝐽𝐽𝑎𝑎𝑛𝑛𝑛𝑛𝑛𝑛𝑛𝑛𝑛𝑛𝑛𝑛𝑛𝑛 = 𝑥𝑥5 − 𝑥𝑥1
𝐽𝐽𝑎𝑎𝑛𝑛𝑛𝑛𝑛𝑛𝑛𝑛𝑛𝑛𝑛𝑛𝑛𝑛 = 𝑥𝑥5 − 1 … … … … … … … … … (15.14)
Sementara dari frasa Modus = 9 dapat dipastikan bahwa sekurangkurangnya 2 dari 5 data itu bernilai 9, karena Modus adalah nilai atau angka yang paling banyak muncul. Karenanya, dengan | 365
membuat permisalan x4 = 9 dan memperhatikan persamaan (15.12), persamaan (15.13) dan persamaan (15.14), akan diperoleh: 1 + 𝑥𝑥2 + 9 + 9 + 𝑥𝑥5 = 35 𝑥𝑥2 + 𝑥𝑥5 = 35 − 19
𝑥𝑥2 + 𝑥𝑥5 = 16 … … … … … … … … … … … … … … … . … . . (15.15)
Selanjutnya,
membuat
tabel
kemungkinan
yang
memenuhi
persamaan (15.15), sebagai berikut: X2 1 2 3 4 5 6 7 8 9
X5 15 14 13 12 11 10 9 8 7
(X5 -1 ) 15 – 1 = 14 14 – 1 = 13 13 – 1 = 12 12 – 1 = 11 11 – 1 = 10 10 – 1 = 9 9–1=8 8–1=7
Keterangan Tidak memenuhi, karena 2 Modus Jangkauan terbesar yang mungkin
Tidak memenuhi, x2 harus <x3
Dari tabulasi kemungkinan tersebut di atas terlihat bahwa kelompok 5 data dengan ciri rataan 7, Modus = Median = 9, dipenuhi oleh deretan bilangan bulat positip: 1, 2, 9, 9, 14. Dan jangkauan terbesar yang mungkin adalah sama dengan 13. Demikian keterangan tentang soal nomor 16 dan maksud dari ‘tidak hanya Statistika, tapi juga soal soal kecermatan logika berfikir’ itu, Kawan-kawan,”pungkas Iman. Kali ini Ikram, Yodha, Fateh, Saver, Bily dan Sabiq tampak mengangguk-anggukkan kepada mereka. “Selamat Kawan, kau sudah bisa menembus batas kebiasaan dan memasuki wilayah kreatif dalam matematika,”puji Sabiq.
| 366
“Biq, aku juga menemukan urutan angka seperti itu. Tap, dengan coba-coba,”tiba-tiba Bintang memotong diskusi. “Mungkin bisa kau ceritakan proses penemuan urutan angka dengan cara coba-coba itu, Kawan!”pinta Sabiq pada Bintang. “Baik. dari frasa, ‘terdiri dari 5 data bilangan bulat positip dengan rataan = 7, aku menyusun 5 angka 7 sedemikian rupa, sehingga susunan 7, 7, 7, 7, 7, memenuhi 5 data dengan rata-rata 7. Tapi, dari frasa, bahwa ‘Median = Modus = 9’, kombinasi di atas tidak mungkin, karena urutan angka yang di tengah haruslah angka 9! Karenanya, aku ubah deretannnya dengan mengambil masingmasing 1 angka dari angka 7 sebelah kiri angka 9, sehingga susunannya menjadi: 6, 6, 9, 7, 7. Susunan ini pun memenuhi syarat 5 data , rata-rata 7 dan median 9. Selanjutnya, dari frasa yang menyebut bahwa nilai Modus = 9, susunan tadi tak memenuhi. Susunan itu, sekurang-kurangnya harus menempatkan 2 angka Sembilan, sehingga susunan angkanya menjadi: 5, 5, 9, 9, 7, dan masih memenuhi ketentuan 5 data, rata-rata 7 dan median 9, modus 9,”terang Bintang bersemangat. Tampak kawan-kawannya pun mulai serius dengan cara cobacoba Bintang. Bintang melanjutkan keteragannya. “Tapi nilai diperhatikan frasa tentang jangkauan terbesar, maka susunan di atas tidak memenuhi. Karenanya, angka di kiri Sembilan, 5 dan 5, harus paling kecil nilainya, yaitu 1 dan 2. Dan angka di kanan angka 9 harus paling besar nilainya. Karenanya, pengurangan atas angka di sebelah kirn angka 9 ditambahkan ke angka di sebelah kanan 9, sehingga menjadi: | 367
𝐴𝐴𝑛𝑛𝑔𝑔𝑔𝑔𝑔𝑔 𝑑𝑑𝑖𝑖 𝐾𝐾𝑖𝑖𝑟𝑟𝑟𝑟 9 = (5 − 4) 𝑑𝑑𝑎𝑎𝑛𝑛 (5 − 3)
Dan
𝐴𝐴𝑛𝑛𝑔𝑔𝑔𝑔𝑔𝑔 𝑑𝑑𝑖𝑖 𝐾𝐾𝑖𝑖𝑟𝑟𝑟𝑟 9 = 1 𝑑𝑑𝑎𝑎𝑛𝑛 2
𝐴𝐴𝑛𝑛𝑔𝑔𝑔𝑔𝑔𝑔 𝑑𝑑𝑖𝑖 𝐾𝐾𝑎𝑎𝑛𝑛𝑛𝑛𝑛𝑛 9 = 7 + 4 + 3
𝐴𝐴𝑛𝑛𝑔𝑔𝑔𝑔𝑔𝑔 𝑑𝑑𝑖𝑖 𝐾𝐾𝑎𝑎𝑛𝑛𝑛𝑛𝑛𝑛 9 = 14
Sehingga, susunan kelima angkanya menjadi: 1, 2, 9, 9, 14. Setelah dilakukan pencocokan, ternyata susunan ini memenuhi persyaratan soal yaitu terdiri atas 5 data, nilai adalah rata-rata 7 dan mediannya sama dengan modus, yaitu 9! Bagaimana menurut kawankawan?”tanya Bintang. “Subhanallah! Sebuah upaya coba-coba yang tertata dan sangat logis, Kawan!”puji Yodha dan Ikram pada Bintang hampir bersamaan. “Aku sependapat dengan Yodha dan Ikram,”ujar Sabiq. Tepuk tangan meriah kembali membahana kali ini. Tepukan hangat dari kawan kepada kawannya, yang berhasil menyajikan yang terbaik yang bisa dia sajikan. Dan kali ini tepukan untuk 2 sahabat mereka Iman dan Bintang. “Giliran siapakah yang akan maju berikutnya, Kawankawan?”tany Sabiq “Aku akan mecobanya, Kram!”jawab Ikram singkat. “Silahkan, Kawan!”balas Sabiq. “Kawan-kawan, soal nomor 17 ini sangat punya kemiripan dengan soal peluang nomor 13 sebelumnya, tentang bola. Hanya saja kali ini untuk tiga kali pengambilan,”terang Ikram membuka persoalan yang akna dijelaskannya. | 368
“Seperti disebutkan oleh persoalan bahwa, ‘di dalam suatu keranjang terdapat 12 Apel Malang, 2 diantaranya diketahui busuk. Jika diambil 3 apel secara acak (random), maka peluang tepat satu di antaranya busuk adalah …’. Terhadap persoalan ini kita berusaha mengungkapkan kemungkinan-kemungkinannya, yaitu: Banyaknya Apel Banyaknya Apel Busuk
= 12 buah (S)
Banyaknya Apel Baik (B)
=2 = 10
Sehingga peluang tepat satu diantaranya terambil adalah busuk, kemungkinannya sebagai berikut (a) Apel busuk terambil pada pengambilan Pertama, sehingga: 2 10 9 𝑥𝑥 𝑥𝑥 12 11 10 3 𝑃𝑃(𝑆𝑆𝐵𝐵𝐵𝐵) = 22 (b) Apel busuk terambil pada pengambilan Kedua, sehingga: 𝑃𝑃(𝑆𝑆𝐵𝐵𝐵𝐵) =
10 2 9 𝑥𝑥 𝑥𝑥 12 11 10 3 𝑃𝑃(𝐵𝐵𝑆𝑆𝐵𝐵) = 22 (c) Apel busuk terambil pada pengambilan Ketiga, sehingga: 𝑃𝑃(𝐵𝐵𝑆𝑆𝐵𝐵) =
10 9 2 𝑥𝑥 𝑥𝑥 12 11 10 3 𝑃𝑃(𝐵𝐵䡄𝑆𝑆) = 22 Jadi, peluang tepat terambil 1 Apel busuk dalam pengambilan 3 Apel 𝑃𝑃(𝐵𝐵𝐵𝐵𝑆𝑆) =
adalah: �
3 3 3 3 9 + + � = 3𝑥𝑥 = 22 22 22 22 22
| 369
Seperti itu hasil yang dapat aku sajikan kepada Kawan-kawan sekalian,”jelas Ikram mengakhiri penjelasannya. “Akhirnya, Fateh pun punya teman dalam Kombinatorika! Terimas kasih, Ikram!”tiba-tiba terdengar suara khas Amer dari sudut kanan ruangan. “Plok, plok, plok, plok, plok,”terdengar tepuk tangan dari semuanya. …∑πχαΩ…
Semilir angin sore yang masuk ke dalam ruangan diskusi tak ber-AC itu terasa semakin dingin. Tapi kesebelas anak muda yang tengah membahas soal-soal OSN Kabupaten dan Kota itu seperti tak merasakannya. Mereka justru merasakan kehangatan dan suasana panas yang sering muncul dari perdebatan mereka. “Baik, kita sudah selesaiakan persoalan nomor 18, selanjutnya siapa yang akan menyelesaikan soal kesembilan belas ini?”tanya Sabiq. Tak ada yang menjawab. Suasana hening. Tiba-tiba, dari deratan kursi tengah sisi kanan ada yang mengangkat tangan. “Ver, kau akan kerjakan soal ini? Silahkan Kawan!”ujar Sabiq. “Tidak kawan. Aku ingin menyampaikan hal lain. Soal ini bukan giliran siapa-siapa, aku justru usulkan soal ini kau yang selesaikan dan jelaskan pada kita semua, Kawan. Bagaiamana, kawan-kawan?”ucap dan usul Saver. “Setuju. Seratus persen setuju!”terdengar jawaban hampir bersamaan dari kawan-kawan Sabiq di ruangan itu. | 370
“Maksudnya
aku
mengerjakan
soal
ini?”tanya
Sabiq
menegaskan. “Ya! Bukankah sejak nomor 11 tadi, kita bergiliran mengerjakan soal-soal ini dengan pola yang tanpa sadar berdasar urutan Abjad. Amer, Ayra, Bily, Denan, Fateh, Iman, dan Ikram, sudah maju dan menyelesaikan tugasnya. Nah, kalau polanya diikuti, sekarang adalah giliranmu, Sabiq!”jwab Saver. “Ternyata kita cukup awas dengan pola. Sebuah kemajuan yang berarti. Baiklah, aku akan menyelesaikan bagian tugasku karena
polanya
memang
harus
begitu,”ujar
Sabiq
sambil
meninggalkan kursinya menuju ke arah papan tulis. Sabiq menggambar dua bangun. Dia menggambar Kubus dan Tabung. Di memberi beberapa angka-angka yang sama pada kedua bangun ruang itu. “Kawan-kawan, mohon sama-sama kita perhatikan kedua gambar ini,”ujar Sabiq sambil menarik garis-garis.
“Sebelum aku bacakan soalnya, mari sama-sama kita perhatikan kedua gambar di atas! Bahwa angka-angka yang ada di dalam bangun kubus adalah sama dengan angka-angka pada bagian silinder. Dengan kata lain, posisi titik-titik pada Gambar 1 yang | 371
terdapat
pada
Kubus,
sama
dengan
posisi
titik-titik
pada
Silinder,”terang Sabiq lagi. “Gambar 2 adalah gambar Silinder yang dilihat dari sisi kanan Kubus pada Gambar 1, yaitu bidang BCFG. Sehingga, bila kita baca persoalan yang menyatakan bahwa ‘sebuah silinder tegal diletakkan di dalam kubus ABCD.EFGH dengan panjang sisi kubus 2m. Selanjutnya silinder dipancung oleh bidang miring yang melalui titik A, B, dan T, di mana T adalah titik perpotongan diagonal bidang CDHG. Volume terbesar silinder terpancung ini adalah...m3’, terlihat bahwa: (1) Titik T adalah titik tengah bidang CDHG yang juga merupakan titik tengah dari tinggi Silinder yang dipancung; (2) Karena dipancung miring melalui bidang miring ABT, maka yang terpancung adalah setengah (½) dari setengah (½) bagian volume Silinder. Dengan
kata lain menjadi
seperempat (¼) bagian Silinder. Sehingga yang tersisa adalah: 1 – ¼ = ¾ bagian Silinder; (3) Volume Silinder terbesar jika dan hanya jika tinggi Silinder sama dengan tinggi Kubus, di mana jari-jari alas dan jari-jari tutup Silinder menyentuh sisi Kubus; dengan kata lain, (4) Silinder dengan volume terbesar adalah Silinder yang menyinggung semua sisi Kubus, ”jelas Sabiq “Kapan kau mulai menghitung, Biq?”tanya Amer tak sabar. “Cool Man! Kita akan segera berhitung. Dengan syarat mengingat ketiga hal tersebut di atas,”jawab Sabiq. Sabiq kembali mencorat-coret di papan tulis. | 372
“Kawan-kawan kita berhitung sekarang! Karena panjang sisi kubus = 2 meter, maka dari angka (3) dan (4) di atas kita peroleh: 1 𝑟𝑟𝑠𝑠𝑖𝑖𝑙𝑙𝑙𝑙𝑙𝑙𝑙𝑙𝑙𝑙𝑙𝑙 = 𝑥𝑥𝑠𝑠𝑖𝑖𝑠𝑠𝑖𝑖 𝑘𝑘𝑢𝑢𝑏𝑏𝑏𝑏𝑏𝑏 2 1 𝑟𝑟𝑠𝑠𝑖𝑖𝑙𝑙𝑙𝑙𝑙𝑙𝑙𝑙𝑙𝑙𝑙𝑙 = 𝑥𝑥 2 = 1 𝑚𝑚 2
𝑉𝑉𝑜𝑜𝑙𝑙𝑙𝑙𝑙𝑙𝑙𝑙 𝑆𝑆𝑖𝑖𝑙𝑙𝑙𝑙𝑙𝑙𝑙𝑙𝑙𝑙𝑙𝑙 = 𝜋𝜋𝑟𝑟 2 𝑡𝑡
𝑉𝑉𝑜𝑜𝑙𝑙𝑙𝑙𝑙𝑙𝑙𝑙 𝑆𝑆𝑖𝑖𝑙𝑙𝑙𝑙𝑙𝑙𝑙𝑙𝑙𝑙𝑙𝑙 = 𝜋𝜋(1)2 (2) 𝑚𝑚3 𝑉𝑉𝑜𝑜𝑙𝑙𝑙𝑙𝑙𝑙𝑙𝑙 𝑆𝑆𝑖𝑖𝑙𝑙𝑙𝑙𝑙𝑙𝑙𝑙𝑙𝑙𝑙𝑙 = 2𝜋𝜋 𝑚𝑚3
Dari angka (2), karena bagian bawah Silinder yang dipancung adalah ¼ bagian dari volume Silinder, maka: 1 𝑉𝑉𝑜𝑜𝑙𝑙𝑙𝑙𝑙𝑙𝑙𝑙 𝑆𝑆𝑖𝑖𝑙𝑙𝑙𝑙𝑙𝑙𝑙𝑙𝑙𝑙𝑙𝑙 𝑇𝑇𝑒𝑒𝑟𝑟𝑟𝑟𝑟𝑟𝑟𝑟𝑟𝑟𝑟𝑟𝑟𝑟𝑟𝑟 = �1 − � 2𝜋𝜋 𝑚𝑚3 4 3 𝑉𝑉𝑜𝑜𝑙𝑙𝑙𝑙𝑙𝑙𝑙𝑙 𝑆𝑆𝑖𝑖𝑙𝑙𝑙𝑙𝑙𝑙𝑙𝑙𝑙𝑙𝑙𝑙 𝑇𝑇𝑒𝑒𝑟𝑟𝑟𝑟𝑟𝑟𝑟𝑟𝑟𝑟𝑟𝑟𝑟𝑟𝑟𝑟 = � � 2𝜋𝜋 𝑚𝑚3 4 2 𝑉𝑉𝑜𝑜𝑙𝑙𝑙𝑙𝑙𝑙𝑙𝑙 𝑆𝑆𝑖𝑖𝑙𝑙𝑙𝑙𝑙𝑙𝑙𝑙𝑙𝑙𝑙𝑙 𝑇𝑇𝑒𝑒𝑟𝑟𝑟𝑟𝑟𝑟𝑟𝑟𝑟𝑟𝑢𝑢𝑛𝑛æ = 𝜋𝜋 𝑚𝑚3 3 Nah, seperti itu hasil hitung-hitunganku, Kawan-kawan. Dan aku sudah menyelesaikan giliranku. Siapa berikutnya?”pungkas Sabiq menutup keterangannya. Tak terdengar jawaban suara, selain tepuk tangan meriah untuk beberapa saat. Memang, mendaulat Sabiq menjadi pimpinan Kesebelesan
Genk
OSN
ini
bagi
kawan-kawannya
dengan
pertimbangan matang. Meskipun sesungguhnya kemampuannya ‘biasa-biasa saja’. Tapi, luas dan beragamnya bacaan Sabiq menjadi hal lain yang disadari oleh kawan-kawannya. Suara tepukan dan suara diskusi kecil antar anak masih terdengar sampai Saver kembali mengangkat tangan kanannya. | 373
“Sekarang giliranku. Aku memilih nomor 20,”jawab Saver singkat. “Lho, kok Nomor 20? Bagaimana dengan nomor 19, Kawan?”tanya Denan. “Nomor
itu,
Yodha
yang
paling
tepat
dan
indah
menjelaskannya, Kawan-kawan,”balas Saver mantap. Saver bergerak ke depan, ia mengambil alat tulis dan mendekati papan tulis. Ia tampaknya mantap dengan pilihan soal nomor 20 itu. “Soal nomor 20 ini soal paling mendasar dalam konsep Kombinatorika, yaitu Kaidah Perkalian atau sering disebut juga sebagai Aturan Pengisian Tempat (filling slots),”jelas Saver membuka penjelasannya. “Berdasar atas Kaidah Perkalian disebutkan bahwa bila ada n tempat tersedia dengan k1 adalah banyaknya cara mengisi tempat pertama, k2 adalah bayaknya cara mengisi tempat kedua, dan seterusnya hingga kn adalah banyaknya cara mengisi tempat ke-n. Maka banyaknya cara mengisi tempat adalah: 𝐾𝐾𝑎𝑎𝑖𝑖𝑖𝑖𝑖𝑖ℎ 𝑃𝑃𝑒𝑒𝑟𝑟𝑟𝑟𝑟𝑟𝑟𝑟𝑟𝑟𝑟𝑟𝑟𝑟 = 𝑘𝑘1 𝑥𝑥𝑘𝑘2 𝑥𝑥 … 𝑥𝑥𝑘𝑘𝑛𝑛−1 𝑥𝑥𝑘𝑘𝑛𝑛
Sehingga, soal nomor 20 yang menyatakan bahwa, ‘beberapa bilangan empat angka memiliki angka-angka penyusun tak nol yang saling berbeda dan berjumlah 10. Banyaknya bilangan yang dimaksud adalah...”, dapat diselesaikan dengan Kaidah Perkalian ini, Kawan-kawan,”papar Saver lagi. “Dan hasilnya adalah sebanyak: 1𝑥𝑥2𝑥𝑥3𝑥𝑥4 = 24. Kawan-
kawan,”pungkas Saver.
| 374
“Kok, gitu aja?”tanya Amer heran. “Ya, memang begitu saja perhitungannya, Kawan!”jawab Saver. “Mungkin kau bisa jelaskan lebih detail sampai bisa terdapat 24 bilangan yang dapat disusun itu, Kawan?”Fateh menengahi. “Baik, memang itu pula maksudku, Kawan,”balas Saver sedikit tersenyum. Tampak Amer masih bingung dengan senyum Saver itu. “Baiklah, Kawan-kawan. Kita akan lihat bagaimana muncul angka 24 itu dengan bukti, bukti melalui pembuatan tabel bilanganbilangan penyusunnya,”terang Saver lagi. “Dari frasa bilangan empat angka ...dan berjumlah 10, kita dapat menyusun kemungkinan, yaitu: 9100, 8110, 7210, 6211, 5320, dan 4321 dan variasinya,”terang Saver. “Tapi, dari frasa memiliki angka-angka penyusun tak nol, kita bisa menghilangkan angka-angka 9100, 8110, 7210, 5320, sehingga tersisa angka 6211 dan 4321 dan variasinya,”terang Saver lagi. “Tunggu dulu Kawan! Apa maksudmu dengan kata variasinya itu?”sela Iman “Ya, variasi dari 6211 dan 4321! Misalnya 2116 atau 3142,”jelas Saver. “Ooh begitu. Oke!”kata Iman. “Baik, aku lanjutkan. Tapi, dengan adanya frasa yang menyebut bahwa yang saling berbeda, sehingga dari keempat angka penyusun bilangan empat angka itu, tidak boleh ada yang sama. Karenanya, kita bisa mengeluarkan 6211! Dan tinggallah 4321 dan | 375
variasinya. Nah, bilangan empat angka 4321 dan variasinya itulah yang harus kita buktikan berjumlah 24, Kawan-kawan,”terang Saver lagi. Suasana jadi hening, karena tiba-tiba semua kawan Saver menjadi begitu serius mendengarkan penjelasannya. “Untuk membuktikannya, kita akan membuat tabel. Dan tabel itu adalah, seperti ini bentuknya!”kata Saver sambil menuliskan sesuatu di papan tulis. 4321 4312 4231 4213 4132 4123 3421 3412 3241 3214 3142 3124 2431 2413 2341 2314 2143 2134 1432 1423 1342 1324 1243 1234 Semua pasang mata yang hadir tertuju ke tabel yang dibuat oleh Saver. Tabel yang berisi bilangan empat angka yang masingmasing angkanya berbeda dan jumlahnya adalah 10. “Kawan-kawan, dari tabel di atas terlihat bahwa terdapat 24 kemungkinan penyusunan letak 4 angka berbeda membentuk satu bilangan yang terdiri atas 4 angka. Sehingga, baik perhitung dengan Kaidah Perkalian maupun Tabel adalah sama, yaitu 24!”jelas Saver mengakhiri penjelasannya. “Subhanallah! Ini salah satu bukti bahwa Metoda Tabulasi atau Metode Tabel yang diajukan oleh Fateh merupakan sebelumnya, merupakan salah satu jalan penyelesaian masalah, Kawankawan,”Sabiq menyela dengan penjelasan. Tiba-tiba semua kawannya mengarahkan pandangan ke arah Fateh dan Denan. “Baiklah, sebuah jalan yang indah, apresiasi kita untuk Saver!”kata Sabiq lagi. | 376
Terdengar tepuk tangan meriah kembali memecah keheningan desa di pegunungan yang semakin dingin udara sekitarnya itu. “Dan yang terakhir untuk Bagian A ini giliran Yodha. Silahkan kawanku!”pinta Sabiq pada Yodha. Yodha berjalan santai. Ia sudah siap dengan alat tulis di tangannya. Sejak terjadi perubahan nomor, ia yang tadinya memersiapkan diri untuk nomor 19 harus mengerjakan nomor 20 juga. Untungnya soal Geometri, hingga dia tak perlu bersusah payah. “Soal Nomor 19 ini murni soal geometri bidang sisi banyak dan penyusunnya. Lebih sering disebut sebagai polygon. Dalam hal ini
polygon
berbentuk
segi
delapan
beraturan
dan
penyusunnya,”Yodha memulai penjelasannya. Yodha kembali melukis beberapa garis dan tanda dalam polygon yang sudah buatnya. “Mohon perhatikan gambar ini, Kawan-kawan! Seperti disebutkan oleh soal bahwa ‘jika gambar di bawah adalah segi delapan beraturan, maka perbandingan luas antara derah yang diarsir dan luas segi delapan beraturan adalah...’. Karenanya, mutlak bagi kita untuk mengetahui luas bidang arsiran dan luas segi delapan beraturan itu sendiri,”terang Yodha lagi.
| 377
“Dari gambar ini terlihat bahwa polygon segi delapan beraturan terdiri atas: 4 buah segitiga (ST) sama kaki pada bagian tepi, 4 persegi panjang (PP) juga di bagian tepi, dan 1 bujursangkar (BS) di tengah-tengahnya. Sementara daerah yang diarsir pada bagian segi delapan beraturan itu adalah luasan yang terdiri atas: 1 persegi panjang (PP) dan 2 segitiga (ST) sama kaki,”papar Yodha. “Jika kita lakukan pemecahan atas polygon PQRSTUVW, dan memerhatikan ∆PQX, dimana PQ = a dan QX = XP = b, maka: 𝑄𝑄𝑋𝑋 2 + 𝑋𝑋𝑃𝑃2 = 𝑃𝑃𝑄𝑄2
𝑏𝑏 2 + 𝑏𝑏 2 = 𝑎𝑎2
2𝑏𝑏 2 = 𝑎𝑎2
𝑏𝑏 =
Sehingga,
dan,
serta,
𝑎𝑎√2 … … … … … … … (15.16) 2
1 𝑙𝑙𝑢𝑢𝑎𝑎𝑎𝑎 𝑠𝑠𝑒𝑒𝑔𝑔𝑔𝑔𝑔𝑔𝑔𝑔𝑔𝑔𝑔𝑔 𝑠𝑠𝑎𝑎𝑚𝑚𝑚𝑚 𝑘𝑘𝑎𝑎𝑘𝑘𝑘𝑘 = 𝑥𝑥𝑎𝑎𝑙𝑙𝑙𝑙𝑙𝑙𝑙𝑙𝑙𝑙𝑙𝑙𝑙𝑙𝑙𝑙𝑙𝑙𝑙𝑙 2 1 𝑙𝑙𝑢𝑢𝑎𝑎𝑎𝑎 𝑠𝑠𝑒𝑒𝑔𝑔𝑔𝑔𝑔𝑔𝑔𝑔𝑔𝑔𝑔𝑔 𝑠𝑠𝑎𝑎𝑚𝑚𝑚𝑚 𝑘𝑘𝑎𝑎𝑘𝑘𝑘𝑘 = 𝑏𝑏 2 2 𝑙𝑙𝑢𝑢𝑎𝑎𝑎𝑎 𝑝𝑝𝑒𝑒𝑟𝑟𝑟𝑟𝑟𝑟𝑟𝑟𝑟𝑟 𝑝𝑝𝑎𝑎𝑛𝑛𝑗𝑗𝑎𝑎𝑛𝑛𝑛𝑛 = 𝑎𝑎𝑏𝑏
𝑙𝑙𝑢𝑢𝑎𝑎𝑎𝑎 𝑏𝑏𝑢𝑢𝑗𝑗𝑗𝑗𝑗𝑗𝑗𝑗𝑗𝑗𝑗𝑗𝑗𝑗𝑗𝑗𝑗𝑗𝑗𝑗 = 𝑎𝑎2
Akhirnya, kita dapat menentukan perbandingan luas antara daerah yang
diarsir
(DIARSIR)
dan
luas
segi
delapan
beraturan
(BERATURAN), sebagai: =
2 𝐿𝐿𝑢𝑢𝑎𝑎𝑎𝑎 𝑆𝑆𝑒𝑒𝑔𝑔𝑔𝑔𝑔𝑔𝑔𝑔𝑔𝑔𝑔𝑔 + 𝑎𝑎𝑏𝑏 4 𝐿𝐿𝑢𝑢𝑎𝑎𝑎𝑎 𝑆𝑆𝑒𝑒𝑔𝑔𝑔𝑔𝑔𝑔𝑔𝑔𝑔𝑔𝑔𝑔 + 4 𝑎𝑎𝑏𝑏 + 𝐿𝐿𝑢𝑢𝑎𝑎𝑎𝑎 𝐵𝐵𝑢𝑢𝑗𝑗𝑗𝑗𝑗𝑗𝑗𝑗𝑗𝑗𝑗𝑗𝑗𝑗𝑗𝑗𝑗𝑗𝑟𝑟
| 378
1 2 2 𝑏𝑏 2 + 𝑎𝑎𝑏𝑏 ԅ𝐼𝐼 𝐴𝐴𝑅𝑅𝑆𝑆𝐼𝐼𝐼𝐼 = 𝐵𝐵𝐸𝐸𝑅𝑅𝐴𝐴𝑇𝑇𝑈𝑈𝑈𝑈𝐴𝐴𝑁𝑁 4 1 𝑏𝑏 2 + 4 𝑎𝑎𝑏𝑏 + 𝑎𝑎2 2 Dengan mengingat (15.16) bahwa
Maka,
𝑏𝑏 =
𝑎𝑎√2 2
𝑏𝑏 2 + 𝑎𝑎𝑏𝑏 𝐴𝐴𝑅𝑅𝑆𝑆𝐼𝐼𝐼𝐼 荂𝐼𝐼 = 𝐵𝐵𝐸𝐸𝑅𝑅𝐴𝐴𝑇𝑇𝑈𝑈𝑈𝑈𝐴𝐴𝑁𝑁 2 𝑏𝑏 2 + 4 𝑎𝑎𝑏𝑏 + 𝑎𝑎2 𝐴𝐴𝑅𝑅𝑆𝑆𝐼𝐼𝐼𝐼 荂𝐼𝐼 = 𝐵𝐵𝐸𝐸𝑅𝑅𝐴𝐴𝑇𝑇𝑈𝑈𝑈𝑈𝐴𝐴𝑁𝑁
𝑎𝑎√2 2 𝑎𝑎√2 ) + 𝑎𝑎 2 2 𝑎𝑎√2 2 𝑎𝑎√2 2 ( 2 ) + 4 𝑎𝑎 2 + 𝑎𝑎2 (
1 2 √2 𝑎𝑎 + 𝑎𝑎2 𝐴𝐴𝑅𝑅𝑆𝑆𝐼𝐼𝐼𝐼 致𝐼𝐼 2 2 = 𝐵𝐵𝐸𝐸𝑅𝑅𝐴𝐴𝑇𝑇𝑈𝑈𝑈𝑈𝐴𝐴𝑁𝑁 𝑎𝑎2 + 2 𝑎𝑎2 √2 + 𝑎𝑎2 1 √2 𝑎𝑎2 ( + ) 𝐼𝐼 𝑅𝑅𝑆𝑆𝐼𝐼𝐼𝐼 𝐴𝐴 2 2 = 2 𝐵𝐵𝐸𝐸𝑅𝑅𝐴𝐴𝑇𝑇𝑈𝑈𝑈𝑈𝐴𝐴𝑁𝑁 2 𝑎𝑎 (1 + √2)
1 √2 𝑎𝑎2 ( + ) 2 蒴𝐼𝐼 𝐴𝐴𝑅𝑅𝑆𝑆𝐼𝐼𝐼𝐼 2 2 𝑥𝑥 = 𝐵𝐵𝐸𝐸𝑅𝑅𝐴𝐴𝑇𝑇𝑈𝑈𝑈𝑈𝐴𝐴𝑁𝑁 2 𝑎𝑎2 (1 + √2) 2 𝑎𝑎2 (1 + √2) 𝐴𝐴𝑅𝑅𝑆𝑆𝐼𝐼𝐼𝐼 〱𝐼𝐼 = 𝐵𝐵𝐸𝐸𝑅𝑅𝐴𝐴𝑇𝑇𝑈𝑈𝑈𝑈𝐴𝐴𝑁𝑁 4 𝑎𝑎2 (1 + √2)
𝑎𝑎2 (1 + √2) 𝐴𝐴𝑅𝑅𝑆𝑆𝐼𝐼𝐼𝐼 𝐼𝐼 = = 1: 4 𝐵𝐵𝐸𝐸𝑅𝑅𝐴𝐴𝑇𝑇𝑈𝑈𝑈𝑈𝐴𝐴𝑁𝑁 4 𝑎𝑎2 (1 + √2)
Jadi, perbandingan luas antara daerah yang diarsir dan luas segi delapan beraturan adalah 1 : 4. Seperti itu yang dapat aku jelaskan untuk kita,”pungkas Yodha. | 379
Tak ada keraguan, tepuk tangan meriah membahana dalam ruang itu. Tepukan hangat nan meriah dari kawan buat kawan, khususnya Yodha yang spesialis Geometri itu. “Alhamdulillah,
kita
sudah
menyelesaikan
Bagian
A.
Sebaiknya kita istirahat sejenak, 10 menit untuk melemaskan syaraf dan menikmati sore dengan Ubi Goreng yang baru disajikan itu!”kata Sabiq sambil menunjuk ke arah meja makan. Tampak telah tersedia 3 piring besar Ubi Goreng dan Seteko Teh Manis Panas. …∑πχαΩ…
Udara sore yang semakin dingin itu mulai dirasakan oleh kesebelas anak muda yang sedang berdisuks. Untungnya, sajian antara yang disiapkan oleh penjaga penginapan itu adalah sesuatu yang bisa menghangatkan. Bahkan bukan sekadar itu, menyegarkan bagi penikmatnya. Setelah sajian Bandrek saat rehat pertama, kali ini kombinasi Ubi Goreng dan Teh Manis Panas, dan kental, adalah kombinasi yang pasti menarik bagi siapapun yang sedang berada di daerah dingin. Tampak sebelas kawan itu begitu menikmati sajian yang disiapkan. Dan semua aktivitas itu tak lepas dari sepasang mata tajam yang sejak entah kapan terus mengamati dari kejauhan. Sepasang mata yang berpadu dengan jenggot yang mulai memutih. “Baik, kawan-kawan, kita mulai lagi tahap akhir pembahasan kita tentang soal-soal OSN Kabupaten-Kota. Sekarang Bagian B tentang Isian Singkat. Aku harap kita bisa agak lebih cepat
| 380
menyelesaikannya, agar setelah makan malam kita bisa pulang!”ujar Sabiq pada kawan-kawannya. “Biq, izinkan aku mengerjakan pertama kali untuk bagian ini!”tiba-tiba Ayra mengajukan diri. Semua kawannya saling pandang. “Kenapa tidak! Silahkan Ayra!’jawab Sabiq singkat. Ayra bangkit dan berjalan menuju papan tulis. Ia tak banyak berkata. Diraihnya alat tulis. Dia menggambar sesuatu di apapun tulis. Tampaknya dia menggambar sebuah tangga dengan seseorang yang sedang berada di tengah-tengahnya. “Kawan-kawan, soal nomor 1 bagian Isian Singkat ini tentang Aljabar
sederhana
dan
logika.
Aku
berusaha
menyajikan
penyelesaiannya dengan dua cara, yaitu: Gambar dan ilustrasi serta persamaan linier,”terang Ayra. Tampak kawan-kawan Ayra tak sabar menunggu dua cara penyelesaian yang dijanjikan oleh Ayra itu. “Kawan-kawan, seperti disebutkan oleh soal bahwa, ‘Tino sedang memanjat tangga dan sekarang dia berada tepat di tengah tangga. Jika ia naik 3 anak tangga ke atas, kemudian turun 5 anak tangga, serta naik kembali 10 anak tangga, maka Tino akan sampai di puncak tangga. Banyak anak tangga yang dimiliki tangga tersebut adalah…’. Karenanya, mohon perhatikan gambar di bawah ini!”pinta Ayra sambil menunjukan sesautu di papan tulis. “Aku mencoba membuat ilustrasi sedemikian rupa, sehingga tampak bahwa posisi Tino berada di tengah. Misalkan T adalah
| 381
posisi Tino di tengah, naik berarti positif dan turun berarti negatif maka akan diperoleh: 𝑇𝑇 = +3 − 5 + 10 = 8
Dari gambaran terlihat bahwa langkah 1 terdiri atas 3 langkah ke atas, selanjutnya turun 5 langkah ke bawah untuk langkah 2, dan berikutnya langkah 3 naik 10 langkah ke atas. Hal ini berarti Tino naik 8 langkah dari posisi semula,”terang Ayra. Kawan-kawannya masih tampak serius mengamati. “Karena ke atas anak tangganya ada 8 dan Tino (T) berada di tengah-tengah, maka pastilah terdapat 8 anak tangga pula ke bawah. Sementara 1 anak tangga di tengah-tengah diinjak oleh Tino. Sehingga jumlah anak tangga adalah: � 𝐴𝐴𝑛𝑛𝑎𝑎𝑎𝑎 𝑇𝑇𝑎𝑎𝑛𝑛𝑛𝑛𝑛𝑛𝑛𝑛 = 8 (𝑎𝑎𝑡𝑡𝑎𝑎𝑎𝑎) + 8 (𝑏𝑏𝑎𝑎𝑤𝑤𝑤𝑤ℎ) + 1 (𝑑𝑑𝑖𝑖𝑖𝑖𝑖𝑖𝑖𝑖𝑖𝑖𝑖𝑖)
� 𝐴𝐴𝑛𝑛𝑎𝑎𝑎𝑎 𝑇𝑇𝑎𝑎𝑛𝑛𝑛𝑛𝑛𝑛𝑛𝑛 = 17
Itu cara pertama, dengan gambar. Selanjutnya, aku coba membuat cara kedua dengan Aljabar sederhana,”terang Ayra lagi. Ayra menuliskan beberapa baris di papan tulis. | 382
“Kawan-kawan, jika kita misalkan banyaknya anak tangga = t, dan Tino berada di tengah-tengah, maka banyaknya anak tangga adalah ganjil. Selanjutnya: 𝑇𝑇𝑎𝑎𝑛𝑛𝑛𝑛𝑛𝑛𝑛𝑛 𝑇𝑇𝑒𝑒𝑛𝑛𝑛𝑛𝑛𝑛ℎ =
𝑡𝑡 + 1 2
Sehingga, berdasarkan informasi soal dimana posisi Tino, �
𝑡𝑡 + 1 � 2
Selanjutnya, ia bergerak 3 langkah ke atas (+3), kemudian turun 5 langkah (– 5), dan ke atas 10 langkah (+10) untuk mencapai puncak (x) dapat dituliskan, sebagai: 𝑡𝑡 + 1 + 3 − 5 + 10 = 𝑡𝑡 2 𝑡𝑡 + 1 + 8 = 𝑡𝑡 2 𝑡𝑡 + 1 + 16 = 2𝑡𝑡
𝑡𝑡 = 17
Jadi, banyak anak tangga yang dimiliki tangga tersebut adalah 17 buah. Kawan-kawan, ternyata hasil dari penggambaran dan perhitungan persamaan Aljabar sederhana adalah sama, yaitu 17 anak tangga. Itu penjelasanku, Kawan-kawan,”pungkas Ayra. “Congrats, 4 menit 9 detik! Selamat Ayra!”ucap Sabiq. Terdengar tepukan meriah dari Kawan-kawan Ayra. “Biq, sebelum kau bertanya, izinkan kuselesaikan nomor 2!”terdengar suara Denan dari sisi Kanan. Sabiq tak bisa berkata-kata. Ia hanya mempersilahkan dengan bahasa tubuh. Ia ulurkan tangan kanannya dengan telapak tangan menghadap ke langit-langit. | 383
Denan berjalan cepat. Ia menerima alat tulis dari Ayra. “Soal ini menyebutkan bahwa ‘Ani mempunyai uang Rp 16.500,00. Sejumlah uang itu akan dihabiskan untuk membeli 6 buah peralatan sekolah. Ia membeli beberapa pensil dengan harga Rp 2.000,00 per pensil. Ia membeli beberapa buku dengan harga Rp 2.500,00 per buku, dan ia juga membeli beberapa kotak pensil dengan harga Rp 4.000,00 per kotak pensil. Banyak buku yang dibeli Ani adalah …’. Ini adalah bentuk persamaan linier dengan 3 variabel. Dari soal kita bisa membuat persamaan sebagai berikut!”kata Denan sambil menuliskan sesuatu di papan tulis. “Bila kita misalkan: Pensil = p; Buku = b; dan Kotak Pensil = k, maka dari frasa, Sejumlah uang itu akan dihabiskan untuk membeli 6 buah peralatan sekolah, dapat dibuat persamaan: 𝑝𝑝 + 𝑏𝑏 + 𝑘𝑘 = 6 … … … … … … . (15.17)
Dan dari frasa membeli pensil dengan harga Rp 2.000,00 per pensil, buku dengan harga Rp 2.500,00 per buku, dan kotak pensil dengan harga Rp 4.000,00 per kotak pensil, dapat dibuat persamaan: 2000𝑝𝑝 + 2500𝑏𝑏 + 4000𝑘𝑘 = 16.500
20𝑝𝑝 + 25𝑏𝑏 + 40𝑘𝑘 = 165
4𝑝𝑝 + 5𝑏𝑏 + 8𝑘𝑘 = 33 … … … … … … . . (15.18)
Dari persamaan (15.17) dan persamaan (15.18) dapat dituliskan: 4𝑝𝑝 + 5𝑏𝑏 + 8𝑘𝑘 = 33
4𝑝𝑝 + 4𝑏𝑏 + 4𝑘𝑘 = 24 –
𝑏𝑏 + 4𝑘𝑘 = 9 … … … … … … … … … (15.19)
Nah, akhirnya kita mendapatkan 1 persamaan dengan 2 variabel. Tidak ada petunjuk lain. Sehingga cara penyelesaian yang paling | 384
mungkin adalah dengan cara coba-coba atau Metoda Tabulasi alias Tabel,”ujar Denan. “Tampaknya Metoda Tabulasi atau Tabel itu telah menjadi pola yang umum, Kawan,”sindir Ikram. “Ya, aku berusaha menerapkan kemungkinan pola itu. Karenanya, akan dibuat tabel yang memenuhi persamaan (15.19), yaitu pasangan penjumlahan yang hasilnya adalah 9. Tentuk dengan memerhatikan variabel-variabel yang terlibat. Sehingga akan dihasilkan tabel sebagai berikut: b 1 2 3 4 5 6 7 8
4k 8 7 6 5 4 3 2 1
Keterangan (k>0; b>0; dan k Ɛ Bil. Asli) Memenuhi dengan k = 2 Tidak memenuhi karena k=(7/4 ) Tidak memenuhi karena k = (3/2) Tidak memenuhi karena k = (5/4) Memenuhi dengan k = 1 Tidak memenuhi karena k = (3/4) Tidak memenuhi karena k = (1/2) Tidak memenuhi karena k = (1/4)
Terlihat bahwa nilai b yang memenuhi adalah saat b = 1 dengan k = 2 atau b = 5 dengan k = 1. Namun dengan mengingat persamaan (1) bahwa 𝑝𝑝 + 𝑏𝑏 + 𝑘𝑘 = 6, maka nilai b yang memenuhi adalah untuk b
= 1 dengan k = 2. Dan pada saat itu pula, jumlah pensil yang dibeli oleh Ani adalah p = 3 buah. Jadi, banyaknya buku yang dibeli Ani adalah 1 buah saja.”pungkas Denan.
“Bagaimana caramu mengetahuinya, Nan?”terdengar suara Iman dari sebelah kiri. “Ya, kita substitusikan nilai barang ke dalam harga harus memenuhi jumlah uang Ani. Karenanya, untuk p = 3; b = 1, k =2: | 385
𝐽𝐽𝑢𝑢𝑚𝑚𝑚𝑚𝑚𝑚ℎ 𝑈𝑈𝑎𝑎𝑛𝑛𝑛𝑛 𝐴𝐴𝑛𝑛𝑖𝑖 = 2000(3) + 2500(1) + 4000(2)
𝐽𝐽𝑢𝑢𝑚𝑚𝑚𝑚𝑚𝑚ℎ 𝑈𝑈𝑎𝑎𝑛𝑛𝑛𝑛 𝐴𝐴𝑛𝑛𝑖𝑖 = 6000 + 2500 + 8000 𝐽𝐽𝑢𝑢𝑚𝑚𝑚𝑚𝑚𝑚ℎ 𝑈𝑈𝑎𝑎𝑛𝑛𝑛𝑛 𝐴𝐴𝑛𝑛𝑖𝑖 = 16.500
Dan ternyata sesuai dengan jumlah uang dimiliki oleh Ani mulamula. Itu yang bisa aku jelaskan, Kawan-kawan,”ujar Denan mengakhiri. “Hebat! Waktumu 4 menit 5 detik, Nan”seru Sabiq menyampaikan waktu Denan menyelesaikan soal nomor 2. “Berikutnya, siapa yang akan mengerjakan?”tanya Sabiq. Bily mengangkat tangannya. Ia langsung maju. Tampaknya ia tak mau ketinggalan waktu dari Ayra dan Denan. “Aku akan mencoba Biq!”kata Bily. “Silahkan, Kawanku Bily!”balas Sabiq. “Kawan-kawan, soal nomor 3 ini tentang faktor dan keterbagian. Disebutkan bahwa, ‘banyak bilangan positif n sehingga 2013 𝑛𝑛 2 −3
berupa bilangan bulat positif adalah …’. Maka, langkah yang
harus dilakukan adalah menemukan faktor-faktor dari 2013!”papar Bily memulai. “Selanjutnya, agar
2013 𝑛𝑛 2 −3
berupa bilangan bulat positif, untuk
bilangan positif n maka harus memenuhi: 𝑛𝑛2 − 3 = (𝑓𝑓𝑎𝑎𝑘𝑘𝑘𝑘𝑘𝑘𝑘𝑘 𝑑𝑑𝑎𝑎𝑟𝑟𝑟𝑟 2013)
𝑛𝑛2 = (𝑓𝑓𝑎𝑎𝑘𝑘𝑘𝑘𝑘𝑘𝑘𝑘 𝑑𝑑𝑎𝑎𝑟𝑟𝑟𝑟 2013) + 3
Selanjutnya, karena faktor dari 2013 adalah: {1, 3, 11, 33, 61, 183, 671, 2013}, maka akan diperoleh bilangan positif sebagai berikut: (𝑓𝑓𝑎𝑎𝑘𝑘𝑘𝑘𝑘𝑘𝑘𝑘 𝑑𝑑𝑎𝑎𝑟𝑟𝑟𝑟 2013) + 3 = 𝑛𝑛2
| 386
1 + 3 = 4 ⇒ 𝑛𝑛 = 2
3 + 3 = 6 ⇒ 𝑛𝑛 = √6
11 + 3 = 14 ⇒ 𝑛𝑛 = √14 33 + 3 = 36 ⇒ 㐷 = 6
61 + 3 = 64 ⇒ 𝑛𝑛 = 8
183 + 3 = 186 ⇒ 𝑛𝑛 = √186
671 + 3 = 674 ⇒ 𝑛𝑛 = √674
2013 + 3 = 2016 ⇒ 𝑛𝑛 = √2016
Sehingga, banyak bilangan positif n adalah sebanyak 8 buah, Kawan-kawan!”papar Bily. “Subhanallah! Kau selesaikan soal ini hanya dalam waktu 3 menit 55 detik, Kawan!”kata Sabiq. “Tapi, masalahnya adalah, bagaimana kita dapat dengan cepat menentukan faktor-faktor dari 2013 itu, Bil?”tiba-tiba terdengar pertanyaan dari sudut kanan, Bintang. Semua mata tertuju ke Bintang. Dan sejurus kemudian ke arah Bily. “Cukup sederhana! Kita tinggal membuat pohon pembagian sedemikian rupa. Mari sama-sama kita perhatikan gambar di papan tulis ini!”jawab Bily sambil berjalan mendekati papan tulis. Bily menuliskan angka 2103 yang sedemikian rupa, kemudian dia membuat garis-garis yang dihubungkan dengan angka-angka pada masing-masing ujung garis itu. Dan…
| 387
“Kawan-kawan, silahkan perhatikan gambar berikut! Ternyata, bilangan 2013 dapat dibagi oleh 4 pembagi utama, yaitu: a. Dibagi oleh 1 menghasilkan dirinya sendiri 2013; b. Dibagi oleh 3 menghasilkan 671; c. Dibagi oleh 11 menghasilkan 183, dan d. Dibagi oleh 33 menghasilkan 61. Sehingga fakktor-faktornya seperti tadi, yaitu: {1, 3, 11, 33, 61, 183, 671, 2013},”terang Bily menjawab pertanyaan Bintang sekaligus mengakhiri penjelasannya.
Tepukan meriah kembali dialamatkan oleh kawan pada kawannya. Dan kali ini untuk Bily. “Siapa berik…” kalimat Sabiq terpotong karena Iman telah berdiri sambil mengangkat tangan dan berjalan menuju papan tulis. “Aku, aku Biq,”kata Iman. Iman tak bicara, dia hanya menggambar sesuatu di papan tulis. Tak sampai 1 menit, dia berbalik. “Kawan-kawan,
dalam
persoalan
disebutkan
bahwa,
‘diberikan tabel bilangan berikut:
| 388
Jika diketahui bahwa jumlah masing-masing baris, kolom, dan diagonal adalah sama, maka nilai x + y adalah …’. Terhadap soal ini kita bisa langsung menghubungkan beberapa persamaan,”terang Iman. ‘Tapi,
kenapa
bisa
begitu,
Kawan?”tiba-tiba
terdengar
pertanyaan dari Amer. “Karena disebutkan bahwa ‘jumlah masing-masing baris, kolom, dan diagonal adalah sama, Mer! Karenanya, dengan memerhatikan kolom kesatu dan kolom ketiga, misalnya, maka: −7 + 2𝑦𝑦 + 𝑥𝑥 − 2 = −8 − 4 + 𝑦𝑦
−9 + 2𝑦𝑦 + 𝑥𝑥 = −12 + 𝑦𝑦
2𝑦𝑦 + 𝑥𝑥 − 𝑦𝑦 = −12 + 9 𝑦𝑦 + 𝑥𝑥 = −3
Atau dengan memerhatikan baris ketiga dan kolom kesatu, akan diperoleh persamaan: 𝑥𝑥 − 2 − 10 + 𝑦𝑦 = −7 + 2𝑦𝑦 + 𝑥𝑥 − 2 𝑥𝑥 − 2 − 𝑥𝑥 + 2 − 10 + 7 = −𝑦𝑦 + 2𝑦𝑦
𝑦𝑦 = −3
Selanjutnya dicari untu nilai x, perhatikan kolom dan baris kedua, sehingga: 𝑥𝑥 − 5 − 10 = 2𝑦𝑦 − 5 − 4
| 389
𝑥𝑥 − 5 − 10 = 2(−3) − 5 − 4
𝑥𝑥 = 0
Karenanya,
𝑦𝑦 + 𝑥𝑥 = −3 + 0 = −3
Hanya itu yang bisa aku sampaikan sebagai penjelasan atas soal nomor 4 bagian ini, Kawan-kawan,”tutup Iman. Beberapa kawan tampak geleng-geleng kagum, sementara beberapa lainnya menganguk-angguk. Yodha, Sabiq dan Ikram senyum-senyum. “Selanjutnya, siapa kawan-kawan?”tanya Sabiq. “Aku, Biq!”jawab Saver lugas. “Silahkan, Kawanku Saver!”balas Sabiq lagi. “Langsung saja, Kawan-kawan! Persoalan yang diajukan pada nomor 5 ini biasa dikenal, saat kita belajar himpunan, sebagai terapan dari Prinsip Inklusi-Eksklusi dalam Teori Himpunan,” Saver memulai penjelasannya. Kawan-kawannya mulai serius mendengar. Sabiq dan Yodha pun begitu. “Prinsip Inklusi dan Eksklusi adalah bentuk paling umum dari prinsip penambahan pada himpunan. Dan salah satu cara terbaik untuk menjelaskan tentang himpunan adalah melalui Diagram Venn,”terang Saver lagi. “Diagram Venn, Ver?”tiba-tiba Fateh menyela. “Ya, Diagram Venn. Diagram yang diperkenalkan tahun 1880 oleh John Venn, dan menunjukkan semua kemungkinan hubungan logika dan hipotesis di antara sekelompok himpunan benda atau | 390
objek. Biasanya, bentuknya terdiri atas persegi sebagai semesta pembicaraan dan dilambangkan dengan huruf S dan himpunan di dalam persegi yang dilambangkan dengan lingkaran-lingkaran dalam kotak yang saling berinteraksi sedemikian rupa,”jelas Saver. Saver menggambar sesuatu di papan tulis. “Kawan-kawan, mohon perhatikan gambar ini! Gambar pertama adalah bagaimana cara untuk menggambarkan himpunan A, sementara gambar berikutnya adalah cara untuk menggambarkan hinpunan B dalam Diagram Venn. Di mana keduanya memiliki hubungan sedemikian rupa”terang Saver.
“Dari kedua ilustrasi gambar kita dapat melihat bahwa: Misalkan S adalah suatu himpunan terhingga dengan A dan B merupakan sembarang dua himpunan bagian dari S. Maka untuk mencacah (menghitung) banyaknya unsur di dalam gabungan A dan B atau sering ditulis dengan A∪B, kita dapat melakukannya dengan mencacah banyaknya unsur himpunan A dan himpunan B−A dan menjumlahkannya,”terang Saver. “Selanjutnya perhatikan gambar ketiga berikut:
| 391
Daerah yang diarsir (berwarna hitam) merupakan daerah bersama (intersection) antara himpunan A dan himpunan B. Dengan kata lain, unsur (anggota) pada himpunan A juga merupakan unsur (anggota) pada himpunan B. Dalam istilah, sering disebut sebagai “Himpunan A beririsan dengan Himpunan B” atau “A beririsan B”, dan ditulis sebagai A∩B,”terang Saver. Saver masih berdiri di dekat papan tulis. “Pertanyaannya, bagaimana menggambarkan Himpunan A gabung dengan Himpunan B? Perhatikan kembali ketiga gambar di atas! Karena ⏐B − A⏐ = ⏐B⏐ − ⏐A∩B⏐ maka ⏐A∪B⏐ = ⏐A⏐ + ⏐B⏐ − ⏐A∩B⏐. Penulisan ⏐A∪B⏐ kadang-kadang, dan lebih sering ditulis dengan n(A∪B), dan menjelaskan tentang banyaknya unsur himpunan gabungan dari himpunan A dan himpunan B. Sehingga dapat dirumuskan bahwa banyaknya unsur dari gabungan antara himpunan A dan himpunan B: 𝑛𝑛(𝐴𝐴 ∪ 𝐵𝐵) = 𝑛𝑛(𝐴𝐴) + 𝑛𝑛(𝐵𝐵) − 𝑛𝑛(𝐴𝐴 ∩ 𝐵𝐵)
Yang kira-kira artinya, ‘ketika mencacah unsur-unsur A dan B sendiri-sendiri, unsur-unsur irisan A dan B tercacah dua kali sehingga untuk mengatasi pencacahan ganda ini, kita harus mengurangkan hasil pencacahan dari n(A) + n(B) dengan n(A∩B) sekali. Dan gambarannya seperti ini!”terang Saver. | 392
“Bagaimana konkretnya, Kawan?”terdengar pertanyaan serius dari Ikram. “Konkretnya, kita merujuk pada apa yang ditanyakan oleh soal nomor 5. Soal itu menyebutkan bahwa ‘jika himpunan A mempunyai anggota sebanyak x dan himpunan B mempunyai anggota sebanyak y, x ≤ y, maka himpunan A∪B mempunyai anggota (maksimum)
sebanyak...’. Kita bisa menjawabnya dengan rumusan tentang banyaknya anggota (unsur) himpunan gabungan, yaitu: 𝑛𝑛(𝐴𝐴 ∪ 𝐵𝐵) = 𝑛𝑛(𝐴𝐴) + 𝑛𝑛(𝐵𝐵) − 𝑛𝑛(𝐴𝐴 ∩ 𝐵𝐵)
Dari frasa ‘himpunan A∪B mempunyai anggota (maksimum)’, maka: ⟺ 𝑛𝑛(𝐴𝐴 ∩ 𝐵𝐵) = 0
Sehingga,
𝑛𝑛(𝐴𝐴 ∪ 𝐵𝐵) = 𝑥𝑥 + 𝑦𝑦 − 0
𝑛𝑛(𝐴𝐴 ∪ 𝐵𝐵) = 𝑥𝑥 + 𝑦𝑦, 〲𝑎𝑎𝑟𝑟𝑒𝑒𝑒𝑒𝑒𝑒 𝑥𝑥 ≤ 𝑦𝑦, 𝑎𝑎𝑘𝑘𝑎𝑎𝑎𝑎 𝑚𝑚𝑎𝑎𝑘𝑘𝑘𝑘𝑘𝑘𝑘𝑘𝑘𝑘𝑘𝑘 𝑥𝑥 = 𝑦𝑦
Oleh karena itu,
𝑛𝑛(𝐴𝐴 ∪ 𝐵𝐵) = 𝑦𝑦 + 𝑦𝑦 𝑛𝑛(𝐴𝐴 ∪ 𝐵𝐵) = 2𝑦𝑦
Jadi, himpunan A∪B mempunyai anggota (maksimum) sebanyak 2y, Kawan-kawan. Terima kasih atas perhatiannya,”pungkas Saver.
Terdengar apresiasi berupa applause yang meriah untuk Saver. Saver hanya tersenyum dan sedikit membungkukkan kepalanya sebagai ucapan terima kasih. Ia pun segera kembali ke kursinya. | 393
“Baik, kita sudah selesaikan 5 soal. Siapa berikutnya yang akan mencoba?”tanya Sabiq pada kawan-kawannya. “Aku, aku mau mencoba nomor 6, Kawan,”terdengar suara Fateh. “Silahkan, Fateh!”ujar Sabiq. Fateh bergerak cepat. Ia tidak menuju papan tulis, tapi langsung menghadap kawan-kawannya. Dia menarik nafas agak dalam. “Kawan-kawan, soal berikut ini masih seputar Teori Bilangan. Kali ini tentang Bilangan Prima. Disebutkan dalam persoalan bahwa, ‘semua bilangan asli n yang memenuhi sifat bahwa 6n2 + 5n – 4 adalah bilangan prima adalah…’. Yang perlu kita pahami adalah konsep dasarnya, yaitu konsep tentang bilangan prima,”terang Fateh memulai penjelasannya. “Seperti sama-sama ita ketahui bahwa bilangan prima adalah bilangan asli yang hanya mempunyai 2 faktor, yaitu 1 dan bilangan itu sendiri. Sehingga kita bisa menyelesaikan persamaan di atas dengan 2 cara, yaitu faktorisasi atau pun substitusi langsung. Kita mulai dengan substitusi langsung untuk beberapa nilai n, sehingga terhadap persamaan: 6𝑛𝑛2 + 5𝑛𝑛 − 4, untuk :
𝑛𝑛 = 1 ⇔ 6(1)2 + 5(1) − 4 = 7 ⇒ 𝒃𝒃𝒊𝒊𝒍𝒍𝒍𝒍𝒍𝒍𝒍𝒍𝒍𝒍𝒍𝒍 𝒑𝒑𝒓𝒓𝒊𝒊𝒊𝒊𝒊𝒊
𝑛𝑛 = 2 ⇔ 6(2)2 + 5(2) − 4 = 30 ⇒ 𝒃𝒃𝒖𝒖𝒌𝒌𝒌𝒌𝒌𝒌 𝑏𝑏𝑖𝑖𝑙𝑙𝑙𝑙𝑛𝑛𝑔𝑔𝑎𝑎𝑛𝑛 𝑝𝑝𝑟𝑟𝑖𝑖𝑖𝑖𝑖𝑖
𝑛𝑛 = 3 ⇔ 6(3)2 + 5(3) − 4 = 65 ⇒ 𝒃𝒃𝒖𝒖𝒌𝒌𝒌𝒌𝒌𝒌 𝑏𝑏𝑖𝑖𝑙𝑙𝑙𝑙𝑙𝑙𝑙𝑙𝑙𝑙𝑙𝑙 𝑝𝑝𝑟𝑟𝑖𝑖𝑖𝑖𝑖𝑖
Dan untuk n seterusnya hasilnya akan memiliki faktor yang lebih dari 2, dengan kata lain bukan merupakan bilangan prima. Sehingga | 394
dengan metode substitusi langsung bilangan asli n yang memenuhi adalah 1,”terang Fateh. Fateh tak langsung menjelaskan tentang metode faktorisasi. Dia berhenti sejenak, kembali menarik nafas dalam-dalam. “Selanjutnya, dengan metode faktorisasi, kita akan melihat bahwa: 6𝑛𝑛2 + 5𝑛𝑛 − 4 = 𝑃𝑃
Di mana P merupakan bilangan prima. Bila difaktorkan sisi kirinya akan diperoleh persamaan: (3𝑛𝑛 + 4)(2𝑛𝑛 − 1) = 𝑃𝑃
Sehingga berdasarkan definisi dari bilangan prima, yaitu, ‘bilangan asli yang hanya mempunyai 2 faktor, yaitu 1 dan bilangan itu sendiri’, maka dapat dipastikan bahwa perkalian kedua ruas yang difaktorkan adalah bilangan itu sendiri dan 1. Sehingga nilai n yang memenuhi dapat dicari dengan menentukan nilai masing-masing ruas =1. Karenanya: (3𝑛𝑛 + 4) = 1 ⇔ 𝑛𝑛 = −1 (𝑡𝑡𝑖𝑖𝑑𝑑𝑑𝑑𝑑𝑑 𝑚𝑚𝑒𝑒𝑚𝑚𝑒𝑒𝑛𝑛𝑢𝑢ℎ𝑖𝑖) (2𝑛𝑛 − 1) = 1 ⇔ 𝑛𝑛 = 1 (𝑚𝑚𝑒𝑒𝑚𝑚𝑚𝑚𝑚𝑚𝑚𝑚ℎ𝑖𝑖)
Jadi, bilangan asli n, yang memenuhi sifat bahwa 6𝑛𝑛2 + 5𝑛𝑛 − 4
adalah bilangan prima, adalah 1. Karenanya, baik melalui metode substitusi langsung maupun metode faktorisasi hasilnya adalah sama, yaitu
untuk
n=1,
Kawan-kawan!”terang
Fateh
menutup
penjelasannya. “Wah, semakin maju saja kau, Kawan. Ada saja cara-cara yang kau gunakan untuk menemukan penyelesaian itu,”puji Amer pada Fateh. | 395
Fateh hanya tersenyum. Dan kawan-kawan mereka yang lain saling bertepuk tangan memberi penghormatan kepada hasil kerja kawannya, Fateh! “Oke, semakin cepat saja kawan-kawan menyelesaikan persoalan yang diajukan. Siapa yang akan menjelaskan soal nomor 7?”tanya Sabiq. “Aku, Biq!”terdengar jawaban singkat dari Bintang. “Baik, silahkan ke depan, Kawan!”pinta Sabiq. Bintang berjalan cepat. Ia segera meraih alat tulis dari tangan Sabiq dan menuju ke papan tulis. “Soal nomor tujuh ini soal logika yang dikaitkan dengan deret bilangan sebagai hasil dari proses. Disebutkan bahwa ‘jika S1 = 1, S2 = S1 – 3, S3 = S2 + 5, S4 = S3 – 7, S5 = S4 + 9 ........ adalah suku-suku suatu barisan bilangan, maka S2013 = ...’. Terhadap bentuk soal seperti ini, akan lebih baik bila kita tulis ulang satu per satu sampai kelihatan polanya,”Bintang memulai paparannya. Bintang menuliskan sesuatu di papan tulis. Tampaknya dalam bentuk berurutan dan membentuk pola yang unik. “Berdasarkan informasi soal yang diberikan, bila kita buat urutannya, akan tampak sebagai berikut: 𝑆𝑆1 = (+)1
𝑆𝑆2 = 𝑆𝑆1 − 3 = (−)2
𝑆𝑆3 = 𝑆𝑆2 + 5 = −2 + 5 = (+)3
𝑆𝑆4 = 𝑆𝑆3 − 7 = 3 − 7 = (−)4
𝑆𝑆5 = 𝑆𝑆5 + 9 = −4 + 5 = (+)5
Sehingga, bila diurutkan dari S1, …, S5, akan terbentuk barisan: | 396
𝑆𝑆1 , 𝑆𝑆2 , 𝑆𝑆3 , 𝑆𝑆4 , 𝑆𝑆5 … . , … = +1, −2, +3, −4, +5, … , … ..
Tampak bahwa:
(1) Terjadi pergantian tanda positif dan negatif untuk setiap urutan bilangan: +, - , +, -, +, -, ….; (2) Nilai bilangan yang terbentuk sama dengan nilai urutan setiap sukunya: S1 =1; S2=2; S3=3; S4=4; S5=5,…; Kondisi (1) terjadi dimana tanda positif terletak pada saat posisi n=ganjil, sementara pada saat posisi n=genap, tanda bilangan adalah tanda negatif. Sementara kondisi (2) berdampak pada keadaan untuk n=2013 ⇔ Sn =S2103, =2013. Dan karena 2013 merupakan bilangan
ganjil, maka tandanya harus positif. Berdasarkan keadaan di atas, maka S2013 = 2013. Demikian penjelasanku, Kawan-kawan,”kata Bintang menutup paparannya. “Hebat! Bintang hanya memerlukan waktu 3 menit 02 detik menyelesaikan soal nomor 7. Sebuah kemajuan yang sangat berarti kawan-kawan. Kalau hitunganku tak salah, kita baru menghabiskan 28 menit 39 detik untuk 7 soal Bagian Isian Singkat ini,”tegas Sabiq. “Kalau begitu, aku segera selesaikan soal nomor berikutnya, Kawan!”terdengar suara khas Amer. Semua heran. Semua temannya melihat ke arah Amer. “Inikan soal Geometri, Mer. Kau…”tanya Iman. “Kau serius, Kawan?”timpal Ayra. “Ini kejutan besar!”komentar Bily. Kawan-kawan lainnya masih memandang Amer penasaran.
| 397
“Ya, ini memang soal tentang Geometri, dan aku serius ingin menyelesaikannya. Karena aku ingin mengejutkan bukan hanya kalian, Kawan-kawan. Tapi, diriku sendiri,”tegas Amer. Amer maju dengan mantap. Ia meraih alat tulis dari Bintang yang belum sempat duduk di kursinya. Dan sesaat kemudian, Amer membungkukan sedikit kepalanya ke arah Yodha. Tradisi a la Jepang untuk menghormati seseorang yang pantas dihormati. Sabiq hanya senyum-senyum. Yodha pun membalas bentuk penghormatan Amer. “Kawan-kawan, aku yakin kalian pasti under estimate atas kemampuanku memecahkan persoalan Geometri. Aku pun, awalnya begitu. Tapi setelah mendapat pencerahan tentang perbandingan dan kesebangunan dari Yodha, aku memberanikan diri. Dan rasanya aku bisa,”Amer memulai pembukaannya. Amer langsung menuju papan tulis. Dia menggambar segitiga sedemikian rupa. Dan tak lama ia berbalik menghadap ke arah kawan-kawannya. “Soal nomor 8 menyetakan bahwa, ‘pada ΔABC terdapat titik D pada BC sehingga BD : DC = 1:3. Titik L pada AD sehingga AL : LD = 1:4. Perbandingan luas ΔACL dan ΔBDL adalah …’. Sesuai prinsip dasar Geometri, aku menggambarkan berdasarkan informasi soal, sehingga akan tampak lukisan segitiga seperti ini:
| 398
Selanjutnya, kita akan melakukan perbandingan-perbandingan seperti disebutkan oleh soal, yaitu: Perhatikan ΔBLC! Karena BD : DC = 1 : 3, maka: Luas ΔBDL ∶ Luas ΔDLC = 1 ∶ 3
1 Luas ΔBDL = Luas ΔDLC 3
Perhatikan ΔADC! Karena AL : LD = 1 : 4, maka: Luas ΔACL ∶ Luas ΔDLC = 1 ∶ 4
1 Luas ΔACL = Luas ΔDLC 4 Sehingga,
1 Luas ΔDLC 3 Luas ΔACL =4 = 1 Luas ΔBDL Luas ΔDLC 4 3 Jadi, perbandingan luas ΔACL dan ΔBDL adalah 3 : 4, Kawan-
kawan!”pungkas Amer sambil memberi hormat kembali seperti hormat semula ke arah Yodha. “Plok, plok, plok, plok, plok, plok, plok, plok, plok,”terdengar tepuk tangan yang panjang sesaat setelah Amer menyelesaikan penjelasannya. Dan entah kenapa, mereka pun segera mengalihkan pandangan ke arah Yodha. “Apa yang dijelaskan oleh Amer, seratus persen aku setuju! Dan Amer sungguh-sungguh telah memahami bukan hanya teori kesebangunan, tetapi juga kesebandingan dalam Geometri. Selamat kawan!”puji Yodha. “Terima kasih, Suhu Yodha!”balas Amer.
| 399
Dan kalimat terakhir Amer itu membuat tawa kawankawannya meledak, memecah keheningan sore yang tampak mulai kemerah-merahan di ufuk barat. “Bukan hanya fantastis dan mendebarkan, tapi apa yang dikerjakan Amer tidak lebih dari 3 menit, Kawan-kawan!”tegas Sabiq. Kembali terdengar tepukan meriah atas apa yang telah dikerjakan oleh Amer itu. Sementara Amer tampak sedang menyeka keringat yang mengucur deras di wajahnya. “Ada apa dengan wajahmu, Kawan?”tanya Fateh heran. “Aku takut salah saat menjelaskan Geometri tadi. Aku agak nervous, Kawan,’terang Amer. “Ternyata kau bisa nervous juga, Kawan!”timpal Ikram. “Ha…ha….ha,..”terdengar tawa lepas sebelas anak muda di tengah ambang sore yang cerah itu. “Semakin sore semakin panas cepat rasanya kawan-kawan menyelesaiakn setiap soalnya. Siapa berikutnya yang akan menyelesaiakan soal nomor 9?”tanya Sabiq. “Insyaallah aku, Biq!”jawab Ikram sambil mengangkat tangannya. “Silahkan, Kawanku!”balas Sabiq. Ikram maju ke depan. Dia berjalan santai saja. Tapi, tampak jari telunjuk tangan kanannya seperti bergerak-gerak menuliskan sesuatu. Ia sampai di depan, ia mendekat ke papan tulis. “Kawan-kawan, soal nomor 10 ini soal tentang Permutasi. Hanya saja, bagian dari permutasi khusus selain Permutasi Siklis | 400
yang pernah kita bahas. Tapi, ini jenis permutasi di mana terdapat sejumlah objek yang sama. Sering disebut sebagai Permutasi Yang Memuat Beberapa Unsur Yang Sama,”papar pembuka Ikram. Suasana hening. Entah kenapa untuk bahasan kombinatorika dan geometri kesebelas kawan ini begitu serius dan bersemangat. “Sebagaimana kita ketahui bahwa secara umum Permutasi membahas tentang susunan berurutan dari semua atau sebagian elemen yang diketahui, dimana urutannya diperhatikan. Namun, dalam keadaan khusus dimana dari n unsur yang tersedia, terdapat k unsur yang sama, l unsur yang sama dan m unsur yang sama, maka banyak permutasi dari n unsur itu ditentukan dengan aturan, sebagai berikut: 𝑃𝑃 =
𝑛𝑛! 𝑘𝑘! 𝑙𝑙! 𝑚𝑚!
Sehingga, soal nomor 9 yang menyatakan bahwa, ‘suatu string terdiri dari 10 angka 0, 1, atau 2. Bobot string didefinisikan sebagai jumlah angka-angka dalam string tersebut. Sebagai contoh, string 0002002001 mempunyai bobot 5. Banyak string dengan bobot 4 adalah …’, harus dilihat dari sisi Permutasi Yang Memuat Beberapa Unsur Yang Sama itu,”terang Ikram. “Namun, sebelum menghitung banyak string bobot 4 itu, perlu diperhatikan kemungkinan-kemungkinan terbentuknya pola string bobot 4 itu, untuk selanjutnya dihitung permutasinya,”terang Ikram. “Dari 10 angka yang disusun sedemikian rupa, yang mungkin membentuk string bobot 4 adalah pola-pola angka sebagai berikut: Pertama
:
1111000000
⇔ 1+1+1+1
= 4; | 401
Kedua
:
1210000000
Ketiga
:
2200000000
⇔ 1+2+1 ⇔ 2+2
= 4; = 4;
Karenanya, untuk masing-masing pola dapat dihitung permutasinya sebagai berikut: 10! 4! 6! 1𝑥𝑥2𝑥𝑥3𝑥𝑥4𝑥𝑥5𝑥𝑥6𝑥𝑥7𝑥𝑥8𝑥𝑥910 = 210 𝑃𝑃1 = 1𝑥𝑥2𝑥𝑥3𝑥𝑥4𝑥𝑥1𝑥𝑥2𝑥𝑥3𝑥𝑥4𝑥𝑥5𝑥𝑥6 10! 𝐾𝐾𝑒𝑒𝑑𝑑𝑑𝑑𝑑𝑑 = 𝑃𝑃2 = 1! 2! 7! 1𝑥𝑥2𝑥𝑥3𝑥𝑥4𝑥𝑥5𝑥𝑥6𝑥𝑥7𝑥𝑥8𝑥𝑥9𝑥𝑥10 = 360 𝑃𝑃2 = 1𝑥𝑥1𝑥𝑥2𝑥𝑥1𝑥𝑥2𝑥𝑥3𝑥𝑥4𝑥𝑥5𝑥𝑥6𝑥𝑥7
𝑃𝑃𝑒𝑒𝑟𝑟𝑟𝑟𝑟𝑟𝑟𝑟𝑟𝑟 = 𝑃𝑃1 =
𝐾𝐾𝑒𝑒
| 402
“Alhamdulillah. Kita sudah masuk pada soal terakhir. Tinggal aku dan Yodha yang belum mendapat giliran. Tapi, sebagai penghormatan, aku minta Yodha selesaikan nomor 20 ini,”pinta Sabiq. Yodha bangkit. Ia berjalan biasa saja ke depan. Ia menghampiri papan tulis. Tak langsung bicara. Ia terus menggambar kotak-kotak di papan tulis, dengan tulisan dan kombinasi huruf-huruf sedemikian rupa. “Baik kawan-kawan. Soal terakhir ini soal yang relatif sederhana tentang Peluang. Dan menurutku, cara yang paling mudah menyelesaikannya dengan bantuan tabel, “kata Yodha membuka diskusi. “Seperti disebutkan bahwa ‘Tita memiliki tetangga baru yang memiliki 2 anak. Jika salah satu anak tetangga baru tersebut adalah perempuan, maka besar peluang anak yang lain adalah laki-laki adalah …’. Terhadap soal ini, seperti aku sebut, kita mendekatinya dengan tabel. Sehingga bila kita buat tabel kemungkinannya adalah sebagai berikut: Laki-laki/ Perempuan Laki-Laki
Laki-Laki Perempuan Laki-laki, Laki-laki, Laki-laki Perempuan Perempuan Perempuan, Perempuan, Laki-laki Perempan Dari tabel di atas, terlihat bahwa kemungkinan munculnya laki-laki terdapat pada 3 kotak dari 4 kotak yang tersedia sebagai ruang sampel. Namun, informasi soal menyebutkan bahwa tetangga baru itu, ‘memiliki 2 anak, salah satu anak tetangga baru tersebut adalah | 403
perempuan’. Karenanya, tinggal 2 kotak (warna hitam) saja yang memungkinkan lahir laki-laki dari 3 kotak ruang sampel. Karena 1 ruang sampel yang terdiri atas Laki-Laki dan Laki-laki tidak memenuhi, sehingga ruang sampelnya menjadi: 𝑛𝑛(𝑆𝑆) = 3 𝑃𝑃(𝑆𝑆) =
𝑃𝑃(𝑆𝑆) =
𝑛𝑛(𝑆𝑆) 𝑛𝑛(𝐾𝐾𝑜𝑜𝑡𝑡𝑡𝑡𝑡𝑡) 2 3
Jadi, besar peluang anak yang lain adalah laki-laki adalah 2/3,”jelas Yodha mengakhiri penjelasannya. “Ternyata sederhana sekali, Yodha?”tanya Amer. “Ya, asal kita mampu memahami maksud soalnya dan membuat tabel kemungkinannya, Mer”terang Yodha. Kembali terdengar tepuk tangan meriah atas keterangan Yodha terhadap soal terakhir pada bagian Isian Singkat OSN tingkat Kabupaten dan Kota. “Alhamdulillah kawan-kawan, kita telah menyelesaikan semua persoalan yang diajukan. Dengan selesainya pembahasan sore ini, maka seluruh rangkaian kegiatan diskusi kita pun selesai. Malam ini, selepas makan kita bisa kembali ke rumah. Bis kita sudah tiba dan sejak pukul 16.00 tadi, dan sekarang parker di halaman ruang utama. Kita sepakati kumpul di ruang utama untuk sama-sama berangkat pulang pukul 20.30! Dan sebelum kita berpisah, mari sama-sama kita membaca hamdalah dan doa penutup majelis,”ujar Sabiq. …∑πχαΩ…
| 404
16
Lima Lontar yang Mengharukan “Baik, kita mulai dengan Lontar Pertama. Ada 3 persoalan yang diajukan. Ini yang pertama, ‘three numbers are in arithmetic progression, three others numbers are in geometric progression. Adding of corresponding terms of these two progressions successively, we obtain: 85, 76, 84 respectively. And adding all three terms of arithmetic progression, we obtain 126. Find the terms both progressions!”Sabiq membaca teks soal dengan lancar. “Dan kira-kira maksud dari soal di atas adalah terdapat 3 bilangan yang membentuk Aritmetika, dan tiga bilangan lain yang membentuk Deret Geometri. Bila masing-masing suku pada Deret Aritmetika dan Deret Geometri padda posisi yang sama dijumlahkan menghasilkan angka-angka: 85, 76, dan 84. Sementara jumlah ketiga bilangan pada Deret Aritmetika adalah 126. Kita diminta menentukan bilangan-bilangan pada masing-masing deret itu, Kawan,”terang Sabiq. “Biarkan aku mencoba, kawan!”Iman menawarkan diri. “Silahkan, Man!”kata Sabiq Iman tampak berkonsentrasi. Ia menatap tajam ke arah soal yang ada di lontar, dan sejurus kemudian dia mulai menulis di papan tulis. | 405
“Andai kita buat permisalan bahwa bilangan-bilangan yang membentuk Deret Aritmetika itu adalah: (𝑎𝑎 − 𝑏𝑏), 𝑎𝑎, (𝑎𝑎 + 𝑏𝑏)
Di mana b adalah beda antara suku berikutnya dengan sebelumnya:
Dan
𝑏𝑏 = 𝑈𝑈𝑛𝑛 − 𝑈𝑈𝑛𝑛−1
bilangan-bilangan
pada
Deret
Geometri
itu,
dengan
perbandingan tetap (r) antara suku sesudah dan sebelumnya akan memenuhi adalah: 𝑔𝑔 � � , 𝑔𝑔, (𝑔𝑔𝑟𝑟) 𝑟𝑟 Di mana r dirumuskan sebagai perbandingan tetap antara suku sesudah dan sebelumnya, yaitu: 𝑟𝑟 =
𝑈𝑈𝑛𝑛 𝑈𝑈𝑛𝑛−1
Maka, dari informasi soal kita peroleh beberapa persamaan, yaitu: 𝑔𝑔 (𝑎𝑎 − 𝑏𝑏) + � � = 85 … … … … … … … . . (16.1) 𝑟𝑟 𝑎𝑎 + 𝑔𝑔 = 76 … … … … … … … … … … . … . (16.2)
(𝑎𝑎 + 𝑏𝑏) + 𝑔𝑔𝑟𝑟 = 84 … … . … … … … … . . . (16.3) (𝑎𝑎 − 𝑏𝑏) + 𝑎𝑎 + (𝑎𝑎 + 𝑏𝑏) = 126 … … … . . (16.4)
Sehingga dari persamaan (16.4) diperoleh: 3𝑎𝑎 = 126 ⟺ 𝑎𝑎 = 42
Karenanya, dari persamaan (16.2) 𝑎𝑎 + 𝑔𝑔 = 76
42 + 𝑔𝑔 = 76 𝑔𝑔 = 34
Bila kita jumlahkan persamaan (16.1) dan persamaan (16.3), diperoleh: | 406
𝑔𝑔 (𝑎𝑎 − 𝑏𝑏) + � � = 85 𝑟𝑟 (𝑎𝑎 + 𝑏𝑏) + 𝑔𝑔𝑟𝑟 = 84 +
1 2𝑎𝑎 + 𝑔𝑔 �𝑟𝑟 + � = 169 𝑟𝑟 1 2(42) + 34 �𝑟𝑟 + � = 169 𝑟𝑟 1 34 �𝑟𝑟 + � = 169 − 84 𝑟𝑟 1 34 �𝑟𝑟 + � = 85 … … … … … … … (16.5) 𝑟𝑟
Selanjutnya, bila persamaan (16.5) kita kalikan kedua ruasnya dengan r, maka: 34𝑟𝑟 2 + 34 = 85𝑟𝑟
34𝑟𝑟 2 − 85𝑟𝑟 + 34 = 0
Diperoleh persamaan kuadrat yang memenuhi bentuk umum 𝑎𝑎𝑟𝑟 2 + 𝑏𝑏𝑟𝑟 + 𝑐𝑐 = 0, dengan a = 34, b = - 85, dan c = 34. Sehingga,
untuk menemukan akar-akar persamaan kuadrat di atas (r1, r2), dapat digunakan perumusan: 𝑟𝑟1,2 =
−𝑏𝑏 ± √𝑏𝑏 2 − 4𝑎𝑎𝑐𝑐 2𝑎𝑎
𝑟𝑟1,2 =
−(−85) ± �(−85)2 − (4𝑥𝑥34𝑥𝑥34) 2(34)
𝑟𝑟1,2 =
85 ± √2601 68
karenannya,
𝑟𝑟1,2 =
85 ± √7225 − 4624 68
| 407
𝑟𝑟1,2 =
Sehingga,
𝑟𝑟1 =
85 ± 51 68
85 + 51 136 = =2 68 68
⟺ (𝑎𝑎 + 𝑏𝑏) + 𝑔𝑔𝑟𝑟1 = 84
⟺ 42 + 𝑏𝑏 + 34(2) = 84
⟺ 𝑏𝑏 = 84 − 68 − 42 ⟺ 𝑏𝑏 = −26
Deret Aritmetikanya menjadi: (𝑎𝑎 − 𝑏𝑏), 𝑎𝑎, (𝑎𝑎 + 𝑏𝑏) = (42 − (−26)), 42, (42 − 26) = 68, 42, 16 (𝑎𝑎 − 𝑏𝑏), 𝑎𝑎, (𝑎𝑎 + 𝑏𝑏) = 68, 42, 16
dan Deret Geometrinya adalah:
34 𝑔𝑔 � � , 𝑔𝑔, (𝑔𝑔𝑟𝑟) = � � , 34, (34𝑥𝑥2) = 17, 34, 68 2 𝑟𝑟
Sementara untuk nilai, 𝑟𝑟2 =
85 − 51 34 1 = = 68 68 2
⟺ (𝑎𝑎 + 𝑏𝑏) + 𝑔𝑔𝑟𝑟2 = 84
1 ⟺ 42 + 𝑏𝑏 + 34 � � = 84 2
⟺ 𝑏𝑏 = 84 − 17 − 42 ⟺ 𝑏𝑏 = 25
Deret Aritmetikanya menjadi: (𝑎𝑎 − 𝑏𝑏), 𝑎𝑎, (𝑎𝑎 + 𝑏𝑏) = (42 − 25)), 42, (42 + 25) = 17, 42, 67 dan Deret Geometrinya adalah:
34 34 𝑔𝑔 � � , 𝑔𝑔, (𝑔𝑔𝑟𝑟) = � � , 34, � � = 68, 34, 17 1 2 𝑟𝑟 2
| 408
Sehingga pada akhirnya, bilangan-bilangan pada masing-masing deret itu adalah: (1) untuk Deret Aritmetika 68, 42, dan 16; maka Deret Geometrinya 17, 34, dan 68; dan (2) untuk Deret Aritmetika 17, 42, dan 67; maka Deret Geometrinya 68, 34, dan 17 Itu hal yang bisa aku pahami dari persoalan kedua deret di atas, Kawan-kawan,”pungkas Iman. “Sebuah kombinasi persoalan yang indah, Man!”komentar Yodha. “Ya, aku sepakat denganmu, Kawan!”timpal Ikram. “Baik, apakah ada yang ingin memecahkan misteri pada lontar-lontar itu?”tantang Sabiq “Biarkan aku memecahkan soal “superball” itu, Biq!”jawab Denan. Denan bergerak cepat, ia membaca singkat soal itu kembali, dan mulai mengoreskan garis-garis naik-turun beberapa kali. Dan... “Bila kita perhatikan informasi soal, yang menyebutkan bahwa, ‘it takes two seconds for a “superball” to hit the ground and at each successive bounce it takes 9/10 of the time previous fall. How long will it take for the ball to stop bounces?’ kira-kira seperti gambaran inilah lintasan “superball” itu sampai berhenti. Dan selanjutnya kita dapat mencatat waktu yang dibutuhkan oleh superball selama melayang, adalah sebagai berikut:
| 409
𝑡𝑡𝐴𝐴𝐵𝐵1 = 2 𝑑𝑑𝑒𝑒𝑒𝑒𝑒𝑒𝑒𝑒
9 9 𝑡𝑡𝐵𝐵1𝐵𝐵2 = 2 � 𝑥𝑥2� = 4 � � 𝑑𝑑𝑒𝑒𝑒𝑒𝑒𝑒𝑒𝑒 10 10
9 9 9 2 𝑡𝑡𝐵𝐵2𝐵𝐵3 = 2 � 𝑥𝑥 𝑥𝑥2� = 4 � � 𝑑𝑑𝑒𝑒𝑒𝑒𝑒𝑒𝑒𝑒 10 10 10 9 3 𝑡𝑡𝐵𝐵3𝐵𝐵4 = 4 � � 𝑑𝑑𝑒𝑒𝑒𝑒𝑒𝑒𝑒𝑒 10
Dan seterusnya, sampai “superball” itu berhenti. Sehingga waktu total yang dibutuhkan oleh “superball” untuk melayang sampai berhenti akan membentuk deretan sebagai berikut: 𝑡𝑡𝑡𝑡𝑜𝑜𝑜𝑜𝑜𝑜𝑜𝑜
9 9 2 9 3 = 2 + 4 � � + 4 � � + 4 � � + ⋯ 𝑑𝑑𝑒𝑒𝑒𝑒𝑒𝑒𝑒𝑒 10 10 10
= 2 + 4�
9 9 2 9 3 9 � �1 + � � + � � + � � + ⋯ � 𝑑𝑑𝑒𝑒𝑒𝑒𝑒𝑒𝑒𝑒 10 10 10 10
Terlihat bahwa suku-suku dalam tanda kurung [ ] membentuk
Deret Geometri tak hingga dengan suku pertama, a = 1 dan perbandingan antara suku, r = (9/10). Sehingga dapat ditulis menjadi: 9 1 𝑡𝑡𝑡𝑡𝑜𝑜𝑜𝑜 𝑎𝑎𝑙𝑙 = 2 + 4 � � � � 𝑑𝑑𝑒𝑒𝑒𝑒𝑒𝑒𝑒𝑒 10 1 − 9 10
| 410
9 𝑡𝑡𝑡𝑡𝑜𝑜𝑜𝑜𝑜𝑜𝑜𝑜 = 2 + 4 � � 10 𝑑𝑑𝑒𝑒𝑒𝑒𝑒𝑒𝑒𝑒 10 𝑡𝑡𝑡𝑡𝑜𝑜𝑡𝑡𝑡𝑡𝑡𝑡 = 38 𝑑𝑑𝑒𝑒𝑡𝑡𝑡𝑡𝑡𝑡
Akhirnya dapat disebutkan bahwa total waktu yang dibutuhkan oleh “superball” untuk berhenti melayang adalah sebanyak 38 detik. Demikian penjelasanku, Kawan-kawan,”kata penutup Denan. “Alhamdulillah, tampaknya Denan pun bukan hanya sudah pandai bahasa Inggeris, tapi paham maksud pertanyaan dan persoalan intinya,”puji Sabiq. “Masih dalam Lontar Pertama, kita permudah saja, siapa berikutnya?”tantang Sabiq. “Bagaimana kalau soal dalam Lontar yang ini bagianku, Kawan?”tiba-tiba Amer menawarkan diri. “Silahkan, sepenuhnya waktu pengerjaan sejak saat ini adalah milikmu, Kawanku,”balas Sabiq. “Aku tak paham betul maksud dari pertanyaannya, yang menyebutkan bahwa, ‘a right-angled triangle ABC has perimeter 624 cm and area 6864 cm2. Find the length of its hypotenuse’, tapi kira-kira soal ini bertanya tentang panjang sisi miring dari suatu segitiga siku-siku yang diketahui luas dan kelilingnya,”Amer memulai. “Ya, memang seperti itu maksud dari pertanyaannya Kawan. Tepat sekali apa yang kau maksud itu,”tegas Sabiq pada Amer. “Nah, kalau seperti itu maksudnya, aku coba gambarkan ilustrasinya.
Kira-kira
seperti
ini,”kata
Amer
lagi
sambil
menggambar sebuah segitiga di papan tulis.
| 411
Gambar itu adalah gambar dari segitiga siku-siku yang lengkap dengan keterangan panjang dari masing-masing sisinya.
“Dari keterangan soal, maka gambar segitiga siku-siku di atas memberi informasi sebagai berikut: 𝐾𝐾𝑒𝑒𝑒𝑒𝑒𝑒𝑒𝑒𝑒𝑒𝑒𝑒𝑒𝑒 (𝐾𝐾) = 𝑎𝑎 + 𝑏𝑏 + 𝑐𝑐 𝐿𝐿𝑢𝑢𝑢𝑢𝑢𝑢 (𝐿𝐿) =
𝑎𝑎𝑏𝑏 ⇔ 4𝐿𝐿 = 2𝑎𝑎𝑏𝑏 2
Dan dari dalil Phytagoras kita ketahui, bahwa: 2
𝑐𝑐 2 = 㕣 + 𝑏𝑏 2
Selanjutnya, jika Keliling (K) kita kuadratkan, akan dihasilkan: 𝐾𝐾 2 = (𝑎𝑎 + 𝑏𝑏 + 𝑐𝑐)2 = 𝑎𝑎2 + 𝑏𝑏 2 + 𝑐𝑐 2 + 2𝑎𝑎𝑐𝑐 + 2𝑎𝑎𝑏𝑏 + 2𝑏𝑏𝑐𝑐 𝐾𝐾 2 = 𝑐𝑐 2 + 𝑐𝑐 2 + 2𝑎𝑎𝑐𝑐 + 2𝑏𝑏𝑐𝑐 + 4𝐿𝐿 𝐾𝐾 2 = 2𝑐𝑐(𝑎𝑎 + 𝑏𝑏 + 𝑐𝑐) + 4𝐿𝐿
𝐾𝐾 2 = 2𝑐𝑐𝐾𝐾 + 4𝐿𝐿
𝐾𝐾 2 − 4𝐿𝐿 2𝐾𝐾 Dengan memasukkan nilai K = 624 dan L = 6864, maka: 𝑐𝑐 =
6242 − 4𝑥𝑥6864 2𝑥𝑥624 624𝑥𝑥624 4𝑥𝑥6864 𝑐𝑐 = − 2𝑥𝑥624 2𝑥𝑥624
𝑐𝑐 =
𝑐𝑐 = 312 − 22 𝑐𝑐 = 290
| 412
Sehingga, jawaban atas ‘the length of its hypotenuse’ adalah 290 sentimer, Kawan-kawan!”pungkas Amer. “Selamat, Mer! Semakin mantap saja kau dalam hal Geometri ini,”puji Fateh. Amer hanya tersenyum kecil sambil berjalan kembali ke posisinya. Beberapa orang kawannya memberi tepukan. Amer hanya mengangguk-anggukan kepalanya. “Alhamdulillah, Lontar Pertama sudah selesai kita kerjakan. Ada yang mau mengajukan solusi atas Lontar lainnya?”tanya Sabiq. “Biq, aku bukan ingin mengerjakan. Justru bertanya serius, tentang Lontar yang diletakkan di depan pintu kami. Setelah kami baca. Ada simbol-simbol aneh, yang, jujur belum pernah kami temukan dalam pelajaran kita di kelas selama ini,”tiba-tiba Ayra berkata serius. “Maksudmu, Ayra? Ada simbol Matematika yang tak kau pahami? Bisa kau bacakan atau gambarkan buat kita?”pinta Sabiq “Baik. aku gambar dulu simbolnya, selanjutnya aku bacakan soal-soalnya,”balas Ayra sambil bererak ke dekat papan tulis. Ayra mengambar sesuatu. Ia menuliskan huruf X dan Y berdampingan, agar besar. Tapi, setelah itu, kedua huruf itu diberi seperti pembatas di kedua sisinya. Nah, inilah simbol itu! ⌊𝑋𝑋⌋ ⌈𝑌𝑌⌉
“Ada tanda seperti ini dalam beberapa huruf. Kami tak tahu maksudnya, Kawan-kawan!”terang Ayra. Semua pasang mata seperti tersihir. Serius mengamati kedua tanda yang saling berkebalikan itu, “⌊ ⌋" 𝑑𝑑𝑎𝑎𝑎𝑎 "⌈ ⌉". Dan entah | 413
kenapa tiba-tiba seperti dikomando, semua pasang mata itu beralih ke satu arah, Sabiq. Sabiq membalas tatapan kawan-kawannya itu. Dia menoleh ke arah Yodha. Yodha menggeleng. Dia menoleh ke arah Ikram, juga menggeleng. Akhirnya, Sabiq hanya bisa menarik nafas dalamdalam. “Sebaiknya kita nikmati dulu secangkir Teh dan penganan kecil yang sudah disajikan di meja oleh Mamakku. Soal itu kita bahas
setelahnya,”Sabiq
mengalihkan
pembicaraan
kawan-
kawannya. “Ah, kalau itu alasannya, aku setuju Kawan!”respons Amer. Memang di meja sederhana itu telah tersedia seteko sedang Teh Manis Hangat dan jajanan pasar seperti kue Cucur, Apem, Nagasari, Lemper, dan kue Lapis. Melihat jajanan pasar yang beragam dan menarik itu, kesebelas kawan itu tampaknya bisa melupakan sejenak ketegangan mereka. Tapi, tidak dengan Sabiq! Anak laki-laki ini tampaknya sedang berusaha mengingat dan berfikir keras. Entah apa yang dipikirkannya. …∑πχαΩ…
Terjadi kegaduhan kecil dalam bis yang mengantar sebelas anakanak jenius kampong itu pulang dari wisata edukasi. Berawal dari laporan Iman, yang terkejut, karena sesaat akan masuk kamar setelah makan malam, tepat di depan pintu masuk kamar mereka tergeletak selembar daun lontar yang unik bentuknya. Setelah diperiksa ternayata berisi tulisan-tulisan aneh. | 414
“Kami periksa tulisan di lontar itu, ternyata berisi 3 soal Matematika, Kawan-kawan!”laporan Iman pada kawan-kawannya. Kegaduhan terjadi. Suasana semakin ramai, karena ternyata bukan hanya Iman, Denan, dan Ikram saja yang mendapatkannya. Ayra juga menemukan Lontar yang mirip diceritakan oleh Iman. Yodha dan Amer juga menemukan Lontar yang berisi soal Matematika. “Aku dan Bintang sampai berfikir siapa yang iseng menaruh Lontar
di
depan
pintu?”kata
Saver
tentang
Lontar
yang
ditemukannya. “Ya, ini bagian dari rencana Allah. Insyaallah semua akan baik-baik saja pada akhirnya,”terdengar sepenggal kalimat Sabiq memecah
keributan
dalam
Bis
yang
mulai
melaju
turun
meninggalkan kampung itu. “Apakah kau tidak mendapatinya di depan pintu kamarmu, Biq?”tanya Ayra. Sabiq hanya diam, setidaknya tak menjawab untuk beberapa saat. “Bukan hanya mendapat Lontar, Ayra. Justru aku sedang berfikir keras siapa yang menaruhnya begitu rapi di bagian dalam pintu kamarku. Ini Lontar itu!”ujar Sabiq sambil menunjukkan selembar Lontar yang agak besar dan panjang. Berbeda dengan ukuran milik kawan-kawannya. Dan menurutku isinya sangat menarik. Suasana menjadi hening. Semuanya berfikir tentang Lontar. “Sudahlah, Kawan-kawan. Tak penting siapa yang menaruh, kenapa bisa di depan pintu kamar kita Lontar-Lontar itu. Yang | 415
penting apakah Lontar-Lontar itu ada manfaatnya buat kita?”tiba-tiba Amer memecah keheningan di bis yang semakin malam bergerak semakin cepat. “Tepat! Itu yang sedang aku pikirkan, Kawan-kawan!”tibatiba Yodha angkat bicara. “Maksudmu apa Yodha?”tanya Iman balik. “Bukankah tadi Sabiq berkata bahwa ini bagian dari rencana Allah. Insyaallah semua akan baik-baik saja pada akhirnya! Pertanyaannya, bagaimana kita bisa menemukan kebaikannya adalah lebih utama,”balas Yodha. “Dan kau punya pendapat tentang hal itu, Kawan?”tanya Ikram pada Yodha. “Ada. Itu pun kalau kawan-kawan bersedia dengan ide ini,”jawab Yodha lagi. “Apa idemu itu, Kawan?”kejar Denan. “Kita sepakat membuat satu pertemuan tambahan untuk membahas apa yang ditulis dalam Lontar-Lontar ini. Dan usulku kita lakukan di rumah Sabiq. Kapan? Pekan depan, tepatnya, Ahad pagi, kita Dhuha bersama di Musholla di dekat rumah Sabiq!”usul Yodha. “Setuju! Aku setuju dengan usulmu, Kawan!”Amer langsung merespons usul Yodha. “Aku juga sependapat denganmu, Yodha,”kata Ayra. “Aku tak punya pendapat selain apa yang kau utarakan itu, Yodha!”terdengar ucapan Iman. “Dan, kalau mengikuti kebiasan kita, lebih-cepat lebih-baik usul itu disepakati,”komentar Fateh. | 416
“Ha..ha….ha. Sekarang kau terdesak Biq. Kau tak bisa menolak. Tugasmu hanya satu, sampaikan ke Bibi, Mamakmu, agar memersiapkan makanan yang banyak dan uuennak, Kawan!”tiba-tiba Amer bersuara cukup keras, sampai membuat Pak Sopir pun tersenyum-senyum. “Baik, Kawan-kawan. Insyaallah kita bertemu di rumahku Ahad pagi pekan depan, tepatnya pukul 08.30. Oke!”respons Sabiq atas usul kawan-kawannya. “Insyaallah kami datang. Hati-hati, Kesebelasan Genk OSN akan menyerbu rumahmu, Biq!”terdengar ucapan serempak hampir seluruh anak yang ada dalam bis yang melaju semakin cepat meninggalkan pedesaan yang dingin itu. …∑πχαΩ…
“Baiklah kawan-kawan, setelah menikmati secangkir Teh Hangat dan jajanan pasar, rasanya kita siap melanjutkan pembahasan kita terhadap persoalan pada Lontar-Lontar itu,”ujar Sabiq pada kawan-kawannya. “Dan kita fokus pada soal simbol aneh ini, Biq!”tegas Ayra. “Ya, kita akan fokus pada persoalan itu, kawan-kawan,”balas Sabiq. Tiba-tiba suasana hening. Semua menunggu apa yang akan disampaikan oleh Sabiq tentang simbol aneh dalam Matematika itu. “Kawan-kawan,
sebelumnya
aku
mohon
maaf
atas
keteranganku ini. Selama rehat tadi, aku coba membuka beberapa catatan, dan memang kita belum pernah memelajari fungsi seperti ini,”terang Sabiq | 417
“Belum pernah? Berarti kita akan pernah, maksudmu Biq?”tanya Ikram “Ya, memang kita akan belajar fungsi ini. Simbol seperti ini masuk dalam materi Bilangan dalam Matematika. Lambang “⌊ ⌋" disebut sebagai Fungsi Tangga sementara lambang "⌈ ⌉"
dikenal sebagai Fungsi Ceiling. Apa istimewanya kedua fungsi ini? Mari sama-sama kita perhatikan,”terang Sabiq lagi. “Kawan-kawan, jika ditemukan fungsi tangga, di mana fungsi y = f(x) = ⎣x⎦, maka dengan tanda ⎣x⎦ menyatakan bilangan bulat
terbesar kurang dari atau sama dengan x. Apa artinya ini? Artinya adalah bahwa: a. Jika x bernilai 3,7 maka y = ⎣3,7⎦ = 3; b. Jika x bernilai 4 maka y = ⎣4⎦ = 4;
c. Jika x bernilai −2,5 maka y = ⎣−2,5⎦ = −3;
d.
Jika 4 ≤ x < 5 maka y = f(x) = 4;
e. Jika 5 ≤ x < 6 maka y = f(x) = 5,
Dan seterusnya, dan seterusnya. Apa uniknya? Uniknya adalah bahwa jika fungsi tangga tersebut digambarkan dalam koordinat kartesian maka akan terbentuk seperti tangga, seperti gambar berikut,”terang Sabiq
| 418
Sabiq kembali menuju papan tulis dan dia menggambar sesuatu di sana. “Selain fungsi tangga itu, ada juga yang disebut fungsi ceiling yang merupakan kebalikan dari fungsi tangga. Karenanya, jika fungsi y = f(x) = ⌈𝑥𝑥⌉, maka tanda ⌈x⌉ menyatakan bilangan bulat terkecil
lebih dari atau sama dengan x. Apa pula artinya ini? Artinya bahwa: a. Jika x bernilai 3,7 maka y = ⌈3,7⌉= 4; b. Jika x bernilai 4 maka y = ⌈4⌉ = 4;
c. Jika x bernilai −2,5 maka y = ⌈−2,5⌉= −2;
d. Jika 4 ≤ x < 5 maka y = f(x) = 5; e. Jika 5 ≤ x < 6 maka y = f(x) = 6.
Dan seterusnya, dan seterusnya,”terang Sabiq lagi. “Susah sekali mengingatnya. Bisa terbalik-balik ingatanku, Kawan,”potong Saver. “Nah, aku punya satu kiat untuk mengingatnya,”balas Sabiq. “Kiat apa itu? Jembatan Keledai? Atau…?”tanya Amer. “Kira-kira seperti ini Fungsi Tangga, ⌊𝑥𝑥⌋, menekan ke bawah,
sementara Fungsi Ceiling , ⌈𝑥𝑥⌉, menarik ke atas!”tegas Sabiq.
Tampak setiap anak yang hadir di ruang itu berfikir, mereka
sedang menelaah apa yang baru saja disampaikan oleh Sabiq. “Exactly!
Kau
menemukan
satu
kalimat
yang
bisa
menggambarkan kedua fungsi itu, Kawan!”seru Yodha. “Biq, terlalu rumit teorinya. Apa bisa langsung kita terapkan dalam persoalan konkret. Apa gunanya kedua fungsi ini. Dan yang paling penting adalah, apakah ada atau pernah dalam soal-soal OSN, kedua atau salah satu fungsi ini ditanyakan?”tanya Ayra fokus. Sabiq tak langsung menjawab. Dia tampaknya berfikir keras. | 419
“Baik sekaligus menjawab semua pertanyaan Ayra, kita akan melihat contoh sederhana. Dan ini ternyata adalah soal OSN di tingkat Provinsi. Begini kalimatnya, ‘Notasi ⎣x⎦ menyatakan bilangan bulat terbesar yang lebih kecil dari atau sama dengan x. Sebagai contoh ⎣7/2⎦ = 2, ⎣-1/2⎦ = -1, maka hubungan yang benar di
antara dua bilangan bulat s= ⎣√2 – √3⎦ dan t = ⎣√2⎦ – ⎣√3⎦
adalah..’,”kata Sabiq pada teman-temannya.
“Ternyata ada! Dan... bagaimana kita menyelesaikannya, Kawan?”tanya Ayra. “Sangat sederhana, Ayra. Ingat prinsip tadi bahwa..,”tiba-tiba kalimat Sabiq terpotong karena ada suara lain yang lebih keras. “Fungsi Tangga, ⌊𝑥𝑥⌋, menekan ke bawah, sementara Fungsi
Ceiling , ⌈𝑥𝑥⌉, menarik ke atas!”terdengar suara khas Amer.
“Lantas, bagaimana menerapkannya?”kali ini Fateh pun
penasaran. “Berapa kira-kira nilai dari √2, Fateh?”tanya Sabiq. “Sekitar 1,4 lebih…, Kawan!”jawab Fateh. “Kalau nilai √3, kira-kira berapa, Man?”tanya Sabiq pada Iman. “Satu koma tujuh lebih, Biq!”jawab Iman. “Nah, kita terapkan saja prinsipnya, sehingga: √2 − √3 = 1,4 … − 1,7 … = −0,3 ….
Dari soal, kita diminta menghitung nilai dari s, dimana: 𝑠𝑠 = ⎣√2– √3⎦ = ⎣ − 0,3⎦
Karena merupakan fungsi tangga berarti, tekan ke bawah. Sehingga: 𝑠𝑠 = ⎣ − 0,3⎦ = −1
| 420
Sementara terhadap nilai t, bila kita hitung: 𝑡𝑡 = ⎣√2⎦ − ⎣√3⎦
𝑡𝑡 = ⎣1,4 … ⎦ − ⎣1,7 … ⎦
Karena juga merupakan fungsi tangga berarti, tekan ke bawah. Maka: 𝑡𝑡 = 1 − 1 = 0
Sehingga, hubungan yang benar di antara dua bilangan bulat s dan t adalah: 𝑠𝑠 + 𝑡𝑡 = −1 + 0 = −1
Itu penyelesaian untuk kasus di atas, Kawan-kawan,”terang Sabiq. “Sederhana, memang sederhana,”komentar Saver. “Tapi, apakah hanya itu saja fungsi dari Fungsi Tangga dan Fungsi Ceiling itu, Biq?”kali ini Denan bertanya serius. “Tidak, ada beberapa. Fungsi lainnya, bahwa tanda ⎣x⎦ yang
berarti fungsi tangga itu sendiri, dapat digunakan untuk menghitung pangkat tertinggi bilangan prima dari suatu bilangan n! di mana tanda “!” menyatakan faktorial,”terang Sabiq lagi. “Nah, kalau ini menarik. Contohnya, Biq?”tanya Bintang. “Seperti ini misalnya! ‘Jika bilangan 2010! = 1x 2x3x ⋅⋅⋅
x2010, dan habis dibagi oleh 7k untuk suatu bilangan asli k tertentu. Tentukan nilai maksimal dari k! Soal ini bisa didekati dengan Fungsi Tangga,”terang Sabiq lagi. “Bagaimana mendekatinya, Kawan?”tanya Bily. Sabiq kembali fokus. “Kawan-kawan, begini caranya. Di antara 2010 bilangan 1, 2, 3, ⋅⋅⋅, 2010 terdapat �
2010 � 7
= 287 bilangan yang habis dibagi 71. Jika | 421
kmaks=287, maka akan ada bilangan kelipatan 72 (=49) yang faktor 7nya hanya dihitung satu kali. Maka nilai k tersebut haruslah ditambahkan dengan �
2010 � 49
= 41. Tetapi faktor 7 dari bilangan
kelipatan 73 = 343 hanya dihitung dua kali padahal seharusnya tiga 2010
kali. Maka hasil sebelumnya harus ditambahkan dengan � � = 5. 343
Selanjutnya, karena tidak ada bilangan kelipatan 74 dari 2010
bilangan tersebut, maka perhitungan telah lengkap. Sehingga: 𝑘𝑘𝑚𝑚 𝑎𝑎𝑎𝑎𝑎𝑎 = �
2010 2010 2010 �+� 2 �+� 3 � 1 7 7 7
𝑘𝑘𝑚𝑚 𝑎𝑎𝑎𝑎𝑎𝑎 = 287 + 41 + 5 = 333
Itu fungsi yang tampaknya paling menonjol dalam Teori Bilangan kaitannya dengan Fungsi Tangga, Kawan-kawan,”tegas Sabiq lagi. Tampak kawan-kawannya begitu antusias mengikuti cerita tentang Fungsi Tangga dan Fungsi Ceiling ini. “Oke, ini kasus terakhir yang mungkin menarik tentang fungsi tangga. Ada persoalan seperti ini. ‘Untuk setiap bilangan real α, kita definisikan ⎣α⎦ sebagai bilangan bulat yang kurang dari atau sama dengan α. Sebagai contoh ⎣9,4⎦ = 9 dan ⎣7⎦ = 7. Jika x dan y
bilangan real sehingga ⎣√x⎦ = 9 dan ⎣√y⎦ = 12, maka nilai terkecil yang mungkin dicapai oleh ⎣y−x⎦ adalah?’ Adakah kawan-kawan yang mau coba?”pancing Sabiq.
“Biar aku mencoba hasil dari penjelasanmu, Kawan,”Ikram memberanikan dirinya. Ikram berdiri dan terus berjalan menu papan tulis. “Seperti sama-sama kita ketahui tentang fungsi dan nilai dari tanda akar, maka: | 422
(1) Karena √81 = 9 dan √100= 10, maka ⎣√x ⎦= 9 dipenuhi oleh 81 ≤ x < 100;
(2) Karena √144 = 12 dan √169 = 13, maka ⎣√y⎦ = 12 dipenuhi oleh 144 ≤ y < 169
Sehingga, ⌊𝑦𝑦 − 𝑥𝑥⌋𝑚𝑚 𝑖𝑖𝑖𝑖 = ⌊𝑦𝑦𝑚𝑚 𝑖𝑖𝑖𝑖 − 𝑥𝑥𝑚𝑚 𝑎𝑎𝑎𝑎 𝑠𝑠 ⌋
⌊𝑦𝑦 − 𝑥𝑥⌋𝑚𝑚 𝑖𝑖𝑖𝑖 = ⌊144 − 99,99 … ⌋
⌊𝑦𝑦 − 𝑥𝑥⌋𝑚𝑚 𝑖𝑖𝑖𝑖 = ⌊44, … 1⌋ ⌊𝑦𝑦 − 𝑥𝑥⌋𝑚𝑚 𝑖𝑖𝑖𝑖 = 44
Hasil akhirnya adalah 44, Kawan-kawan!”ucap Ikram bersemangat. “Wah, cepat sekali kau bisa memahami, Kawan?”tanya Amer heran. Ikram hanya tersenyum dan kembali ke kursinya. “Terima kasih, Biq. Rasa penasaranku telah terjawab dengan keterangan dan contoh soal ini,”kata Ayra. “Sama-sama Ayra . Kawan-kawan, seperti itu beberapa hal tentang Fungsi Tangga dan Fungsi Ceiling yang aku tahu. Dan aku pikir kita cukupkan
saja,
selanjutnya
kita
beranjak
ke
Lontar
berikutnya,”pungkas Sabiq. “Setuju, dan Lontar yang ini tampaknya begitu serius, soal Geometri, Kawan!”ucap Yodha “Baik. Tapi, apakah ada soal yang aneh-aneh, Kawan?”tanya Sabiq. “Rasanya tidak. Hanya kombinasi beberapa hal saja,”balas Yodha tenang. | 423
“Kalau begitu sebaiknya memang kau saja yang harus menjelaskannya kepada kita semua, Kawanku Yodha!”kata Saver. Yodha hanya mengangguk pelan. “Ini isi Lontar yang diletakkan di depan pintu kamar kami, kawan-kawan! ‘Angle ABC of ΔABC is a right angle. The sides of ΔABC are the diameters of semicircles as shown. The area of the semicircle on 𝐴𝐴𝐵𝐵 equals 8π, and the arc of the semicircle on AC has length 8,5 π. What is the radius of the semicircle on BC? Terhadap soal
jenis
ini,
kita
perlu
perhatikan
apa
saja
yang
diinformasikan,”kalimat pembuka Yodha. Yodha menggambar sesuatu di papan tulis. Sebuah segitiga siku-siku yang dikelilingi oleh 3 buah setengah lingkaran.
“Soal di atas menanyakan panjang jari-jari dari setengah lingkaran yang terbentuk pada garis BC pada gambar di atas. Dari informasi soal bahwa ‘the area of the semicircle on 𝐴𝐴𝐵𝐵 equals 8’ kita bisa menghitung:
1 2 𝜋𝜋𝑟𝑟 = 8𝜋𝜋 ⇔ 𝑟𝑟𝐴𝐴𝐵𝐵 = √16 = 4 2 𝐴𝐴𝐵𝐵 𝑟𝑟𝐴𝐴𝐵𝐵 = 4 ⇔ 𝐴𝐴𝐵𝐵 = 8
Berikutnya, dari informasi tentang panjang busur, yaitu the arc of the semicircle on AC has length 8,5 π kita bisa menghitung panjang AC, sebagai berikut: | 424
1800 𝑥𝑥2𝜋𝜋𝑟𝑟𝐴𝐴𝐶𝐶 = 8,5𝜋𝜋 ⇔ 𝑟𝑟𝐴𝐴𝐶𝐶 = 8,5 3600 𝑟𝑟𝐴𝐴𝐶𝐶 = 8,5 ⇔ 𝐴𝐴𝐶𝐶 = 17
Selanjutnya, bila diperhatikan ΔABC, adalah merupakan segitiga siku-siku di B, sehingga dengan Phytagoras: 𝐴𝐴𝐶𝐶 2 = 𝐴𝐴𝐵𝐵2 + 𝐵𝐵𝐶𝐶 2
𝐵𝐵𝐶𝐶 2 = 𝐴𝐴𝐶𝐶 2 − 𝐴𝐴𝐶𝐶 2
𝐵𝐵𝐶𝐶 2 = 𝐴𝐴𝐶𝐶 2 − 𝐴𝐴𝐵𝐵2
𝐵𝐵𝐶𝐶 2 = 172 − 82
𝐵𝐵𝐶𝐶 2 = (17 + 8)(17 − 8)
𝐵𝐵𝐶𝐶 = √25𝑥𝑥9 = 15
Sehingga, jawaban atas pertanyaan ‘what is the radius of the semicircle on BC?’ adalah: 𝐵𝐵𝐶𝐶 15 = = 7,5 2 2 Jadi, panjang jari-jari setengah lingkaran BC adalah sama dengan 7,5 𝑟𝑟𝐵𝐵𝐶𝐶 =
satuan. Begitu hasil perhitunganku, Kawan-kawan,”ujar Yodha. “Yodha, bagaimana dengan soal berikutnya, masih dalam Lontar yang sama, di mana disebutkan bahwa ‘titik-titik sudut suatu segitiga memiliki koordinat (0,0), (4,3) dan (7,-1). Maka luas segitiga tersebut adalah...’,”tanya Ikram. “Nah, ini soal menarik. Menurut yang aku ketahui, ada beberapa cara untuk menjawabnya. Oke, sama-sama kita perhatikan gambar berikut!”ujar Yodha sambil kembali menggambar di papan tulis.
| 425
“Kalau
begitu
kau
teruskan
saja
menjelaskannya,
Yodha!”pinta Sabiq. “Baik. gambar ini aku usahan mewakili posisi titik-titik sudut segitiga itu. Cara pertama cara biasa, yaitu mencari luas segitiga dengan rumusan yang sudah dipahami, sebagai: 1 𝐿𝐿𝑢𝑢𝑢𝑢𝑢𝑢 ∆= 𝑥𝑥𝑎𝑎𝑎𝑎𝑎𝑎𝑎𝑎𝑎𝑎𝑎𝑎𝑎𝑎𝑎𝑎𝑔𝑔𝑔𝑔𝑖𝑖 2
Nah, masalahnya adalah bagaimana menentukan yang mana alas dan bagian mana tingginya, serta berapa panjang alas dan berapa tingginya?”terang Yodha. Tampak kawan-kawannya sangat antusias dan bersemangat. Entah kenapa setiap kali Yodha membahas tentang Geometri, seperti ada daya magnet yang menarik kawan-kawannya untuk serius memerhatikan. “Untuk mengatasi hal ini, kita harus memahami satu prinsip bahwa garis tinggi pastilah tegak lurus terhadap alasnya. Sehingga yang kita lakukan adalah menemukan perpotongan garis yang memungkinkan saling tegak lurus,”terang Yodha. “Memang ada caranya, Yodha?”tanya Ayra. “Ada Ayra. Prinsipnya adalah bahwa setiap garis memiliki kemiringan atau yang disebut dengan istilah gradien. Nah, dua garis | 426
garis lurus dikatakan saling tegak lurus jika perkalian antara gradien garisnya sama dengan -1. Itu prinsipnya,”terang Yodha lagi. Yodha kembali menulis sesuatu di papan tulis. “Sebentar Yodha! Sebelum kau teruskan, kita kembali ke gradien lagi. Bagaimana cara untuk mengetahui gradien dari suatu garis lurus?”tanya Denan. “Baik. seperti kita ketahui bahwa sebuah persamaan garis lurus biasanya dituliskan dalam bentuk: 𝑦𝑦 = 𝑚𝑚𝑥𝑥 + 𝑐𝑐
Nah, huruf m itulah yang disebut sebagai gradien. Secara sederhana bisa kita sebut bahwa m atau gradien adalah perbandingan antara koefisien x pada persamaan garis dengan koefisien y. Dalam kasus persamaan y = mx + c itu, berarti perbandingannya adalah: 𝑔𝑔𝑟𝑟𝑟𝑟𝑟𝑟𝑟𝑟𝑟𝑟𝑟𝑟 =
𝐾𝐾𝑜𝑜𝑜𝑜𝑜𝑜𝑜𝑜𝑜𝑜𝑜𝑜𝑜𝑜𝑜𝑜 𝑥𝑥 𝑚𝑚 = = 𝑚𝑚 𝐾𝐾𝑜𝑜𝑜𝑜𝑜𝑜𝑜𝑜𝑜𝑜𝑜𝑜𝑜𝑜𝑜𝑜 𝑦𝑦 1
Terlihat bahwa m adalah gradien garis,”terang Yodha. “Baik aku teruskan dulu, Kawan-kawan. Perhatikan ΔOAB pada gambar! Terdapat 3 garis yang membentuk segitiga itu, yaitu: Garis OA, Garis AB dan Garis OB. Sebagaimana kita pelajari di kelas bahwa persamaan garis yang melalui titik (x1, y1) dan (x2, y2) adalah: 𝑦𝑦 − 𝑦𝑦1 𝑥𝑥 − 𝑥𝑥1 = 𝑦𝑦2 − 𝑦𝑦1 𝑥𝑥2 − 𝑥𝑥1
Sehingga persamaan Garis OA dengan O(0,0) = (x1,y1) dan A (4,3) = (x2,y2), menjadi: 𝐺𝐺𝑎𝑎𝑎𝑎𝑎𝑎𝑎𝑎 𝑂𝑂𝐴𝐴 ⇔
𝑥𝑥 − 𝑥𝑥1 𝑦𝑦 − 𝑦𝑦1 = 𝑦𝑦2 − 𝑦𝑦1 𝑥𝑥2 − 𝑥𝑥1
| 427
𝑦𝑦 − 0 驂 − 0 = 3−0 4−0 3 𝐺𝐺𝑎𝑎𝑎𝑎𝑎𝑎𝑎𝑎 𝑂𝑂𝐴𝐴 ⇔ 𝑦𝑦𝑂𝑂𝐴𝐴 = 𝑥𝑥 4 Dari persamaan YOA=(¾)X ini, berapa gradien garis YOA, 𝐺𝐺𝑎𝑎𝑎𝑎𝑎𝑎𝑎𝑎 𝑂𝑂𝐴𝐴 ⇔
Denan?”pancing Yodha. “Karena gradien merupakan perbandingan koefisien x dengan koefisien y, berarti ya ¾, Yodha!”jawab Denan. “Tepat. Jadi garis yOA memiliki gradien sebesar ¾,”tegas Yodha. “Dengan cara yang sama kita bisa menghitung persamaan Garis AB, dengan memisalkan A(4,3)=(x1,y1) dan B(7,-1) = (x2,y2). Sehingga: 𝐺𝐺𝑎𝑎𝑟𝑟𝑖𝑖𝑠𝑠 𝐴𝐴𝐵𝐵 ⇔
𝐺𝐺𝑎𝑎𝑎𝑎𝑎𝑎𝑎𝑎 𝐴𝐴𝐵𝐵𝑜𝑜
𝑦𝑦 − 𝑦𝑦1 𝑥𝑥 − 𝑥𝑥1 = 𝑦𝑦2 − 𝑦𝑦1 𝑥𝑥2 − 𝑥𝑥1
𝑥𝑥 − 4 𝑦𝑦 − 3 = −1 − 3 7 − 4
𝐺𝐺𝑎𝑎𝑎𝑎𝑎𝑎𝑎𝑎 𝐴𝐴𝐵𝐵 ⇔ 3(𝑦𝑦 − 3) = (−4)(𝑥𝑥 − 4)
𝐺𝐺𝑎𝑎𝑎𝑟𝑟𝑖𝑖𝑠𝑠 𝐴𝐴𝐵𝐵 ⇔ 3𝑦𝑦 − 9 = −4𝑥𝑥 + 16
𝐺𝐺𝑎𝑎𝑎𝑎𝑎𝑎𝑎𝑎 𝐴𝐴𝐵𝐵 ⇔ 3𝑦𝑦 = −4𝑥𝑥 + 16 + 9 4 25 𝐺𝐺𝑎𝑎𝑎𝑎𝑎𝑎𝑎𝑎 𝐴𝐴𝐵𝐵 ⇔ 𝑦𝑦 = − 𝑥𝑥 + 3 3
Kawan-kawan, kita sudah punya 2 garis lurus, yang berarti pula bahwa kita memiliki 2 gradien atau kemiringan garis. Seperti tadi aku sebutkan bahwa dua garis garis lurus dikatakan saling tegak lurus jika perkalian antara gradien garisnya sama dengan -1, maka kita bisa menguji kedua garis itu, sehingga: | 428
3 3 𝐺𝐺𝑎𝑎𝑎𝑎𝑎𝑎𝑎𝑎 𝑂𝑂𝐴𝐴 ⇔ 𝑌𝑌𝑂𝑂𝐴𝐴 = 𝑋𝑋 ⇔ 𝑚𝑚𝑂𝑂𝐴𝐴 = 4 4 4 25 4 ⇔ 𝑚𝑚𝐴𝐴𝐵𝐵 = − 𝐺𝐺𝑎𝑎𝑎𝑎𝑎𝑎𝑎𝑎 𝐴𝐴𝐵𝐵 ⇔ 𝑌𝑌𝐴𝐴𝐵𝐵 = − 𝑋𝑋 + 3 3 3
Jika kedua gradien garis kita kalikan, akan diperoleh: 3 4 𝑚𝑚𝑂𝑂𝐴𝐴 𝑥𝑥𝑚𝑚𝐴𝐴𝐵𝐵 = 𝑥𝑥 �− � 4 3
𝑚𝑚𝑂𝑂𝐴𝐴 𝑥𝑥𝑚𝑚𝐴𝐴𝐵𝐵 = −1
3 4
Sehingga dapat dipastikan bahwa garis 𝑌𝑌𝑂𝑂𝐴𝐴 = 𝑋𝑋 tegak lurus 4 3
terhadap garis 𝑌𝑌𝐴𝐴𝐵𝐵 = − 𝑋𝑋 +
25 ,”terang 3
Yodha bersemangat.
“Dan…”tiba-tiba Yodha menghentikan kalimatnya karena ada
suara yang memotong. “Dan kau ingin mengatakan selanjutnya kita bisa menentukan alas dan tinggi dari segiitiga yang akan ditentukan luasnya! Begitu, Yodha?”tebak Iman. “Tepat, Kawanku Iman! Karena kedua garis saling tegak lurus, kita bebas menentukan salah satu menjadi alas atau pun tinggi dari segitiga yang yang akan ditentukan luasnya,”terang Yodha. “Tapi, sebelum sampai pada hal itu, perlu disebutkan bahwa segitiga OAB yang terbentuk adalah segitiga siku-siku di...”kalimat Yodha terhenti. “Siku-siku di A! Karena garis OA dan garis AB saling tegak lurus. Bukankah begitu, Yodha!”sela Iman “Exactly, my Brother!”balas Yodha sambil melemparkan senyum
| 429
Ada perasaan bahagia dalam diri Yodha. Ia bahagia karena kawan-kawannya bisa menerima dan memahami dengan baik apa yang disampaikannya. “Kawan-kawan, sebelum kita menentukan luas segitiga itu, satu hal yang harus kita selesaikan, setelah kita mengetahui 2 garis yang tegak lurus dan menentukan kedua garis itu sebagai alas dan tinggi, adalah…”Yodha sengaja berhenti. Ia menarik nafas panjang. Seperti menunggu sesuatu. “Mencari dan menemukan berapa panjang kedua garis itu, yang akan ditetapkan sebagai alas dan tinggi pada segitiga adalah langkah berikutnya. Bukankah begitu, Yodha?”tiba-tiba Ikram menyela. “Tepat kawanku Ikram. Dan untuk mengetahui panjangnya, kita kembali ingat pada pelajaran tentang mencari jarak antara dua titik, Kawan-kawan!”terang Yodha. Kawan-kawan Yodha saling pandang. Mereka senang dengan cara Yodha menjelaskan step by step. “Ada yang masih ingat, Kawan-kawan?”tanya Yodha. “Jika diketahi koordinat dari titik-titik A dan B secara berturut-turut adalah (x1, y1) dan (x2, y2), maka jarak antara titik A dan titik B atau panjang garis AB adalah: 𝐽𝐽𝑎𝑎𝑎𝑎𝑎𝑎𝑎𝑎 𝐷𝐷𝑢𝑢𝑢𝑢 𝑇𝑇𝑖𝑖𝑖𝑖𝑖𝑖𝑖𝑖 = �(𝑥𝑥2 − 𝑥𝑥1 )2 + (𝑦𝑦2 − 𝑦𝑦1 )2
Begitukan, Yodha!”tiba-tiba terdengar suara Bintang.
“Tepat sekali, Kawan. Sehingga bila kita terapkan untuk menghitung jarak antara titik O dan titik A yang berarti juga panjang Garis OA, dan jarak antara titik A dan titik B yang berarti juga panjang Garis AB, maka: | 430
𝐽𝐽𝑎𝑎𝑎𝑎𝑎𝑎𝑎𝑎 𝑂𝑂𝐴𝐴 = �(4 − 0)2 + (3 − 0)2
𝐽𝐽𝑎𝑎𝑎𝑎𝑎𝑎𝑎𝑎 Ǥ𝑂𝑂ǤǤ𝐴𝐴 = √25
𝐽𝐽𝑎𝑎𝑎𝑎𝑎𝑎𝑎𝑎 𝑂𝑂𝐴𝐴 = 5 𝑠𝑠𝑎𝑎𝑎𝑎𝑎𝑎𝑎𝑎𝑎𝑎 Ϭ𝑎𝑎𝑛𝑛𝑛𝑛𝑛𝑛𝑛𝑛𝑛𝑛
𝐽𝐽𝑎𝑎𝑎𝑎𝑎𝑎𝑎𝑎 𝐴𝐴𝐵𝐵 = �(7 − 4)2 + (−1 − 3)2 𝐽𝐽𝑎𝑎𝑎𝑎𝑎𝑎𝑎𝑎 𝐴𝐴𝐵𝐵 = √25
𝐽𝐽𝑎𝑎𝑟𝑟𝑎𝑎𝑎𝑎 𝐴𝐴𝐵𝐵 = 5 𝑠𝑠𝑎𝑎𝑎𝑎𝑎𝑎𝑎𝑎𝑎𝑎 𝑝𝑝𝑎𝑎𝑎𝑎𝑎𝑎𝑎𝑎𝑎𝑎𝑎𝑎
Sehingga, pada akhirnya terbentuk segitiga yang bukan hanya sikusiku di A tetapi juga sama kaki, sebagai berikut:
Dan dapat dihitung luasnya menjadi: 1 𝐿𝐿𝑢𝑢𝑢𝑢𝑢𝑢 ∆𝐵𝐵𝐴𝐴𝐴𝐴 = 𝑥𝑥5𝑥𝑥5 2
𝐿𝐿𝑢𝑢𝑢𝑢𝑢𝑢 ∆𝐵𝐵𝐴𝐴𝐴𝐴 = 12,5 𝑠𝑠𝑎𝑎𝑎𝑎𝑎𝑎𝑎𝑎𝑎𝑎 𝑙𝑙𝑢𝑢𝑢𝑢𝑢𝑢
Kawan-kawan itulah hasil akhir dari perhitungan luas segitiga dengan
cara
normal,
menghitung
semua
variabel
yang
diperlukan,”kata Yodha. “Penghitungan dengan cara normal, katamu Yodha? Apakah ada cara yang tak-normal untuk menemukan luas sebuah segitiga yang hanya diketahui titik-titik sudutnya?”kejar Ayra. “Ya, bukan cara tak-normal. Tapi cara lain alias jalan berbeda! seperti tadi tadi aku katakan ada beberapa cara untuk menentukan luas suatu segitiga. Dalam hal persoalan ini, yang hanya diberikan | 431
koordinat masing-masing titik-titiknya, kita bisa menggunakan cara penghitungan luas segitiga dengan bantuan Determinan atau cross vector,”terang Yodha. “Konkretnya, seperti apa Kawan?”tanya Ikram. “Konkretnya dipenuhi dengan prinsip aturan perkalian tertutup seperti ini, ‘jika A (x1,y1), B(x2,y2), dan C(x3,y3) adalah titik-titik koordinat dimana A, B, C tidak terletak segaris (non collinear), maka luas segitiga ABC itu memenuhi persamaan: 1 𝐿𝐿𝑢𝑢𝑢𝑢𝑢𝑢 ∆𝐴𝐴𝐵𝐵𝐵𝐵 = |(𝑥𝑥1 𝑦𝑦2 + 𝑥𝑥2 𝑦𝑦3 + 𝑥𝑥3 𝑦𝑦1 ) − (𝑥𝑥2 𝑦𝑦1 + 𝑥𝑥3 𝑦𝑦2 + 𝑥𝑥1 𝑦𝑦3 )| 2
Dan aturan perkalian tertutup itu, digambarkan dalam bentuk seperti gambar berikut,”terang Yodha sambil melukiskan sesuatu di papan tulis.
“Wah, tampaknya akan sederhana, Kawan. Tapi, apakah hasilnya akan sama, Yodha?”tanya Ayra. “Coba saja substitusikan dan operasikan, Ayra!”pancing Yodha. Ayra maju ke depan, dia mulai mengoperasikan perkalian tertutup titik-titik segitiga untuk menghitung luas segitiga yang terbentuk. “Kalau titik O(0,0), A(4,3), dan B(7,-1) kita masukkan ke dalam operasi perkalian tertutup, maka: | 432
1 𝐿𝐿𝑢𝑢𝑢𝑢𝑢𝑢 ∆𝑂𝑂𝐴𝐴𝐵𝐵 = |(𝑥𝑥𝑜𝑜 𝑦𝑦𝑎𝑎 + 𝑥𝑥𝑎𝑎 𝑦𝑦𝑏𝑏 + 𝑥𝑥𝑏𝑏 𝑦𝑦𝑜𝑜 ) − (𝑥𝑥𝑎𝑎 𝑦𝑦0 + 𝑥𝑥𝑏𝑏 𝑦𝑦𝑎𝑎 + 𝑥𝑥0 𝑦𝑦𝑏𝑏 )| 2 1 𝐿𝐿𝑢𝑢𝑎𝑎𝑎𝑎 ∆𝑂𝑂𝐴𝐴𝐴𝐴 = |(0.3 + 4(−1) + 7.0) − (4.0 + 7(3) + 0. (−1))| 2 1 𝐿𝐿𝑢𝑢𝑢𝑢𝑢𝑢 ∆𝑂𝑂𝐴𝐴𝐴𝐴 = |(−4) − (21)| 2 1 𝐿𝐿𝑢𝑢𝑢𝑢𝑢𝑢 ∆𝑂𝑂𝐴𝐴𝐴𝐴 = |−25| 2 𝐿𝐿𝑢𝑢𝑢𝑢𝑢𝑢 ∆𝑂𝑂𝐴𝐴𝐴𝐴 = −12,5
Tapi, kenapa hasilnya negatif 12,5 Yodha?”Ayra heran. “Hitunganmu sudah tepat Ayra! Tanda 2 garis tegak yang membatasi operasi di atas itu, | |, berarti memositifkan semua
operasi di dalamnya, memutlakan bahasa lainnya. Sehingga, perhitunganmu seharusnya adalah: 1 𝐿𝐿𝑢𝑢𝑢𝑢𝑢𝑢 ∆𝑂𝑂𝐴𝐴𝐴𝐴 = |−25| 2 25 = 12,5 𝑠𝑠𝑎𝑎𝑎𝑎𝑎𝑎𝑎𝑎𝑎𝑎 𝑙𝑙𝑢𝑢𝑎𝑎𝑎𝑎 𝐿𝐿𝑢𝑢𝑢𝑢𝑢𝑢 ∆𝑂𝑂𝐴𝐴𝐴𝐴 = 2 Terlihat bahwa baik melalui perhitungan cara biasa yang konvensional, maupun dengan perkalian tertutup determinan, hasilnya menunjukan bahwa luas segitiga yang dicari adalah sama, yaitu 12,5 satuan luas. Begitu penjelasanku, Kawan-kawan,”pungkas Yodha. “Plok, plok, plok, plok, plok, plok, plok,”terdengar tepuk tangan bersahutan dari teman-teman Yodha. Sabiq, Fateh, Ikram dan Ayra tersenyum puas dan ada rasa bangga terpancar dari wajah mereka. “Tinggal 2 Lontar lagi. Bagaimana kawan-kawan?”tanya Sabiq | 433
“Kita selesaikan saja, Biq. Mumpung belum terlalu siang. Mudah-mudhan sebelum Dzuhur kita bisa menyelesaikan semua persoalan dalam Lontar-Lontar ini,”usul Fateh. “Baiklah kalau begitu. Lontar yang mana sekarang?”tanya Sabiq lagi. “Lontar kami saja, Biq”kata Saver yang didampingi oleh Bintang. “Oke, silahkan bacakan persoalan dalam Lontar kalian, Kawan!”pinta Sabiq. “Kami mendapatkan Lontar yang agak lebar. Karenanya, ada 4 persoalan tertulis disana. Dari keempat soal itu, aku dan Bintang hanya bisa memahami 2 persoalan, sementara 2 persoalan lainnya kami belum paham, Kawan-kawan,”buka Saver. Kawan-kawannya serius mendengar. “Tak apa, sampaikan saja keempat soal itu, yang bisa kalian pahami tolong jelaskan kepada kita semua, sementara 2 soal lainnya, mungkin
ada
diantara
kita
yang
bisa
membantu
untuk
sharing,”jawab Amer. Kawan-kawan lainnya memandang ke arah Amer. Tak biasanya bahasanya lemah lembut dan tenang. Hmm…. “Baik ini keempat persoalan itu: Pertama, 16 dapat dinyatakan sebagai 3x + 7y sebab jika x diganti dengan 3 dan y diganti dengan 1 diperoleh 3.3+7.1 yang bernilai 16. Tujuh bilangan antara 100 dan 122 yang dapat dinyatakan ke dalam bentuk 6x +9y adalah ....’; Kedua, H adalah himpunan yang didefinisikan oleh {x Ɛ B │x2 < 10, x -1 < 2} dengan B adalah himpunan bilangan Bulat. Banyaknya himpunan bagian yang tak | 434
kosong dari H adalah ’ Ketiga, Bilangan 45 dapat dinyatakan sebagai selisih dari bilangan kuadrat a2 – b2 dengan a dan b adalah bilangan asli. Semua pasangan bilangan asli a dan b yang memenuhi persamaan a2 – b2 = 45 adalah ...’ dan Keempat, ‘pada suatu kubus ABCD.EFGH, ruas garis AG adalah diagonal ruang dari kubus tersebut. Ada berapa carakah perjalanan terpendek dari titik sudut G ke titik sudut A dengan syarat perjalanan tersebut hanya melalui rusuk-rusuk kubus tanpa ada yang dilalui lebih dari satu kali?’ Itu keempat persoalan yang tertera di Lontar kami,”terang Saver lagi. “Bagian yang mana bisa kalian jelaskan, Kawan?”tanya Denan. “Persoalan pertama dan keempat, Nan!”jawab Saver. Bintang maju mendekati papan tulis. Ia mengambil alat tulis dan mulai melukiskan sesuatu di papan tulis. “Aku mengerjakan soal keempat. Jika informasi pada soal itu kita tuangkan dalam bentuk gambar, akan diperoleh gambaran seperti ini!”terang Bintang.
“Kawan-kawan berdasar persayaratan perjalanan tersebut hanya melalui rusuk-rusuk kubus tanpa ada yang dilalui lebih dari satu kali, maka dari titik G ada tiga jalur yang dapat dilalui yaitu melalui titik C, F, dan H, seperti ditunjukkan oleh arah panah. | 435
Sedangkan dari masing-masing 3 titik tersebut, ada 2 jalur yang dapat ditempuh, yaitu: (1) Dari titik H bisa melalui D atau melalui E; (2) Dari titik C bisa melalui D atau melalui B; (3) Dari titik F bisa melalui E atau melalui B; Sehingga banyaknya cara perjalanan terpendek dari titik G ke A sebanyak 3 x 2 = 6 cara. Dalam hal ini, terdapat Aturan Perkalain yang menyebutkan bahwa: ‘jika suatu peristiwa terjadi dengan m cara yang berbeda dan ada peristiwa lain terjadi dengan n cara yang berbeda maka kedua peristiwa itu dapat terjadi dengan: (m × n) cara yang berbeda. Sehingga, cara yang ditempuh itu adalah: (1) GCBA; (2) GCDA; (3) GFBA; (4) GFEA; (5) GHEA; (6) GHDA. Dan selanjutnya, bila kita gambarkan jalur yang ditempuh, akan terlihat sebagai berikut:
| 436
Demikian penjelasanku, Kawan-kawan,”pungkas Bintang. “Subhanallah! Indah dan lengkap sekali, Kawan!”puji Fateh. “Terima kasih, Fateh!”balas Bintang. Selanjutnya adalah giliran Saver. ia pun mengambil posisi dekat papan tulis. “Kawan-kawan, inti dari persoalan yang ditanya pada peroalanku adalah terdapat pada frasa ‘tujuh bilangan antara 100 dan 122 yang dapat dinyatakan ke dalam bentuk 6x +9y ... Karenanya, kita persempit persoalan pada itu saja,”kalimat pembuka Saver. “Bila bentuk 6x + 9y kita sederhanakan kembali,dapat ditulis ualang menjadi: 6𝑥𝑥 + 9𝑦𝑦 = 3(2𝑥𝑥 + 3𝑦𝑦)
Bentuk di atas mengandung arti bahwa bentuk 6x + 9y merupakan kelipatan dari 3. Selanjutnya, Bila kita misalkan bahwa: 6𝑥𝑥 + 9𝑦𝑦 = 3(2𝑥𝑥 + 3𝑦𝑦) = 𝑀𝑀
⇔ 𝑀𝑀 = 3𝑘𝑘
Dengan memerhatikan frasa, ‘tujuh bilangan antara 100 dan 122 yang dapat dinyatakan ke dalam bentuk 6x +9y..., maka dapat dituliskan pertidaksamaan: 100 < 𝑀𝑀 < 122
100 < 3𝑘𝑘 < 122
100 122 < 𝑘𝑘 < 3 3 1 2 33 < 𝑘𝑘 < 40 3 3
| 437
Sehingga nilai k dapat disebutkan dalam bahasa yang sederhana adalah bilangan bulat positif yang lebih besar dari 33 tapi kurang dari 41. Sehingga yang memungkinkan adalah: 𝑘𝑘 = {34, 35, 36, 37, 38, 39, 40}
Karenanya:
(1) Untuk k=34, diperoleh M = 3k = 102, dan 102 dapat dinyatakan dalam 6x + 9y sebagai 6. 2 + 9. 10 = 102; (2) Untuk k=35, diperoleh M = 3k = 105, dan 105 dapat dinyatakan dalam 6x + 9y sebagai 6. 10 + 9. 5 = 105; (3) Untuk k=36 , diperoleh M = 3k = 108, dan 108 dapat dinyatakan dalam 6x + 9y sebagai 6. 3 + 9. 10 = 108; (4) Untuk k=37, diperoleh M = 3k = 111, dan 111 dapat dinyatakan dalam 6x + 9y sebagai 6. 5 + 9. 9 = 111; (5) Untuk k=38, diperoleh M = 3k = 114, dan 114 dapat dinyatakan dalam 6x + 9y sebagai 6. 4 + 9. 10 = 114; (6) Untuk k=39, diperoleh M = 3k = 117, dan 117 dapat dinyatakan dalam 6x + 9y sebagai 6. 15 + 9. 3 = 117; (7) Untuk k=40, diperoleh M = 3k = 120, dan 120 dapat dinyatakan dalam 6x + 9y sebagai 6. 14 + 9. 4 = 120 . Jadi, tujuh bilangan antara 100 dan 122 yang dapat dinyatakan ke dalam bentuk 6x +9y adalah 102, 105, 108, 111, 114, 117, dan 120,”terang Saver menyudahi penjelasannya. Terdengar applause meriah dari Kawan-kawan Saver. “Satu hal yang kau luput, Kawan!”tiba-tiba Bily yang sedari tadi diam berkomentar. “Apa itu, Bily?”tanya Saver. | 438
“Akan konsisten bila kau merujuk pada apa yang sebelumnya kau terangkan bahwa 6x + 9y merupakan kelipatan dari 3. Apakah ketujuh bilangan bulat yang kau peroleh itu sudah kau pastikan merupakan kelipatan dari 3, Kawan?”tanya Bily. “Kalau boleh aku sebut, ya! Ketujuh bilangan bulat itu adalah bilangan yang merupakan kelipatan 3, Kawan!”Bily menjawab sendiri pertanyaannya. “Tapi begaiamana kita tahu, Kawan?”sambung Denan. “Bukankah salah satu ciri suatu bilangan habis dibagi 3, atau 3 merupakan salah satu faktor daris uatu bilangan, adalah apabila jumlah angka-angkanya habis dibagi 3. Karenanya, 102, 105, 108, 111, 114, 117, dan 120, merupakan bilangan bulat yang habis dibagi 3, karena jumlah masing-masing angkanya berturut-turut adalah: 102 = 34 3 105 = 35 105 = (1 + 0 + 5 = 6) ⇔ 3 108 = 36 108 = (1 + 0 + 8 = 9) ⇔ 3 111 111 = (1 + 1 + 1 = 3) ⇔ = 37 3 114 = 38 114 = (1 + 1 + 4 = 6) ⇔ 3 117 117 = (1 + 1 + 7 = 9) ⇔ = 39 3 120 = 40 120 = (1 + 2 + 0 = 3) ⇔ 3
102 = (1 + 0 + 2 = 3) ⇔
⇔ 102 = 3𝑥𝑥34 ⇔ 105 = 3𝑥𝑥35 ⇔ 108 = 3𝑥𝑥36 ⇔ 111 = 3𝑥𝑥37 ⇔ 114 = 3𝑥𝑥38 ⇔ 117 = 3𝑥𝑥39 ⇔ 120 = 3𝑥𝑥40
| 439
Terlihat bahwa hasil penjumlahan angka-angknya adalah 3, 6, 9, 3, 6, 9, yang semuanya habis dibagi oleh 3. Itu tambahan dariku, Kawan!”terang Bily. Kembali tepuk tangan meriah dan hangat terdengar. “Oke. dua dari 4 soal pada Lontar pada Bintang dan Saver telah terjawab. Apakah ada yang ingin mengerjakan nomor 2 atau nomor 3?”tantang Sabiq. “Aku selesaikan nomor 2, Biq!”kata Bily mantap. “Silahkan, Bil!”balas Sabiq. Bily tak sempat undur diri. Dia masih di depan. Dan kali ini memang permintaannya. “Terhadap persoalan yang menyatakan bahwa, ‘H adalah himpunan yang didefinisikan oleh {x Ɛ B │x2 < 10, x -1 < 2 } dengan B adalah himpunan bilangan Bulat. Banyaknya himpunan bagian yang tak kosong dari H adalah …’. Soal ini sekurang-kurangnya menyimpan satu kata kunci. Dan kata kunci itu adalah, ‘himpunan bagian yang tak kosong’,”terang Bily. “Bila pertidaksamaan kita selesaikan, maka: {𝑥𝑥 Ɛ 𝐵𝐵 │𝑥𝑥 2 < 10, 𝑥𝑥 − 1 < 2}
Kita selesaikan ruas kanan: 𝑥𝑥 < 2 + 1
𝑥𝑥 < 3 … … … … … … … … … … … … . . (16.6)
Selanjutnya kita selesaikan bagian kiri 𝑥𝑥 2 < 10
𝑥𝑥 2 − 10 < 0
�𝑥𝑥 + √10��𝑥𝑥 − √10� < 0 … … … … . (16.7)
| 440
Dari pertidaksamaan (16.6) dan (16.7), himpunan bilangan Bulat yang memenuhi adalah seperti pada gambaran garis bilangan berikut:
Sehingga anggota himpunan H yang memenuhi adalah {-3, -2, -1, 0, 1, 2 } ⇔ n(H) = 6. Karenanya:
𝐵𝐵𝑎𝑎𝑎𝑎𝑎𝑎𝑎𝑎𝑘𝑘𝑛𝑛𝑛𝑛𝑛𝑛 ℎ𝑖𝑖𝑚𝑚𝑚𝑚𝑚𝑚𝑚𝑚𝑚𝑚𝑚𝑚 𝑏𝑏𝑎𝑎𝑔𝑔𝑔𝑔𝑔𝑔𝑔𝑔 𝑑𝑑𝑎𝑎𝑎𝑎𝑎𝑎 𝐻𝐻 = 2𝑛𝑛(𝐻𝐻) 2𝑛𝑛(𝐻𝐻) = 26 = 64
Tapi karena ada frasa ‘yang tak kosong’, maka banyaknya himpunan bagian yang tak kosong dari H adalah: 2𝑛𝑛(𝐻𝐻) − 1 = 26 − 1
= 64 − 1 = 63,
Itu penjelasanku, Kawan-kawan,”tutup Bily. “Alhamdulillah, tinggal satu persoalan pada Lontar Saver dan Bintang. Apakah ada yang bersedia menyelesaikannya?”tanya Sabiq. “Biar aku coba selesaikan, Biq!”terdengar suara Fateh pelan. Fateh langsung ke depan menuju papan tulis. Ia menuliskan sesuatu di sana. Kemudian dia berbalik, menghadap kawankawannya. “Kawan-kawan, soal ini sepintas akan merepotkan. Apalagi dilakukan dengan coba-coba. Tapi, dengan mengingat sifat dari selisih kuadrat, kita akan sangat tertolong,”Fateh membuka keterangannya. “Kalau kita perhatikan bahwa disebutkan, ‘bilangan 45 dapat dinyatakan sebagai selisih dari bilangan kuadrat a2 – b2 dengan a | 441
dan b adalah bilangan asli. Semua pasangan bilangan asli a dan b yang memenuhi persamaan a2 – b2 = 45 adalah...’. Kata kuncinya pada pada bentuk a2 – b2! Sehingga dengan mengingat bahwa: 𝑎𝑎2 − 𝑏𝑏 2 = (𝑎𝑎 + 𝑏𝑏)(𝑎𝑎 − 𝑏𝑏)
Sehingga persamaan soalannya dapat dituliskan menjadi: 𝑎𝑎2 − 𝑏𝑏 2 = (𝑎𝑎 + 𝑏𝑏)(𝑎𝑎 − 𝑏𝑏) = 45 (𝑎𝑎 + 𝑏𝑏)(𝑎𝑎 − 𝑏𝑏) = 45
Persamaan terakhir ini memberi arti bahwa (a + b) dan (a – b) merupakan sepasang faktor-faktor yang tentu dari 45, dimana (a + b) > ( a – b ) > 0. Sepasang faktor-faktor dari 45 itu adalah (45, 1), (9,5) dan (15, 3), sehingga terdapat tiga pasang (a, b) yang memenuhi persamaan di atas. Karenanya, Untuk faktor (45,1); 𝑎𝑎 + 𝑏𝑏 = 45
𝑎𝑎 − 𝑏𝑏 = 1 +
2𝑎𝑎 = 46 ⇔ 𝑎𝑎 = 23; 𝑏𝑏 = 22
Untuk faktor (9,5) 𝑎𝑎 + 𝑏𝑏 = 9
𝑎𝑎 − 𝑏𝑏 = 5 +
2𝑎𝑎 = 14 ⇔ 𝑎𝑎 = 7; 𝑏𝑏 = 2
Untuk faktor (15,3) 𝑎𝑎 + 𝑏𝑏 = 15
𝑎𝑎 − 𝑏𝑏 = 3 +
2𝑎𝑎 = 18 ⇔ 𝑎𝑎 = 9; 𝑏𝑏 = 6
Karenanya, pasangan (a, b) yang memenuhi persamaan soal adalah (23, 22), (7, 2) dan (9, 6), Kawan-kawan!”tutup Fateh. | 442
“Subhanalah! Kita sudah menyelesaikan soal-soal yang diajukan dan ditulis di Lontar-Lontar. Tinggal yang ada di Lontarku yang belum terselesaikan. Ada usul?”tanya Sabiq “Kami berharap kau yang jelaskan pada kami tentang soal-soal yang ada di Lontarmu itu, Biq?”usul Ikram dan diamini oleh Yodha dan Fateh. Setidaknya dalam bentuk anggukan setuju. “Begini saja, aku mulai kerjakan nomor 1 dulu. Nomor selanjutnya, bila ada yang bersedia atau mau melatih kemampuan, silahkan saja,”balas Sabiq. Seakan tak mau menyia-nyiakan waktu, Sabiq langsung menyambar alat tulis dan menggambar beberapa coretan di papan tulis. “Kawan-kawan ini adalah soal pertama di Lontar yang diletakan di pintu bagian dalam kamarku itu. ‘Diberikan bentuk suku ke-n sebagai berikut: 1 2 1 2 𝑎𝑎𝑛𝑛 = �1 + �1 + � + �1 + �1 − � , 𝑛𝑛 ≥ 1 𝑛𝑛 𝑛𝑛
Hitung dan buktikan bahwa
1 1 1 1 + + + ⋯+ 𝑎𝑎20 𝑎𝑎1 𝑎𝑎2 𝑎𝑎3
merupakan bilangan bulat!”kata Sabiq lagi. “Terhadap bentuk soal seperti ini, kita harus mengupayakan bentuk sederhananya, kawan-kawan. Yaitu, karena bentukya adalah penjumlahan
dua
bilangan
di
bawah
tanda
akar,
maka
penyederhanaannya dengan mengalikan bilangan itu dengan sekawan atau conjugate-nya,”kata Sabiq. Kemudin Sabiq membuat beberapa coretan lagi di papan tulis. | 443
“Karenanya, bila kita lakukan perkalian sekawan, akan terlihat bahwa bentuk an akan menjadi: 2
2
1 1 ��1 + �1 + � + �1 + �1 − � � 𝑛𝑛 𝑛𝑛
1 2 1 2 ��1 + �1 + � − �1 + �1 − � � 𝑛𝑛 𝑛𝑛 1 2 1 2 ��1 + �1 + � − �1 + �1 − � � 𝑛𝑛 𝑛𝑛
Dengan mengingat sifat perkalian bahwa: (𝑎𝑎 + 𝑏𝑏)(𝑎𝑎 − 𝑏𝑏) = 𝐶𝐶 2 − 𝑏𝑏 2
Maka bentuk,
1 2 1 2 1 2 1 2 ��1 + �1 + 𝑛𝑛� + �1 + �1 − 𝑛𝑛� � 𝑥𝑥 ��1 + �1 + 𝑛𝑛� − �1 + �1 − 𝑛𝑛� � 2 2 �1 + �1 + 1 � − �1 + �1 − 1 � 𝑛𝑛 𝑛𝑛
Bisa menjadi lebih sederhana, yaitu:
= = =
1 2 1 2 �1 + �1 + 𝑛𝑛� � �1 + �1 − 𝑛𝑛� �
2 2 �1 + �1 + 1 � − �1 + �1 − 1 � 𝑛𝑛 𝑛𝑛
1 2 1 2 �1 + �1 + 𝑛𝑛� � − �1 + �1 − 𝑛𝑛� �
2 2 �1 + �1 + 1 � − �1 + �1 − 1 � 𝑛𝑛 𝑛𝑛 2 1 2 1 �1 + (1 + + 2 )� − �1 + (1 − + 2 )� 𝑛𝑛 𝑛𝑛 𝑛𝑛 𝑛𝑛
2 2 �1 + �1 + 1 � − �1 + �1 − 1 � 𝑛𝑛 𝑛𝑛 4 𝑛𝑛 = 2 2 �1 + �1 + 1 � − �1 + �1 − 1 � 𝑛𝑛 𝑛𝑛
Akhirnya,
| 444
𝑛𝑛 1 2 1 2 1 � � = � 1 + �1 + � − 1 + �1 − � � 𝑛𝑛 𝑛𝑛 𝑎𝑎𝑛𝑛 4 1 2 𝑛𝑛
2 𝑛𝑛
Karena 1 + �1 + � = 1 + �1 + +
1 � 𝑛𝑛 2
= 2𝑛𝑛2 + 2𝑛𝑛 + 1, maka:
1 1 = ��2𝑛𝑛2 + 2𝑛𝑛 + 1 − �2𝑛𝑛2 − 2𝑛𝑛 + 1� 𝑎𝑎𝑛𝑛 4
Bila masing-masing nilai a1, a2, a3,..., a20 dimasukkan, diperoleh: 1 1 = ��2(1)2 + 2(1) + 1 − �2(1)2 − 2(1) + 1� Ѻ1 4 1 = (√5 − √1) 4
1 1 = (�2(2)2 + 2(2) + 1 − �2(2)2 − 2(2) + 1) 𝑎𝑎2 4 1 = (√13 − √5) 4
1 1 = (�2(3)2 + 2(3) + 1 − �2(3)2 − 2(3) + 1) 𝑎𝑎3 4 1 = (√25 − √13) 4
1 1 = (�2(4)2 + 2(4) + 1 − �2(4)2 − 2(4) + 1) 𝑎𝑎4 4 ...
1 = (√41 − √25) 4
1 1 1 = �√800 + 40 + 1 − √800 − 40 + 1� = �√841 − √761� 4 𝑎𝑎20 4
Sehingga, jika dijumlahkan akan menghasilkan suku yang saling menghilangkan sedemikian rupa: 1 1 1 1 1 + + +. . + = (√841 − √1) 𝑎𝑎20 4 𝑎𝑎1 𝑎𝑎2 𝑎𝑎3
| 445
1 1 1 1 1 + + +. . + = (29 − 1) 𝑎𝑎20 4 𝑎𝑎1 𝑎𝑎2 𝑎𝑎3
1 1 1 28 1 + + +. . + = =7 𝑎𝑎20 4 𝑎𝑎1 𝑎𝑎2 𝑎𝑎3
Karenanya, terbukti bahwa hasil penjumlahan merupakan bilangan bulat yang nilainya adalah 7. Begitu yang bisa aku jelaskan, Kawankawan,”tutup Sabiq. “Fantastik! Indah sekali jalan penyelesaianmu, Kawan!”puji Amer. “Terima kasih, Mer!”balas Sabiq sambil tersenyum. “Bagaimana
dengan
soal
kedua
dalam
Lontar
itu,
Kawan?”tanya Ayra. “Masih soal pembuktian perkalian pecahan yang berurutan, Ayra”jawab Sabiq. “Atau
mungkin
ada
di
antara
kita
yang
mau
membantu?”pancing Sabiq. “Bukannya masih bagianmu, Kawan!”tegas Amer. “Ya, mungkin saja ada yang mau mencoba,”tawar Sabiq. “Tergantung soalnya, Biq!”tiba-tiba Fateh menyela. Sabiq lalu menulis di papan tulis bentuk dan apa yang diminta oleh soal kedua dalam Lontarnya. Semua kawannya saling pandang. Semua serius. Sampai tibatiba terdengar suara pelan. “Beri kesempatan aku untuk mencoba menyelesaikan soal itu, Biq!”terdengar kalimat optimis dari belakang, suara Iman. Sontak, semua mengarahkan pandangannya ke Iman. Sabiq dan Yodha hanya tersenyum. | 446
“Silahkan, Kawanku!”ucap Sabiq. Iman maju mendekat ke papan tulis. Ia berkonsenterasi. Dan mulai menuliskan beberapa coretan di di papan tulis. “Disebutkan
dalam
soal
bahwa
kita
diminta
untuk
membuktikan. Tertulis, ‘show that: 1 3 5 899 1 𝐴𝐴 = 𝑥𝑥 𝑥𝑥 𝑥𝑥 … 𝑥𝑥 𝑖𝑖𝑠𝑠 𝑠𝑠𝑚𝑚𝑚𝑚𝑚𝑚𝑚𝑚𝑚𝑚𝑚𝑚 𝑡𝑡ℎ𝑎𝑎𝑛𝑛 2 4 6 900 30 898 1 2 4 6 𝑖𝑖𝑠𝑠 𝑏𝑏𝑖𝑖𝑖𝑖𝑖𝑖𝑖𝑖𝑖𝑖 𝑡𝑡ℎ𝑎𝑎𝑛𝑛 𝐵𝐵 = 𝑥𝑥 𝑥𝑥 𝑥𝑥 … 𝑥𝑥 899 30 3 5 7 Terhadap persoalan ini, kita bisa melakukan
beberapa
pendekatan,”papar Iman memulai penjelasannya. “Jika kedua besaran A dan B kita kalikan, akan dihasilkan persamaan baru, sebagai: 𝐴𝐴𝑥𝑥𝑥𝑥 =
1𝑥𝑥2𝑥𝑥3𝑥𝑥4𝑥𝑥 … 𝑥𝑥898𝑥𝑥899 1 = 2𝑥𝑥3𝑥𝑥4𝑥𝑥 … 𝑥𝑥898𝑥𝑥899𝑥𝑥900 900
Selanjutnya, sebagaimana diketahui bahwa: 1 3 3 4 5 6 897 898 < ; < ; < … < 2 4 4 5 6 7 898 899
Karenanya, pastilah berlaku pula bentuk:
1 3 5 897 2 4 6 898 𝑥𝑥 𝑥𝑥 𝑥𝑥 … 𝑥𝑥 < 𝑥𝑥 𝑥𝑥 𝑥𝑥 … 𝑥𝑥 2 4 6 898 3 5 7 899
Dan juga berakibat pada:
900 900 1 3 5 899 𝐴𝐴 = � 𝑥𝑥 𝑥𝑥 𝑥𝑥 … 𝑥𝑥 � < 𝐵𝐵 899 899 2 4 6 900 900 𝐴𝐴 ⇔ > 899 Padahal, 900 𝐴𝐴 > 𝐴𝐴 899
| 447
Sehingga, 𝐵𝐵 > 𝐴𝐴 ⇔ 𝐴𝐴 < 𝐵𝐵
Selanjutnya, karena:
Maka,
𝐴𝐴𝑥𝑥𝑥𝑥 = 𝐴𝐴 <
1 𝑑𝑑𝑎𝑎𝑎𝑎 𝐴𝐴 < 𝐵𝐵 900
1 1 𝑑𝑑𝑎𝑎𝑎𝑎 𝐵𝐵 > 30 30
Akhirnya, terbuktilah apa yang diminta bahwa: 1 30 1 𝐵𝐵 𝑖𝑖𝑠𝑠 𝑏𝑏𝑖𝑖𝑖𝑖𝑖𝑖𝑖𝑖𝑖𝑖 𝑡𝑡ℎ𝑎𝑎𝑛𝑛 30
𝐴𝐴 𝑖𝑖𝑠𝑠 𝑠𝑠𝑚𝑚𝑚𝑚𝑚𝑚𝑚𝑚𝑚𝑚𝑚𝑚 𝑡𝑡ℎ𝑎𝑎𝑛𝑛
Kawan-kawan,”kalimat penutup Iman yang dibarengi dengan senyuman. “Wow, keren! Sederhana lugas, dan tepat sasaran!”komentar Amer. Beberapa kawan Iman tampak belum hilang kagetnya. Mereka seakan tak percaya Iman mampu secepat itu membuktikan yang diminta oleh soal. “Bagaimana bisa secepat itu, Man?”tanya Denan. “Hasil dari diskusi dan latihan kita selama ini, Kawan,”jawab Iman datar. Sabiq dan Yodha hanya senyum-senyum. Kali ini Ayra dan Fateh juga ikut tersenyum. Hanya Amer yang masih tampak heran. “Alhamdulillah, soal kedua dalam lontar itu telah terpecahkan. Dan ini adalah soal ketiga, Kawan-kawan. Disebutkan jelas dengan ilustrasi gambar sebagai berikut: ‘What is the sum of the | 448
circumferences and the areas of the infinite stack of circles inside the triangle in the picture below? Disertakan gambar seperti tertera di papan tulis,”kata Sabiq
Sabiq berhenti agak lama. Ia tak langsung melanjutkan. “Apakah ada kawan-kawan yang akan mengerjakan bagian ini? Yodha, kau…?”pancing Sabiq. “Tidak, Biq. Kami juga ingin melihat kau menyelesaikan soalsoal Geometri, Kawan!”balas Yodha sambil tersenyum. “Aku setuju betul dengan usulmu itu, Yodha!”terdengar Amer bicara keras. “Asyik juga melihat kemampuanmu dalam memecahkan persoalan Geometri, Biq”timpal Ikram. Ayra, Denan, Iman, Fateh, dan Bily pun mengangguk. Tampaknya mereka setuju dengan sikap Yodha. “Kalau semua sudah seperti itu, aku pun memang tak bisa menghindar lagi, Kawan-kawan,”tegas Sabiq. Sabiq bergerak kembali mendekat ke papan tulis. Kali ini dia menambahi beberapa garis dan titik pada gambar yang disebutkan oleh soal dalam Lontar itu.
| 449
“Kawan-kawan, gambar di atas merupakan modifikasi terhadap gambar dalam soal. Menurutku, inti dari pertanyaan soal di atas ada dua. Pertama, menghitung jumlah keliling lingkaran yang dapat disusun dalam segitiga, dan Kedua, menghitung luas keseluruhan dari lingkaran yang ada dalam segitiga tadi. Dan untuk itu kita mulai dari hal sederhana, yaitu tinggi segitiga,”terang Sabiq. “Dengan bantuan rumusan Phytagoras, kita bisa menghitung tinggi segitiga ABT, yaitu TR! Perhatikan ∆ART, maka akan terdapat hubungan: 𝐴𝐴𝑇𝑇 2 = 𝐴𝐴𝑅𝑅 2 + 𝑅𝑅𝑇𝑇 2
𝑅𝑅𝑇𝑇 2 = 𝐴𝐴𝑇𝑇 2 − 𝐴𝐴𝑅𝑅 2
Dengan mengingat bahwa panjang alas AR = ½ AB = 5, maka: 𝑅𝑅𝑇𝑇 2 = (13)2 − (5)2
𝑅𝑅𝑇𝑇 = �(13)2 − (5)2 𝑅𝑅𝑇𝑇 = 12
Selanjutnya kita bisa menentukan dan menetapkan bahwa jumlah panjang diameter dari seluruh lingkaran yang ada dalam segitiga itu adalah 12,”kata Sabiq. | 450
“Kenapa bisa begitu, Kawan?”tanya Amer. “Ya, perhatikan gambar tadi! Lingkaran yang terbentuk itu semua bersinggungan satu sama lain, dari yang paling besar sampai yang terkecil. Hal itu berarti bahwa diameter semua lingkaran saling terhubung dari dasar, alas segitiga, sampai puncak, yaitu tinggi segitiga. Dengan kata lain, diameter seluruh lingkaran yang terbentuk adalah sama dengan tinggi segitiga itu sendiri,”jelas Sabiq. “Ah, benar kau Biq,”jawab Amer. “Sehingga dengan mengingat bahwa keliling sebuah lingkaran adalah ditemukan dengan cara: 𝐾𝐾𝑒𝑒𝑒𝑒𝑒𝑒𝑒𝑒𝑒𝑒𝑒𝑒𝑒𝑒 = 2𝜋𝜋𝑥𝑥 𝑗𝑗𝑎𝑎𝑎𝑎𝑎𝑎 − 𝑗𝑗𝑎𝑎𝑎𝑎𝑎𝑎
dan,
maka,
𝐾𝐾𝑒𝑒𝑒𝑒𝑒𝑒𝑒𝑒𝑒𝑒𝑒𝑒𝑔𝑔 = 2𝜋𝜋𝑟𝑟
𝐷𝐷𝑖𝑖𝑖𝑖𝑖𝑖𝑖𝑖𝑖𝑖𝑖𝑖𝑖𝑖 (𝑑𝑑) = 2𝑥𝑥 𝑗𝑗𝑎𝑎𝑎𝑎𝑎𝑎 − 𝑗𝑗𝑎𝑎𝑎𝑎𝑎𝑎 = 2𝑟𝑟 𝐾𝐾𝑒𝑒𝑒𝑒𝑒𝑒𝑒𝑒𝑒𝑒𝑒𝑒𝑒𝑒 = 𝜋𝜋𝑑𝑑
𝐾𝐾𝑒𝑒𝑒𝑒𝑒𝑒𝑒𝑒𝑒𝑒𝑒𝑒𝑒𝑒 = 12𝜋𝜋
So, jawaban atas the sum of the circumferences of the infinite stack of circles inside the triangle adalah 12π, Kawan-kawan!”terang Sabiq. Satu persoalan terselesaikan. Sabiq masih berdiri tegak di samping papan tulis. Tampaknya dia akan kesulitan untuk tahap berikutnya, menemukan jumlah luas dari seluruh lingkaran dalam segitiga, yang kelilingnya sudah diketahui 12π itu. “Kawan-kawan, andai saja seluruh lingkarannya adalah sama besar dan bentuknya, dan saling bersinggungan dalam bentuk barisan, maka kita dengan mudah bisa menghitung luas seluruhnya | 451
setelah diketahui keliling dan diameternya. Tapi, …”Sabiq menghentikan kalimatnya. “Ya, kau benar, Biq. Luasnya akan sama dengan π(12/2)2 atau 36π. Tapi, ....tapi apa, Biq?”tanya Yodha serius. “Apakah kau tidak menemukan keanehannya, Yodha?”tanya Sabiq langsung. Sementara Kawan-kawan mereka pun serius. Kalau Yodha sampai serius bertanya tentang soal Geometri, itu berarti ada yang penting dalam persoalan itu. “Ya, keanehan itu misalnya jumlah luas lingkaran yang 36π satuan luas akan lebih besar daripada luas segitiga yang hanya 60 satuan luas, Kawan!”jawab Yodha. “Aku juga menemukan kesulitan yang sama, Kawankawan!”tiba-tiba Ayra menyela. “Begitupun aku!”terdengar suara Fateh. Ikram, Saver, Bily, Iman, Denan, dan yang lain juga mengangguk-anggukkan kepalanya. “Bukankah harusnya kita menemukan luasan masing-masing lingkaran, kemudian menjumlahkannya, Kawan?”kata Amer. “Tepat, Mer! Bisa kau beri tahu, dari mana jalan untuk memulainya?”tanya Sabiq. Amer menggeleng. “Mungkin dari lingkaran terbesar yang paling bawah, Kawan?”kata Bily. “Kita akan menghadapi persoalan menemukan jari-jarinya karena hanya setengah alas informasi yang diketahui, Bil!”jawab Yodha. | 452
Suasana jadi sangat serius. Dan keseriusan itu tampaknya sedari awal tak lepas dari sepasang mata yang terus mengamati aktivitas kesebelas anak itu. Sepasang mata dengan jenggot yang mulai memutih di dagu. Tapi, kali ini wajah pemilik jenggot itu tampak ceria. Ada kepuasan terpancar dari wajahnya. “Jadi, bagaimana kau memecahkannya, Biq?”kali ini Amer bertanya dengan suara khasnya dan agak keras. “Kita menggunakan jalan lain, jalan perantara, Kawan!”jawab Sabiq enteng. “Ada cara lain, maksudmu Kawan?”tanya Ayra. “Ya,
bukankah
pernah
aku
bilang
bahwa
banyak
jalan...”kalimat Sabiq terpotong, karena terdengar suara Amer menyela. “Banyak jalan menuju Mekkah, Kawan!”sela Amer. “Benar, Mer. Banyak jalan menuju Mekkah. Karenanya, akan banyak jalan untuk menemukan jumlah seluruh luas lingkaran dalam segitiga itu,”tegas Sabiq. Sabiq kembali menulis di papan tulis. Kali ini dia menuliskan persamaan dalam bentuk penjumlahan, dan... “Kawan-kawan, kalau kita perhatikan lebih teliti, maka segitiga itu dapat dipecah menjadi 3 buah segitiga yang menyusunnya,
yaitu: ∆ATO, ∆BTO,
dan ∆ABO,
sehingga
sesungguhnya: 𝐿𝐿𝑢𝑢𝑢𝑢𝑢𝑢 ∆𝐴𝐴𝑇𝑇𝑇𝑇 = 𝐿𝐿𝑢𝑢𝑢𝑢𝑢𝑢 ∆𝐴𝐴𝑇𝑇𝑇𝑇 + 𝐿𝐿𝑢𝑢𝑢𝑢𝑢𝑢 ∆𝐵𝐵𝑇𝑇𝑇𝑇 + 𝐿𝐿𝑢𝑢𝑢𝑢𝑢𝑢 ∆𝐴𝐴𝐵𝐵𝐵𝐵
Dimana,
1 𝐿𝐿𝑢𝑢𝑢𝑢𝑢𝑢 ∆𝐴𝐴𝑇𝑇𝑇𝑇 = (10)(12) = 60 𝑠𝑠𝑎𝑎𝑎𝑎𝑎𝑎𝑎𝑎𝑛𝑛 𝑙𝑙𝑢𝑢𝑢𝑢𝑢𝑢 2
| 453
1 𝐿𝐿𝑢𝑢𝑢𝑢𝑢𝑢 ∆𝐴𝐴𝑇𝑇𝑇𝑇 = (13)(𝑟𝑟) 2 1 𝐿𝐿𝑢𝑢𝑢𝑢𝑢𝑢 ∆𝐵𝐵𝑇𝑇𝑇𝑇 = (13)(𝑟𝑟) 2 1 𝐿𝐿𝑢𝑢𝑢𝑢𝑢𝑢 ∆𝐴𝐴𝐵𝐵𝐵𝐵 = (10)(𝑟𝑟) 2 karenanya, dapat kita susun ulang persamaan luasnya menjadi: 1 1 1 (13)(𝑟𝑟) + (13)(𝑟𝑟) + (10)(𝑟𝑟) = 60 2 2 2 18𝑟𝑟 = 60
10 3 Sehingga, kita bisa menghitung luas lingkaran besar, Lingkaran I, 𝑟𝑟 =
yang bersentuhan langsung dengan alas segitiga, yaitu: 𝐿𝐿𝑢𝑢𝑢𝑢𝑢𝑢 𝐿𝐿𝑖𝑖𝑖𝑖𝑖𝑖𝑖𝑖𝑖𝑖𝑖𝑖𝑖𝑖𝑖𝑖 𝐼𝐼 = 𝜋𝜋𝑟𝑟 2
10 2 𝐿𝐿𝑢𝑢𝑢𝑢𝑢𝑢 𝐿𝐿𝑖𝑖𝑖𝑖𝑖𝑖𝑖𝑖𝑖𝑖𝑖𝑖𝑖𝑖𝑖𝑖 𝐼𝐼 = 𝜋𝜋 � � 𝑠𝑠𝑎𝑎𝑎𝑎𝑎𝑎𝑎𝑎𝑎𝑎 𝑙𝑙𝑢𝑢𝑢𝑢𝑢𝑢 3
Itu nilai untuk luas lingkaran pertama. Bagaimana dengan yang kedua dan seterusnya?”tanya Sabiq tiba-tiba. Kawan-kawannya belum sempat mengapresiasi sudah timbul masalah berikutnya. “Ya, kembali kita gunakan cari pertama untuk menghitung jari-jari kedua, Biq!”jawab Ikram. “Ya, dengan begitu kita bisa menemukan luasan untuk lingkaran kedua, ketiga dan seterusnya, Kawan!”timpal Denan. “Bagaimana menurutmu, Man?”tanya Sabiq pada Iman. “Tidak gampang cari itu diterapkan kembali. Karena semua alas pada segitiga kedua itu tidak diketahui, Kawan-kawan!”jawab Iman. | 454
Semua yang mendengar jawaban Iman sontak kaget. Hanya Yodha dan Sabiq tampak senyum-senyum. “Tepat! Analisis yang akurat, Man. Untuk itulah, kemahiran Yodha yang paling tepat digunakan di sini,”kata Sabiq sambil melirik ke arah Yodha. Yodha hanya tersenyum dan menggeleng. “Kau harus selesaikan permainan ini sendiri, Kawan!”jawab Yodha singkat. “Baiklah Kawan-kawan, tampaknya kita harus menggunakan jalan lain, jalan perantara lagi, untuk menemukan jari-jari lingkaran berikutnya dan seterusnya,”kata Sabiq. Sabiq kembali ke papan tulis. Kali ini dia menulis tentang sesuatu yang sangat dikenal oleh Amer. “Tunggu Kawan! Kau sedang menuliskan sifat kesebangunan segitiga. Betulkan!”tanya Amer. “Tepat, Mer. Kita akan menggunakan sifat dasar Geometri khususnya tentang kesebangunan dalam segitiga, Kawan-kawan. Perhatikan gambar, terlihat bahwa ∆AOR sebangun dengan ∆CPQ. Sehingga:
Dan
𝑂𝑂𝑅𝑅 𝑃𝑃𝑄𝑄 = 𝐴𝐴𝑂𝑂 𝐶𝐶𝑃𝑃 𝐴𝐴𝑅𝑅 𝐶𝐶𝑄𝑄 = 𝐴𝐴𝑂𝑂 𝐶𝐶𝑃𝑃
Karenanya,
Dan
𝑟𝑟2 𝑟𝑟2 𝐶𝐶𝑃𝑃 𝑟𝑟 = ⇔ = … … … … … (16.8) 𝑟𝑟 𝐴𝐴𝑂𝑂 𝐴𝐴𝑂𝑂 𝐶𝐶𝑃𝑃 | 455
𝐶𝐶𝑃𝑃 𝐶𝐶𝑄𝑄 𝐴𝐴𝑅𝑅 𝐶𝐶〱 = ⇔ = … … … … … (16.9) 𝐶𝐶𝑃𝑃 𝐴𝐴𝑂𝑂 𝐴𝐴𝑅𝑅 𝐴𝐴𝑂𝑂 Dari kesebangunan (16.8) dan (16.9) akan diperoleh, bahwa: 𝑟𝑟2 𝐶𝐶𝑄𝑄 = … … … … … … … … … … . . (16.20) 𝐴𝐴𝑅𝑅 𝑟𝑟 Selanjutnya perhatikan kesebangunan berikutnya: ∆TCQ sebangun dengan ∆TAR, sehingga:
𝐶𝐶𝑄𝑄 𝐴𝐴𝑅𝑅 𝐶𝐶𝑄𝑄 𝑇𝑇𝑄𝑄 = ⇔ = … … … … . (16.21) 𝑇𝑇𝐶𝐶 𝑇𝑇𝑅𝑅 𝐴𝐴𝑅𝑅 𝑇𝑇𝑅𝑅 Akhirnya, dari (16.20) dan (16.21) kita peroleh: 𝑟𝑟2 𝑇𝑇𝑄𝑄 = 𝑇𝑇𝑅𝑅 𝑟𝑟 𝑇𝑇𝑄𝑄 𝑟𝑟2 = � � 𝑟𝑟 𝑇𝑇𝑅𝑅
Sementara,
𝑇𝑇𝑄𝑄 = 𝑇𝑇𝑅𝑅 − 2𝑟𝑟
10 𝑇𝑇𝑄𝑄 = 12 − 2 � � 3 16 𝑇𝑇𝑄𝑄 = 3 Sehingga, 16 𝑇𝑇𝑄𝑄 � 3 � 4 = = 12 9 𝑇𝑇𝑅𝑅
Karenanya,
4 𝑟𝑟2 = � � 𝑟𝑟 9
4 4 4 2 4 𝑟𝑟3 = � � 𝑟𝑟2 = � � � � 𝑟𝑟 = � � 𝑟𝑟 9 9 9 9
4 4 4 4 4 3 𝑟𝑟4 = � � 𝑟𝑟3 = � � � � � � 𝑟𝑟 = � � 𝑟𝑟 9 9 9 9 9
| 456
Dan seterusnya sampai jari-jari lingkaran terakhir. Sehingga luas seluruh lingkaran dalam segitiga itu akan mengikuti perubahan pada jari-jarinya, yaitu: 10 2 𝐿𝐿𝑢𝑢𝑎𝑎𝑠𝑠 𝐼𝐼 = 𝜋𝜋 � � 3
2
4 10 10 2 4 2 𝐿𝐿𝑢𝑢𝑢𝑢𝑢𝑢 𝐼𝐼𝐼𝐼 = 𝜋𝜋(𝑟𝑟2)2 = 𝜋𝜋 �� � � �� = 𝜋𝜋 � � � � 9 3 3 9 2
4 2 10 10 2 4 4 𝐿𝐿𝑢𝑢𝑎𝑎𝑎𝑎 𝐼𝐼𝐼𝐼𝐼𝐼 = 𝜋𝜋(𝑟𝑟3)2 = �� � � �� = 𝜋𝜋 � � � � 9 3 3 9
Dan seterusnya,....
Sehingga akan terbentuk deretan luas lingkaran sebagai berikut: 𝐿𝐿𝑢𝑢𝑢𝑢𝑠𝑠 𝐾𝐾𝑒𝑒𝑒𝑒𝑒𝑒𝑒𝑒𝑒𝑒𝑒𝑒𝑒𝑒ℎ𝑎𝑎𝑛𝑛 = 𝐿𝐿𝑢𝑢𝑢𝑢𝑢𝑢 + 𝐿𝐿𝑢𝑢𝑢𝑢𝑢𝑢 𝐼𝐼𝐼𝐼 + 𝐿𝐿𝑢𝑢𝑢𝑢𝑢𝑢 𝐼𝐼𝐼𝐼𝐼𝐼 + ⋯ ..
10 2 4 2 10 2 4 4 10 2 𝐿𝐿𝑢𝑢𝑢𝑢𝑢𝑢 𝐾𝐾𝑒𝑒𝑒𝑒𝑒𝑒𝑒𝑒𝑒𝑒𝑒𝑒𝑒𝑒ℎ𝑎𝑎𝑛𝑛 = 𝜋𝜋 � � + 𝜋𝜋 � � � � + 𝜋𝜋 � � � � 3 9 3 9 3
4 2 4 4 10 2 𝐿𝐿𝑢𝑢𝑢𝑢𝑢𝑢 𝐾𝐾𝑒𝑒𝑒𝑒𝑒𝑒𝑒𝑒𝑒𝑒𝑒𝑒𝑒𝑒ℎ𝑎𝑎𝑛𝑛 = 𝜋𝜋 � � �1 + � � + � � + ⋯ . � 9 9 3 Perhatikan, Kawan-kawan! Bahwa suku-suku: 4 2 4 4 4 8 �1 + � � + � � + � � … . � 9 9 9
Adalah rangkaian suku yang membentuk Deret Geometri Tak Berhingga, dengan suku pertama adalah a = 1 dan ratio, r-nya (4/9)2, sehingga:
Maka,
𝑆𝑆~ =
𝑎𝑎 1 − 𝑟𝑟
| 457
4 2 4 4 4 8 �1 + � � + � � + � � … . � = 9 9 9
Sehingga,
10 2 𝐿𝐿𝑢𝑢𝑢𝑢𝑢𝑢 𝐾𝐾𝑒𝑒𝑒𝑒𝑒𝑒𝑒𝑒𝑢𝑢𝑟𝑟𝑟𝑟ℎ𝑎𝑎𝑛𝑛 = 𝜋𝜋 � � � 3
1
4 2 1−� � 9 1
4 2 1 − �9�
�
10 2 81 100𝑥𝑥81 𝐿𝐿𝑢𝑢𝑢𝑢𝑢𝑢 𝐾𝐾𝑒𝑒𝑒𝑒𝑒𝑒𝑒𝑒𝑒𝑒𝑒𝑒𝑒𝑒ℎ𝑎𝑎𝑛𝑛 = 𝜋𝜋 � � � � = 𝜋𝜋 3 65 9𝑥𝑥65 180𝜋𝜋 𝑠𝑠𝑎𝑎𝑎𝑎𝑎𝑎𝑎𝑎𝑎𝑎 𝑙𝑙𝑢𝑢𝑢𝑢𝑢𝑢 𝐿𝐿𝑢𝑢𝑢𝑢𝑢𝑢 𝐾𝐾𝑒𝑒𝑒𝑒𝑒𝑒𝑒𝑒𝑒𝑒𝑒𝑒𝑒𝑒ℎ𝑎𝑎𝑛𝑛 = 13
Itu hasil yang kita dapatkan, Kawan-kawan!”pungkas Sabiq. Spontan tepuk tangan meriah tak terbendung. Bahkan dari beberapa wajah tampak rasa haru. Yodha, Sang Ahli Geometri pun tampak menitikan air mata. Ada kebanggaan atas capaian ini. Terasa beban yang menyesakan dadanya hilang seketika. Lain lagi dengan Amer, laki-laki yang baru mencintai Geometri itu, kesebangunan khususnya, tak bisa menutupi rasa harunya, kalimat takbir, “Allahu Akbar!” terlontar dari mulutmya. Ayra pun begitu, perempuan satu-satunya dalam Kesebelasan Genk OSN ini tampak berkaca-kaca. Iman, laki-laki yang menyimpan banyak kejutan ini tampak tertunduk
sambil
mulutnya
tak
putus
mengucap
Istighfar,
“Astaghfirullah!” dan Denan, Ikram, Bily, Fateh serta Saver hanya berujar, “Subhanallah!”. Tampaknya, persoalan terakhir Lontar Sabiq ini mengguncang mereka. Namun, kebersamaan, kesederhanaan, ketekunan dan
| 458
ketelitian yang dilandasi oleh ketulusan menemukan kebenaran telah menuntun mereka pada tujuan yang diharapkan. “Biq, maafkan ucapan kasar dan gelar-gelar yang telah kuberikan padamu. Aku tarik gelar Dukun dan Ban Serep darimu. Kau adalah Kawan Sejati. Kita semua Kawan. Kau tetaplah Sastrawan-Penerjemah. Itu cukup bagimu,”ucap Amer pelan sambil mengusap bagian atas pipinya dan berjalan mendekat Sabiq dan memeluknya dengan hangat. “Tak apa Kawan, what’s in a name? Apalah arti sebuah nama? Bukankah begitu Kawan. Indahnya namaku Sabiq Sang Dukun, Penerjemah, Sastrawan, dan Ban Serep!”canda Sabiq. Terdengar tawa yang memecah siang menjelang Dzuhur itu, dari sebuah rumah sederhana di Desa Terpadat dan Kecamatan terbesar yang terletak 45 kilometer arah Timur dari pusat kota itu. Tawa yang berasal dari sebelas anak jenius kampong yang menggenggam cita-cita besar. Cita-cita perubahan yang melampaui usia dan zamannya. …∑πχαΩ…
| 459
17 Akhirnya........... “Pak, Bapak, Bangun Pak!”tiba-tiba aku mendengar suara cukup keras di telingaku. Bukan hanya itu, kurasakan seseorang menggoyang-goyangkan bahuku dengan keras. “Astaghfirullah al-Adhiem, Paman tertidur anak-anak!”kalimat pertama yang kuucapkan saat kubuka mata dan kulihat tujuh anak muda duduk rapi bersila, tepat di depanku. “Ya, Paman. Tadi kami lihat Paman pulas betul tidurnya. Kami tak berani membangunkan. Kami menunggu, Paman Bangun. Tapi, kata
Bibi,
bangunkan
saja.
Ya,
akhirnya,
Rayhan
yang
membangunkan Paman,”terang Saffana. “Paman mohon maaf, ya anak-anak. Memang beberapa hari ini Paman sering tertidur bila sedang duduk atau bersandar. Apalagi ada hembusan angin sepoi-sepoinya,”terangku. Aku membetulkan letak dudukku. Kulihat ketujuh anak itu sudah siap dengan dua lembar kertas. Selembar di tangan Saffana, dan selembar lagi di tangan Zaidan. “Oh iya, Paman hampir lupa. Kalian sudah selesai mengerjakan kedua soal itu? Siapa yang akan maju dulu? Rey, siapkan papan tulis dan alat tulisnya”kataku. “Sudah, Pak. Kami sudah siapkan semuanya,”jawab anakku. | 460
“Baiklah
kalau
begitu,
kelompok
berapa
yang
akan
memulai?”tanyaku. “Kelompok kami, Paman. Kelompok 2 dan Zaidan adalah Juru Bicaranya, Paman!”jawab Saed. Zaidan maju ke depan. Kuperhatikan anak itu cukup tenang. Beberapa saat dia berhenti di depan papan tulis. Akhirnya, dia pun memulai kalimatnya. “Soal yang diberikan Paman kepada kelompok kami adalah soal tentang bilangan, khususnya bilangan kuadrat sempurna. Disebutkan bahwa ‘the five-digit number 2xy89 is the square of an integer. Find the two-digit number xy! Terhadap soal ini kami memulainya dengan pemisalan,”kalimat pembuka Zaidan. Aku seperti menemukan semangat sebelas sahabat, anak-anak jenius kampong dulu dalam diri Zaidan. “Misalkan bilangan itu adalah n2, maka dari informasi soal, dapat diketahui bahwa: 𝑛𝑛2 = 2𝑥𝑥𝑥𝑥89
Dari angka yang dihasilkan kita dapat menebak bahwa n mestilah sebuah bilangan yang terdiri atas 3 angka. Bukan hanya itu, ketiga angka itu haruslah dimulai dengan angka 1 dan diakhiri oleh angka 3 atau angka 7!”papar Zaidan. Perasaanku semakin terguncang. “Persis!”pikirku. Zaidan kembali melanjutkan keterangannya, sementara temantemannya memerhatikan dengan serius. “Karenanya, kemungkinan bentuk dan susunan angka adalah 1p3 atau 1p7. Selanjutnya, kami melakukan coba-coba atas beberapa | 461
nilai
kuadrat
dari
bilangan
tiga
angka,
Paman.
Sehingga
kemungkinan bilangan kuadrat yang menghasilkan bilangan lima angka 2xy89 adalah terletak di antara 1402 = 19.600 dan 1802 =
32.400. Bila dibuat dalam rentang nilai, maka bilangan lima angka
2xy89 itu akan memenuhi 1402 = 19.600 < 2𝑥𝑥𝑥𝑥89 < 32.400 = 1802 ”terang Zaidan.
Aku menikmati paparan Zaidan ini. Kembali Zaidan yang
ditunjuk sebagai Juru Bicara itu menulis di papan tulis. Dan gaya bicaranya serta kegemarannya atas tabel pun mengingatkanku pada seseorang, dulu! “Dari batasan minimal pada 1402 dan 1802 itu, kami hubungkan dengan bentuk yang mungkin di awal, yaitu 1p3 atau 1p7, selanjutnya, kami kembali membuat tabel kemungkinan untuk angka satuan 3 (1p3), dan untuk angka satuan 7 (1p7). Hasil dari tabel itu adalah sebagai berikut!”terang Zaidan sambil menggambar sebuah tabel.
Dari beberapa kemungkinan nilai kuadrat bilangan tiga angka di atas, maka kami menemukan bahwa ternyata bilangan kuadrat tiga angka yang memenuhi adalah 167, karena 1672 = 27.889. Sehingga, jawaban atas ‘find the two-digit number xy dari the five-digit number
| 462
2xy89 adalah 78! Seperti itu yang kami kerjakan, Paman,”pungkas Zaidan. Sekali lagi aku hanya bisa mengangguk-angguk, dan tersenyum. “Berikan applause untuk Kelompok 2 anak-anak!”pintaku. Terdengar tepuk tangan meriah pagi itu di rumahku. Dan aku seperti merasakan aroma yang sama etpuka seperti itu, dulu. “Baik,
Kelompok
1
sudah
siap?
Siapa
yang
akan
mewakili?”tanyaku. “Alhamdulillah, kami sudah siap, Paman. Dan Saffana akan mewakili kelompok kami,”kata Yaseen. “Silahkan, Yaseen!”balasku. Anak laki-laki berkulit putih dan bertubuh kurus ini berjalan pelan. Tapi, kemantapan terpancar jelas di wajahnya. “Mohon maaf sebelumnya, Paman. Mungkin apa yang akan kami kerjakan ini tidak pas atau kurang berkenan,”kalimat pembuka Yaseen. Ternyata kelembutan dan kesantunan adalah bagian melekat dalam diri anak muda yang juga berwajah culun alias baby face ini. “Kelompok kami diminta untuk menentukan angka berikutnya setelah deretan angka-angka. Disebutkan bahwa ‘predict the next term in the sequence: 1, 14, 51, 124, 245, 426, ...”. Terhadap pertanyaan ini, kami menggunakan apa yang disebut sebagai Tabel Perbedaan Bertingkat,”terang Yaseen. Kali ini aku agak terkejut, saat anak muda ini menyebut Tabel Perbedaan Bertingkat. Tapi aku berusahan menyembunyikan keterkejutanku itu dalam senyum tipis yang mengembang di | 463
wajahku. Kulihat istriku yang duduk agak menjauh dari anak-anak senyum-senyum dan mengangguk-angguk sambil menatapku. “Apa yang Yaseen maksud dengan Tabel Perbedaan Bertingkat itu?”pancingku. “Sebenarnya ‘bukan tabel yang berbentuk kolom dan baris’, Paman. Hanya istilah saja. Yaitu angka-angka yang berisi perbedaan dari setiap suku dengan suku berikutnya. Kenapa kami sebut bertingkat? Karena proses seperti itu akan terus kami lakukan sampai perbedaan antara suku-suku sebelum dan sesudahnya adalah konstan atau tetap. Karenanya sangat mungkin bisa satu tahap, dua tahap, bahkan tiga tahap. Tahapan-tahapan ini yang kami sebut sebagai bertingkat, Paman!”terang Yaseen. Aku merasa puas dengan keterangan anak ini. “Karenanya, setelah kami susun ulang angka-angka yang diberikan,
dan
kami
menemukan
pola
seperti
gambar
di
berikut!”terang Yaseen lagi, dan kali ini ia menggambar sesuatu di papan tulis.
“Setelah kami urutkan bilangan yang diberikan, kami menemukan perbedaan antara suku pertama dengan suku berikutnya pada deretan itu beragam, yaitu: 13, 37, 73, 121, dan 181. Karenanya | 464
kami berkesimpulan belum terbentuk pola. Selanjutnya, kami melakukan langkah yang sama terhadap deretan bilangan yang merupakan perbedaan dari bilangan-bilangan pada deretan bilang yang pertama. Hasilnya, kami menemukan perbedaan yang berbeda meskipun sudah menunjukkan peningkatan yang konstan, yaitu: 24, 36, 48, 60. Karenanya kami melakukan hal yang sama terhadap deretan hasil perbedaan dari perbedaan sebelumnya. Dan ternyata kami menemukan bahwa perbedaan antara satu bilangan dengan bilangan berkutnya adalah konstan yaitu 12. Karenanya, kami berkesimpulan bahwa prosesnya sudah selesai dan hasilnya bisa diketahui,”terang Yaseen. Aku sungguh-sungguh menikmati penjelasan anak ini. “Dan akhirnya, kami melakukan perhitungan ke atas terhadap bilangan terakhir dan beda antara bilangan yang diketahui, sehingga: a. Dengan menambahkan 12 pada angka keempat perbedaan kedua, yaitu 60, akan menghasilkan angka 72; b. Dengan menambahkan 72 pada angka kelima perbedaan pertama, yaitu 181, akan menghasilkan 253; c. Dengan menambahkan 253 pada angka terakhir pada barisan bilangan, yaitu 426, akan menghasilkan 679. Akhirnya, jawaban atas pertanyaan predict the next term in the sequence: 1, 14, 51, 124, 245, 426, ...adalah 679. Begitu yang bisa kami sajikan, Paman!”tutup Yaseen. “Subhanallah! Kalian memang anak-anak yang pintar. Selamat, selamat, selamat! Dan tadi itu penyelesaian yang tepat seluruhnya, baik Kelompok 1 maupun Kelompok 2,”pujiku pada anak-anak. | 465
“Alhamdulillah!”terdengar ucapan hamdalah berbarengan. “Jadi bagaimana, Paman?”tiba-tiba Saed bertanya padaku. Aku tak langsung menjawab. Aku diam beberapa saat. Kupandangi istriku. Tampak ia tersenyum dan mengangguk. Aku menarik nafas panjang. “Kalian tunggu sebentar!”pintaku. Aku beranjak. Kali ini tujuanku adalah lemari klasik tempat penyimpanan buku-buku lawasku. Kupandangi kembali buku-buku tua itu. Dan akhirnya pandanganku berhenti pada sebuah buku ukuran setengah kertas A-4 Tidak itu bukan hanya buku, lebih tepatnya sebuah Novel. Kuambil, dan aku kembali ke anak-anak yang setia menunggu sambil duduk bersila. “Anak-anak, sebagai langkah awal untuk niat tulus dan citacita kalian itu, Paman punya hadiah. Mulailah dari Novel ini! Saffana, kau bertanggung jawab atas Novel ini. Baca, telaah, pelajari, dan diskusikan! Insyaallah, Ada Jalan!”ujarku singkat. “Terima kasih, Paman. ‘Olimpiade? Insyaallah, Ada Jalan! {Cerita Sebelas Sahabat} Novel Matematika’. Wah, ini artinya Paman bersedia membimbing kita, teman-teman!”tiba-tiba Saffana bersorak dan meninggikan suaranya. Aku hanya tersenyum dan menganggukkan kepala. Kulihat istriku pun tersenyum di sudut ruang pertemuan itu.
T
A
M
A
T
| 466
Sumber Inspirasi Aha! Solutions by Martin Erickson. American Mathematics Competition 8 A Prime for Mathematics Competitions by Alexander Zawaira and Gavin Hitchcock.
Competitions Mathematics by Haese, Haese, Webber and Bruce.
Diktat Pembinaan Olimpiade Matematika Materi Dasar by Eddy Hermanto Elementary Number Theory with Applications by Thomas Koshy.
Gazeta Matematica A Bridge Over Three Centuries by Vasile Berinde and Eugen Paltanea.
Junior Balkan Mathematical Olympiads by Dan Branzel, Ioan Serdean and Vasile Serdean.
Kumpulan Soal Kompetisi Matematika Nalaria Realistik SMP oleh Ridwan Hasan Saputra Lecture Notes on Mthematical Olympiad Courses by Xu Jiagu.
Mathcounts Mathematics Sport by Mohammad Tohir. Mathematical Olympiad Treasures by Titu Andreescu and Bogdan Enescu.
Mathematical Excursions by Aufman, Lockwood, Nation and Clegg.
Math Olympiad Contest Problems for Elementary and Middle Schools by George Lenchner. Olimpiade Matematika untuk SMU oleh Suwah Sembiring.
Pintarmatematika.net by Tutur Widodo The Holy Qur’an by Abdullah Yusuf Ali. The Art and Craft of Problem Solving by Paul Zeitz The Standford Mathematics Problem Book With Hints & Solutions by G. Poyla and J. Kilpatrick. The Art of Problem Solving by Sandor Lehoczky and Richard Rusczyk.
| 467